316 Pharm Question bank

Ace your homework & exams now with Quizwiz!

The nurse mixes a short-acting and an intermediate-acting insulin in the same syringe. List the actions in the order the nurse will perform them. 1. Put air into the intermediate-acting insulin vial. 2. Put air into the short-acting insulin vial. 3. Withdraw the prescribed amount of short-acting insulin. 4. Withdraw the prescribed amount of intermediate-acting insulin.

1. Put air into the intermediate-acting insulin vial. 2. Put air into the short-acting insulin vial. 3. Withdraw the prescribed amount of short-acting insulin. 4. Withdraw the prescribed amount of intermediate-acting insulin.

The nurse is preparing to administer insulin to a client with diabetes. In which order will the nurse perform the actions associated with insulin administration? 1. Wash hands with soap and water. 2. Rotate the vial of insulin between the palms of the hands. 3. Wipe the top of the insulin vial with an alcohol swab. 4. Instill air into the vial of insulin equal to the desired dose. 5. Withdraw the correct amount of insulin from the inverted vial.

1. Wash hands with soap and water. 2. Rotate the vial of insulin between the palms of the hands. 3. Wipe the top of the insulin vial with an alcohol swab. 4. Instill air into the vial of insulin equal to the desired dose. 5. Withdraw the correct amount of insulin from the inverted vial.

The nurse is educating an older adult client about newly prescribed levofloxacin for the treatment of pneumonia. The nurse should teach the client that which side effect is a priority for the client to report to the provider? A . Joint tenderness B. Diarrhea C. Dizziness D. Difficulty sleeping

A . Joint tenderness Rationale: There is a black box warning for fluoroquinolones alerting health professionals not only to the increased disabling risk of tendinitis and tendon rupture but also to the significant risk of peripheral neuropathy, central nervous system and cardiac effects, and dermatologic and hypersensitivity reactions. Signs of tendonitis and tendon rupture include pain and tenderness in the affected limb or joint. The medication must be stopped immediately. The other options are common side effects and while reportable, are not a priority.

The nurse is preparing to administer doxycycline to a client to treat syphilis. Which lab results should the nurse review before administering this medication? A . Pregnancy test B. Hematocrit C. Sodium level D. Arterial blood gas

A . Pregnancy test Rationale: Tetracyclines, such as doxycycline, may cause fetal harm and should not be administered during pregnancy. It is important to know the client's pregnancy status prior to administration. Reviewing hematocrit, serum sodium level, and ABGs may be a part of the client's assessment, but these do not affect the prescription for doxycycline.

The nurse in the neurology office is reviewing information about levetiracetam with a 30-year-old female client with a history of seizures. Which instruction about the medication should the nurse make sure to include? A. "Call the office immediately if you feel like hurting or killing yourself." B. "You should avoid becoming pregnant while taking this medication." C. "You should stay away from large crowds and sick children." D. "You might experience irregular menses and intermittent bleeding."

A. "Call the office immediately if you feel like hurting or killing yourself." Rationale: Levetiracetam is an anti-convulsant medication used to prevent seizures. One of the significant side effects is behavioral changes and suicidal ideations. It is important to notify the provider's office immediately if the client experiences these thoughts. The other instructions do not apply to this particular medication.

The triage nurse at a health clinic receives a call from a client. The client states that they have been experiencing flu-like symptoms for the past 24 hours. The client asks for a prescription for zanamivir. How should the triage nurse respond? A. "Come in right away so we can start treating you." B. "Do you have trouble swallowing big pills?" C. "We will call your pharmacy for an antibiotic prescription for you." D. "Call back tomorrow when you are sure you have the flu."

A. "Come in right away so we can start treating you." Antiviral influenza treatment with zanamivir should be initiated within 48 hours of onset of symptoms, thus it is important to get treatment started as soon as possible. The medication won't cure the disease, it will only shorten the time frame that someone is sick and may reduce the severity of the illness. It is administered by oral inhalation. Antibiotics are not an appropriate treatment for the flu.

Which instruction regarding nutrition will the nurse give a client discharged after a short hospitalization for an episode of a transient ischemic attack (TIA) related to hypertension who is on a regimen that includes chlorothiazide? A. "Eat more dark green, leafy vegetables such as spinach." B. "Substitute a potassium-based salt substitute for table salt." C. "Return to previous eating habits." D. "Increase intake of dairy products."

A. "Eat more dark green, leafy vegetables such as spinach." The client should increase the dietary intake of potassium because of potassium loss associated with chlorothiazide. Leafy green vegetables are high in potassium and should be encouraged. Salt substitutes should only be used if prescribed by the provider; otherwise, they should be discouraged because electrolyte abnormalities may occur without close monitoring. Returning to previous eating habits may be unsafe for those who do not consume a nutritional diet; the client should be taught about medication-induced deficiencies and how to try to prevent future TIAs. Dairy products should be limited, unless fat-free, because they are high in saturated fats.

The nurse is caring for a female client who is requesting hormonal contraceptives. Which of the following questions should the nurse ask to assess for contraindications? A. "Have you ever had a blood clot?" B. "How many children do you have?" C. "Do you drink alcohol?" D. Did you experience acne in adolescence?

A. "Have you ever had a blood clot?" Rationale: A history of thromboembolic disorders is a contraindication to hormonal contraceptives; therefore, any history of thrombus should be assessed. The number of children/pregnancies and use of alcohol are probable history questions but are not contraindications to this method. Acne is a side effect of oral contraceptives but not a contraindication.

A nurse is administering an intravenous piggyback infusion of penicillin. Which client statement would require the nurse's immediate attention? A. "I am itching all over." B. "I have soreness and aching in my muscles." C. "I have cramping in my stomach." D. "I have a burning sensation when I urinate."

A. "I am itching all over." Allergic reactions to medications can include itching all over. This can be further supported by the presence of hives or welts. Abdominal pain or cramping could indicate a side effect of the penicillin. The other symptoms of muscle soreness and painful urination are not as urgent as the itching.

Which client statement demonstrates an understanding of cyanocobalamin (vitamin B 12) prescribed for pernicious anemia? A. "I should have a vitamin B 12 injection every month." B. "I'll take vitamin B 12 supplements every morning with my breakfast." C. "I'll eat a diet high in green vegetables." D. "I will increase my intake of processed foods fortified with vitamin B 12."

A. "I should have a vitamin B 12 injection every month." Vitamin B 12 is administered via injection on a weekly or monthly basis. For the client with pernicious anemia, there is inadequate intrinsic factor for adequate absorption of vitamin B 12. Green vegetables are not an important source of vitamin B 12. Vitamin B 12 is found primarily in meat, fish, poultry, and eggs. Although there is an abundance of foods fortified with vitamin B 12, for the client with pernicious anemia, the vitamin will not be absorbed in adequate amounts secondary to lack of intrinsic factor.

Which statement by a client with metastatic melanoma who is being treated with interferon gamma 1b indicates that teaching was understood? A. "I will increase my fluid intake to several liters (quarts) every day." B. "I need to discard any reconstituted solution at the end of the week." C. "I can continue driving my car as before as long as I have the stamina." D. "I should be able to continue my usual activity while taking this medication." Rationale

A. "I will increase my fluid intake to several liters (quarts) every day." Increasing fluid intake to several liters (quarts) every day helps flush the kidneys and prevent nephrotoxicity, especially during the early phase of treatment. Reconstituted solution can be stored in the refrigerator for 1 month. Confusion, dizziness, and hallucinations are adverse effects of this medication; the client should avoid hazardous tasks, such as driving or using machinery. Activity may have to be altered because fatigue and other flu-like symptoms are common with this medication.

A client has been taking rosuvastatin for six weeks as part of a treatment plan to reduce hyperlipidemia. The clinic nurse is reviewing and reinforcing information about the medication with the client. Which statements by the client indicates an understanding about the medication? Select all that apply. A. "I will need to call my doctor if I have any muscle weakness or pain, especially in my legs." B. "I will need to come back to have my liver and kidney labs checked." C. "I need to be careful when I get up because this medication can make my blood pressure drop." D. "I add some nuts and fresh fruit to my oatmeal in the morning and I can't remember when I last ate a steak." E. "This medication has to be taken first thing in the morning, before I eat breakfast."

A. "I will need to call my doctor if I have any muscle weakness or pain, especially in my legs." B. "I will need to come back to have my liver and kidney labs checked." D. "I add some nuts and fresh fruit to my oatmeal in the morning and I can't remember when I last ate a steak." Clients taking rosuvastatin need to be monitored for alteration in liver function. An adverse effect of rosuvastatin is muscle pain and weakness (rhabdomyolysis). Left untreated, rhabdomyolysis can lead to renal impairment. The medication does not affect blood pressure or cause orthostatic hypotension. The client should be taught to follow a low-cholesterol diet, which includes increasing intake of whole grains and limiting intake of foods high in saturated fats, trans fats and dietary cholesterol. The medication is ordered once a day. The client can take it at any time of day, preferably at the same time of day each day, before or after eating.

A client has been prescribed alendronate for osteoporosis. Which statements indicate that the client understands how to safely take this medication? Select all that apply. A. "I will notify my doctor if I experience worsening heartburn." B. "I will take the pill with an antacid to prevent stomach upset." C. "I will swallow the pill with a full glass of water." D. "I will stand or sit quietly for 30 minutes after taking the pill." E. "I will always eat breakfast before taking the pill."

A. "I will notify my doctor if I experience worsening heartburn." C. "I will swallow the pill with a full glass of water." D. "I will stand or sit quietly for 30 minutes after taking the pill." Alendronate is a bisphosphonate used to treat osteoporosis. It can cause esophagitis or esophageal ulcers unless precautions are followed. The client must sit upright or stand for at least 30 minutes after taking the medication. The client should take the medication with a full glass of water, at least 30 minutes before eating or drinking anything or taking any other medication. Antacids will interfere with absorption and should not be taken at the same time.

The nurse is providing discharge education to a client diagnosed with coronary artery disease. The client is prescribed to use a nitroglycerin transdermal patch at home. Which statement by the client indicates a correct understanding of safe medication administration? A. "I will remove the old patch and cleanse the area before applying a new patch." B. "This drug can lead to hypertension. So, I will monitor my blood pressure at home." C. "I will keep a record of chest pain occurrences now that I have this patch." D. "I can place this patch on broken skin. It will absorb better."

A. "I will remove the old patch and cleanse the area before applying a new patch." Numerous administration errors have been reported with nitroglycerin paste and patches. The errors include improper storage and basic administration. The client should be taught to remove the previous patch before applying the new patch and to properly label the tube of nitroglycerin paste and keep it out of the reach of children. When selecting an area to place the patch, the skin should be intact and show no signs of irritation. Nitroglycerin paste has been used erroneously as lotion and caused toxic effects. Nitroglycerin causes vasodilation, which increases the blood supply through the coronary arteries. This may cause hypotension in clients. Some other common side effects include lightheadedness, nausea, dizziness, headache and redness or irritation of the skin covered by the patch.

How will the nurse respond to a client with a new diagnosis of type 1 diabetes who becomes agitated and says, "I am scared of shots. If that is my only option, I'll just have to go into a coma and die!" when told that lifelong insulin will be needed? A. "Injections are not the only option available for insulin." B. "It won't be so bad; you will get used to it if you will only try." C. "This is one of those times when you need to act like an adult." D. "Clients have the right to refuse treatment, but I need you to sign this form that removes us from liability for your decision."

A. "Injections are not the only option available for insulin." An insulin nasal spray was approved by the Food and Drug Administration (FDA) in 2014 and is available for clients who do not want insulin injections. The nurse should use therapeutic communication in interacting with clients. Intimidating the client by suggesting that actions are childlike and suggesting that the client's concerns are not significant are not therapeutic responses. The nurse's primary concern should be for the client's well-being, not protection from liability.

The nurse is reinforcing teaching for a client with chronic kidney disease about the prescribed aluminum hydroxide. Which is the best statement by the nurse about this medication? A. "It decreases phosphate levels." B. "It increases urine output." C. "It reduces potassium levels." D. "It controls stomach acid secretions."

A. "It decreases phosphate levels." Phosphates tend to accumulate in the client with chronic kidney disease due to decreased filtration capacity of the kidneys. Antacids that contain aluminum such as aluminum hydroxide (Amphojel) are commonly used to lower phosphate levels. Aluminum binds phosphates in the gastrointestinal tract and prevents their absorption. Aluminum hydroxide neutralizes stomach acid already present, but does not control gastric acid production or secretion. It does not affect potassium absorption or levels.

The caregiver of a client with Alzheimer's disease asks the nurse for information about different treatment options that can help with memory or behavior problems. Which of the following responses by the nurse are correct? Select all that apply. A. "Music therapy has been found to help some clients." B. "Ginkgo biloba may help with memory." C. "Acupuncture may be very relaxing." D. "Donepezil (Aricept) may help slow cognitive decline." E. "Garlic may help with this disease."

A. "Music therapy has been found to help some clients." B. "Ginkgo biloba may help with memory." D. "Donepezil (Aricept) may help slow cognitive decline." Some complementary and integrative health therapies may help with the symptoms of Alzheimer's disease. Music, art and dance therapies can help with behavior issues. Ginkgo biloba may be used to improve memory. Acupuncture may be a frightening experience for someone with Alzheimer's disease. Garlic is not a treatment for Alzheimer's disease. Donepezil (Aricept) is used to ease the symptoms associated with Alzheimer's disease.

A nurse is giving instructions to the parents of a newborn infant with oral candidiasis. Which statement made by a parent is incorrect and indicates a need for more teaching? A. "The therapy can be discontinued when the spots disappear." B. "I will boil the nipples and pacifiers for 20 minutes." C. "I will use a dropper to place the medicine on each side of my baby's mouth." D. "Nystatin should be given four times a day after my baby eats."

A. "The therapy can be discontinued when the spots disappear." The therapy should be continued for a week, even if lesions have disappeared within a few days. If the mother is breast-feeding, mother and baby should be treated at the same time to prevent re-infection.

Which rationale will the nurse provide to a client with Crohn's disease who asks why the prescribed vitamins have to be given intravenously (IV) rather than by mouth? Select all that apply. One, some, or all responses may be correct. A. "They provide more rapid action results." B. "They decrease colon irritability." C. "Oral vitamins are less effective." D. "Intestinal absorption may be inadequate." E. "Allergic responses are less likely to occur."

A. "They provide more rapid action results." C. "Oral vitamins are less effective." D. "Intestinal absorption may be inadequate." Absorption through the gastrointestinal (GI) tract is impaired, and parenteral administration goes directly into the intravascular compartment. Disease of the GI tract hampers absorption. Because the mucosa of the intestinal tract is damaged, its ability to absorb vitamins taken orally is greatly impaired. IV vitamins do not decrease colonic irritability. Route of administration does not affect allergic response.

A client calls the clinic and states to the triage nurse: "I had an upset stomach and took Pepto-Bismol and now my tongue looks black. What's happening to me?" What would be the nurse's best response? A. "This is a common and temporary side effect of this medication." B. "How long have you had an upset stomach?" C. "Come to the clinic so you can be seen by the health care provider." D. "Are your stools also black?"

A. "This is a common and temporary side effect of this medication." The best response would be to explain that a dark tint of the tongue is a common and temporary side effect of bismuth subsalicylate (Pepto-Bismol). Although it may also turn stools a darker color, do not confuse this with black, tarry stools, which is a sign of bleeding in the intestinal tract. After addressing the client's initial concern and the reason for the call, the nurse can ask about the upset stomach and then ask the client to come to the clinic if necessary.

The nurse is educating a client with end-stage renal failure about newly prescribed aluminum hydroxide. Which statement should the nurse include in the teaching? A. "This medication binds with phosphates from food to decrease absorption." B. "This medication is used to decrease urea to prevent urticaria." C. "This medication will coat the lining of the stomach to decrease acid production." D. "This medication treats hyperkalemia by exchanging sodium for potassium in the intestines."

A. "This medication binds with phosphates from food to decrease absorption." Rationale: Hyperphosphatemia occurs in end-stage renal failure when kidneys can no longer filter out phosphorus. Treatment of hyperphosphatemia may include the administration of aluminum hydroxide as a phosphate-binding agent. The aluminum binds with phosphates which are excreted in the feces. Sodium polystyrene is used to treat hyperkalemia by exchanging sodium for potassium in the intestines. Dialysis is used to remove urea from the blood, and diphenhydramine is used to treat urticaria. Sucralfate is a medication that coats the stomach lining to decrease acid production.

Which statement indicates that a female client who is receiving rifampin for tuberculosis understands the teaching? Select all that apply. One, some, or all responses may be correct. A. "This medication may be hard on my liver, so I must avoid alcoholic drinks while taking it." B. "This medication may reduce the effectiveness of the oral contraceptive." C. "I cannot take an antacid within 2 hours before." D. "My health care provider must be called immediately if my eyes and skin become yellow."

A. "This medication may be hard on my liver, so I must avoid alcoholic drinks while taking it." B. "This medication may reduce the effectiveness of the oral contraceptive I am taking." D. "My health care provider must be called immediately if my eyes and skin become yellow." Alcohol may increase the risk of hepatotoxicity. Rifampin has teratogenic properties and may reduce the effectiveness of oral contraceptives. Yellowing of the eyes and skin are signs of hepatitis and should be reported immediately. An antacid may be taken 1 hour before taking the medication.

A client with hyperthyroidism is being treated with propylthiouracil (PTU). Which instruction will the nurse include in the teaching plan regarding this medication? Select all that apply. One, some, or all responses may be correct. A. 'Avoid abrupt discontinuation of the medication.' B. 'Monitor your weight, pulse, and mood routinely.' C. 'You can expect an immediate response to this medication.' D. 'Also take an iodine replacement to aid metabolism of the medication.' E. 'Report side effects, such as sore throat, fever, joint pain, or oral lesions.'

A. 'Avoid abrupt discontinuation of the medication.' B. 'Monitor your weight, pulse, and mood routinely.' E. 'Report side effects, such as sore throat, fever, joint pain, or oral lesions.' Abrupt discontinuation of the medication may result in thyroid crisis. PTU blocks the synthesis of T 3 (triiodothyronine) and T 4 (thyroxine). The therapeutic effect of the medication should result in increased weight, decreased pulse, and stability of mood. Sore throat, joint pain, fever, or oral lesions may indicate infection caused by medication-induced blood dyscrasias, such as leukopenia and agranulocytosis. The response to this medication may take up to 3 weeks. Over-the-counter medications and seafood containing iodine should be avoided.

Trimethoprim-sulfamethoxazole is prescribed for a client with cystitis. Which instruction would the nurse include when providing medication teaching? A. 'Drink eight to ten glasses of water daily.' B. 'Take this medication with orange juice.' C. 'Take the medication with meals.' D. 'Take the medication until symptoms subside.'

A. 'Drink eight to ten glasses of water daily.' A urinary output of at least 1500 mL daily should be maintained to prevent crystalluria (crystals in the urine). Taking the medication with orange juice provides no advantage. Also, orange juice produces an alkaline ash, which results in an alkaline urine that supports the growth of bacteria. Trimethoprim-sulfamethoxazole should be taken 1 hour before meals for maximum absorption. A prescribed course of antibiotics must be completed to eliminate the infection, which can exist on a subclinical level after symptoms subside.

The nurse teaches a client who is scheduled for a kidney transplant about the need for immunosuppressive medications. The nurse determines that the client understands the teaching when the client says that medications must be taken for which period of time? A. 'For the rest of my life.' B. 'Until the surgery is over.' C. 'Until the surgery heals.' D. 'During the intraoperative period.'

A. 'For the rest of my life.' These medications must be taken continuously for life to prevent rejection of the transplanted organ.

Hydrochlorothiazide (HCTZ) has been prescribed for a client with hypertension. The client reports hearing that furosemide is more effective and requests a prescription change. How will the nurse respond? A. 'HCTZ has fewer side effects.' B. 'HCTZ does not cause dizziness.' C. 'HCTZ is only taken when needed.' D. 'HCTZ does not cause dehydration.'

A. 'HCTZ has fewer side effects.' Side effects from thiazides generally are minor and rarely result in discontinuation of therapy. Dizziness is a side effect of all diuretics. There is a potential for dehydration with all diuretics. All diuretic medications are taken regularly as directed.

Which statement by the client indicates to the nurse a need for further teaching on rifampin therapy? A. 'I can expect my skin to turn yellow.' B. 'I can expect my sweat to change color.' C. 'I can expect my urine to turn red-orange.' D. 'I can expect my contact lenses to stain orange.'

A. 'I can expect my skin to turn yellow.' The skin turning yellow indicates jaundice, a serious unexpected adverse effect of rifampin therapy that needs to be reported to the prescriber. Sweat, urine, saliva, and tears (which may stain contact lenses) may turn to a red-orange color during rifampin therapy, which is expected.

Which client statement indicates understanding of the side effects of nitroglycerin ointment? A. 'I may experience a headache.' B. 'Confusion is a common adverse effect.' C. 'A slow pulse rate in an expected side effect.' D. 'Increased blood pressure readings may occur initially.'

A. 'I may experience a headache.' The most common side effect of nitroglycerin is a headache. Additional cardiovascular side effects are hypotension, not hypertension; tachycardia, not bradycardia; and dizziness, not confusion.

The nurse provides instruction when the beta-blocker (BB) atenolol is prescribed for a client with moderate hypertension. Which client statement indicates to the nurse that further teaching is needed? A. 'I must take the medication before going to bed.' B. 'This medication will make me feel drowsy.' C. 'I need to count my pulse before taking the medication.' D. 'I will move slowly when changing positions from sitting to standing.'

A. 'I must take the medication before going to bed.' This medication should be taken early in the morning to maximize its therapeutic effect. Orthostatic hypotension is a side effect of BBs, and the client should change positions slowly from sitting to standing to prevent dizziness and falls. Drowsiness is a side effect of BBs, and the client should be taught precautions to prevent injury. The pulse rate should be taken before administration because ventricular dysrhythmias and heart block may occur with BBs.

The nurse provides teaching regarding vitamin B 12 injections to a client with pernicious anemia. Which statement by the client indicates that teaching was understood? A. 'I must take this medication for the rest of my life.' B. 'I should take this vitamin, as needed, when feeling fatigued.' C. 'Once my symptoms subside, I can stop taking this vitamin.' D. 'I need to have this available for use during exacerbations of anemia.'

A. 'I must take this medication for the rest of my life.' Because the intrinsic factor does not return to gastric secretions even with therapy, B12 injections will be required for the remainder of the client's life. Vitamin B 12 must be taken on a regular basis for the rest of the client's life.

A client is receiving epoetin for the treatment of anemia associated with chronic renal failure. Which client statement indicates to the nurse that further teaching about this medication is necessary? A. 'I realize it is important to take this medication because it will cure my anemia.' B. 'Because I am at risk for seizures, I need to avoid hazardous activities.' C. 'I recognize that I may still need blood transfusions if my hemoglobin is very low.' D. 'I understand that I will still have to take supplemental iron therapy with this medication.'

A. 'I realize it is important to take this medication because it will cure my anemia.' Epoetin will increase a sense of well-being, but it will not cure the underlying medical problem; this misconception needs to be corrected. Seizures are a risk during the first 90 days of therapy, especially if the hematocrit increases more than 4 points in a 2-week period. A dose adjustment may be necessary. Blood transfusions may still be necessary when the client is severely anemic. Supplemental iron therapy is still necessary when receiving epoetin because the increased red blood cell production still requires iron.

Which statement regarding treatment with interferon indicates that the client understands the nurse's teaching? A. 'I will drink 2 to 3 liters [2-3 quarts] of fluid a day.' B. 'Any reconstituted solution must be discarded in 1 week.' C. 'I can continue driving my car as long as I have the stamina.' D. 'While taking this medicine I should be able to continue my usual active lifestyle.'

A. 'I will drink 2 to 3 liters [2-3 quarts] of fluid a day.' Adequate fluid intake helps prevent nephrotoxicity, especially during the early phase of treatment. Reconstituted solution may be stored in the refrigerator for 1 month. Confusion, dizziness, and hallucinations are side effects of this medication; the client should avoid hazardous tasks, such as driving or using machinery. Activity may have to be altered because fatigue and other flu-like symptoms are common with this medication.

Potassium supplements are prescribed for a client receiving diuretic therapy. Which client statement indicates that the teaching about potassium supplements is understood? A. 'I will report any abdominal distress.' B. 'I should use salt substitutes with my food.' C. 'The medication must be taken on an empty stomach.' D. 'The dosage is correct if my urine output increases.'

A. 'I will report any abdominal distress.' Potassium supplements can cause gastrointestinal ulceration and bleeding. Most salt substitutes contain potassium, and their use with potassium supplements can cause hyperkalemia. Because they can be irritating to the stomach, potassium supplements should not be taken on an empty stomach. An increase in urine output is the therapeutic effect of diuretic therapy, not potassium supplements. An adverse effect of potassium supplements is oliguria.

The nurse provides client teaching on the use of oral contraceptives. Which statement made by the client indicates to the nurse that teaching was effective? A. 'I will take my pill at the same time every day.' B. 'I can stop the pill and try to get pregnant right away.' C. 'I may miss two periods and not worry about being pregnant.' D. 'I am so glad we won't have to use condoms even if I miss just one pill during the month.'

A. 'I will take my pill at the same time every day.' Taking the pill at the same time every day makes it more effective, and the client should be instructed to do so. A woman should wait 2 to 3 months after stopping the oral contraceptive pill before attempting to become pregnant. If two consecutive menstrual cycles are missed, the client should stop the contraceptive pill and perform a pregnancy test. The client should use a barrier method of contraception for the first month of pill use and when a pill is missed to help prevent conception.

The nurse teaches a client about side effects that necessitate discontinuation of oral contraceptives. Which statement made by the client indicates that the teaching was effective? A. 'I'll stop taking the pills if I have chest pain.' B. 'I'll stop taking the pills if I start to retain fluid.' C. 'I'll stop taking the pills if I have white discharge from the vagina.' D. 'I'll stop taking the pills if I have pain during the middle of my cycle.'

A. 'I'll stop taking the pills if I have chest pain.' Oral contraceptives should be discontinued with any symptom that may be related to a pulmonary embolus. Fluid retention is a common side effect of increased estrogen and progestin; discontinuation of the contraceptive is unnecessary. Leukorrhea may be a sign of infection, not a side effect of oral contraceptives. Abdominal pain in the middle of the menstrual cycle is not expected while an oral contraceptive is being taken. Abdominal pain in the area of an ovary that occurs midway during the menstrual cycle (mittelschmerz) usually indicates ovulation.

Which instructions will the nurse include in the teaching plan for a client with hyperlipidemia who is being discharged with a prescription for cholestyramine? A. 'Increase your intake of fiber and fluid.' B. 'Take the medication before you go to bed.' C. 'Check your pulse before taking the medication.' D. 'Contact your health care provider if your skin turns yellow.'

A. 'Increase your intake of fiber and fluid.' Fiber and fluids help prevent the most common adverse effect of constipation and its complication, fecal impaction. The medication should be taken with meals. The pulse is not affected. Cholestyramine binds bile in the intestine; therefore it reduces the incidence of jaundice.

A client has a prescription for nitrofurantoin 50 mg orally every evening to manage recurrent urinary tract infections. Which instruction would the nurse give to the client? A. 'Increase your intake of fluids.' B. 'Strain your urine for crystals and stones.' C. 'Stop taking the medication if your urinary output increases.' D. 'This may turn your urine green.'

A. 'Increase your intake of fluids.' To prevent crystal formation, the client should have sufficient intake to produce 1000 to 1500 mL of urine daily while taking this medication. Straining urine is not indicated when the client is taking a urinary anti-infective. If fluids are encouraged, the client's output should increase. Nitrofurantoin turns urine dark yellow to brown, not green.

Which statement by the nurse is most appropriate regarding the greatest advantage of using an insulin pump? A. 'Independence is fostered.' B. 'Fear of daily injections is allayed.' C. 'Dietary restrictions are minimized.' D. 'Blood glucose monitoring can be eliminated.'

A. 'Independence is fostered.' Continuous insulin therapy allows the child to become independent of parental control and anxiety regarding insulin injections. The pump can be programmed to give a bolus of insulin, which corresponds to food eaten, rather than the child needing an injection because of a sudden increase in blood glucose. The pump requires a subcutaneous needle insertion site that needs periodic changing (e.g., every third day or as necessary). The child must still adhere to the recommended diet; dietary control minimizes the amount of exogenous insulin needed. Blood glucose monitoring is required regardless of the method of insulin administration.

An adolescent with leukemia is receiving vincristine. The mother reports that the child is complaining of feeling 'tingles' all over. Which response by the nurse is most appropriate regarding the effect of this medication? A. 'It is a neurological side effect.' B. 'It is caused by an autoimmune reaction.' C. 'The skin becomes sensitive with chemotherapy.' D. 'The central nervous system has become hyperactive.'

A. 'It is a neurological side effect.' Neurotoxicity is an anticipated side effect of vincristine sulfate. Some children report it as 'tingles' or feeling 'funny all over.' It is not usually permanent. Vincristine causes leukopenia, which increases susceptibility to infection; it does not cause an autoimmune reaction. Skin sensitivity is not the reason that the child feels tingly. Hyperactivity of the central nervous system is not a factor in the development of this neurological finding.

Which statement by the nurse reflects teaching for a client recently initiated on anticonvulsants? Select all that apply. One, some, or all responses may be correct. A. 'It is important to take the medication at the same time every day with meals.' B. 'It is important to not drink excessive amounts of caffeine-containing beverages or alcohol.' C. 'Avoid driving or hazardous activities until any side effects such as drowsiness can be determined.' D. 'Some anticonvulsants interfere with vitamin and mineral absorption, so you may need a supplement.' E. 'Oral hygiene, such as gum massage and tooth brushing, is important to combat the gingival hyperplasia that some anticonvulsant medication can cause.'

A. 'It is important to take the medication at the same time every day with meals.' B. 'It is important to not drink excessive amounts of caffeine-containing beverages or alcohol.' C. 'Avoid driving or hazardous activities until any side effects such as drowsiness can be determined.' D. 'Some anticonvulsants interfere with vitamin and mineral absorption, so you may need a supplement.' E. 'Oral hygiene, such as gum massage and tooth brushing, is important to combat the gingival hyperplasia that some anticonvulsant medication can cause.' When initiating anticonvulsant medication, it is important to maintain consistent levels of the medication, so the patient would take it at the same time every day and with meals for absorption. It is important not to drive or perform any hazardous activities as new medications can cause drowsiness. There is some evidence that caffeinated and alcoholic beverages lower seizure threshold. Adding a multivitamin supplement is recommended, especially if the anticonvulsant is the kind that interferes with absorption. Frequent dental examinations and good oral hygiene are the first line of defense against the gingival hyperplasia caused by some anticonvulsants.

The nurse is caring for a 6-year-old child who has undergone craniotomy. The parents ask what effect mannitol has. Which response by the nurse is most appropriate? A. 'It relieves cerebral pressure.' B. 'It increases the bladder's filtration rate.' C. 'It reduces glucose excretion in the urine.' D. 'It decreases the peripheral retention of fluid.'

A. 'It relieves cerebral pressure.' Mannitol is an osmotic diuretic used to relieve cerebral edema. The bladder is a storage basin and is not involved with filtration; mannitol acts in the kidneys. Mannitol is an osmotic diuretic that affects neither the body's excretion of glucose nor peripheral edema.

) A child is prescribed insulin glargine before breakfast. Which instruction is most appropriate for the nurse to give the parents regarding a bedtime snack? A. 'Offer a snack to prevent hypoglycemia during the night.' B. 'Give the child a snack if signs of hyperglycemia are present.' C. 'Avoid a snack because the child is being treated with long-acting insulin.' D. 'Keep a snack at the bedside in case the child gets hungry during the night.'

A. 'Offer a snack to prevent hypoglycemia during the night.' Insulin glargine is released continuously throughout the 24-hour period; a bedtime snack will prevent hypoglycemia during the night. Providing a snack when signs of hyperglycemia are present is unsafe because it intensifies hyperglycemia; if hyperglycemia is present, the child needs insulin. Because insulin glargine is a long-acting insulin, bedtime snacks are recommended to prevent a hypoglycemic episode during the night. When hypoglycemia develops, the child will be asleep; the child should eat the snack before going to bed.

A 28-year-old woman who is a smoker seeks advice about oral contraceptives. Which response by the nurse is appropriate? A. 'Oral contraceptives can cause thrombophlebitis.' B. 'Oral contraceptives must be used with other methods.' C. 'Some oral contraceptives can be used without concern.' D. 'Some oral contraceptives are safe, but others are not safe.'

A. 'Oral contraceptives can cause thrombophlebitis.' Studies have shown that women who smoke at least a pack of cigarettes a day are more prone to cardiovascular problems such as thrombophlebitis. Using oral contraceptives with other methods of contraception is not necessary if there are no contraindications; oral contraceptives are effective used alone. There is no 'safe' oral contraceptive for all women or one that may be used without concern; any client at risk should be informed of the potential consequences of taking an oral contraceptive.

The nurse is preparing a teaching plan for a client prescribed nitroglycerin sublingual. Which would the nurse include in the teaching? A. 'Place the tablet under the tongue or between the cheek and gums.' B. 'It takes 30 to 45 minutes for the nitroglycerin to achieve its effect.' C. 'If dizziness occurs, take a few deep breaths and lean the head back.' D. 'To facilitate absorption, drink a large glass of water after taking the medication.'

A. 'Place the tablet under the tongue or between the cheek and gums.' Nitroglycerin sublingual tablets should not be chewed, crushed, or swallowed. They work much faster when absorbed through the lining of the mouth. Clients are instructed to place the tablet under the tongue or between the cheek and gums and let it dissolve. The client should not eat, drink, smoke, or use chewing tobacco while a tablet is dissolving; this will decrease the effectiveness of the medication. If taken with water, the tablet is washed away from the site of absorption or may be swallowed. Nitroglycerin sublingual tablets usually give relief in 1 to 5 minutes. If a client experiences dizziness or lightheadedness, the client is instructed to take several deep breaths and bend forward with the head between the knees. This position promotes blood flow to the head.

A biphasic antiovulatory medication of combined progestin and estrogen is prescribed for a client. Which instruction would the nurse include when teaching about this oral contraceptive? A. 'Report irregular vaginal bleeding.' B. 'Restrict sexual activity temporarily.' C. 'Have regular bimonthly Pap smears.' D. 'Increase dietary intake of calcium.'

A. 'Report irregular vaginal bleeding.' Antiovulatory medications suppress menstruation. Breakthrough bleeding is not expected with biphasic medications. The medication is given for 21 days, and menstrual flow does not occur during this time. Sexual activity is not restricted when one is taking oral contraceptives. There is no indication for increased frequency of Pap smears; one a year is sufficient. Increased calcium intake is not relevant to the administration of oral contraceptives.

The health care provider prescribes finasteride for a client with benign prostatic hyperplasia (BPH). The client would like to take saw palmetto instead of the finasteride. Which information would the nurse provide to the client about this herbal supplement? A. 'Research has shown that saw palmetto is no better than a placebo.' B. 'You can take both; saw palmetto doesn't require a prescription.' C. 'The herbal supplement will relieve symptoms by altering the size of the prostate.' D. 'Substituting saw palmetto is a good option to avoid all the bad side effects of finasteride.'

A. 'Research has shown that saw palmetto is no better than a placebo.' Rigorous research has demonstrated no significant difference between saw palmetto and a placebo. The health care provider must be consulted regarding the client's desire to change the prescribed therapy. Saw palmetto should be taken with food to limit gastrointestinal side effects. Saw palmetto does not alter the size of the prostate gland. Substituting something that is ineffective is not a good solution regardless of issues surrounding side effects.

A client with tuberculosis takes combination therapy with isoniazid, rifampin, pyrazinamide, and streptomycin. The client says, 'I've never had to take so much medication for an infection before.' How would the nurse respond? A. 'The bacteria causing this infection are difficult to destroy.' B. 'Streptomycin prevents the side effects of the other medications.' C. 'You only need to take the medications for a couple of weeks.' D. 'Aggressive therapy is needed, the infection is well advanced.'

A. 'The bacteria causing this infection are difficult to destroy.' Multiple medications are administered because of concerns regarding medication resistance. Streptomycin sulfate is an antibiotic; it does not prevent the side effects of other medications used in therapy. Multiple antitubercular medications are necessary for an extended period, approximately 6 to 8 months depending on the individual. Multiple dose therapy is needed regardless of whether the disease is advanced.

The health care provider prescribes peak and trough levels after initiation of intravenous antibiotic therapy. The client asks why these blood tests are necessary. Which reason would the nurse provide? A. 'They determine if the dosage of the medication is adequate.' B. 'They detect if you an allergic reaction to the medication.' C. 'The tests permit blood culture specimens obtained when the medication is lowest.' D. 'These allow comparison of your fever to changes in the antibiotic level.'

A. 'They determine if the dosage of the medication is adequate.' Medication dose and frequency are adjusted according to peak and trough levels to enhance efficacy by maintaining therapeutic levels. Peak and trough levels reveal nothing about allergic reactions. Blood cultures are obtained when the client spikes a temperature; they are not related to peak and trough levels of an antibiotic. A sustained decrease in fever is the desired outcome, not a reduction just at peak serum levels of the medication.

Furosemide has been prescribed as part of the medical regimen for a client with hypertension. Which client statement indicates a need for medication education? A. 'This can decrease my vitamin K level.' B. 'I will take the medication in the morning.' C. 'I will contact my health care provider if I notice muscle weakness.' D. 'I plan to take the medication even when my blood pressure is normal.'

A. 'This can decrease my vitamin K level.' Furosemide can produce hypokalemia, not vitamin K deficiency. A well-balanced diet should provide all the necessary vitamins and nutrients. Further teaching is necessary. The morning is the desirable time to take furosemide; early administration prevents nocturia. The client's statement to call the health care provider at signs of muscle weakness is appropriate because muscle weakness may indicate hypokalemia. The client's response to take the medicine even when the blood pressure is normal demonstrates an understanding that the medication should be taken as prescribed, independent of how the client feels, because hypertension is often asymptomatic.

The nurse teaches an adolescent who has completed chemotherapy for acute lymphocytic leukemia (ALL) about the administration of mercaptopurine. Which statement by the adolescent indicates teaching has been effective? A. 'This will help prevent a relapse.' B. 'I guess I'll need an intravenous line for this medication.' C. 'I guess this medication is a substitute for brain radiation.' D. 'This will stop the cancer from spreading to my stomach.'

A. 'This will help prevent a relapse.' Mercaptopurine is given as maintenance therapy to prevent relapses. Mercaptopurine is an oral medication. Oral chemotherapy is an adjunct to other therapies in childhood leukemia, not an alternative for other therapies. The prime site of metastasis of ALL is the central nervous system.

) The nurse is reviewing discharge instructions with the parent of an infant with cystic fibrosis. Which statement indicates the parents know how to administer the pancreatic enzyme replacement? A. 'We should give the medication with feedings.' B. 'We should put crushed enteric-coated pills in the formula.' C. 'We need to give the medication every 6 hours, even during the night.' D. 'We should feed the granules from the capsule in applesauce every morning.'

A. 'We should give the medication with feedings.' Pancreatic enzyme replacement is given just before or with every meal to aid digestion. Breaking up and dissolving the medication will hasten its degradation by gastric secretions and interfere with its efficiency. The medication must be given just before or with every meal, not every 6 hours or every morning, to aid digestion.

Nitroglycerin sublingual tablets are prescribed for a client with the diagnosis of angina. The nurse advises the client to anticipate pain relief will begin within which period of time? A. 1 to 3 minutes B. 4 to 5 seconds C. 30 to 45 seconds D. 10 to 15 minutesmedication."

A. 1 to 3 minutes The onset of action of sublingual nitroglycerin tablets is rapid (1-3 minutes); duration of action is 30 to 60 minutes. If nitroglycerin is administered intravenously, the onset of action is immediate, and the duration is 3 to 5 minutes. It takes longer than 30 to 45 seconds for sublingual nitroglycerin tablets to have a therapeutic effect. Sustained-release nitroglycerin tablets start to act in 20 to 45 minutes, and the duration of action is 3 to 8 hours.

According to developmental norms for a 5-year-old child, the nurse would hold digoxin if an apical heart rate falls below which number? A. 70 beats/min B. 80 beats/min C. 90 beats/min D. 100 beats/min

A. 70 beats/min The purpose of digoxin is to slow and strengthen the apical rate. The apical rate for a healthy child of 5 years is 70 to 110 beats/min. If the apical rate is slow, administration of the medication may lower the apical rate to an unsafe level.

To which nursing home resident could a nurse safely administer tricyclic antidepressants without questioning the health care provider's order? A. A client with mild hypertension B. A client with narrow-angle glaucoma C. A client with coronary artery disease (CAD) D. A client with benign prostatic hypertrophy (BPH)

A. A client with mild hypertension Tricyclics can be safely administered to the hypertensive client. The expected anticholinergic effects of tricyclic antidepressants include difficulty in urination, which is why TCAs are contraindicated with BPH. TCAs are also contraindicated in narrow-angle glaucoma (they can cause elevated pressure in the eyes) and for certain heart abnormalities.

A nurse is reviewing laboratory data prior to administering methotrexate to a client with breast cancer. Which clinical finding will the nurse report to the healthcare provider before administering the medication? A. ALT of 55 IU/mL B. WBC of 12,000/mm³ C. AST of 34 U/L D. HGB of 11.5 g/dL

A. ALT of 55 IU/mL Rationale: Alanine transaminase (ALT) is a liver enzyme that is released into the bloodstream when liver damage is present. Methotrexate is an antineoplastic used in the treatment of various carcinomas. Methotrexate is contraindicated in clients with hepatic impairment. A higher than normal white blood cell (WBC) count is an expected finding in a client with carcinoma. Aspartate aminotransferase (AST) is a liver enzyme used to assess hepatic function. An AST level of 34 U/L is a normal finding. Anemia (low hemoglobin) is an expected finding in a client with carcinoma.

The nurse is preparing to administer 0600 medications to a client. The client is prescribed levothyroxine 125 mcg PO daily for hypothyroidism. The medication package states levothyroxine tablet 0.125 mg. Which action is appropriate? A. Administer the medication B. Call the pharmacy and ask them to deliver the correct dose C. Hold the medication until the healthcare provider arrives D. Call the healthcare provider and request that the time of administration be changed

A. Administer the medication Rationale: Levothyroxine is taken in the morning on an empty stomach. Administering the medication later in the day reduces the efficacy. A typical dose of levothyroxine ranges from 100-200 mcg/day. Often the medication packages display the dose in milligrams, so the dose must be converted. In this case, it is safe to administer the medication as provided. It is important to check the dosage as part of the checks/rights of medication administration (right medication, right patient, right dosage, right route, and right time).

How can the nurse prevent vomiting in a client who reports feeling nauseated after cataract surgery? A. Administer the prescribed antiemetic medication. B. Provide some dry crackers for the client to eat. C. Explain that this is expected after surgery. D. Teach how to breathe deeply until the nausea subsides.

A. Administer the prescribed antiemetic medication. An antiemetic will prevent vomiting; vomiting increases intraocular pressure and should be avoided. Providing some dry crackers for the client to eat, explaining that this is expected after surgery, and teaching how to breathe deeply until the nausea subsides are unsafe; vomiting increases intraocular pressure, and aggressive intervention is required.

The client is diagnosed with tuberculosis (TB). The nurse understands that the treatment plan for this client will involve what type of drug therapy? A. Administering two antituberculosis drugs B. Aminoglycoside antibiotics C. An anti-inflammatory agent D. High doses of B complex vitamins

A. Administering two antituberculosis drugs In order to prevent drug-resistant strains of TB, clients are always prescribed at least two different anti-tubercular medications. Rifampin and isoniazid are the most effective drugs used to treat TB and are always used together, for at least six months. Additional medications, such as pyrazinamide and either streptomycin or ethambutol, may also be prescribed. Vitamin B6 is usually prescribed to help prevent expected side effect of isoniazid.

An adolescent is to begin a chemotherapeutic medication regimen. Which side effect of vincristine is most important for the nurse to prepare the adolescent to expect? A. Alopecia B. Constipation C. Loss of appetite D. Peripheral neuropathy

A. Alopecia A side effect of vincristine is alopecia. To adolescents, who are very concerned with identity, hair loss represents a tremendous threat to self-image. Constipation, although very serious, is not as important to the adolescent as a side effect that affects appearance. Although anorexia will be a concern while the adolescent is undergoing chemotherapy, it is not as important before the start of the regimen. Although neurologic side effects are serious, they are not as important to the adolescent before the start of chemotherapy.

A client's cardiac monitor indicates multiple multifocal premature ventricular complexes (PVCs). Which medication is indicated for treatment of ventricular dysrhythmias? A. Amiodarone B. Epinephrine C. Methyldopa D. Hydrochlorothiazide

A. Amiodarone Amiodarone has an antiarrhythmic action that stabilizes cell membranes of the heart, reducing cardiac excitability; it is used for acute ventricular dysrhythmias. Methyldopa is used to treat hypertension, not PVCs. Epinephrine increases the contractibility of the heart; the effect is opposite of that which is needed. Hydrochlorothiazide is a diuretic used for hypertension, not for correcting multiple PVCs.

Which manifestation is an adverse effect of intravenous lorazepam? Select all that apply. One, some, or all responses may be correct. A. Amnesia B. Drowsiness C. Sleep driving D. Blurred vision E. Respiratory depression

A. Amnesia B. Drowsiness C. Sleep driving D. Blurred vision E. Respiratory depression Benzodiazepines such as lorazepam have a range of side effects, many of which are related to central nervous system depression. Anterograde amnesia, drowsiness, sleep driving, blurred vision, and respiratory depression are all potential adverse effects of lorazepam.

Which assessment will the nurse conduct before administering digoxin to a client? A. Apical heart rate B. Radial pulse C. Difference between carotid and radial pulses D. Difference between apical and radial pulses

A. Apical heart rate Because digoxin slows the heart rate, the apical pulse should be counted for 1 minute before administration. If the apical rate is below a preset parameter (usually 60 beats/minute), digoxin should be withheld because its administration may further decrease the heart rate. Some protocols permit waiting for 1 hour and retaking the apical rate; the result determines if it is administered or if the health care provider is notified. Obtaining the radial pulse on the left side is not as accurate as an apical pulse; the client also may have an atrial dysrhythmia, which cannot be detected through a radial rate alone. Obtaining the radial pulse in both right and left arms is not as accurate as an apical pulse; the client also may have an atrial dysrhythmia, which cannot be detected through a radial rate alone. Obtaining the difference between apical and radial pulses is a pulse deficit, not a pulse rate.

An adult client is given a prescription for a scopolamine patch (Transderm Scop) to prevent motion sickness while on a cruise. Which information should the nurse provide to the client? A. Apply the patch at least 4 hours prior to departure. B. Change the patch every other day while on the cruise. C. Place the patch on a hairless area at the base of the skull. D. Drink no more than 2 alcoholic drinks during the cruise.

A. Apply the patch at least 4 hours prior to departure. Scopolamine, an anticholinergic agent, is used to prevent motion sickness and has a peak onset in 6 hours, so the client should be instructed to apply the patch at least 4 hours before departure on the cruise ship. The duration of the transdermal patch is 72 hours. Scolopamine blocks muscarinic receptors in the inner ear and to the vomiting center, so the best application site of the patch is behind the ear. Anticholinergic medications are CNS depressants, so the client should be instructed to avoid alcohol while using the patch.

A client with Hodgkin's disease is to receive the cyclic antineoplastic vincristine as part of a therapy protocol. Which mechanism of action would the nurse associate with this medication? A. Arresting mitosis in metaphase B. Inhibiting the synthesis of thymidine C. Alkylating nucleic acids needed for mitosis D. Inactivating DNA while inhibiting RNA synthesis

A. Arresting mitosis in metaphase Vincristine is a plant alkaloid that is cell-cycle specific. It affects cell division during metaphase by interfering with spindle formation and causing cell death. Inhibiting the synthesis of thymidine is the typical action of antimetabolites, not plant alkaloids. Alkylating nucleic acids needed for mitosis is typical of the action of alkylating agents, not plant alkaloids. Inactivating DNA and RNA synthesis is the typical action of antineoplastic antibiotics, not plant alkaloids.

There is an order to administer an intramuscular influenza vaccine to an adult client. What actions should the nurse take before administration of the injection? Select all that apply. A. Ask if the client ever had an adverse reaction to the flu vaccine B. Have the client sign the vaccination consent form C. Check the expiration date D. Provide the client with the a vaccine information E. Record the site and time of injection F. Record the client's reaction to the injection

A. Ask if the client ever had an adverse reaction to the flu vaccine B. Have the client sign the vaccination consent form C. Check the expiration date on the vaccination bottle D. Provide the client with the a vaccine information statement Prior to administration, the nurse should identify the expiration date on the bottle and give a current copy of the vaccine information statement to the client. The nurse should also verify any allergies or previous reactions to the vaccine, prior to administering the vaccine. A signed consent is required for vaccinations. Observing for a reaction to the injection and recording the site and time of injection should be performed after administering the vaccine.

Which is an appropriate nursing action when caring for a client taking benazepril for hypertension? A. Assess for dizziness. B. Assess for dark, tarry stools. C. Administer the medication after meals. D. Monitor the electroencephalogram (EEG).

A. Assess for dizziness. Dizziness may occur during the first few weeks of therapy until the client adapts physiologically to the medication. Dark, tarry stools are not a side effect of benazepril. Administering the medication after meals is unnecessary; however, if nausea occurs, the medication may be taken with food or at bedtime. The blood pressure should be monitored before and after administration. An EEG is unnecessary. Cardiac monitoring may be instituted because of possible dysrhythmias.

Propranolol is prescribed for a client with coronary artery disease (CAD). The nurse should consult with the health care provider (HCP) before giving this medication when the client reports a history of which condition? A. Asthma B. Deep vein thrombosis C. Myocardial infarction D. Peptic ulcer disease

A. Asthma Non-cardioselective beta-blockers such as propranolol block b1- and b2-adrenergic receptors and can cause bronchospasm, especially in clients with a history of asthma. Beta-blockers will have no effect on the client's peptic ulcer disease or risk for DVT. Beta-blocker therapy is recommended after an MI.

A client has been prescribed cholestyramine (Questran) in addition to other medications for coronary artery disease and hyperlipidemia. When should the nurse instruct the client to take the cholestyramine? A. At least 1 to 2 hours after other medications B. At least 1 hour before meals C. Anytime is acceptable D. Early in the morning on an empty stomach

A. At least 1 to 2 hours after other medications Rationale: Cholestyramine is a bile acid sequestrant used to reduce LDL cholesterol levels. They are used primarily as adjuncts to statin therapy. Benefits derive from blocking cholesterol synthesis in the liver. The bile-acid sequestrants can form insoluble complexes with other drugs. Medications that undergo binding cannot be absorbed, and hence are not available for systemic effects. Drugs known to form complexes with the sequestrants include thiazide diuretics, digoxin, warfarin, and some antibiotics. To reduce the formation of sequestrant-drug complexes, oral medications that are known to interact should be administered either 1 to 2 hours before the sequestrant or 4 hours after. Cholestyramine works best when taken with meals.

The nurse is caring for a child with an exacerbation of leukemia. The nurse would plan to administer the prescribed analgesic for bone pain at which time? A. At scheduled intervals B. When the child asks for it C. When pain becomes severe D. Before the pain becomes severe

A. At scheduled intervals For maximal benefit, the analgesic should be administered at scheduled intervals that are individualized for the child; routine administration manages the pain before it becomes too intense. The goal is to keep the child pain free; by the time the child asks for the analgesic, the pain has returned. It is insensitive to allow the child to be in pain; there should be no pain.

A client suspected of having myasthenia gravis is scheduled for an edrophonium chloride test. To treat a common complication associated with the test, the nurse will have which medication available? A. Atropine B. Phenytoin C. Neostigmine D. Diphenhydramine

A. Atropine Atropine, an anticholinergic, should always be available to treat a cholinergic crisis (sudden, severe episode of muscle weakness that affects breathing and swallowing) should the edrophonium chloride test trigger this response. Phenytoin is an anticonvulsant that will not avert or treat complications resulting from an edrophonium test. Neostigmine is a cholinergic that has the same action as edrophonium chloride; it is contraindicated if a cholinergic crisis occurs. Diphenhydramine is an antihistamine that will not avert complications or effectively treat a cholinergic crisis.

A client is admitted to the hospital with pancytopenia as a result of chemotherapy. Which information will the nurse provide to minimize the risk for complications? A. Avoid activities that risk traumatic injuries and exposure to infection. B. Perform frequent mouth care with a firm toothbrush. C. Increase oral fluid intake to a minimum of 3 L daily. D. Report any unusual muscle cramps or tingling sensations in the extremities.

A. Avoid activities that risk traumatic injuries and exposure to infection. Reduced platelets increase the likelihood of uncontrolled bleeding; reduced lymphocytes increase susceptibility to infection. Aggressive oral hygiene can precipitate bleeding from the gums. Although fluids may be increased to flush out the toxic by-products of chemotherapy, this has no effect on pancytopenia. Muscle cramps or tingling sensations in the extremities are adaptations to hypocalcemia; hypocalcemia is unrelated to pancytopenia.

A nurse is providing dietary instructions to a client who is taking prescribed amiloride. Which information will the nurse include in the teaching? A. Avoid eating foods that are rich in potassium such as bananas B. It is important to control high-sodium foods such as canned soups C. Eat plenty of foods that contain calcium such as milk D. Choose foods that are high in iron content such as shellfish

A. Avoid eating foods that are rich in potassium such as bananas Rationale: Amiloride is a potassium-sparing diuretic used in the treatment of edema, hypertension, and potassium loss caused by other diuretic medications. Amiloride may cause hyperkalemia, so the client should be informed to limit their potassium intake. Sodium, calcium, and iron are not affected by the use of amiloride.

Which medications would the nurse identify as being used as preanesthetic agents? Select all that apply. One, some, or all responses may be correct. A. Barbiturates B. Benzodiazepines C. Antiepileptic agents D. Atypical antipsychotics E. Mood stabilizing agents

A. Barbiturates B. Benzodiazepines Barbiturates and benzodiazepines are sedative-hypnotics that may also be used to decrease effects of anxiety in presurgical clients. Antiepileptic agents are used to treat seizure activity and manage bipolar disorder. Atypical antipsychotics are mainly used to treat bipolar disorder and psychosis. Mood stabilizing agents are used to treat bipolar disorder.

The nurse in a primary care clinic is reviewing the medical record of a client with chronic gastroesophageal reflux disease (GERD). Which findings are risk factors for developing GERD? Select all that apply. A. Being overweight or obese B. Diabetes mellitus type 2 C. Helicobacter pylori infection D. Taking a calcium channel blocker E. Smoking F. Essential hypertension

A. Being overweight or obese C. Helicobacter pylori infection D. Taking a calcium channel blocker E. Smoking Gastroesophageal reflux disease or GERD occurs as a result of backward flow of stomach contents into the esophagus. The most common cause of GERD is excessive relaxation of the lower esophageal sphincter (LES), which allows the reflux of gastric contents into the esophagus and exposure of the esophageal mucosa to acidic gastric contents. A number of factors can decrease LES pressure including calcium channel blockers, smoking and alcohol. Clients who are overweight or obese are at highest risk for development of GERD because increased weight increases intra-abdominal pressure, which contributes to reflux of stomach contents into the esophagus. Helicobacter pylori infection can also contribute to GERD. Diabetes and hypertension are not considered risk factors for GERD.

Which mechanism of action explains how aluminum hydroxide decreases serum phosphorus? A. Binding with phosphorus in the intestine B. Promoting excretion of phosphorus C. Promoting excretion of excessive urinary phosphates D. Dissolving stones as they pass through the urinary tract

A. Binding with phosphorus in the intestine Aluminum hydroxide binds phosphorus in the intestine, preventing its absorption; this decreases serum phosphorus. Promoting excretion of phosphorus, promoting excretion of excessive urinary phosphates, and dissolving stones as they pass through the urinary tract are not actions of this medication.

Oral contraceptives are prescribed for a client who smokes heavily. Which side effect would the nurse warn the client might occur? A. Blood clots B. Cervical cancer C. Ovarian cancer D. Risk of coronary heart disease later in life

A. Blood clots Heavy smoking is a major risk factor for an increased risk of thrombosis or blood clots. Cervical cancer is associated with human papillomavirus infection, not oral contraceptive use. Oral contraceptives have a protective effect against ovarian cancer. Although there is an increased risk of coronary heart disease while taking an oral contraceptive, this risk abates when it is no longer taken and does not carry over into later life.

A client is receiving hydrochlorothiazide. Which physiological alteration will the nurse monitor to best determine the effectiveness of the client's hydrochlorothiazide therapy? A. Blood pressure B. Decreasing edema C. Serum potassium level D. Urine specific gravity

A. Blood pressure Diuretics promote urinary excretion, which reduces the volume of fluid in the intravascular compartment, thus lowering blood pressure. The measure of blood pressure is the best determination of effectiveness because it is a direct measure of the desired outcome. A reduction in edema reflects effectiveness; however, multiple physiological processes, including venous competence, gravity, and disuse, maintain a significant degree of edema even when the diuretic is optimally effective. A lowered potassium level would indirectly indicate that the medication is working; however, this does not provide a good measure of effectiveness. Although specific gravity decreases with increased urinary output, and thus would demonstrate that the medication is working, it is not a direct measure of the desired outcome. A measure of the reduction in intravascular pressure is preferable.

Which lifestyle advice does the nurse give to a client when oral digoxin therapy is initiated? Select all that apply. One, some, or all responses may be correct. A. Bran can decrease digoxin absorption. B. Digoxin should not be taken with hawthorn supplements. C. Ginseng may cause a dangerous increase in digoxin levels in the blood. D. St. John's Wort can increase digoxin levels in the blood. E. Medications that lower serum potassium or magnesium can cause digoxin toxicity.

A. Bran can decrease digoxin absorption. B. Digoxin should not be taken with hawthorn supplements. C. Ginseng may cause a dangerous increase in digoxin levels in the blood. E. Medications that lower serum potassium or magnesium can cause digoxin toxicity. Consuming large amounts of bran can decrease the absorption of digoxin. Hawthorn may potentiate the effects of digoxin and should be avoided. Ginseng might increase levels of digoxin. St. John's Wort can reduce levels of digoxin in the blood.

The nurse recognizes that hormonal therapy (HT) increases the risk of which condition in postmenopausal women? A. Breast cancer B. Rapid weight loss C. Accelerated bone loss D. Vaginal tissue atrophy

A. Breast cancer There is a relationship between HT that combines estrogen and progesterone compounds and an increased incidence of invasive breast cancer. One side effect of HT is weight gain with ankle and foot edema. Bone loss is slowed with HT. Vaginal tissue maintains turgor and lubrication with HT.

Which food would the nurse encourage a client to eat while receiving treatment to prevent hypokalemia? A. Broccoli B. Oatmeal C. Fried rice D. Canned carrots

A. Broccoli Potassium is plentiful in green leafy vegetables; broccoli provides 207 mg of potassium per half cup. Oatmeal provides 73 mg of potassium per half cup. Rice provides 29 mg of potassium per half cup. Cooked fresh carrots provide 172 mg of potassium per half cup; canned carrots provide only 93 mg of potassium per half cup.

Which medication is derived from a natural source and may be prescribed for the treatment of osteoporosis? A. Calcitonin B. Raloxifene C. Clomiphene D. Bisphosphonates

A. Calcitonin Calcitonin is derived from natural sources such as fish; this medication may be prescribed to prevent osteoporosis. Raloxifene is prescribed to prevent postmenopausal osteoporosis. Clomiphene is prescribed to induce ovulation. Bisphosphonates are prescribed to treat osteoporosis; this medication is not derived from natural sources.

Which medication will the nurse expect the health care provider to prescribe to a client who had a thyroidectomy and is pale with spasms of the hand when taking the blood pressure? A. Calcium B. Magnesium C. Bicarbonate D. Potassium chloride

A. Calcium These signs may indicate calcium depletion as a result of accidental removal of parathyroid glands during thyroidectomy. Symptoms associated with hypomagnesemia include tremor, neuromuscular irritability, and confusion. Symptoms associated with metabolic acidosis include deep, rapid breathing, weakness, and disorientation. Symptoms associated with hypokalemia include muscle weakness and dysrhythmias.

Which medications may be used to correct severe hyperkalemia resulting from intravenous (IV) administration? Select all that apply. One, some, or all responses may be correct. A. Calcium chloride B. Sodium chloride C. Calcium gluconate D. Sodium bicarbonate E. Dextrose solution with insulin

A. Calcium chloride C. Calcium gluconate D. Sodium bicarbonate E. Dextrose solution with insulin Hyperkalemia resulting from IV administration might be treated with calcium chloride, calcium gluconate, sodium bicarbonate, and dextrose solution with insulin. These substances lead to the rapid shifting of intracellular potassium ions, thereby reducing potassium concentration. Sodium chloride is primarily used to prevent or treat sodium losses.

A client with Parkinson disease is admitted to the hospital. Which medication is prescribed to improve the physical manifestations of Parkinson disease? A. Carbidopa-levodopa B. Isocarboxazid C. Dopamine D. Pyridoxine (vitamin B 6)

A. Carbidopa-levodopa Levodopa crosses the blood-brain barrier and converts to dopamine, a substance depleted in Parkinson disease. Isocarboxazid is a monoamine oxidase inhibitor used for the treatment of psychological symptoms associated with severe depression, not physiological symptoms of Parkinson disease. Dopamine is not prescribed for this purpose because it does not cross the blood-brain barrier. Pyridoxine can reverse the effects of some antiparkinsonian medications and is contraindicated.

The client with a seizure disorder receives intravenous (IV) phenytoin. The nurse will monitor closely for which condition? A. Cardiac dysrhythmias B. Hypoglycemia C. Polycythemia D. Paradoxical excitation

A. Cardiac dysrhythmias IV phenytoin was once used to treat dysrhythmias until better medications were developed. It depresses both atrial and ventricular conduction, and so it can cause significant dysrhythmias. It can also cause hyperglycemia (not hypoglycemia) and pancytopenia (not polycythemia). Paradoxical excitation is not a known issue; it has a depressant effect resulting in drowsiness.

At 6 weeks' gestation a client is found to have gonorrhea. For which medication would the nurse anticipate preparing a teaching plan? A. Ceftriaxone B. Levofloxacin C. Sulfasalazine D. Trimethoprim/sulfamethoxazole

A. Ceftriaxone Ceftriaxone, a broad-spectrum antibiotic, is preferred during pregnancy. Levofloxacin, although listed for unlabeled use against gonococcal infection, should not be prescribed during pregnancy. Sulfonamides, such as Sulfasalazone, may cause hemolysis in the fetus. Trimethoprim/sulfamethoxazole contains a sulfonamide and is contraindicated during pregnancy.

Atenolol is prescribed for a client with moderate hypertension. Which information would the nurse include when teaching the client about this medication? Select all that apply. One, some, or all responses may be correct. A. Change to standing positions slowly. B. Take the medication before going to bed. C. Count the pulse before taking the medication. D. Mild weakness and fatigue are common side effects. E. It is safe to take over-the-counter (OTC) medications.

A. Change to standing positions slowly. C. Count the pulse before taking the medication. D. Mild weakness and fatigue are common side effects. A side effect of this medication is orthostatic hypotension. The client should be advised to move to a standing position slowly to allow the body to adjust to the new position. The rate of the pulse should be taken before administering the medication; slower rates due to ventricular dysrhythmias and heart block may occur. Mild weakness and fatigue are side effects of this medication. The blood pressure decreases when the client is sleeping; the medication usually is prescribed to be administered earlier in the day. The medication should be taken with food. No OTC medication should be taken without consulting the prescribing health care provider; decreased or increased effects can occur when there is an interaction with another medication.

A nurse is assessing a client diagnosed with diabetic ketoacidosis. The client is on a prescribed regular insulin infusion at 0.1 units/kg/hr. The client appears restless and verbalizes tingling to the extremities. Which action does the nurse perform next? A. Check the client capillary blood glucose B. Stop the regular insulin infusion C. Increase the infusion to 0.15 units/kg/hr D. Give the client 4 oz of fruit juice

A. Check the client capillary blood glucose Rationale: The client is experiencing symptoms of hypoglycemia. Prior to decreasing the dose of the infusion, the nurse should assess the client's blood glucose level to confirm the hypoglycemia. Prior to stopping the infusion, the nurse needs to assess the client's blood sugar level and notify the healthcare provider of the results. Increasing the infusion will cause further hypoglycemia. Prior to performing an intervention to correct the hypoglycemia, the nurse needs to assess the blood glucose level first.

The nurse is caring for a child receiving furosemide for pulmonary edema. Which nursing intervention(s) would the nurse implement? Select all that apply. One, some, or all responses may be correct. A. Checking the child's weight every day B. Administering the medication on an empty stomach C. Calculating the dose of medication as carefully as possible D. Exposing the child to sunlight for increasing periods E. Assessing the child regularly to help prevent electrolyte loss

A. Checking the child's weight every day C. Calculating the dose of medication as carefully as possible E. Assessing the child regularly to help prevent electrolyte loss The child's weight should be checked and recorded daily to aid in the assessment of therapeutic and adverse effects. Pediatric doses should be calculated carefully to prevent an accidental overdose. Pediatric clients are at greater risk of electrolyte loss; therefore they require closer and more cautious assessment to help prevent hypertension and stroke. Furosemide may cause stomach upset if it is taken on an empty stomach; the child should be given the medication with food to help prevent gastric upset. A child taking diuretics should not be exposed to sunlight for long periods, because this action may precipitate fluid volume loss and heatstroke.

A client is scheduled to begin chemotherapy 2 weeks after surgery for colon cancer. Which explanation would the nurse give to explain the delay after surgery? A. Chemotherapy interferes with cell growth and delays wound healing. B. Because chemotherapy causes vomiting, it endangers the integrity of the incisional area. C. Chemotherapy decreases red blood cell production, and the resultant anemia will add to postoperative fatigue. D. Chemotherapy increases edema in areas distal to the incision by blocking lymph channels with destroyed lymphocytes.

A. Chemotherapy interferes with cell growth and delays wound healing. Chemotherapeutic agents can attack healthy as well as malignant cells; they generally interfere with protein synthesis and cell division in all rapidly dividing cells, including those regenerating traumatized tissue (as in wound healing), bone marrow, and cutaneous and alimentary tract epithelial tissue. Vomiting should not disturb the integrity of the area. Decreased red blood cell levels caused by bone marrow depression can be corrected with transfusions. Chemotherapy should not cause a blockage of lymph channels, with destroyed lymphocytes increasing edema.

During morning rounds, a healthcare provider informs a client with hypertension that a calcium channel blocker will be added to their treatment regimen. The nurse notes a new prescription for amiloride 10 mg PO daily. Which action does the nurse perform next? A. Clarify the prescription with the healthcare provider B. Educate the client on the new prescription C. Administer the medication with food D. Assess the client blood pressure

A. Clarify the prescription with the healthcare provider Rationale: The nurse should clarify the new prescription. Amiloride is a potassium-sparing diuretic. It is also a look-alike/sound-alike medication commonly confused with amlodipine, a calcium channel blocker. Educating the client on a new prescription, administering the medication with food, and assessing the blood pressure are important interventions for amiloride. However, this prescription should be clarified.

Following the administration of sublingual nitroglycerin to a client experiencing an acute anginal attack, which assessment finding indicates to the nurse that the desired effect has been achieved? A. Client states chest pain is relieved. B. Client's pulse decreases from 120 to 90. C. Client's systolic blood pressure decreases from 180 to 90. D. Client's SaO2 level increases from 92% to 96%.

A. Client states chest pain is relieved. Nitroglycerin reduces myocardial oxygen consumption which decreases ischemia and reduces chest pain.

Two weeks after starting phenobarbital sodium, the client develops new symptoms. Which symptom is probably caused by the medication? A. Cognitive impairment B. Frequent urination C. Abdominal pain D. Mania

A. Cognitive impairment Phenobarbital can cause significant lethargy, depression, and cognitive impairment. Frequent urination and abdominal pain are not typically associated with this medication. Depression, not mania, is a common adverse effect for this medication.

The nurse is caring for a client with a diagnosis of cardiogenic shock who has been prescribed dobutamine infusion. Which action should the nurse take first? A. Compare the packaging of the medication to the prescription B. Prime the IV tubing with the medication C. Set the infusion pump for the correct infusion rate D.Increase the frequency of blood pressure and heart rate monitoring on the

A. Compare the packaging of the medication to the prescription Rationale: The Food and Drug Administration and Institute for Safe Medication Practices has maintained a list of drug name pairs and trios that look and sound similar. These medications are called out with tall man lettering on dissimilar lettering on the packaging (capital letters). Dobutamine is often confused with dopamine and is included in the list. Ensuring the use of the right medication is a part of the rights of medication administration. This should be done before priming the tubing with the medication and setting the infusion pump. While frequent vital signs are important for the client with shock, it isn't the most important action in this scenario.

Which finding would lead the nurse to recheck the blood glucose level of a diabetic client before administering a mealtime insulin dose? Select all that apply. One, some, or all responses may be correct. A. Confusion B. Drowsiness C. Diaphoresis D. Nervousness E. Heart rate 110 beats/min

A. Confusion B. Drowsiness C. Diaphoresis D. Nervousness E. Heart rate 110 beats/min Signs of hypoglycemia include confusion, drowsiness, diaphoresis, nervousness, tachycardia, and headache. The nurse would recheck the blood glucose level of a diabetic client with these symptoms to avoid worsening hypoglycemia caused by administration of additional insulin.

Which manifestation would the nurse include when teaching a client about ketoacidosis? Select all that apply. One, some, or all responses may be correct. A. Confusion B. Hyperactivity C. Excessive thirst D. Fruity-scented breath E. Decreased urinary output

A. Confusion C. Excessive thirst D. Fruity-scented breath Diabetic ketoacidosis signs and symptoms often develop quickly, sometimes within 24 hours. Diabetic ketoacidosis is a serious complication of diabetes that occurs when the body produces high levels of ketones (blood acids). Diabetic ketoacidosis develops when the body is unable to produce enough insulin. Without enough insulin, the body begins to break down fat as an alternative fuel. This process produces a buildup of ketones (toxic acids) in the bloodstream, eventually leading to diabetic ketoacidosis if untreated. Signs and symptoms include confusion, excessive thirst, fruity-scented breath, frequent urination, nausea and vomiting, abdominal pain, weakness or fatigue, and shortness of breath. Weakness or fatigue, not hyperactivity, is a symptom. Frequent urination, not decreased urination, is a symptom.

A nurse is preparing to administer intravenous mannitol to a client with increased intracranial pressure. Which action will the nurse perform prior to administering the medication? A. Connect an in-line filter to the infusion tubing B. Dilute the medication with lactated ringers C. Prepare an infusion warmer D. Ensure the client has a patent central line

A. Connect an in-line filter to the infusion tubing Rationale: The nurse should connect a filter to the infusion tubing prior to administering mannitol. Mannitol is an osmotic diuretic that may contain crystals within the solution. The in-line filter prevents the administration of particulates into the bloodstream. Mannitol should be administered undiluted. An infusion warmer is not required for the administration of mannitol. Mannitol can be administered through a peripheral line.

The nurse is preparing a client for discharge following inpatient treatment for pulmonary tuberculosis. Which instruction should be given to the client? A. Continue taking medications as prescribed. B. Continue taking medications until symptoms are relieved. C. Avoid contact with children, pregnant women or immunosuppressed persons. D. Take medication with aluminum hydroxide if epigastric distress occurs.

A. Continue taking medications as prescribed. Early cessation of treatment may lead to development of drug-resistant tuberculosis (TB). Active TB is usually treated with a combination of four different antibiotics (Isoniazid, rifampin, ethambutol and pyrazinamide) and can now take anywhere from 6-12 months to completely kill the bacteria. As with any antibiotics, clients should continue to take medications even after they begin to feel better. There is no reason to avoid contact with children, pregnant women or immunosuppressed persons once discharged from the hospital as long as the client is adhering to medication schedules. Isoniazid should be taken on an empty stomach; ethambutol can be taken with food to avoid stomach upset. If taken with TB medications, aluminum hydroxide will interfere with absorption of these medications.

The nurse is preparing to administer newly prescribed intravenous phenytoin to a client. When reviewing the client's medical record, which prescription should the nurse question? A. Continuous infusion of dextrose 5% in 0.9% saline B. NPH insulin 40 units before meals C. Labetalol 100 mg orally twice per day D. Ketorolac 15 mg IV push as needed for pain

A. Continuous infusion of dextrose 5% in 0.9% saline Rationale: Phenytoin is not compatible with most IV fluids, especially those with dextrose. If the nurse observes a continuous infusion of a fluid that contains dextrose, they should understand that incompatibilities are likely and should not administer the medication as prescribed. Insulin, labetalol, and ketorolac do not have potential incompatibilities.

An 18-month-old toddler is being treated with intravenous diazepam every 4 hours for generalized tetanus. Which response to the medication would the nurse anticipate? A. Control of hypertonicity and prevention of seizures B. Control of laryngospasms and neck and jaw rigidity C. Prevention of excess oxygen and caloric expenditure D. Prevention of restlessness and resistance to assisted ventilation

A. Control of hypertonicity and prevention of seizures Diazepam is commonly used to manage generalized muscular spasms. Laryngospasm and nuchal rigidity are responses to the exotoxin and are treated with tetanus immune globulin. Diazepam is not administered to decrease the metabolic rate. Pancuronium bromide, an acetylcholine antagonist, is given to children who do not respond to sedatives and muscle relaxants and resist ventilatory assistance.

Which assessment would the nurse perform before administering a dose of vancomycin to a client? Select all that apply.One, some, or all responses may be correct. A. Creatinine B. Trough level C. Hearing ability D. Intravenous site E. Blood urea nitrogen

A. Creatinine B. Trough level C. Hearing ability D. Intravenous site E. Blood urea nitrogen ALL are correct. Two major adverse effects of vancomycin are nephrotoxicity and ototoxicity. The nurse would assess the client's creatinine and blood urea nitrogen levels to determine renal function. The nurse would also assess the vancomycin trough levels to determine if the client's kidneys are clearing the medication. The nurse would assess for changes in hearing as a result of ototoxicity. Vancomycin can cause phlebitis, so the nurse would assess the intravenous site before initiating the infusion.

Which fact about ceftriaxone medication therapy will the nurse emphasize when teaching a client diagnosed with gonorrhea? A. Cures the infection B. Prevents complications C. Controls its transmission D. Reverses pathologic changes

A. Cures the infection Ceftriaxone, followed by doxycycline, is specific for Neisseria gonorrhoeae and eradicates the microorganism; other treatment regimens are available for resistant strains. If the disease progresses before the diagnosis is made, complications such as sterility, heart valve damage, or joint degeneration may occur. Transmission is not controlled; the organism is eliminated. If tubal structures, heart valves, or joints degenerate, the pathologic changes will not be reversed by antibiotic therapy.

The client with congestive heart failure is receiving furosemide 80 mg once daily. Which data collection assessment would be performed to evaluate medication effectiveness? Select all that apply. One, some, or all responses may be correct. A. Daily weight B. Intake and output C. Monitor for edema D. Daily pulse oximetry E. Auscultate breath sounds

A. Daily weight B. Intake and output C. Monitor for edema D. Daily pulse oximetry E. Auscultate breath sounds Daily weight at the same time, on the same scale, and in the same clothing is important as it is an indication of fluid gains or losses. The nurse would also record daily intake and output and report intake exceeding output. The nurse would monitor for peripheral edema and document the findings. It is important to obtain and record vital signs and daily pulse oximetry as improving results relate to effectiveness of furosemide. The nurse would also auscultate breath sounds, look for jugular venous distension, and report abnormal data.

A client reports to the nurse, 'I've been using St. John's wort to try and feel more like myself again. I'm not sure whether it's going to work.' Which symptom would the nurse further assess? A. Depression B. Sleep disturbances C. Diminished cognitive ability D. Sensory-perceptual disturbances

A. Depression St. John's wort is an herb marketed as a natural way to improve mood and ease feelings of depression. Because St. John's wort is considered a dietary supplement, it is not regulated by the Food and Drug Administration as medications are. It has not been shown to exert positive effects in people with sleep disturbances, diminished cognitive abilities, or sensory-perceptual disturbances.

An ambulatory client with relapsing-remitting multiple sclerosis is to receive every-other-day injections of interferon beta-1a. Which adverse effects would the nurse explain may occur when taking this medication? Select all that apply. One, some, or all responses may be correct. A. Depression B. Polycythemia C. Flu-like symptoms D. Increased risk for infection E. Decreased perspiration

A. Depression C. Flu-like symptoms D. Increased risk for infection Central nervous system effects include depression that may lead to suicide attempts. Interferon immune modifier causes flu-like symptoms, such as fever, muscle aches, and lethargy. Myelosuppression can cause leukopenia, thus increasing the risk for infection. It also increases the risk for anemia because of decreased production of red blood cells (RBCs); this is the opposite of polycythemia (too many RBCs). An integumentary response to this medication is sweating, not lack of perspiration (anhidrosis).

A client is taking thyroxine to manage hypothyroidism. Which developments indicate to the nurse that the dosage should be reduced? Select all that apply. One, some, or all responses may be correct. A. Diaphoresis B. Weight gain C. Tachycardia D. Nervousness E. Cold intolerance

A. Diaphoresis C. Tachycardia D. Nervousness Diaphoresis, tachycardia, and nervousness are signs of hyperthyroidism, which indicate that too much medication is being taken. Weight gain and cold intolerance indicate that the medication has not yet been effective.

Which common side effect would the nurse expect in a client with acquired immunodeficiency syndrome (AIDS) who is on a treatment protocol that includes a protease inhibitor? A. Diarrhea B. Hypoglycemia C. Paresthesias of the extremities D. Seeing yellow halos around lights

A. Diarrhea Diarrhea, nausea, and vomiting are common side effects; clients should take these medications with a meal or light snack. These medications may cause hyperglycemia, not hypoglycemia. Circumoral (perioral), not peripheral, paresthesias may occur with protease inhibitors; peripheral paresthesias may occur with nucleoside reverse transcriptase inhibitors. Seeing yellow halos around lights does not occur with protease inhibitors; it may occur with digoxin toxicity.

For which side effects will the nurse assess a client with cancer who is being treated with chemotherapeutic agents? Select all that apply. One, some, or all responses may be correct. A. Diarrhea B. Leukocytosis C. Bleeding tendencies D. Lowered sedimentation rate E. Increased hemoglobin levels

A. Diarrhea C. Bleeding tendencies Most chemotherapeutic agents interfere with mitosis. The rapidly dividing cells of the mucous membranes of the gastrointestinal tract are affected, causing stomatitis and diarrhea. Bone marrow depression often causes thrombocytopenia, resulting in bleeding tendencies. The bone marrow consists of rapidly dividing cells, and its activity is depressed. Leukopenia, not leukocytosis, can occur. The erythrocyte sedimentation rate generally increases in the presence of tissue inflammation or necrosis. Hemoglobin and hematocrit levels may decrease because of an inadequate number of red blood cells related to bone marrow depression.

A client is admitted to the acute medical unit for severe amphetamine intoxication. Which medications are indicated to counteract the effects of stimulant intoxication? Select all that apply. One, some, or all responses may be correct. A. Diazepam B. Propranolol C. Benztropine D. Bupropion E. Amitriptyline

A. Diazepam B. Propranolol Because stimulants act by increasing both adrenaline and dopamine, seizures may occur. Diazepam can reduce the chance of seizures. Because amphetamines act by increasing adrenaline, which can stimulate the heart, propranolol, a beta-blocker, will decrease this adrenergic stimulation. Benztropine, a cholinergic blocker, is not indicated as a treatment for stimulant intoxication. Bupropion is contraindicated because it increases dopamine and adrenaline, which will exacerbate stimulant intoxication. Amitriptyline is contraindicated because it increases dopamine and adrenaline, which will exacerbate stimulant intoxication.

An infant with congenital heart disease is prescribed digoxin and furosemide upon discharge. Which sign would the nurse instruct the parents to be alert for? A. Difficulty feeding with vomiting B. Cyanosis during periods of crying C. Daily naps lasting more than 3 hours D. A pulse rate faster than 100 beats/min

A. Difficulty feeding with vomiting Vomiting and feeding issues are early signs of digoxin toxicity. Cyanosis is expected in a crying infant with heart disease because the energy expenditure exceeds the body's ability to meet the oxygen demand. Long naps are expected; infants routinely require several naps, and an infant with heart disease requires long rest periods. The pulse rate of an infant receiving digoxin should remain faster than 100 beats/min.

A child being treated with cardiac medications developed vomiting, bradycardia, anorexia, and dysrhythmias. The nurse understands which medication toxicity is responsible for these symptoms? A. Digoxin B. Nesiritide C. Dobutamine D. Spironolactone

A. Digoxin Digoxin helps improve pumping efficacy of the heart, but an overdose can cause toxicity leading to nausea, vomiting, bradycardia, anorexia, and dysrhythmias. The side effects of nesiritide may include effects such as headache, insomnia, and hypotension. Dobutamine does not cause nausea or vomiting but may cause hypertension and hypotension. Spironolactone may cause edema.

A client who takes multiple medications complains of severe nausea, and the client's heartbeat is irregular and slow. The nurse determines that these signs and symptoms are toxic effects of which medication? A. Digoxin B. Captopril C. Furosemide D. Morphine sulfate

A. Digoxin Signs of digoxin toxicity include cardiac dysrhythmias, anorexia, nausea, vomiting, and visual disturbances. Although nausea and heart block may occur with captopril, these symptoms rarely are seen; drowsiness and central nervous system disturbances are more common. Toxic effects of morphine are slow, deep respirations, stupor, and constricted pupils; nausea is a side effect, not a toxic effect. Toxic effects of furosemide are renal failure, blood dyscrasias, and loss of hearing.

The client with hypokalemia reports nausea, vomiting, and seeing a yellow light around objects. Which of the client's medications is the likely cause of the client's symptoms? A. Digoxin B. Furosemide C. Propranolol D. Spironolactone

A. Digoxin These are signs of digitalis toxicity, which is more likely to occur in the presence of hypokalemia. Although furosemide most likely contributed to the hypokalemia, the client's symptoms are consistent with digitalis toxicity. Although propranolol can cause nausea, vomiting, and blurred vision, the presence of hypokalemia and yellow vision are more suggestive of digitalis toxicity. A side effect of spironolactone is hyperkalemia, not hypokalemia.

One week after being hospitalized for an acute myocardial infarction, a client reports nausea and loss of appetite. Which of the client's prescribed medications would be withheld and the health care provider notified? A. Digoxin B. Propranolol C. Furosemide D. Spironolactone

A. Digoxin Toxic levels of digoxin stimulate the medullary chemoreceptor trigger zone, resulting in anorexia, nausea, and vomiting. Although anorexia, nausea, and vomiting may be side effects of furosemide, propranolol, and spironolactone, they do not indicate toxicity.

) A nurse is preparing to withdraw insulin into a syringe in the medication room. The nurse notes an open, full vial of regular insulin with no labeled expiration date. Which action does the nurse take? A. Discard the vial and request a new one from the pharmacy B. Label the vial with the current date and withdraw the medication C. Withdraw the medication and discard the vial D. Store the medication in the refrigerator and notify the pharmacy

A. Discard the vial and request a new one from the pharmacy Rationale: Opened vials of medication should have a labeled expiration date. Multi-dose vials expire 28 days after being opened. The nurse should discard the vial and request a new one since there is no way of knowing when the vial was opened. Labeling the medication with the current date or withdrawing the medication are not safe practices. There is no way to verify how long the medication has been opened. The vial should not be stored without an expiration date label.

A client with left ventricular heart failure and supraventricular tachycardia is prescribed digoxin 0.25 mg daily. Which changes would the nurse expect to find if this medication is therapeutically effective? Select all that apply. One, some, or all responses may be correct. A. Diuresis B. Tachycardia C. Decreased edema D. Decreased pulse rate E. Reduced heart murmur F. Jugular vein distention

A. Diuresis C. Decreased edema D. Decreased pulse rate Digoxin increases kidney perfusion, which results in urine formation and diuresis. The urine output increases because of improved cardiac output and kidney perfusion, resulting in a reduction in edema. Because of digoxin's inotropic and chronotropic effects, the heart rate will decrease. Digoxin increases the force of contractions (inotropic effect) and decreases the heart rate (chronotropic effect). Digoxin does not affect a heart murmur. Jugular vein distention is a specific sign of right ventricular heart failure; it is treated with diuretics to reduce the intravascular volume and venous pressure.

A client is taking an estrogen-progestin oral contraceptive. Which adverse effects from the contraceptive would the nurse teach the client to report to the primary health care provider? Select all that apply. One, some, or all responses may be correct. A. Dizziness B. Chest pain C. Bloating D. Nausea E. Calf tenderness F. Breast tenderness

A. Dizziness B. Chest pain E. Calf tenderness Early side effects of oral contraceptives include bloating, nausea, and breast tenderness. Although they may be bothersome enough to lead to discontinuation of the contraceptive, these side effects usually subside in several months. Dizziness is not a common side effect and should be reported to the provider. Contraceptives have been associated with thrombophlebitis; clinical manifestations of thrombophlebitis include calf tenderness and redness and heat over the affected area. If the clot travels, it could present as a pulmonary embolism, so chest pain should be reported as well.

A client is receiving metoprolol. Which potential effect will the nurse teach the client to expect? A. Dizziness with strenuous activity B. Acceleration of the heart rate after eating a heavy meal C. Flushing sensations after taking the medication D. Pounding of the heart

A. Dizziness with strenuous activity Because metoprolol competes with catecholamines at beta-adrenergic receptor sites, the expected increase in the heart's rate and contractility in response to exercise does not occur. This, combined with the medication's hypotensive effect, may lead to dizziness. Metoprolol decreases the heart rate. Flushing sensations and pounding of the heart do not represent side effects of metoprolol.

A client with hypertension has received a prescription for metoprolol. Which information will the nurse include when teaching this client about metoprolol? A. Do not abruptly discontinue the medication. B. Consume alcoholic beverages in moderation. C. Report a heart rate of less than 70 beats per minute. D. Increase the medication dosage if chest pain occurs.

A. Do not abruptly discontinue the medication. Abrupt discontinuation of metoprolol may cause rebound hypertension and an acute myocardial infarction. Alcohol is contraindicated for clients taking beta-adrenergic blockers such as metoprolol. Clients should never increase medications without medical direction. The pulse rate can go lower than 70 beats per minute as long as the client is asymptomatic.

A nurse has administered oral radioactive iodine to a client with thyroid cancer. What instructions will the nurse provide to the client upon discharge? A. Do not share utensils with your family members B. Remain isolated until instructed by your healthcare provider C. Limit your fluid intake for the first several days D. Use a bedside commode for your elimination needs

A. Do not share utensils with your family members Rationale: Radioactive medications such as iodine remain active until the substance decays. It is important to advise the client to not share food utensils as secretions are radioactive and can be transmitted to another person. The client does not need to be isolated. The nurse should instruct the client to maintain a 6-foot distance from others. Fluid intake should be increased after taking radioactive iodine to aid in waste removal. Bodily fluids and secretions remain radioactive and should be disposed of properly. Using a bedside commode increases the risk of transmission to others.

A primary health care provider prescribes 0.25 mg of alprazolam by mouth three times a day for a client with anxiety and physical symptoms related to work pressures. Which side effect of this medication will the nurse monitor for in this client? A. Drowsiness B. Bradycardia C. Agranulocytosis D. Tardive dyskinesia

A. Drowsiness Alprazolam, a benzodiazepine, potentiates the actions of gamma-aminobutyric acid, enhances presympathetic inhibition, and inhibits spinal polysynaptic afferent pathways. Drowsiness, dizziness, and blurred vision are common side effects. Alprazolam may cause tachycardia, not bradycardia. Agranulocytosis is usually a side effect of the antipsychotics in the phenothiazine group, not benzodiazepines. Tardive dyskinesia occurs after prolonged therapy with antipsychotic medications; alprazolam is an antianxiety medication, not an antipsychotic.

The nurse is teaching a client about tricyclic antidepressants. Which potential side effects would the nurse include? Select all that apply. One, some, or all responses may be correct. A. Dry mouth B. Drowsiness C. Constipation D. Severe hypertension E. Orthostatic hypotension

A. Dry mouth B. Drowsiness C. Constipation E. Orthostatic hypotension Dry mouth is a common anticholinergic side effect of tricyclic antidepressants. Drowsiness can be a common side effect but usually decreases with continued treatment. Constipation is a common side effect that usually can be managed with stool softeners and a high-fiber diet. Orthostatic hypotension is a common side effect of tricyclic antidepressants; the client should be instructed to rise slowly from a sitting to a standing position. Hypertension of any type is not a side effect of tricyclic antidepressants.

A client is prescribed oral disopyramide to manage a ventricular dysrhythmia. Which side effects will the nurse include when teaching the client about this medication? Select all that apply. One, some, or all responses may be correct. A. Dry mouth B. Rhinorrhea C. Constipation D. Hyperglycemia E. Stress incontinence

A. Dry mouth C. Constipation Dry mouth occurs because of its anticholinergic properties. Constipation is a side effect of this nonnitrate antidysrhythmic because of its anticholinergic properties. A thin, watery discharge from nose (rhinorrhea) does not occur with this medication because of its anticholinergic properties. Hypoglycemia, not hyperglycemia, may occur. Urinary hesitancy and retention, rather than stress incontinence, occur.

A client who has returned from surgery reports feeling nauseated and later has an emesis. The nurse administers promethazine per standing orders. In addition to relief from nausea, what other effects of this medication does the nurse expect? Select all that apply. A. Dry mouth B. Sedation C. Pinpoint pupils D. Heart palpitations E. Rhinorrhea

A. Dry mouth B. Sedation D. Heart palpitations Promethazine is used as an antihistamine, sedative and antiemetic. It produces anticholinergic effects, such as dry mouth and reduced nasal congestion, dilated pupils and urinary retention. Although promethazine is a sedative, the nurse should understand that it can cause some people to have heart palpitations and to feel restless and unable to sleep.

administer vaccines to a 4-year-old child including the DTap, IPV, MMR, VAR. Before, what information should the nurse be aware of? Select all that apply. A. Either the deltoid muscle of the arm or anterolateral thigh muscle B. A 20 gauge needle varicella (VAR) vaccine intramuscularly (IM) C. A 5/8 inch needle is used for (SubQ) injections D. The vaccines contain preservative thimerosal E. Multiple should be administered a min of 1 in apart F. The vaccines all contain weakened live viruses

A. Either the deltoid muscle of the arm or anterolateral thigh muscle can be used C. A 5/8 inch needle length is often used for subcutaneous (SubQ) injections E. Multiple immunizations should be administered a minimum of 1 inch apart Vaccinations for a 4 to 6 year-old child include diphtheria, tetanus, and whooping cough (DTaP), Polio (IPV), measles, mumps, and rubella (MMR), and chicken pox (Varicella). DTap is given intramuscularly (IM) and can be administered in either the deltoid muscle of the arm or the anterolateral thigh muscle. The IPV can be administered either subcutaneously (subq) or IM. If multiple vaccinations are to be administered, injections should be spaced a minimum of 1-inch apart. The MMR and Varicella are administered subq using a 5/8 inch, 25-gauge needle. Not all the vaccinations contain live viruses; IPV and DTaP. Vaccines no longer contain thimerosal, which is a form of mercury.

The nurse is caring for a client with sepsis receiving broad-spectrum antibiotics. Which finding might indicate to the nurse the need for a dosage adjustment? A. Elevated creatinine level B. Elevated heart rate C. Decreased white blood cell count D. Decreased platelet count

A. Elevated creatinine level Rationale: Septic shock is the most common type of distributive shock that threatens multi-system organ failure with a rapid onset, which is the leading cause of death in noncoronary ICU patients. Gram-negative bacteria have been the most implicated organism, and broad-spectrum antibiotics are given to help increase the likelihood of increasing tissue perfusion. The majority of broad-spectrum antibiotics are excreted through the kidneys, and an elevated creatine level will indicate the need for dosage adjustments. Elevated lactic acid levels, heart rate, and white blood cell (WBC) levels are all signs of sepsis and need to be monitored closely. Decreased platelet counts are seen when the condition is exacerbated with blood loss but does not affect the antibiotic dosage.

The nurse is assessing a child receiving chemotherapy for treatment of leukemia. Which side effect would the nurse anticipate? A. Epistaxis B. Tachycardia C. Flushed skin D. Increased temperature

A. Epistaxis Nosebleeds (epistaxis) are expected in a child with leukemia who is undergoing chemotherapy because the bone marrow is depressed and the number of platelets decreases substantially. Tachycardia is not expected unless there is severe anemia. Usually children with leukemia have pale skin. An increased temperature occurs only if there is an infection resulting from the leukemia.

A client with a dysrhythmia is prescribed procainamide (Pronestyl) in 4 divided doses over the next 24 hours. Which dosing schedule is best for the nurse to implement? A. Every 6 hours. B. QID. C. AC and bedtime. D. PC and bedtime.

A. Every 6 hours Pronestyl, procainamide, is a class 1A antidysrhythmic. It should be taken around the clock, so that a stable blood level of the drug can be maintained, thereby decreasing the possibility of hypotension (an adverse effect) occurring because of too much of the drug circulating systemically at any particular time of day. Pronestyl may be given with food if GI distress is a problem.

The nurse is developing a plan of care for a client who has developed blisters and sores in the mouth after receiving chemotherapy. Which interventions should the nurse include? Select all that apply. A. Examine your mouth frequently. B. Use strong mouthwashes to kill bacteria. C. Drink 2 or more liters of water per day. D. Suck on ice chips during chemotherapy. E. Visit a dental hygienist weekly. F. Avoid spicy or acidic foods. Correct Answer (Blank)

A. Examine your mouth frequently. C. Drink 2 or more liters of water per day. D. Suck on ice chips during chemotherapy. F. Avoid spicy or acidic foods. Mucositis is a complex, multiphase process at the cellular level started in response to cytotoxic chemotherapy. The epithelial cells in the mouth are very sensitive to chemotherapy due to their high rate of cell turnover. Oral cryotherapy using ice water or ice chips can be used for the prevention of mucositis. It is believed that vasoconstriction caused by the cold temperature decreases exposure of the oral mucous membranes to the mucositis-causing agents. Frequent mouth assessment, and good and frequent oral hygiene are key in managing mucositis. The client should avoid the use of "strong" mouthwashes that often contain alcohol. Mucositis can be managed at home and does not require seeing a dental hygienist. Increased hydration is generally recommended.

The nurse is caring for a client with type 1 diabetes. Which signs or symptoms may indicate that the client has insulin-induced hypoglycemia? Select all that apply. One, some, or all responses may be correct. A. Excessive hunger B. Weakness C. Diaphoresis D. Excessive thirst E. Deep respirations

A. Excessive hunger B. Weakness C. Diaphoresis Hypoglycemia affects the central nervous system, causing weakness. Hypoglycemia affects the sympathetic nervous system, causing diaphoresis. Excessive hunger is associated with hypoglycemia because the body needs glucose for cellular metabolism. Excessive thirst is associated with hyperglycemia because fluid shifts, along with the excess glucose being excreted by the kidneys, result in polyuria. Deep respirations (Kussmaul respirations) are associated with hyperglycemia because the body is attempting to blow off carbon dioxide to compensate for the metabolic acidosis.

A client with hyperlipidemia receives a prescription for niacin (Niaspan). Which client teaching is most important for the nurse to provide? A. Expected duration of flushing. B. Symptoms of hyperglycemia. C. Diets that minimize GI irritation. D. Comfort measures for pruritus.

A. Expected duration of flushing. Flushing of the face and neck, lasting up to an hour, is a frequent reason for discontinuing niacin. Inclusion of this effect in client teaching may promote compliance in taking the medication. While nutrition tips and managing pruritus are worthwhile instructions to help clients minimize or cope with normal side effects associated with niacin (Niaspan), flushing is intense and causes the most concern for the client.

The nurse in an urgent care clinic is preparing discharge instructions for the parents of a 15-month-old child with a first episode of otitis media. Which information is the priority to include? A. Explain that the child should complete the full 10 days of antibiotics B. Describe the tympanocentesis most likely needed to clear the infection C. Offer information on recommended immunizations around the child's second birthday D. Provide a written handout describing the care of myringotomy tubes

A. Explain that the child should complete the full 10 days of antibiotics Otitis media, an inner ear infection, commonly occurs in young children. Although not always caused by bacteria, many ear infections are treated with oral antibiotics. If a client is prescribed antibiotics, the priority is to make sure that they take the full prescription for the prescribed number of days to prevent recurrence or antibiotic resistance.

A client who takes rifampin tells the nurse, 'My urine looks orange.' Which action would the nurse take? A. Explain that this is expected. B. Check the liver enzymes. C. Ask the provider to order a urinalysis. D. Ask what foods were eaten.

A. Explain that this is expected. Rifampin causes a reddish-orange discoloration of secretions such as urine, sweat, and tears. Although liver enzymes should be monitored because of the risk of hepatitis, this action is not addressing the client's statement. A urinalysis is not indicated for an anticipated finding. The medication, not food, is responsible for the urine color.

A client who is taking isoniazid for tuberculosis asks the nurse about the possible side effects of this medication. The nurse informs the client to report which side effect of this medication to the primary health care provider (HCP)? A. Extremity tingling and numbness B. Confusion and light-headedness C. Double vision and visual halos D. Photosensitivity and photophobia

A. Extremity tingling and numbness Peripheral neuropathy is a common side effect of isoniazid and other anti-tubercular medications. Extremity tingling and numbness should be reported to the primary health care provider (HCP). Daily doses of pyridoxine (vitamin B6) may lessen or even reverse peripheral neuropathy due to isoniazid use.

Sildenafil is prescribed for a man with erectile dysfunction. Which side effects of this medication would the nurse mention in teaching? Select all that apply. One, some, or all responses may be correct. A. Flushing B. Headache C. Dyspepsia D. Constipation E. Hypertension

A. Flushing B. Headache C. Dyspepsia Flushing is a common central nervous system response to sildenafil. Headache is a common central nervous system response to sildenafil. Dyspepsia is a common gastrointestinal response to sildenafil. Diarrhea, not constipation, is a common gastrointestinal response to sildenafil. Hypotension, not hypertension, is a cardiovascular response to sildenafil. It should not be taken with antihypertensives and nitrates because medication interactions can precipitate cardiovascular collapse.

Which diuretic would the nurse anticipate administering to a client admitted with acute pulmonary edema? A. Furosemide B. Chlorothiazide C. Spironolactone D. Acetazolamide

A. Furosemide Furosemide acts on the loop of Henle by increasing the excretion of chloride and sodium; is available for intravenous administration; and is more effective than chlorothiazide, spironolactone, and acetazolamide. Although it is used in the treatment of edema and hypertension, chlorothiazide is not as efficacious as furosemide. Spironolactone is a potassium-sparing diuretic; it is less efficacious than thiazide diuretics. Acetazolamide is used in the treatment of glaucoma to lower intraocular pressure.

The nurse is caring for a client who reports sweating, tachycardia, and tremors. The laboratory report of the client reveals serum cortisol less than normal and a blood glucose level of 60 mg/dL. Which medication would be administered to this client? A. Glucagon B. Kayexalate C. Hydrocortisone D. Insulin with dextrose in normal saline

A. Glucagon A decrease in cortisol levels impairs the glucose metabolism. The client's blood glucose level is 60 mg/dL, which is indicative of hypoglycemia. The nurse should administer glucagon as per the prescription to manage the low glucose levels. Kayexalate is a potassium-binding resin that facilitates potassium excretion and is used to manage hyperkalemia. Intramuscular hydrocortisone is given concomitantly every 12 hours as part of hormone replacement in adrenal insufficiency. Insulin with dextrose in normal saline is given to manage hyperkalemia by causing an intracellular shift of potassium.

Which conditions would the nurse identify as decreasing the effectiveness of estrogen therapy? Select all that apply. One, some, or all responses may be correct. A. Habit of smoking B. Use of anticoagulants C. Use of tricyclic antidepressants D. Presence of endometrial cancer E. Presence of thromboembolic disorders

A. Habit of smoking B. Use of anticoagulants The effectiveness of estrogen therapy decreases with smoking and decreases with the use of anticoagulants. The use of tricyclic antidepressants along with estrogen may result in medication toxicity. Estrogen therapy is not recommended in clients with endometrial cancer and thromboembolic disorders because it may increase the risk of these complications.

The nurse is providing discharge instructions to a client with a prescription for sublingual nitroglycerin. The nurse should inform the client to prepare for this most common side effect? A. Headache B. Depression C. Dry mouth D. Anorexia

A. Headache Nitroglycerin is a potent vasodilator and a headache is the most common side effect. The headache comes on suddenly and can be severe, thus the client should be prepared for this effect. The other side effects listed are common side effects of oral medications, but not specifically to nitroglycerin.

A 12-year-old child has just received a dose of epinephrine. Which assessment is the priority after this medication is administered? A. Heart rate B. Glucose level C. Neurological status D. Blood pressure

A. Heart rate Epinephrine is a sympathetic nervous system stimulant that may cause significant tachycardia, so assessment of heart rate is the priority. Assessing for hyperglycemia (glucose level), pupil dilatation (neurological status), and hypertension (blood pressure) are all assessments that the nurse would also perform, but these would be done after assessing heart rate for tachycardia.

A 32-year-old female with human epidermal growth factor receptor 2-positive (HER2-positive) metastatic breast cancer is scheduled to begin therapy with pertuzumab. What information is important for the nurse to reinforce and discuss with the client? Select all that apply. A. Other therapies for cancer treatment are no longer needed. B. Use contraception during and for 6 months following the use of this drug. C. Take the medication at the same time every day on an empty stomach. D. Report shortness of breath, lightheadedness, dizziness, cough or swelling of the feet. E. Report chills, fatigue, or headache during treatment

B. Use contraception during and for 6 months following the use of this drug. D. Report shortness of breath, lightheadedness, dizziness, cough or swelling of the feet. E. Report chills, fatigue, or headache during treatment Pertuzumab (Perjeta) is used in combination with trastuzumab (Herceptin) as a targeted therapy for HER2+ metastatic breast cancer; these medications are used in combination with chemotherapy and radiation. The most common side effects are fatigue, loss of taste, muscle pain and vomiting; sometimes slowing the infusion rate can help. It is best to eat a small meal before receiving the infusion. Serious side effects include birth defects and fetal death; women of child-bearing age must use a form of effective contraception during and for 6 months following treatment. Drugs that block HER2+ activity decrease left ventricular ejection fraction (LVEF) and will worsen symptoms of congestive heart failure; heart function must be tested before and monitored during treatment.

To evaluate the effectiveness of antiretroviral therapy for a client infected with human immunodeficiency virus (HIV), which laboratory test result will the nurse plan to review? A. Western blot test B. Viral load test C. Nucleic acid amplification test D. Rapid HIV antibody test

B. Viral load test Viral load refers to the amount of HIV circulating in the blood. The effectiveness of antiretroviral therapy (ART) is measured by the decrease in the amount of HIV virus, i.e., viral load, detectable in the blood. The goal is for the viral load to be so low that it is deemed undetectable. An undetectable viral load does not mean that the client is cured or can no longer transmit the disease. The other tests are used to detect HIV antibodies, which remain positive even with effective ART. A nucleic acid amplification test (NAAT) is commonly used to diagnose a gonorrhea infection.

Which supplement would the nurse instruct a client taking oral contraceptives to increase? A. Calcium B. Vitamin C C. Vitamin E D. Potassium

B. Vitamin C Oral contraceptives can affect the metabolism of certain vitamins, particularly vitamin C, and supplementation may be required. It is unnecessary to increase the intake of calcium when one is taking oral contraceptives. There is no clinical evidence linking oral contraceptives with a deficiency of vitamin E. There is no interrelationship between oral contraceptives and dietary intake of potassium.

The nurse is monitoring a 6-year-old child for toxicity precipitated by digoxin. Which sign of digoxin toxicity would the nurse monitor for? A. Oliguria B. Vomiting C. Tachypnea D. Splenomegaly

B. Vomiting Vomiting is a sign of digoxin toxicity in children. Oliguria is associated with renal failure, not toxicity. Tachypnea is associated with heart failure, not toxicity. Splenomegaly is associated with heart failure, specifically right ventricular failure.

Which action describes a therapeutic effect of atenolol? A. Heart rate decreases B. Cardiac output increases C. Bronchospasm is relieved D. Pulse oximetry improves

A. Heart rate decreases Atenolol, a beta-blocker, slows the rate of sinoatrial (SA) node discharge and atrioventricular (AV) node conduction, thus decreasing the heart rate; it prevents angina by decreasing the cardiac workload and myocardial oxygen consumption. Cardiac output is not increased and may be decreased. Atenolol may promote bronchospasm, not relieve it. Atenolol does not directly affect gas exchange in the lungs to promote improving oxygenation.

A client is prescribed epoetin injections. To ensure the client's safety, which laboratory value would the nurse assess before administration? A. Hemoglobin B. Platelet count C. Prothrombin time D. Partial thromboplastin time

A. Hemoglobin Epoetin is used to treat anemia by increasing production of red blood cells. The laboratory value the nurse would assess before administration is the hemoglobin because it measures the number of red blood cells. Erythropoietin is specific for increasing red blood cells and does not increase other blood components such as white blood cells or thrombocytes (platelets). The partial thromboplastin time and prothrombin time are measures of the effectiveness of anticoagulant therapy.

Which assessment findings during the administration of intravenous penicillin prompt the nurse to stop the infusion? Select all that apply. One, some, or all responses may be correct. A. Hives B. Itching C. Nausea D. Skin rash E. Shortness of breath

A. Hives B. Itching D. Skin rash E. Shortness of breath Penicillin administration carries a high rate of allergic reaction, so the nurse monitors the client for signs of allergy. Hives, itching, skin rash, and shortness of breath are all indications of allergic reaction and warrant cessation of the infusion and contact with the health care provider. Nausea is not an indication of allergic reaction.

Which side effect would the nurse anticipate in a child receiving chelation therapy? A. Hypocalcemia B. Hyperkalemia C. Hypoglycemia D. Hypernatremia

A. Hypocalcemia Calcium EDTA removes calcium along with lead, so the serum calcium level should be checked periodically. Hyperkalemia, hypoglycemia, and hypernatremia do not occur with chelation therapy.

A client is given a prescription for bumetanide. The nurse will teach the client to watch for symptoms of which condition? A. Hypokalemia B. Hyperchloremia C. Hypernatremia D. Hypoglycemia

A. Hypokalemia Bumetanide is a loop diuretic. Diuretic therapy that affects the loop of Henle increases urinary excretion of sodium, chloride, and potassium. As a result, clients are at risk for hypokalemia, hyponatremia, and hypochloremia. Additionally, hyperglycemia can occur.

An adolescent with type 1 diabetes mellitus is admitted to the intensive care unit in ketoacidosis with a blood glucose level of 170 mg/dL (9.4 mmol/L). A continuous insulin infusion is started. Which adverse reaction to the infusion is most important for the nurse to monitor? A. Hypokalemia B. Hypovolemia C. Hypernatremia D. Hypercalcemia

A. Hypokalemia Insulin moves potassium into the cells along with glucose, thus lowering the serum potassium level. Insulin does not lead to a reduced blood volume. Insulin does not directly alter the sodium levels. Insulin does not affect the calcium levels.

Intravenous fluids and insulin are prescribed to treat a client's diabetic ketoacidosis. The client develops peripheral paresthesias and shortness of breath. The cardiac monitor shows the appearance of a U wave. Which complication would the nurse suspect? A. Hypokalemia B. Hypoglycemia C. Hypernatremia D. Hypercalcemia

A. Hypokalemia These are classic signs of hypokalemia that occur when potassium levels are reduced as potassium reenters cells with glucose. Symptoms of hypoglycemia are weakness, nervousness, tachycardia, diaphoresis, irritability, and pallor. Symptoms of hypernatremia are thirst, orthostatic hypotension, dry mouth and mucous membranes, concentrated urine, tachycardia, irregular heartbeat, irritability, fatigue, lethargy, labored breathing, and muscle twitching or seizures. Symptoms of hypercalcemia are lethargy, nausea, vomiting, paresthesias, and personality changes.

A child infected with human immunodeficiency virus (HIV) is admitted with Pneumocystis jiroveci pneumonia and receives trimethoprim/sulfamethoxazole. Which common side effects would the nurse anticipate? Select all that apply. One, some, or all responses may be correct. A. Jaundice B. Vomiting C. Headache D. Crystalluria E. Photosensitivity

B. Vomiting D. Crystalluria E. Photosensitivity Nausea and vomiting may occur as a result of gastrointestinal irritation. Crystalluria may occur with this medication, especially in the presence of restricted fluid intake secondary to nausea and vomiting. Skin reactions such as photosensitivity are also common. Hepatic side effects such as jaundice may occur but are not common. Central nervous system side effects such as headache are rare adverse reactions.

The home health nurse is teaching a female client about self-administering vancomycin. Which statement by the client demonstrates understanding of the teaching? A. I need to call my provider if my urine changes B. Muscle tingling and weakness is an expected side effect of this medication C. Ringing in the ears is common when taking vancomycin D. I should avoid eating food with active cultures in it

A. I need to call my provider if my urine changes Rationale: Vancomycin is commonly linked to nephrotoxicity, leading to the need for monitoring trough levels. Signs of kidney injury include decreased urination, blood in the urine, and other changes in urine color and clarity. Antibiotic-associated diarrhea (colitis) results from oral or parenteral antibiotic therapy. Another pathogen is Candida albicans, which results in vaginal yeast infection and oral thrush. Probiotics can reduce these risks. Antibiotic-induced neuropathy is a rare complication of several antimicrobial agents. Hypokalemia can result from vancomycin; therefore, muscle weakness and numbness or tingling should be reported. Ototoxicity is a serious complication from vancomycin due to vestibular damage.

The nurse is counseling a client with gastroesophageal reflux disease (GERD) who has been taking prescribed famotidine for two days. Which statement would require immediate follow up by a healthcare provider? A. I take digoxin for my heart failure B. I use calcium carbonate if I have symptoms after meals C. I use alendronate for my osteoporosis D. I'm still having some symptoms of heartburn.

A. I take digoxin for my heart failure Rationale: Most medications for heartburn decrease stomach acid. Histamine blocking drugs such as famotidine (H2 receptor antagonist) are available as both prescription and over-the-counter. It is often advised to take an antacid with an H2RA to relieve pain. Symptoms should be improved after one week. Famotidine does not cause bone loss, unlike proton pump inhibitors, and is an acceptable choice for clients with osteoporosis. Famotidine is used cautiously in clients on digoxin as it decreases absorption. This client needs to have their digoxin level checked, and the dosage may need to be adjusted.

The nurse is providing care for a client prescribed propranolol. Which symptoms should the nurse report to the healthcare provider immediately? A. Headache, hypertension, and blurred vision. B. Wheezing, hypotension, and AV block. C. Vomiting, dilated pupils, and papilledema. D. Tinnitus, muscle weakness, and tachypnea.

B. Wheezing, hypotension, and AV block. Wheezing, hypotension, and AV block represents the most serious adverse effects of beta-blocking agents. AV block is generally associated with bradycardia and results in potentially life-threatening decreases in cardiac output. Additionally, wheezing secondary to bronchospasm and hypotension represent life-threatening respiratory and cardiac disorders.

A nurse is educating a client on insulin administration. Which statement made by the client indicates further teaching is required? A. I will inject the insulin in the same site every day B. The best injection area is around my abdomen C. I will squeeze my skin together to inject the medication D. Gentle pressure should be applied to the site after injection

A. I will inject the insulin in the same site every day Rationale: The nurse should further educate the client on rotating injection sites to prevent lipohypertrophy. Lipohypertrophy is the development of scar tissue under the skin that prevents adequate absorption of the medication. The absorption rate is greater in the subcutaneous tissue of the abdomen. Squeezing or bunching the skin together ensures the medication is administered into the subcutaneous layer. Gentle pressure helps the medication to absorb better.

The nurse is reviewing the laboratory results for a client with cancer who is being treated with chemotherapy and recently started prescribed filgrastim. Which laboratory value indicates the treatment is effective? A. Hemoglobin level of 9.8 g/dL B. White blood cell count (WBC) of 5,200/mm3 C. Platelet count of 200,000/mm D. Red blood cell count (RBC) of 4 million/mm

B. White blood cell count (WBC) of 5,200/mm3 Rationale: The client has a normal white blood cell count indicating that filgrastim has been effective. The action of filgrastim is to increase neutrophil production, thereby increasing the white blood cell (WBC) count. Decreased hemoglobin (Hgb) indicates anemia. The hemoglobin and red blood cell (RBC) count are below normal limits for an adult male. Epoetin alfa is used to treat low RBC counts (anemia) caused by chemotherapy. The platelet count is within normal limits for an adult client.

The nurse evaluates that teaching for the oral pancreatic enzymes pancrelipase is understood when the client identifies which time for medication scheduling? A. At bedtime B. With meals C. One hour before meals D. On arising each morning

B. With meals The pancreatic enzymes (amylase, trypsin, and lipase) must be present when food is ingested for digestion to take place. At bedtime the food eaten for dinner has passed beyond the duodenum, so at bedtime the enzyme is given too late to aid digestion. Taking pancrelipase 1 hour before meals or on arising each morning will have no chyme in the duodenum on which the enzyme can act.

Which time for medication scheduling would a nurse teach to a client prescribed the oral pancreatic enzymes pancrelipase? A. At bedtime B. With meals C. One hour before meals D. On arising each morning

B. With meals The pancreatic enzymes (amylase, trypsin, and lipase) must be present when food is ingested for digestion to take place. At bedtime the food eaten for dinner has passed beyond the duodenum, so at bedtime the enzyme is given too late to aid digestion. Taking pancrelipase 1 hour before meals or on arising each morning will have no chyme in the duodenum on which the enzyme can act.

Which adverse response to isoniazid (INH) in a client with tuberculosis would cause the nurse to determine that prompt intervention is needed? A. Orange feces B. Yellow sclera C. Temperature of 96.8°F (36°C) D. Weight gain of 5 pounds (2.3 kilograms)

B. Yellow sclera An adverse reaction to isoniazid (INH) is hepatitis, resulting in jaundice. Rifampin, an antitubercular medication, can color excretions orange, which is not harmful. A temperature of 96.8°F (36°C) is within expected limits. Weight gain indicates improvement in the client's health status.

When teaching a client about digoxin, which symptom will the nurse include as a reason to withhold the digoxin? A. Fatigue B. Yellow vision C. Persistent hiccups D. Increased urinary output

B. Yellow vision Digoxin toxicity is a common and dangerous effect. Visual disturbances, most notably yellow vision, may be evidence of digoxin toxicity. Fatigue is not a toxic effect of digoxin. Persistent hiccups are not related to digoxin toxicity. An increased urinary output is not a sign of digoxin toxicity; it may be a sign of a therapeutic response to the medication and an improved cardiac output.

The nurse is teaching a client with diabetes about newly prescribed trimethoprim and sulfamethoxazole (TMP-SMX) to treat a urinary tract infection. Which statement by the client indicates understanding? A. I will stop taking this medication if I develop a rash." B. This antibiotic will kill mature bacteria in my urinary tract." C. I should avoid dairy products when taking this medication." D. "My blood sugar will not be affected by this medication."

A. I will stop taking this medication if I develop a rash." Rationale: TMP-SMX is a sulfonamide medication. These drugs are bacteriostatic and therefore, halt the multiplication of new bacteria, but do not kill mature bacteria. Clients using sulfonylureas for the management of diabetes should know that other sulfa drugs may increase the chances of hypoglycemia. The action of metformin is also enhanced. Dairy is avoided when clients are taking tetracyclines. TMP-SMX is the most common cause of erythema multiforme. Sulfonamides are also often implicated in cases of both toxic epidermal necrosis and Stevens-Johnson syndrome, which can be fatal.

A nurse is providing care to a client with ovarian cancer prescribed intravenous topotecan. The nurse expects to administer the medication via which venous access site? A. Implanted port B. PICC C. Central line D. Peripheral

A. Implanted port Rationale: Topotecan is an antineoplastic medication administered over the course of 21 days. Chemotherapy medications are commonly administered via an implanted port. An implanted port is accessed through the skin only when therapy is needed. A peripherally inserted central line (PICC), a central line, and a peripheral line have continuous external access and have a higher risk of infection.

Which instruction the nurse give to a female client who received a prescription for metronidazole (Flagyl) for treatment of trichomonas vaginalis? Select all that apply A. Increase fluid intake, especially cranberry juice. B. Do not abruptly discontinue the medication; taper use. C. Check blood pressure daily to detect hypertension. D. Avoid drinking alcohol while taking this medication. E. Use condoms until treatment is completed. F. Ensure that all sexual partners are treated at the same time.

A. Increase fluid intake, especially cranberry juice. D. Avoid drinking alcohol while taking this medication. E. Use condoms until treatment is completed. F. Ensure that all sexual partners are treated at the same time. Increased fluid intake and cranberry juice are recommended for prevention and treatment of urinary tract infections, which frequently accompany vaginal infections. It is not necessary to taper use of this drug or to check the blood pressure daily, as this condition is not related to hypertension. Flagyl can cause a disulfiram-like reaction if taken in conjunction with ingestion of alcohol, so the client should be instructed to avoid alcohol. All sexual partners should be treated at the same time and condoms should be used until after treatment is completed to avoid reinfection.

The nurse is caring for a client who received digoxin-specific immune fab. Which finding indicates the treatment is having the intended effect? A. Increased heart rate B. Decreased potassium levels C. Decreased blood pressure D. Increased serum digoxin levels

A. Increased heart rate Rationale: Digoxin-specific immune fab is an antidote that binds molecules of digoxin, making them unavailable for binding at their usual sites of action in the body. After administration of the medication, serum digoxin levels may be misleading, as they will be elevated until the drug is excreted by the kidneys. The goal of treatment is to lower digoxin levels and treat symptomatic digoxin toxicity, specifically cardiac dysrhythmias including bradycardia. Potassium levels may be low, triggering digoxin toxicity, and then elevated due to shifts caused by digoxin toxicity, so fluctuating levels are not a sign of effective treatment. Effective treatment of dysrhythmia should raise blood pressure.

Which mechanism of action explains how hydrochlorothiazide increases urine output? A. Increases the excretion of sodium B. Increases the glomerular filtration rate C. Decreases the reabsorption of potassium D. Increases renal perfusion

A. Increases the excretion of sodium Hydrochlorothiazide inhibits sodium reabsorption in the nephrons, causing increased excretion of sodium, which increases urine excretion. The glomerular filtration rate is not affected. The loss of potassium is a side effect, not the mechanism of action. Renal perfusion is not affected.

Which action would the nurse take to avoid red man syndrome when preparing to administer a vancomycin infusion? A. Infuse slowly. B. Change the intravenous (IV) site. C. Reduce the dosage. D. Administer vitamin K.

A. Infuse slowly. Vancomycin should be infused slowly to avoid the occurrence of the reaction known as 'red man syndrome.' Changing the IV site reduces the incidence of thrombophlebitis. Reducing the dosage is done in the setting of renal dysfunction. Administration of vitamin K is done to correct an elevated prothrombin time.

A client with hyperthyroidism is to receive methimazole. Which information would the nurse provide? A. Initial improvement will take several weeks. B. Few side effects are associated with this medication. C. This medication may be taken at any time during the day. D.Large loading doses are used initially to normalize thyroid function

A. Initial improvement will take several weeks. Methimazole blocks thyroid hormone synthesis; it takes several weeks of medication therapy before the hormones stored in the thyroid gland are released and the excessive level of thyroid hormone in the circulation is metabolized. There are many common side effects that include nausea, vomiting, diarrhea, rash, urticaria, pruritus, alopecia, hyperpigmentation, drowsiness, headache, vertigo, and fever. Methimazole should be spaced at regular intervals because blood levels are reduced in approximately 8 hours. Large doses cause toxic side effects that can be life threatening, including nephritis, hepatitis, agranulocytosis, leukopenia, thrombocytopenia, hypothrombinemia, and lymphadenopathy.

The nurse is preparing to administer a client's prescribed insulins and needs to mix NPH and lispro. Which of the following actions should the nurse take first? A. Inject air into the long-acting insulin B. Draw up the short-acting insulin C. Draw up the long-acting insulin D. Inject air into the short-acting insulin

A. Inject air into the long-acting insulin Rationale: When mixing insulins in the same vial, the process should be to inject air into the long-acting insulin, inject air into the short-acting insulin, draw up the short-acting insulin, and then draw up the long-acting insulin.

The nurse plans to teach a client with type 1 diabetes about the use of an insulin pump. Which information will the nurse include in client teaching? A. Insulin pumps mimic the way a healthy pancreas works. B. The insulin pump's needle should be changed every day. C. Pumps are implanted in a subcutaneous pocket near the abdomen. D. The insulin pump's advantage is that it only requires glucose monitoring once a day.

A. Insulin pumps mimic the way a healthy pancreas works. The basal infusion rate mimics the low rate of insulin secretion during fasting, and the bolus before meals mimics the high output after meals. The subcutaneous needle may be left in place for as long as 3 days. Most insulin pumps are external to the body and access the body via a subcutaneous needle. Blood glucose monitoring is done a minimum of four times a day.

The nurse is providing teaching to the client taking metoclopramide. Serious side effects that should be reported to the provider are included in the teaching plan. Which of the following side effects is the priority? A. Involuntary muscle movements B. Report of increased fatigue C. Onset of headaches D. Difficulty with sleep

A. Involuntary muscle movements Rationale: Metoclopramide is a GI stimulant that is effective in reducing headache, nausea, and vomiting. Metoclopramide can cause a serious movement disorder called tardive dyskinesia (TD). This condition is often irreversible. TD is characterized by involuntary movements of the face, tongue, or extremities. The risk of developing TD is increased with longer treatment and increased dosage. To help prevent TD, this drug shouldn't be used for longer than 12 weeks. The more common side effects of metoclopramide can include headache, confusion, drowsiness, dizziness, restlessness, and insomnia.

The nurse concludes that a client has a hypoglycemic reaction to insulin. Which clinical findings support this conclusion? Select all that apply. One, some, or all responses may be correct. A. Irritability B. Glycosuria C. Dry, hot skin D. Heart palpitations E. Fruity odor of breath

A. Irritability D. Heart palpitations Irritability, a neuroglycopenic symptom, occurs when the glucose in the brain declines to a low level. Heart palpitations, a neurogenic symptom, occur when the sympathetic nervous system responds to a rapid decline in blood glucose. Because the blood glucose level is decreased, the renal threshold is not exceeded, and there is no glycosuria. Dry, hot skin is consistent with dehydration, which often is associated with hyperglycemic states. Fruity odor of the breath is associated with hyperglycemia; it is caused by the breakdown of fats as a result of inadequate insulin supply.

A parent of three young children has contracted tuberculosis. Which medication would the nurse anticipate being prescribed for members of the family who have been exposed? A. Isoniazid B. Multiple-puncture test C. Bacille Calmette-Guérin D. Tuberculin purified protein derivative

A. Isoniazid Isoniazid is used as a prophylactic agent for people who have been exposed to tuberculosis; also, it is one of several medications used to treat the disease. Multiple-puncture tests, such as the tine test, are used to test for tuberculosis; these are no longer recommended. They are not a treatment for the prevention or cure of tuberculosis. Bacille Calmette-Guérin is a vaccine that provides limited immunity; it is not recommended for use in the United States. Tuberculin purified protein derivative, the Mantoux test, is a widely used skin test for detecting tuberculosis; it is not a treatment for the prevention or treatment of tuberculosis.

An adolescent who has been prescribed prednisone and vincristine for leukemia tells the nurse that he is constipated. Which reason would the nurse cite as the probable cause of the constipation? A. It is a side effect of the vincristine. B. The spleen is compressing the bowel. C. It is a toxic effect from the prednisone. D. The leukemic mass is obstructing the bowel.

A. It is a side effect of the vincristine. Constipation is a side effect of vincristine because it slows gastrointestinal motility. An enlarged spleen will put pressure on the stomach and diaphragm, not on the large bowel. Constipation is not a toxic effect of prednisone. It is unlikely that leukemia is causing an obstruction.

Which statements would the nurse include when teaching a client who is considering medroxyprogesterone injection as a form of hormonal birth control? Select all that apply. One, some, or all responses may be correct. A. It lasts for 3 months. B. It can cause nausea and vomiting. C. It contains both progestin and estrogen. D. It decreases the risk of age-related loss of bone density. E. Side effects include drowsiness or insomnia and depression. F. The most severe undesirable effects are liver dysfunction and thromboembolic disorders.

A. It lasts for 3 months. B. It can cause nausea and vomiting. E. Side effects include drowsiness or insomnia and depression. F. The most severe undesirable effects are liver dysfunction and thromboembolic disorders. Medroxyprogesterone does last for 3 months. It can cause nausea and vomiting, as well as lethargy or insomnia and depression, although the most undesirable severe effects are liver dysfunction and thromboembolic disorders, including thrombophlebitis, cerebrovascular disorder, pulmonary embolism, and retinal thrombosis. Medroxyprogesterone does not contain both progestin and estrogen; it is progestin-only. Medroxyprogesterone does not decrease the risk of age-related loss of bone density; in fact, it has a black-box warning because it can cause bone mineral density loss that puts women at higher risk for osteoporosis and bone fractures later in life.

Which rationale accurately explains why insulin is prescribed for clients in acute renal failure? A. It promotes transfer of potassium into cells to lower serum potassium levels. B. Insulin is required because the alpha cells of the pancreas cease to function with renal failure. C. It is necessary to manage the elevated blood glucose levels that accompany renal failure. D. Insulin reduces the accumulated toxins by lowering the metabolic rate.

A. It promotes transfer of potassium into cells to lower serum potassium levels. Insulin promotes the transfer of potassium into cells, which reduces the circulating blood level of potassium. Renal failure does not cause pancreatic alpha cells to cease functioning. Blood glucose levels usually are not elevated in acute renal failure. Insulin will not lower the metabolic rate.

Which benefit is provided by intraarterial chemotherapy for cancer of the liver? A. It reduces systemic toxicity. B. It provides for rapid dilution of chemotherapy. C. The medication bypasses the blood-brain barrier. D. The chemotherapy is delivered to the peritoneal cavity.

A. It reduces systemic toxicity. Higher concentrations of the medication can be delivered to the specific site of the tumor, with reduced systemic toxicity. Providing for rapid dilution of chemotherapy is the purpose of central vascular access devices. The ability to pass the blood-brain barrier is the purpose of intrathecal or intraventricular access devices. Delivering chemotherapy to the peritoneal cavity is the purpose of intraperitoneal chemotherapy.

The nurse is providing discharge instructions to an older adult client with heart failure. The client asks, "What is the purpose for taking the furosemide?" How should the nurse respond? A. It will help with decreasing fluid buildup in your lungs. B. It will help with reducing the risk for an irregular heart rhythm. C. It will protect your kidneys from chronic damage. D. It will reverse the damage to your heart muscle.

A. It will help with decreasing fluid buildup in your lungs. Furosemide is a loop diuretic. Diuretics are the first-line drug of choice in older adults with heart failure (HF) and fluid overload. These drugs enhance the renal excretion of sodium and water by reducing circulating blood volume, decreasing preload, and reducing systemic and pulmonary congestion, i.e., decreased fluid buildup in the lungs. The other actions do not pertain to furosemide.

The nurse is assessing a client with tuberculosis who has been taking prescribed pyrazinamide. Which finding reported by the client should the nurse immediately report to the healthcare provider? A. Joint pain B. Fatigue C. Nausea D. Decreased appetite

A. Joint pain Rationale: Joint pain is a symptom of gout, which is a side effect of pyrazinamide. While fatigue, nausea, and loss of appetite are common side effects of the drug, the joint pain is the priority.

The nurse is caring for a client who is receiving intermittent intravenous piggyback (IVPG) doses of vancomycin every 12 hours. The primary health care provider prescribes trough levels of the antibiotic. The nurse schedules the blood sample to be obtained at which time? A. Just before the medication is administered B. Between 30 and 60 minutes after the infusion is completed C. Six hours after the dose is completely infused D. In the morning before the client eats breakfast

A. Just before the medication is administered Trough levels are measured in relation to the time a medication is administered. The trough level for a medication is drawn just before a medication is given, when the medication's level is at its lowest. Any other time would be inaccurate for a medication's trough level. The medication's peak level is drawn 30 to 60 minutes after the infusion is completed. Whether the client eats breakfast does not affect this medication's trough levels, because it is an intravenous infusion.

Which medications would the nurse plan to use when administering chelation therapy to a toddler-age client to decrease the pain associated with intramuscular administration? Select all that apply. One, some, or all responses may be correct. A. LMX-4 B. Fentanyl C. Procaine D. Ibuprofen E. Acetaminophen

A. LMX-4 C. Procaine LMX-4 is a topical medication that can be applied to the skin to decrease the pain associated with chelation therapy. Procaine is a medication that can be mixed with chelation therapy to decrease the pain associated with chelation therapy. Fentanyl, ibuprofen, and acetaminophen are not medications used to decrease the pain associated with chelation therapy.

A client with anemia has a new prescription for ferrous sulfate. When teaching the client about diet and iron supplements, what should the nurse emphasize about taking an iron supplement? A. Lie down for about 10 minutes after taking the pill B. Take the iron tablet with a glass of orange juice C. Take an antacid with the iron supplement to reduce stomach upset D. Take the iron tablet with a glass of low-fat milk

A. Lie down for about 10 minutes after taking the pill Iron is best taken on an empty stomach, one hour before or two hours after meals, with a full glass of water or orange juice (ascorbic acid enhances the absorption of iron.) The client should not take the medication with antacids, dairy products, coffee or tea because these will decrease the effectiveness of the medicine. The client should not lie down for at least 10 minutes after taking the medicine.

Intravenous (IV) insulin is prescribed for a client in. Which insulin can be administered IV? Select all that apply. One, some, or all responses may be correct. A. Lispro insulin B. Aspart insulin C. Regular insulin D. Glargine insulin E. Glulisine insulin

A. Lispro insulin B. Aspart insulin C. Regular insulin E. Glulisine insulin Four insulins are approved for IV administration: regular, aspart, lispro, and glulisine insulin can be administered intravenously. Glargine insulin is long-acting insulin; it is not approved for IV use.

In addition to clients who are receiving insulin for type 1 diabetes, the nurse will assess for signs and symptoms of hypoglycemia in clients who have which diagnosis? A. Liver failure B. Anemia C. Hyperthyroidism D. Stage 3 hypertension

A. Liver failure The client with liver disease is at risk for hypoglycemia—40% of clients with liver failure develop hypoglycemia. This client is limited in mobilizing carbohydrates because of a decreased ability to form glycogen (glycogenesis) and to form glucose from glycogen (glycogenolysis). Anemia is not related to a decreased serum glucose level. Hyperthyroidism is not related to a decreased serum glucose level. Hypertension is not related to a decreased serum glucose level.

Which essential test results will the nurse review before starting antitubercular pharmacotherapy when caring for a client with human immunodeficiency virus (HIV) infection who is diagnosed with tuberculosis? A. Liver function studies B. Pulmonary function studies C. Electrocardiogram and echocardiogram D. White blood cell counts and sedimentation rate

A. Liver function studies Antitubercular medications, such as isoniazid (INH) and rifampin (RIF), are hepatotoxic; liver function should be assessed before initiation of pharmacological therapy. Pulmonary function studies, electrocardiogram, and echocardiogram might be done; the results of these tests are not crucial for the nurse to review before administering antitubercular medications. White blood cell counts and sedimentation will not provide information relative to starting antitubercular therapy or to its side effects.

Which are the characteristics of reactions associated with immunizations for a 2-month-old infant? A. Local or systemic and usually mild B. Often serious, possibly requiring hospitalization C. Sometimes causing ulceration at the injection site D. May be responsible for permanent neurological damage

A. Local or systemic and usually mild Mild reactions consist of redness and induration at the injection site, slight fever, and irritability. Serious reactions are not common. Induration at the injection site may occur, but not ulceration. Permanent brain damage is not likely after an immunization.

A health care provider prescribes bed rest, loperamide, and esomeprazole for a client who just had major surgery. After several days of this regimen, the client complains of diarrhea. Which treatment strategy would the nurse conclude is the cause of the diarrhea? A. Loperamide B. Esomeprazole C. Bed rest D. Diet alteration

A. Loperamide Esomeprazole, a proton-pump inhibitor, may cause diarrhea. Loperamide, an antidiarrheal, may cause constipation, not diarrhea. Immobility causes constipation, not diarrhea. Although diet can affect elimination, no data are presented to support this conclusion.

The nurse provides care for a client with a long history of alcohol abuse. Which medication would the nurse anticipate will be prescribed for the client to prevent symptoms of withdrawal? A. Lorazepam B. Phenobarbital C. Chlorpromazine D. Disulfiram

A. Lorazepam Lorazepam is most effective in preventing the signs and symptoms associated with withdrawal from alcohol. It depresses the central nervous system by potentiating gamma-aminobutyric acid, an inhibitory neurotransmitter. Phenobarbital is used to prevent withdrawal symptoms associated with barbiturate use. Chlorpromazine, an antipsychotic medication, is not used for alcohol withdrawal. Disulfiram does not prevent symptoms; it is aversion therapy that causes symptoms when alcohol is ingested.

A client takes furosemide and digoxin for heart failure. Why would the nurse advise the client to drink a glass of orange juice every day? A. Maintaining potassium levels B. Preventing increased sodium levels C. Limiting the medications' synergistic effects D. Correcting the associated dehydration

A. Maintaining potassium levels Orange juice is an excellent source of potassium. Furosemide promotes excretion of potassium, which can result in hypokalemia. Digoxin toxicity can occur in the presence of hypokalemia. Neither medication increases sodium levels. Digoxin does not potentiate the action of furosemide; therefore the client should not experience dehydration. Orange juice will not prevent an interaction between digoxin and furosemide.

The nurse administers cimetidine to a 75-year-old client diagnosed with a gastric ulcer. The nurse should monitor the client for which adverse reaction? A. Mental status change B. Increased liver enzymes C. Constipation D. Hearing loss

A. Mental status change Cimetidine is a histamine H2-receptor antagonist used to treat gastric ulcers. It has been found to cause confusion in susceptible clients, such as the elderly and debilitated clients. Clients over age 50 or who are severely ill may become temporarily confused while taking H2 blockers, especially cimetidine.

Which substance history of a severe allergic reaction results in avoidance of the cephalosporins such as cefazolin, cefditoren, cefotetan, and ceftriaxone? Select all that apply. One, some, or all responses may be correct. A. Milk B. Aspirin C. Calcium D. Penicillin E. Strawberries

A. Milk B. Aspirin C. Calcium D. Penicillin Use of cephalosporins like cefazolin should be avoided in the client with a history of severe allergic reaction to penicillin because of the potential of cross-sensitivity. The cephalosporin cefditoren should not be administered to the client with a milk allergy because it contains the milk protein caseinate. Bleeding can be magnified with the use of aspirin and the use of the cephalosporins cefotetan or ceftriaxone. The cephalosporin ceftriaxone and calcium should not be administered together because they cause the formation of precipitates.

The nurse is providing preoperative teaching for a client preparing for a thyroidectomy about the medication saturated solution of potassium iodide drops. Which information is important for the nurse to include? A. Mix the medication with juice or milk. B. Take the medication on an empty stomach. C. Store the medication in the refrigerator. D. The medication will enlarge the thyroid gland.

A. Mix the medication with juice or milk. For clients with Grave's disease, saturated solution of potassium iodide (SSKI) is given to control the hyperthyroidism, but also to reduce the amount of blood loss during surgery. The medication is in drop form, and the typical dosing is 1 to 2 drops three times a day mixed in juice or milk for 10 days preoperatively. It is not necessary to take it on an empty stomach. Storing it in the refrigerator may cause crystallization of the solution. The medication will not enlarge the thyroid gland.

Which nursing intervention is important when caring for clients receiving intravenous(IV) digoxin? Select all that apply. One, some, or all responses may be correct. A. Monitor the heart rate closely. B. Check the blood levels of digoxin. C. Administer the dose over 1 minute. D. Monitor the serum potassium level. E. Give the medication with other infusing medications.

A. Monitor the heart rate closely. B. Check the blood levels of digoxin. D. Monitor the serum potassium level. Bradycardia or other dysrhythmias may occur; therefore the heart rate and rhythm should be monitored. Electrocardiogram (ECG) monitoring should be continuous. The digoxin level is checked before administration to avoid toxicity. A low serum potassium level when digoxin is administered can contribute to toxicity. Digoxin should be given over a 5-minute period through a Y-tube or three-way stopcock. There are many syringe, Y-site, and additive incompatibilities; the manufacturer recommends that digoxin not be administered with other medications.

Which independent nursing action would be included in the plan of care for a client after an episode of ketoacidosis? A. Monitoring for signs of hypoglycemia resulting from treatment B. Withholding glucose in any form until the situation is corrected C. Giving fruit juices, broth, and milk as soon as the client is able to take fluids orally D. Regulating insulin dosage according to the client's urinary ketone levels

A. Monitoring for signs of hypoglycemia resulting from treatment During treatment for acidosis, hypoglycemia may develop; careful observation for this complication will be made by the nurse. Withholding all glucose may cause insulin coma. Whole milk and fruit juices are high in carbohydrates, which are contraindicated immediately following ketoacidosis. The regulation of insulin depends on the prescription for coverage; the prescription usually depends on the client's blood glucose level rather than ketones in the urine.

The nurse is providing care for a client admitted to the hospital with a diagnosis of digoxin toxicity. The client reports more than usual urine output over the previous 48 hours, because of the prescribed diuretic. Which assessment finding does the nurse anticipate? A. Muscle weakness or cramping B. Blood in the urine C. Hypertension D. Tinnitus

A. Muscle weakness or cramping The client with heart failure on digoxin and a diuretic is at risk for hypokalemia. The digoxin binds to the potassium receptor of the sodium/potassium ATPase pump. The increased urine output makes hypokalemia likely and thus it is more likely for digoxin toxicity to occur. Symptoms of hypokalemia include muscle weakness and cramping. The digoxin toxicity will not cause blood in the urine, or tinnitus or hypertension.

The nurse teaches the client about effects of carbamazepine that would be reported to the primary health care provider. Which effects would the nurse include? Select all that apply. One, some, or all responses may be correct. A. Nausea B. Dizziness C. Unusual bleeding or bruising D. Sensitivity to bright light or sun E. Breast enlargement

A. Nausea Nausea may be a side effect or it may signal toxicity. The client should be evaluated by the primary health care provider. Carbamazepine can cause severe bone marrow depression; the client should have weekly complete blood counts for the first 4 weeks of therapy and every 3 to 6 months thereafter. Dizziness is a common side effect of carbamazepine that does not require primary health care provider notification. The client should be cautioned not to engage in hazardous activities such as driving a car. Sensitivity to bright light or sun is not a side effect of carbamazepine. Breast enlargement is not associated with carbamazepine.

Digoxin is prescribed for a client with heart failure. The nurse will assess for which signs and symptoms that indicate digoxin toxicity? Select all that apply. One, some, or all responses may be correct. A. Nausea B. Yellow vision C. Irregular pulse D. Increased urine output E. Heart rate of 64 beats/minute

A. Nausea B. Yellow vision C. Irregular pulse Signs and symptoms of digoxin toxicity include bradycardia, headache, dizziness, confusion, nausea, and visual disturbances (blurred vision or yellow vision). In addition, electrocardiogram (ECG) findings may include heart block, atrial tachycardia with block, or ventricular dysrhythmias, all causing an irregular pulse. Increased urine output is an expected effect of improved cardiac output; a pulse rate of 64 beats/minute is an acceptable rate when a client is receiving digoxin.

Which adverse response would a nurse assesses for when carbidopa-levodopa is prescribed for a client with Parkinson disease? Select all that apply. One, some, or all responses may be correct. A. Nausea B. Lethargy C. Bradycardia D. Polycythemia E. Emotional changes

A. Nausea E. Emotional changes Nausea and vomiting may occur; this reflects a central emetic reaction to levodopa. Changes in affect, mood, and behavior are related to the toxic effects of carbidopa-levodopa. Insomnia, tremors, and agitation are side effects that may occur, not lethargy. Tachycardia and palpitations, not bradycardia, occur. Anemia and leukopenia, not polycythemia, are adverse reactions.

Which body function maintained by thiamine (vitamin B 1) and niacin (vitamin B 3) will the nurse monitor when prescribed for a client with alcoholism? A. Neuronal activity B. Bowel elimination C. Efficient circulation D. Prothrombin development

A. Neuronal activity Thiamine and niacin help convert glucose for energy and influence nerve activity. These vitamins do not affect elimination. These vitamins are not related to circulatory activity. Vitamin K, not thiamine and niacin, is essential for the manufacturing of prothrombin.

A client is receiving ABVD (doxorubicin, bleomycin, vinblastine, and dacarbazine) therapy for Hodgkin's disease. When the client reports burning and tingling of the feet, which medication would the nurse determine is the likely cause? A. Neurotoxicity caused by vinblastine B. Hypersensitivity caused by dacarbazine C. Endocrine alterations caused by doxorubicin D. Peripheral vasoconstriction caused by bleomycin

A. Neurotoxicity caused by vinblastine Neurotoxicity manifested by peripheral neuropathy (burning and tingling of the hands and/or feet) is a common and expected side effect of vinblastine. Dacarbazine and doxorubicin also cause peripheral neuropathy; however, the peripheral neuropathy does not occur as part of a hypersensitivity reaction, as was mentioned for dacarbazine, or due to endocrine function alterations, as was mentioned for doxorubicin. Bleomycin is not known to cause neurotoxicity and peripheral neuropathy.

Which medication is classified as an H 2 receptor antagonist? Select all that apply. One, some, or all responses may be correct. A. Nizatidine B. Ranitidine C. Famotidine D. Lansoprazole E. Metoclopramide

A. Nizatidine B. Ranitidine C. Famotidine Nizatidine, ranitidine, and famotidine are H 2 receptor antagonists that reduces gastric acid secretion and provide symptomatic improvement in GERD. Lansoprazole is a proton pump inhibitor that inhibits gastric secretion up to 90% with one dose daily and provides for symptomatic improvement in GERD. Metoclopramide is a prokinetic agent that increases the rate of gastric emptying; it has multiple side effects and is not appropriate for long-term treatment of GERD.

The nurse is teaching a nursing student about tricyclic antidepressant medications. Which statement made by the student indicates the need for further teaching? Select all that apply. One, some, or all responses may be correct. A. Nortriptyline is contraindicated in older adult clients. B. Desipramine is preferred for use in older adult clients. C. Imipramine is used as an adjunct in the treatment of childhood enuresis. D. Tricyclic antidepressant medications are prescribed for clients with seizure disorders. E. Tricyclic antidepressant medications are contraindicated in clients with a history of seizures.

A. Nortriptyline is contraindicated in older adult clients. D. Tricyclic antidepressant medications are prescribed for clients with seizure disorders. Nortriptyline is a preferred tricyclic antidepressant that can be administered safely in older adult clients. Antiepileptic medications are prescribed to clients with seizures. Desipramine and nortriptyline are preferred tricyclic antidepressants for use in older adult clients. Childhood enuresis necessitates the administration of imipramine. Clients with epilepsy should not be prescribed tricyclic antidepressants to avoid the risk of medical complications.

Which instruction would the nurse give an unlicensed assistive personnel (UAP) to perform while caring for a client prescribed captopril? Select all that apply. One, some, or all responses may be correct. A. Obtain blood pressure. B. Measure intake and output. C. Weigh the client every morning. D. Notify the nurse if the client has a dry cough. E. Assist the client to change positions slowly.

A. Obtain blood pressure. B. Measure intake and output. C. Weigh the client every morning. D. Notify the nurse if the client has a dry cough. E. Assist the client to change positions slowly. ACE inhibitors such as captopril are prescribed for the management of hypertension, heart failure, and diabetic nephropathy. The nurse would ask the UAP caring for a client taking captopril to perform several tasks. The UAP would obtain the client's blood pressure. The UAP would also measure the client's intake and output as well as obtain a daily weight in the morning. This data would help the nurse determine the client's fluid volume status and is an important component of heart failure management. The UAP would be aware that a dry cough is a common side effect of ACE inhibitors. Because of the blood pressure-lowering effects of this medication, the nurse would instruct the UAP to assist the client to make sure the client changes positions slowly.

Which advice about the administration of vitamins A, D, E, and K will the nurse teach the parents of a toddler with newly diagnosed cystic fibrosis? A. Offer them in a water-miscible form. B. Give them during meals and snack times. C. The dosage is based on the child's height and weight. D. Present them to the child with fruit juice rather than milk.

A. Offer them in a water-miscible form. Because children with cystic fibrosis do not absorb fat-soluble vitamins effectively, they should be given in a water-miscible form. These vitamins may be given with other vitamins once a day; pancreatic enzymes are administered with meals and snacks. The nurse does not have to base the dosage of these vitamins on the child's height and weight. There is no reason to select juice over milk when administering these vitamins.

Intravenous (IV) potassium is prescribed for a client with a diagnosis of hypokalemia. Which statement about administration of IV potassium is accurate? A. Oliguria is an indication for withholding IV potassium. B. Rapid infusion of potassium prevents burning at the IV site. C. Clients with severe deficits should be given IV push potassium. D. Average IV dosage of potassium should not exceed 60 mEq in 1 hour.

A. Oliguria is an indication for withholding IV potassium. Potassium chloride should not be given unless renal flow is adequate; otherwise, the potassium chloride will accumulate in the body, causing hyperkalemia. Rapid infusion may cause severe pain at the infusion site and precipitate cardiac arrest. Potassium chloride must be well diluted or it will precipitate cardiac arrest. A dose of 60 mEq per hour of potassium chloride is too high.

Which medication would the nurse instruct a client to avoid while taking alprazolam? Select all that apply. One, some, or all responses may be correct. A. Opioids B. Alcohol C. Barbiturates D. Antidepressants E. First-generation antipsychotics

A. Opioids B. Alcohol C. Barbiturates Respiratory depression can occur if a client combines benzodiazepines with opioids, alcohol, or barbiturates. Antidepressants and first-generation antipsychotics are safe to take with benzodiazepines.

A client is concerned about taking hormones for birth control. Which contraceptives, explained to the client by the nurse, have a hormonal component? Select all that apply. One, some, or all responses may be correct. A. Oral contraceptives B. Diaphragm C. Cervical cap D. Female condom E. Foam spermicide F. Transdermal agents

A. Oral contraceptives F. Transdermal agents Oral contraceptives have a hormonal component. Transdermal agents have a hormonal component. The diaphragm, cervical cap, and female condom act as barriers. Foam spermicides kill the sperm; there is no hormonal effect.

A client being discharged home is prescribed an antibiotic with a dosage three times higher than it was administered when the client was in the hospital (IV route). Which route of administration should the nurse anticipate will be prescribed for the greatest first-pass effect? A. Oral. B. Sublingual. C. Intravenous. D. Subcutaneous.

A. Oral. The first-pass effect is a pharmacokinetic phenomenon that is related to the drug's metabolism in the liver. After oral medications are absorbed from the gastrointestinal tract, the drug is carried directly to the liver via the hepatic portal circulation, where hepatic inactivation occurs and reduces the bioavailability (strength/concentration) of the drug.

Which principle explains how loop diuretics promote diuresis? A. Osmosis B. Filtration C. Diffusion D. Active transport

A. Osmosis Loop diuretics inhibit the reabsorption of sodium and water in the ascending loop of Henle. The increased sodium load in the distal tubule causes the passive transfer of water from the glomerular filtrate to urine through the process of osmosis. Filtration refers to solutes; solutes are not being passed into the urine. Diffusion is not specific to fluid; osmosis is. Active transport requires energy; water is passively moved from tubule cells to the urine.

Which statement accurately describes nortriptyline? Select all that apply. One, some, or all responses may be correct. A. Overdosage is often lethal. B. Constipation and urinary retention may occur. C. It is a selective serotonin reuptake inhibitor (SSRI). D. Weight gain is a common side effect. E. It increases effectiveness of monoamine oxidase inhibitors (MAOIs).

A. Overdosage is often lethal. B. Constipation and urinary retention may occur. D. Weight gain is a common side effect. Nortriptyline is notoriously lethal; between 70% and 80% of people who die from overdose do so before reaching the hospital. Nortriptyline can cause constipation and urinary retention because it causes blockage of cholinergic receptors, and it tends to increase appetite and cause weight gain. Nortriptyline is not an SSRI; it is a tricyclic antidepressant and affects the balance of neurotransmitters in the brain. The interaction between nortriptyline and MAOIs is severe and possibly fatal.

Which effect would the nurse monitor to evaluate the effectiveness of carbamazepine in the management of a client's trigeminal neuralgia? A. Pain intensity B. Gait C. Range of motion D. Seizure activity

A. Pain intensity Carbamazepine is administered to control pain by reducing transmission of nerve impulses in clients with trigeminal neuralgia. This neurological condition isn't manifested by problems associated with gait or range of motion. Carbamazepine is not administered to clients with trigeminal neuralgia (tic douloureux) for its anticonvulsant properties because seizures are not present with this disorder.

Sublingual nitroglycerin has been prescribed for a client with unstable angina. Which client response indicates that nitroglycerin is effective? A. Pain subsides as a result of arteriole and venous dilation. B. Pulse rate increases because the cardiac output has been stimulated. C. Sublingual area tingles because sensory nerves are being triggered. D. Capacity for activity improves as a response to increased collateral circulation.

A. Pain subsides as a result of arteriole and venous dilation. Nitroglycerin causes vasodilation, increasing the flow of blood and oxygen to the myocardium and reducing anginal pain. An increased pulse rate does not indicate effectiveness; it is a side effect of nitroglycerin. The tingling indicates that the medication is fresh; relief of pain is the only indicator of effectiveness. Nitroglycerin does not promote the formation of new blood vessels.

A client who has type 1 diabetes and chronic bronchitis is prescribed atenolol for the management of angina pectoris. Which clinical manifestation will alert the nurse to the fact that the client may be developing a life-threatening response to the medication? A. Paroxysmal nocturnal dyspnea B. Supraventricular tachycardia C. Malignant hypertension D. Hyperglycemia

A. Paroxysmal nocturnal dyspnea Atenolol is associated with the adverse reactions of bradycardia, heart failure, and pulmonary edema; these are the most serious responses to atenolol and are often manifested by episodes of paroxysmal nocturnal dyspnea and orthopnea. A decreased, not increased, pulse rate is associated with atenolol so supraventricular tachycardia is not a response. Atenolol decreases, not increases, blood pressure so malignant hypertension is not a response. It also will not cause an increase in blood glucose. It may increase the hypoglycemic response to insulin, causing hypoglycemia. In addition, the medication may mask the clinical manifestations of hypoglycemia.

Which therapy is indicated for a client admitted to the hospital after general paresis develops as a complication of syphilis? A. Penicillin therapy B. Major tranquilizers C. Behavior modification D. Electroconvulsive therapy

A. Penicillin therapy Massive doses of penicillin may limit central nervous system damage if treatment is started before neural deterioration from syphilis occurs. Tranquilizers are used to modify behavior, not to treat general paresis. Behavior, not paresis, is treated with behavior modification. Electroconvulsive therapy is used to treat certain psychiatric disorders.

Which toxic effect would a nurse monitor for in a client who is prescribed vincristine? A. Peripheral paresthesia B. Anginal-type chest pain C. Ophthalmic papilledema D. Bilateral crackles in the lung

A. Peripheral paresthesia Peripheral paresthesia is an indication of toxicity from a plant alkaloid such as vincristine. Anginal-type chest pain, ophthalmic papilledema, and bilateral crackles in the lung are not side effects of this medication.

A client who is receiving phenytoin asks why folic acid was prescribed. Which explanation would the nurse provide? A. Phenytoin inhibits absorption of folate from foods. B. Folic acid potentiates the action of phenytoin. C. Absorption of iron from foods is improved. D. Neuropathy caused by phenytoin is prevented.

A. Phenytoin inhibits absorption of folate from foods. Phenytoin inhibits folic acid absorption and potentiates the effects of folic acid antagonists. Folic acid diminishes, not potentiates, the effects of phenytoin. Absorption of iron from foods and prevention of neuropathy caused by phenytoin are not effects of folic acid.

The nurse is educating a client prescribed metronidazole. Which of the following findings should the nurse include in the education as reportable to the healthcare provider? A. Pinpoint red spots on the skin B. Nausea after beginning the medication C. Metallic taste D. Occasional diarrhea

A. Pinpoint red spots on the skin Rationale: The most common gastrointestinal effects of metronidazole are nausea, vomiting, diarrhea, and metallic taste. Drug-induced immune thrombocytopenia (DITP) is a rare, but serious, adverse effect where medications cause the body to produce antibodies to platelets. The medication must be stopped immediately because DITP can be life-threatening. Heparin-induced thrombocytopenia is one example. Metronidazole is associated with DITP. Petechiae are pinpoint, round spots that appear on the skin as a result of bleeding. The bleeding causes the petechiae to appear red, brown, or purple.

A nurse is reviewing laboratory data for a client taking pramlintide for diabetes management. Which clinical finding indicates medication effectiveness? A. Postprandial glucose of 160 mg/dL B. Hemoglobin A1c of 8.5% C. Fasting blood glucose of 135 mg/dL D. Preprandial glucose of 150 mg/dL

A. Postprandial glucose of 160 mg/dL Rationale: Pramlintide is an antidiabetic medication used in the treatment of diabetes mellitus in conjunction with other hypoglycemic drugs. The therapeutic goal of pramlintide is to achieve postprandial glucose levels below 180 mg/dL. A preprandial blood glucose of 150 mg/dL, a hemoglobin A1c of 8.5%, and a fasting blood glucose of 135 mg/dL are all indicative of poor disease management. The goal is a preprandial glucose level of less than 130 mg/dL, a hemoglobin A1c level of less than 7%, and a fasting blood glucose of less than 100 mg/dL.

A client is admitted to the hospital for a new onset of supraventricular tachycardia (SVT) and is prescribed digoxin. For which laboratory finding should the nurse notify the healthcare provider immediately? A. Potassium level of 3.1 mEq/L. B. Sodium level of 132 mEq/L. C. Calcium level of 8.6 mg/dL. D. Magnesium level of 1.2 mEq/L.

A. Potassium level of 3.1 mEq/L. Hypokalemia affects myocardial contractility and places this client at greatest risk for dysrhythmias that may be unresponsive to drug therapy. Although an imbalance of serum sodium, calcium, and magnesium can effect cardiac rhythm, the greatest risk for a client receiving digoxin is low potassium.

An 80-year-old client who is taking digoxin reports nausea, vomiting, abdominal cramps and halo vision. Which laboratory result should the nurse evaluate first? A. Potassium levels B. Blood pH C. Magnesium levels D. Blood urea nitrogen

A. Potassium levels Nausea, vomiting, abdominal cramps and halo vision are classic signs of digitalis toxicity. The most common cause of digitalis toxicity is a low potassium level. Clients are to be taught that it is important to have adequate potassium intake, especially if taking loop or thiazide diuretics that enhance the loss of potassium.

Which dietary choices will the nurse instruct the client taking spironolactone to avoid increasing? Select all that apply. One, some, or all responses may be correct. A. Potatoes B. Red meat C. Cantaloupe D. Wheat bread

A. Potatoes C. Cantaloupe Spironolactone is potassium-sparing, and beverages and foods containing potassium such as potatoes, cantaloupe, bananas, avocados, oranges, dates, apricots, and raisins should not be increased beyond the client's ordinary consumption to prevent hyperkalemia. Red meat may need to be limited for other reasons not related to spironolactone. Whole grains are associated with prevention of constipation and should not be avoided. Dairy products are rich in sodium and calcium; spironolactone may cause hyponatremia.

Which condition contraindicates the use of ginseng herbal therapy? A. Pregnancy B. Schizophrenia C. Bipolar depression D. Alzheimer disease

A. Pregnancy Pregnancy is contraindicated for ginseng herbal therapy. Schizophrenia, bipolar depression, and Alzheimer disease are contraindicated for St. John's Wort herbal therapy.

Which instruction would the nurse provide to parents of a school-age child who has been on long-term phenytoin therapy to prevent side effects? A. Provide good oral hygiene. B. Administer the medication between meals. C. Watch for a reddish-brown discoloration of urine. D. Supplement the diet with high-calorie foods. Rationale

A. Provide good oral hygiene. These procedures reduce the risk for gingival hyperplasia, a side effect of phenytoin. This medication is strongly alkaline and should be administered with meals to help prevent gastric irritation. Discoloration of the urine may occur during medication excretion; it does not cause physiological problems. Avoiding overeating and overhydration may result in better seizure control.

Tetanus immune globulin is prescribed after a client steps on a rusty nail. Which action would the nurse associate with this medication? A. Provides antibodies B. Stimulates plasma cells C. Produces active immunity D. Facilitates long-lasting immunity

A. Provides antibodies Tetanus immune globulin provides antibodies, which confer immediate passive immunity. It does not stimulate production of plasma cells, the precursors of antibodies. Passive, not active, immunity occurs. Passive immunity, by definition, is not long lasting.

During an assessment the client mentions taking cefotetan and drinking a few cocktails at dinner. Which symptoms might be explained by this medication-alcohol interaction? Select all that apply. One, some, or all responses may be correct. A. Pruritus B. Diaphoresis C. Hypotension D. Hypertension E. Stomach cramps F. Chest pain

A. Pruritus B. Diaphoresis C. Hypotension E. Stomach cramps Individuals taking the antibiotic cefotetan need to avoid alcohol. Drinking alcohol while on this medication causes acute alcohol intolerance, resulting in pruritus, diaphoresis, hypotension, and stomach cramps. Hypertension and chest pain or pressure are not typical symptoms of acute alcohol intolerance and cannot be explained by this medication-alcohol interaction.

A client is prescribed ampicillin sodium (Omnipen) for a sinus infection. The nurse should instruct the client to notify the healthcare provider immediately if which symptom occurs? A. Rash. B. Nausea. C. Headache. D. Dizziness.

A. Rash. Rash is the most common adverse effect of all penicillins, indicating an allergy to the medication that could result in anaphylactic shock, a medical emergency.

Which concern will the nurse keep in mind when a client has been taking a benzodiazepine? A. Rebound insomnia may occur if the medication is discontinued abruptly. B. Lifelong treatment is often required. C. Higher doses are needed to accommodate physiological changes during pregnancy. D. These medications have both analgesic and antidepressant properties.

A. Rebound insomnia may occur if the medication is discontinued abruptly. Benzodiazepine often leads to tolerance and withdrawal; therefore it can cause rebound insomnia when discontinued abruptly. Benzodiazepine is not indicated for lifelong treatment; addiction is a concern. Benzodiazepine should be discontinued if the client becomes pregnant. Benzodiazepines not have analgesic, antidepressant, or antipsychotic properties.

The nurse is caring for a client who was prescribed alprazolam. When educating the client about the new medication, which intended effect should the nurse include? A. Reduce anxiety and provide a calming effect B. Increase coordination and the ability to concentrate C. Alleviate signs and symptoms of spasticity D. Reduce symptoms of depression

A. Reduce anxiety and provide a calming effect Alprazolam is a benzodiazepine which is as an anxiolytic. The medication will not increase coordination and the ability to concentrate or alleviate symptoms associated with nerve damage, such as spasticity. Alprazolam will not reduce symptoms of depression.

The nurse is caring for a client diagnosed with diabetic ketoacidosis who is receiving 50 mEq of sodium bicarbonate in 1 L of dextrose 5% in water via a central venous access device. The client has three new prescriptions for continuously infused medications. Which action is appropriate? A. Refer to an IV compatibility chart B. Request that an additional IV access be inserted C. Use a Y-site connector to infuse two medications in the same port D. Insert a peripheral intravenous access

A. Refer to an IV compatibility chart Rationale: Sodium bicarbonate is incompatible with many other drugs and solutions. The nurse should consult a drug compatibility reference for more information on which drugs can be administered via connection at the most distal IV tubing port. Y tubing should not be added to an IV until compatibility is determined. Y tubing does not prevent the mixing of infusions. Adding an additional access may be unnecessary if compatibility is determined and may pose an unnecessary infection risk to the client. A central line is the preferred access for drugs that have a pH less than 5 and greater than 9. Certain drugs are venous irritants regardless of pH or concentration; therefore, a PIV would be inappropriate.

Diazepam is administered to the client with status epilepticus. In addition to decreasing central neuronal activity, which effect would the nurse anticipate? A. Relaxing of peripheral muscles B. Decreased heart rate C. Dilation of airways D. Hypertension

A. Relaxing of peripheral muscles Diazepam is a benzodiazepine indicated for treatment of anxiety, muscle spasms, and seizures. Peripheral muscles may relax as a result of the antispasmodic effects. Diazepam does not slow the heart rate. Diazepam does not dilate the bronchial airways and may cause bronchoconstriction, though this is uncommon. Diazepam may cause hypotension, not hypertension.

A nurse is reviewing a client's medical history. The client has been newly diagnosed with hypertension and has been prescribed oral losartan as treatment. The nurse will clarify the use of losartan if which comorbidity is noted in the client's medical record? A. Renal stenosis B. Hyperlipidemia C. Atrial fibrillation D. Diabetes

A. Renal stenosis Rationale: Losartan is an angiotensin II receptor blocker used in the treatment of hypertension. Losartan is contraindicated in clients with renal stenosis due to the risk of kidney injury. Hyperlipidemia, atrial fibrillation, and diabetes are not known to be contraindicated in the use of losartan.

A newly admitted client reports taking phenytoin for several months. Which assessment should the nurse include in the admission report? Select all that apply. A. Report of unsteady gait, rash and diplopia B. Report of any seizure activity C. Serum phenytoin levels D. Report of anorexia, numbness and tingling of the extremities

A. Report of unsteady gait, rash and diplopia B. Report of any seizure activity C. Serum phenytoin levels Serious adverse outcomes of antiseizure medications such as phenytoin are unsteady gait, slurred speech, extreme fatigue, blurred vision or feelings of suicide. Clients who are prescribed phenytoin should have their levels monitored on a routine basis. The nurse should include any seizure activity as this may demonstrate lack of a therapeutic level. Increased hunger (not anorexia), increased thirst or increased urination are additional serious side effects.

Which instruction will the nurse include in a teaching plan for a client taking a calcium channel blocker such as nifedipine? Select all that apply. One, some, or all responses may be correct. A. Report peripheral edema. B. Expect temporary hair loss. C. Avoid drinking grapefruit juice. D. Change to a standing position slowly. E. Reduce calcium intake.

A. Report peripheral edema. C. Avoid drinking grapefruit juice. D. Change to a standing position slowly. Changing positions slowly helps reduce orthostatic hypotension. Peripheral edema may occur as a result of heart failure and must be reported. Grapefruit juice affects the metabolism of calcium channel blockers and should be avoided. Reducing calcium intake is unnecessary because calcium levels are not affected. Hair loss does not occur.

Which information would the nurse provide to a client diagnosed with chlamydia and prescribed doxycycline? Select all that apply. One, some, or all responses may be correct. A. Report worsening symptoms. B. Refrain from sexual relations. C. Use barrier protection devices. D. Contact partners to be tested. E. Take the entire course of antibiotics.

A. Report worsening symptoms. B. Refrain from sexual relations. C. Use barrier protection devices. D. Contact partners to be tested. E. Take the entire course of antibiotics. The nurse would instruct clients taking doxycycline for an STI to report worsening symptoms to the health care provider as it could indicate antibiotic resistance. Clients would also be instructed to refrain from sexual relations while the infection is being treated. If they do choose to have sexual relations, they would be instructed on the importance of using barrier protection. The nurse would also instruct clients to contact their sexual partners and inform them of the need to be tested and treated for the STI. Clients should take the entire prescribed course of antibiotics to prevent recurrence of the infection.

A client is given a loading dose of digoxin and placed on a maintenance dose of digoxin 0.25 mg by mouth daily. Which responses would the nurse expect the client to exhibit when a therapeutic effect of digoxin is achieved? A. Resolution of heart failure B. Decreased anginal episodes C. Conversion of atrial fibrillation D. Decreased blood pressure

A. Resolution of heart failure Digoxin improves cardiac output to improve heart failure. Digoxin is not an antianginal medication; if it decreases angina as a result of controlling heart failure, it is a secondary effect. Digoxin may be given to control a rapid ventricular response to atrial fibrillation, but it does not convert the rhythm. Digoxin has a negligible effect on blood pressure; therefore it is not an antihypertensive medication.

Which vitamin deficiency may occur if cholestyramine, an anion exchange resin, to treat a client's persistent diarrhea is needed long-term? A. Retinol (Vitamin A) B. Riboflavin (Vitamin B 2) C. Thiamine (Vitamin B 12) D. Pyridoxine (Vitamin B 6)

A. Retinol (Vitamin A) Cholestyramine is a fat-binding agent; it binds with and interferes with all the fat-soluble vitamins (A, D, E, and K). Riboflavin is not a fat-soluble vitamin and is unaffected. Thiamine is not a fat-soluble vitamin and is unaffected. Vitamin B 6 is not a fat-soluble vitamin and is unaffected.

) A client who takes daily megadoses of vitamins is hospitalized with joint pain, loss of hair, yellow pigmentation of the skin, and an enlarged liver due to vitamin toxicity. Which type of toxicity would the nurse suspect? A. Retinol (vitamin A) B. Thiamine (vitamin B 1) C. Pyridoxine (vitamin B 6) D. Ascorbic acid (vitamin C)

A. Retinol (vitamin A) Retinol is lipid soluble and eliminated by the liver. Joint pain, hair loss, jaundice, anemia, irritability, pruritus, and enlarged liver and spleen are signs of vitamin A toxicity. Thiamine, pyridoxine, and ascorbic acid are water soluble, so they are typically excreted in the urine before toxic blood levels can be achieved. However, excess thiamine may elicit an allergic reaction in some individuals, excess vitamin C (ascorbic acid) may cause diarrhea or renal calculi, and ultrahigh doses (about 800 times the normal dose) of pyridoxine (vitamin B 6) can promote neuropathy. Remember that lipid-soluble vitamins normally take longer to eliminate and accumulate faster than water-soluble vitamins.

The nurse is collecting the health history for a client who reports a sudden onset of generalized weakness and fatigue. The nurse notes the client has a new prescription for spironolactone. Which action should the nurse take first? A. Review the drug formulary for side effects B. Request the health care provider to stop the medication C. Notify the pharmacist of the findings D. Document the findings

A. Review the drug formulary for side effects Rationale: During medication administration, it is important for the nurse to assess knowledge of drugs, including adverse effects and physiologic factors that affect drug action. Information about specific drugs is available in pharmacology texts and drug reference books. Calling the health care provider may be an option after reviewing the drug formulary. The nurse should notify the pharmacist if the medication is the cause of the symptoms. The nurse will document the findings, but the priority is to review the formulary.

The nurse is assessing a client with hypertension who reports experiencing dizziness after taking prescribed diltiazem. It is most important that the nurse assesses for which client characteristic? A. Schedule for taking medication B. Appearance of feet and ankles C. Activity and rest patterns D. Daily intake of potassium

A. Schedule for taking medication A critical focus is whether the client has complied with the prescribed medication schedule and dose. Although diltiazem (Cardizem, Cartia, Dilacor, Diltia, Taztia, Tiazac) can be taken either in the morning or evening, taking the medication in the evening might help with this common side effect.

The nurse is providing discharge education to a client who is prescribed alprazolam for a panic disorder. What concept should the nurse emphasize concerning the drug action? A. Short-term relief can be expected B. The medication acts as a stimulant C. The medication works by suppressing dopamine D. If you miss a dose, double the next scheduled dose

A. Short-term relief can be expected Alprazolam is a short-acting benzodiazepine, which works quickly to control panic symptoms by enhancing the effects of the neurotransmitter Gamma-amino butyric acid (GABA). This produces a calming effect. The drug does not suppress dopamine like dopamine antagonists and some antipsychotic medications. Alprazolam will not be increased as tolerated, the lowest dose that controls the symptoms will be maintained.

Which finding in a menopausal client's health history would prevent the health care provider from prescribing hormone replacement therapy? Select all that apply. One, some, or all responses may be correct. A. Smoking B. Cirrhosis C. Cholecystitis D. Breast cancer E. Deep vein thrombosis

A. Smoking B. Cirrhosis C. Cholecystitis D. Breast cancer E. Deep vein thrombosis Use of estrogens can have major side effects, especially if the client smokes. The nurse would provide information to the client about smoking cessation. Clients with cirrhosis have a decreased ability to break down medications, especially estrogen. Cholecystitis can worsen in clients taking estrogen. Clients at risk for breast and endometrial cancer should not take estrogen because it can further increase the risk. Estrogens can lead to deep vein thrombosis.

Which mechanism of action explains how glyburide decreases serum glucose levels? A. Stimulates the pancreas to produce insulin B. Accelerates the liver's release of stored glycogen C. Increases glucose transport across the cell membrane D. Decreases absorption of glucose from the gastrointestinal system

A. Stimulates the pancreas to produce insulin Glyburide, an antidiabetic sulfonylurea, stimulates insulin production by the beta cells of the pancreas. Accelerating the liver's release of stored glycogen occurs when serum glucose drops below normal levels. Increasing glucose transport across the cell membrane occurs in the presence of insulin and potassium. Antidiabetic medications of the biguanide chemical class improve sensitivity of peripheral tissue to insulin, which ultimately increases glucose transport into cells. Beta cells must have some function to enable this medication to be effective.

Which action would the nurse take when a client develops a maculopapular rash on the upper extremities and audible wheezing during the administration of intravenous vancomycin? A. Stop the infusion. B. Decrease the flow rate. C. Reassess in 15 minutes. D. Notify the health care provider.

A. Stop the infusion. The first action the nurse would take is to stop the infusion immediately. The client may be experiencing an allergic reaction. Decreasing the flow rate is not an appropriate action. Infusions must be stopped if an allergic reaction is suspected. This could be an emergent situation, so reassessing in 15 minutes is not the most appropriate action. The nurse would stop the medication infusion and then notify the health care provider.

A client with a history of tuberculosis reports difficulty hearing. Which medication would the nurse consider is causing this response? A. Streptomycin B. Pyrazinamide C. Isoniazid D. Ethambutol

A. Streptomycin Ototoxicity is an adverse effect of aminoglycosides such as streptomycin. Ototoxicity is not an adverse effect of pyrazinamide, isoniazid, or ethambutol.

The nurse is teaching a 68-year-old, postmenopausal, female client has about adverse effects of newly prescribed tamoxifen for breast cancer with bone metastases. Which information should the nurse include in the teaching? A. Stroke-like symptoms B. Insomnia C. Seizures D. Symptoms of hypocalcemia

A. Stroke-like symptoms Tamoxifen is an antineoplastic drug, commonly prescribed for clients with breast cancer or for clients who are at high risk for developing breast cancer. The most common adverse drug effects (ADEs) are hot flashes, fluid retention, vaginal discharge, nausea, vomiting and menstrual irregularities. In women with bone metastases, tamoxifen may cause transient hypercalcemia. Because of its estrogen agonist actions, tamoxifen poses a small risk of thromboembolic events, including deep vein thrombosis, pulmonary embolism and stroke. Insomnia and seizures are not known ADEs of tamoxifen.

A client is discharged with a prescription for sustained-release nitroglycerin. Which information will the nurse provide to the client? A. Swallow the capsule whole. B. Take the medication with milk. C. Place the capsule under the tongue. D. Crush the capsule and mix with soft food.

A. Swallow the capsule whole. The sustained-release capsule should be swallowed whole on an empty stomach. The capsule should not be chewed or crushed because the 'beads' within the capsule are activated on a time-release schedule. Taking the capsule with milk isn't necessary; a full glass of water is sufficient. The sustained-release capsule is taken on an empty stomach. A sublingual tablet is held under the tongue, not swallowed; sustained-release nitroglycerin is a capsule that needs to be swallowed. A stinging feeling when the medication is under the tongue may occur with a sublingual nitroglycerin tablet; sustained-release nitroglycerin is a capsule that should be swallowed whole.

The nurse is assessing an adolescent after the administration of epinephrine. Which side effect is most important for the nurse to identify? A. Tachycardia B. Hypoglycemia C. Constricted pupils D. Decreased blood pressure

A. Tachycardia Epinephrine is a sympathetic nervous system stimulant that causes tachycardia. Hyperglycemia, not hypoglycemia, may result. The pupils will be dilated, not constricted. Epinephrine is more likely to cause hypertension than hypotension.

Which side effect would the nurse assess for after epinephrine is administered to an infant with severe bronchospasms? A. Tachycardia B. Hypotension C. Respiratory arrest D. Central nervous system depression

A. Tachycardia Epinephrine stimulates beta- and alpha-receptors; its actions include increasing heart rate and blood pressure and inducing bronchodilation. Increased blood pressure, not hypotension, is a potential side effect. Epinephrine relieves respiratory problems; it does not cause respiratory arrest. Epinephrine stimulates, not depresses, the central nervous system.

A client is being treated for osteoporosis with alendronate (Fosamax), and the nurse has completed discharge teaching regarding medication administration. Which morning schedule would indicate to the nurse that the client teaching has been effective? A. Take medication, go for a 30 minute morning walk, then eat breakfast. B. Take medication, rest in bed for 30 minutes, eat breakfast, go for morning walk. C. Take medication with breakfast, then take a 30 minute morning walk. D. Go for a 30 minute morning walk, eat breakfast, then take medication.

A. Take medication, go for a 30 minute morning walk, then eat breakfast. Alendronate (Fosamax) is best absorbed when taken thirty minutes before eating in the morning. The client should also be advised to remain in an upright position for at least thirty minutes after taking the medication to reduce the risk of esophageal reflux and irritation.

The nurse is providing medication teaching for a client who has been prescribed tetracycline. The client regularly takes calcium supplements to prevent osteoporosis. Which statement is appropriate for the nurse to make? A. Take your calcium two hours before you take the antibiotic B. You can take the calcium with the antibiotic to decrease an upset stomach C. Try taking the antibiotic and calcium with orange juice D. It is best to take the antibiotic and calcium on an empty stomach

A. Take your calcium two hours before you take the antibiotic Rationale: All tetracycline derivatives are bacteriostatics, and their concentration in serum should not fall during the therapy below the generally accepted minimum therapeutic concentration. Tetracyclines have a high affinity to form chelates with iron, aluminum, magnesium, and calcium. These complexes are poorly absorbed in the gastrointestinal tract; therefore, an interval between the ingestion of tetracyclines and cations is necessary. Taking tetracyclines with orange juice may increase irritation because the medication itself is also acidic. Additionally, orange juice may have added calcium, which would interact with the antibiotic. It is okay to take tetracyclines with food as long as it doesn't contain dairy. This may reduce stomach-related side effects.

Which action by a client taking alendronate requires correction? Select all that apply. One, some, or all responses may be correct. A. Taking medication twice a week B. Taking medication before rising C. Taking medication with breakfast D. Taking medication before bedtime E. Taking medication with apple juice

A. Taking medication twice a week B. Taking medication before rising C. Taking medication with breakfast D. Taking medication before bedtime E. Taking medication with apple juice Bisphosphonates such as alendronate have specific dosing instructions. The medication is given once a week. Clients must remain in an upright position for 30 minutes after taking the medication. Clients should refrain from eating right before they are given the medication and should take it with a full glass of water because of the risk for esophagitis. Alendronate must be administered in the morning. Clients should take the medication with water.

Parents of a child with sickle cell anemia ask about their child taking iron supplements to help treat the anemia. How will the nurse respond? A. Taking supplements will not help with this condition. B. It is advised that iron be taken with orange juice to aid in absorption. C. An over-the-counter multivitamin with iron should meet the needs of the child. D. It is advised that liquid iron supplements be given through a straw to prevent staining the teeth.

A. Taking supplements will not help with this condition. Taking iron supplements will not help. Sickle cell anemia is not caused by too little iron in the blood; it is caused by destruction of red blood cells, which increases free iron. Taking iron supplements could cause harm, because the extra iron builds up in the body and can damage organs. Although iron is better absorbed when taken with orange juice, in the case of sickle cell anemia supplements are not given. Using a straw when giving liquid iron supplements does prevent staining of the teeth; however, giving iron to this child may be detrimental. A multivitamin may be beneficial for this child; however, the addition of iron could build up in the body.

The nurse is collecting the health history of a client with heart disease who reports experiencing episodes of diarrhea. The client reports taking loperamide at home. Which of the following statements should the nurse make? A. Taking this medication may increase your risk of an abnormal heart rhythm B. It is safe to drink alcohol while using this medication C. Using this medication may cause dependence D. Stop taking this medication if your symptoms do not improve by tomorrow

A. Taking this medication may increase your risk of an abnormal heart rhythm Rationale: Loperamide decreases GI motility and is a nonprescription drug. It does not cause the central nervous system effects associated with opiate derivatives and lacks the potential for abuse. Loperamide should not be continued after 48 hours if improvement has not occurred. Loperamide has a black box warning because torsades de pointes, cardiac arrest, and death have been reported in people using higher than recommended dosages. Alcohol can increase the nervous system side effects of loperamide, such as dizziness, drowsiness, and difficulty concentrating.

A client has completed therapy and requests to discontinue treatment with benzodiazepines. Which information would the nurse provide to prevent injury? A. Taper dose over several months. B. Switch to using an antidepressant. C. Refrain from renewing prescription. D. Monitor for signs of increased anxiety.

A. Taper dose over several months. Abruptly discontinuing the use of benzodiazepines can cause withdrawal symptoms including intense anxiety and worsening of panic disorders. Therefore, the dose will need to be tapered over several months. Switching to an antidepressant does not alleviate withdrawal symptoms when discontinuing benzodiazepines. Refraining from renewing the prescription would lead to abrupt discontinuation of the medication and would be discouraged. Monitoring for signs of increased anxiety would not actively prevent injury.

A 5-year-old child is given fluoroquinolones. Which potential adverse effect unique to pediatric clients would the nurse anticipate? A. Tendon rupture B. Cartilage erosion C. Staining of developing teeth D. Central nervous system toxicity

A. Tendon rupture Fluoroquinolones may cause tendon rupture in children. Nalidixic acid can cause cartilage erosion, and tetracycline can cause staining of developing teeth. Hexachlorophene may cause central nervous system toxicity in infants.

A mother complains that her child's teeth have become yellow in color. The nurse understands that with prolonged use, which medication may be responsible? A. Tetracycline B. Promethazine C. Chloramphenicol D. Fluoroquinolones

A. Tetracycline When administered to neonates and infants, tetracycline may cause staining of developing teeth. Promethazine can cause respiratory depression in children under 2 years of age. Chloramphenicol can cause Gray baby syndrome, and fluoroquinolones may cause tendon rupture in pediatric clients.

A client has been taking alprazolam for three days. For which expected effect of the medication should the nurse evaluate the client? A. The client reports sleeping through the night. B. The client reports feeling less depressed. C. The client denies having auditory hallucinations. D. The client denies having suicide ideation.

A. The client reports sleeping through the night. Antianxiety medications or anxiolytics, such as alprazolam, a benzodiazepine, work quickly. They produce sedative effects and reduce anxiety through effects on the limbic system, a neuronal network associated with emotionality. They also promote sleep through effects on cortical areas and on the brain's sleep-wakefulness "clock." Alprazolam is not used to treat depression or hallucinations.

Which points would the nurse include when counseling a woman on hormone therapy? Select all that apply. One, some, or all responses may be correct. A. The client should use appropriate sun protection. B. The client should monitor any deviations in body weight. C. The client should take the medication on an empty stomach. D. The client should discontinue the medication if adverse effects occur. E. The client should alternate the time of day the medication is taken.

A. The client should use appropriate sun protection. B. The client should monitor any deviations in body weight. Appropriate sun protection should be used because hormones make people sensitive to sunlight. The client's body weight should be monitored during hormonal therapy because abnormal bleeding can lead to weight loss and serious complications. Hormonal medications should be taken with food to reduce gastrointestinal upset. The client should report any side effects to the primary health care provider and seek his or her advice. Hormonal therapy should never be discontinued without the knowledge of the primary health care provider. Oral medications should be administered at the same time every day to maintain the appropriate concentration of serum medication levels.

The nurse is observing a new graduate nurse preparing to administer bumetanide 4 mg orally to a client with heart failure. Which client finding requires the nurse to intervene immediately? A. The client's most recent serum potassium level is 2.9 mEq or mmol/L. B. The client has crackles in both lung bases. C. The client has 4+ pitting edema in both lower legs. D. The client's most recent blood pressure is 96/60 mmHg.

A. The client's most recent serum potassium level is 2.9 mEq or mmol/L. Bumetanide is a powerful, potassium-wasting loop diuretic. It promotes diuresis in clients suffering from heart failure (HF) and fluid retention. Prior to administration, the nurse should verify that the client's potassium level is within normal range (3.5 to 5.0 mEq or mmol/L). A serum potassium level of 2.9 mEq or mmol/L is very low. The new graduate nurse should hold the bumetanide and notify the health care provider (HCP) immediately. Bibasilar crackles and pitting edema are expected findings for a client with HF and are indications for the use of diuretics. Although loop diuretics can cause hypotension related to diuresis, a BP of 96/60 is within acceptable limits for a client with HF.

Which administration guidelines would the nurse follow when administering midazolam to an older client? A. The increments should be smaller, and the rate of injection should be slower. B. The medication should be given as a rapid intravenous push. C. It is important to monitor for spikes in blood pressure elevation during administration. D. During the procedure, the medication should be given as needed for pain management.

A. The increments should be smaller, and the rate of injection should be slower. In an older client, the peak effect may be delayed; increments should be smaller, and the rate of injection should be slower. When used for sedation/anxiolysis/amnesia for a procedure, the dosage must be individualized and titrated. Midazolam should always be titrated slowly; administer over at least 2 minutes, and allow an additional 2 or more minutes to fully evaluate the sedative effect. Titration to effect with multiple small doses is essential for safe administration. Central nervous system depression is the most serious side effect. A sudden rise in blood pressure shortly after administration has not been evidenced. Midazolam is given to induce sedation/anxiolysis/amnesia for a procedure.

An infant with congenital hypothyroidism receives levothyroxine for 3 months. Which finding would indicate to the nurse that the medication is effective? A. The infant is alert and interactive. B. The skin is cool to the touch. C. The baby's fine tremor has ceased. D. The baby's thyroid stimulating hormone level has increased.

A. The infant is alert and interactive. Infants with congenital hypothyroidism are lethargic, and may even need to be awakened and stimulated to nurse; therefore an infant who is alert and interacts appropriately for its age would demonstrate improvement. Cool skin is a clinical sign of hypothyroidism related to a slow basal metabolic rate. Fine hand tremor is related to hyperthyroidism and is not present in an infant with hypothyroidism, even one whose condition is being stabilized with levothyroxine. An increased thyroid stimulating hormone level would indicate inadequate treatment.

The nurse observes a new nurse administering a rectal suppository to a client. Which actions are appropriate for the new nurse to implement? Select all that apply. A. The nurse places the client on the left side during insertion. B. The nurse pushes the suppository in, up to the second knuckle. C. After 10 minutes, the nurse turns the client to the right side. D. The nurse applies water-soluble lubricant to the suppository. E. The nurse instructs clients to hold their breath and bear down.

A. The nurse places the client on the left side during insertion. B. The nurse pushes the suppository in, up to the second knuckle. C. After 10 minutes, the nurse turns the client to the right side. D. The nurse applies water-soluble lubricant to the suppository. Left side-lying position is the optimal position for the client receiving rectal medications. Due to the position of the descending colon, placing the client on their left side allows the medication to be inserted and move along the natural curve of the intestine and facilitates retention of the medication. The suppository should be somewhat melted after 10 to 15 minutes and turning the client to the right side will aid in further absorption. The suppository should be lubricated to ease insertion and reduce discomfort for the client. Bearing down will place pressure on the anal sphincter and may cause the suppository to be expelled. The client should be instructed to breathe slowly and try to relax.

The nurse is teaching a client who has been diagnosed with recurrent genital herpes about newly prescribed valacyclovir. Which statement by the client indicates understanding? A. This medication is preferable because I can take it less often than other antivirals B. I will be free of outbreaks from now on C. This medication will prevent transmission of the virus to my partner D. Starting the medication now will not help speed up healing

A. This medication is preferable because I can take it less often than other antivirals Rationale: Valacyclovir has greater bioavailability than acyclovir does and is administered less frequently. It speeds up the healing process for lesions and reduces discomfort from the lesions, even if they've already developed. While antivirals do reduce the risk of transmitting herpes simplex to partners, it is not eliminated. The number of outbreaks may be reduced but also may not be completed eliminated.

A client who has a habit of smoking is on estrogen therapy. Which condition is the client at most risk of developing? A. Thrombosis B. Gastrointestinal upset C. Risk of developing cancer D. Decreased effectiveness of estrogen

A. Thrombosis Smoking along with estrogen therapy may increase the risk of thrombosis. Estrogen taken on an empty stomach may cause gastrointestinal upset. Estrogen is not prescribed to clients with endometrial or breast cancer. The effectiveness of estrogen decreases with the use of anticoagulants, rifampicin, and St. John wort.

The nurse teaches an adolescent about the side effects of azithromycin. The nurse determines the teaching has been understood when the adolescent identifies which problem as the most common side effect of this medication? A. Tinnitus B. Diarrhea C. Dizziness D. Headache

A. Tinnitus Diarrhea initially is related to gastrointestinal irritation; later it is related to loss of intestinal flora, which may lead to overgrowth of drug-resistant microbes, resulting in superinfection. This also causes diarrhea. Tinnitus, dizziness (vertigo), and headache all may occur, but none is the most common side effect.

A client is diagnosed with acute kidney failure secondary to dehydration. An intravenous (IV) infusion of 50% glucose with regular insulin is prescribed to address which purpose? A. To correct hyperkalemia B. To increase urinary output C. To prevent respiratory acidosis D. To increase serum calcium levels

A. To correct hyperkalemia The 50% glucose and regular insulin infusion treats the hyperkalemia associated with kidney failure; it moves potassium from the intravascular compartment into the intracellular compartment. Insulin will not increase urinary output. Insulin is not a treatment for respiratory acidosis. Insulin and glucose do not increase serum calcium levels.

Which teaching would a nurse give to a client with a prescription for potassium supplements? A. To report any abdominal distress B. To use salt substitutes to season food C. To take the medication on an empty stomach D. To increase the dosage if muscle cramps occur

A. To report any abdominal distress Potassium supplements can cause gastrointestinal ulceration and bleeding. Most salt substitutes contain potassium, and their use with potassium supplements can cause hyperkalemia. Because they can be irritating to the stomach, potassium supplements should not be taken on an empty stomach. Although muscle cramps may indicate hypokalemia, clients should not adjust their own dosage.

Which assessment finding would the nurse identify as an adverse effect of a client's lidocaine infusion? A. Tremors B. Tachypnea C. Tachycardia D. Hypertension

A. Tremors Tremors are a precursor to the major adverse effect of seizures. Respiratory changes may occur, but tachypnea is not typical. Bradycardia, which may lead to heart block, may occur, not tachycardia. Hypotension, not hypertension, may occur.

Which finding in a client with hypothyroidism and hypertension who reports taking an extra dose of levothyroxine indicates the need to obtain a thyroid function panel? Select all that apply. One, some, or all responses may be correct. A. Tremors B. Diaphoresis C. Nervousness D. Temperature 101°F E. Heart rate 116 beats/min

A. Tremors B. Diaphoresis C. Nervousness D. Temperature 101°F E. Heart rate 116 beats/min Clients with hypothyroidism can develop thyrotoxicosis from an acute overdose of thyroid hormone. Tremors, diaphoresis, and nervousness are all signs of thyrotoxicosis. Clients may also be hyperthermic and tachycardic.

Several hours after administering insulin, the nurse assesses the client's response to the insulin. Which client responses are indicative of a hypoglycemic reaction? Select all that apply. One, some, or all responses may be correct. A. Tremors B. Anorexia C. Confusion D. Glycosuria E. Diaphoresis

A. Tremors C. Confusion E. Diaphoresis Confusion is typically the first sign of a hypoglycemic reaction. Tremors are a sympathetic nervous system response that occurs because circulating glucose in the brain decreases. Diaphoresis is a cholinergic response to hypoglycemia. Hypoglycemia causes hunger, not anorexia. Because blood glucose is low in hypoglycemia, the renal threshold is not exceeded and glycosuria does not occur.

Which clinical manifestation exhibited by a client taking levothyroxine for hypothyroidism for 3 months would cause a nurse to suspect that a decrease in dosage is needed? Select all that apply. One, some, or all responses may be correct. A. Tremors B. Bradycardia C. Somnolence D. Heat intolerance E. Decreased blood pressure

A. Tremors D. Heat intolerance Excessive levothyroxine produces adaptations similar to hyperthyroidism, including tremors, heat intolerance, tachycardia, hypertension, and insomnia. These adaptations are related to the increase in the metabolic rate associated with hyperthyroidism. Bradycardia is a sign of hypothyroidism and a need to increase the dose of levothyroxine. Somnolence is a sign of hypothyroidism and a need to increase the dose of levothyroxine. Hypotension is a sign of hypothyroidism and a need to increase the dose of levothyroxine.

Which medication class includes amitriptyline? A. Tricyclics B. Monoamine oxidase inhibitors (MAOIs) C. Selective serotonin reuptake inhibitors (SSRIs) D. Serotonin-norepinephrine reuptake inhibitors (SNRIs)

A. Tricyclics Amitriptyline is one of several tricyclic antidepressants used to treat anxiety disorders. It is not an MAOI, SSRI, or SNRI.

Which nursing assessment would performed by a nurse before administering intravenous (IV) infusion of potassium chloride (KCl) 40 mEq in 100 mL of 5% dextrose and water to be infused over 2 hours? Select all that apply. One, some, or all responses may be correct. A. Urinary output B. Deep tendon reflexes C. Last bowel movement D. Arterial blood gas results E. Last serum potassium level F. Patency of the intravenous access

A. Urinary output E. Last serum potassium level F. Patency of the intravenous access Before administering IV potassium, the urinary output must be normal. If the urine output is low, a potassium infusion may damage renal cells. The last serum potassium level should also be checked to ensure potassium replacement is appropriate. A patent IV access is essential because potassium is very irritating and painful to subcutaneous tissue. The infusion of KCL 40 mEq in 100 mL of 5% dextrose and water has no direct effect on bowel movement patterns, arterial blood gases, or deep tendon reflexes. These items are not required to be assessed before the administration of this medication.

A female client with rheumatoid arthritis takes ibuprofen (Motrin) 600 mg PO 4 times a day. To prevent gastrointestinal bleeding, misoprostol (Cytotec) 100 mcg PO is prescribed. Which information is most important for the nurse to include in client teaching? A. Use contraception during intercourse. B. Ensure the Cytotec is taken on an empty stomach. C. Encourage oral fluid intake to prevent constipation. D. Take Cytotec 30 minutes prior to Motrin.

A. Use contraception during intercourse. Cytotec, a synthetic form of a prostaglandin, is classified as pregnancy Category X and can act as an abortifacient, so the client should be instructed to use contraception during intercourse to prevent loss of an early pregnancy. A common side effect of Cytotec is diarrhea, so constipation prevention strategies are usually not needed. Cytotec and Motrin should be taken together to provide protective properties against gastrointestinal bleeding.

The nurse is providing education to the parent of a pediatric client receiving amoxicillin clavulanate suspension. Which of the following statements is appropriate? A. Use the measuring device provided by the pharmacy B. You should take this medication on an empty stomach C. Avoid shaking the medication before opening D. Take the medication with a glass of juice

A. Use the measuring device provided by the pharmacy Rationale: Take augmentin (amoxicillin clavulanate) with meals to increase absorption and decrease GI upset. Acidic fluids may destroy the drug, so avoid taking the medication with citrus juice. The client should be taught to shake liquid penicillins well as the medication tends to separate out of the suspension. Measure liquid doses carefully. Use the measuring device that comes with this drug. If there is none, ask the pharmacist for a device to measure this drug.

A woman questions the nurse about the effectiveness of oral contraceptives. Which important factor about the effectiveness of oral contraceptives would the nurse include in the response? A. User motivation B. Simplicity of use C. Reliability record D. Identified risk factors

A. User motivation Conception will not be prevented unless the user is motivated to use the method correctly and consistently. No matter how simple, the method must be used consistently. Reliability record is not relevant if the method is not used correctly and consistently by the woman. Risk factors have little influence on the effectiveness of the contraceptive method.

The nurse is caring for a client who has been prescribed vancomycin intravenous infusion for the treatment of methicillin-resistant staphylococcus aureus. Which of the following laboratory values should be immediately reported to the healthcare provider? A. Vancomycin trough of 15 mcg/dl B. Blood urea nitrogen level of 18 mg/dl C. Creatinine level of 1.1 mg d/l D. White blood cell count of 11,500 per microliter

A. Vancomycin trough of 15 mcg/dl Rationale: Vancomycin has a low therapeutic index, with nephrotoxicity and ototoxicity complicating therapy if toxicity develops. In contrast, underdosing (less than the minimum inhibitory concentration) can lead to treatment failure. Nephrotoxicity is associated with a trough level above 10 mcg/dl. The BUN and creatinine in this case are still within a normal range. While the WBC count is elevated, this is an expected finding.

Which medication is indicated for emergency treatment of bleeding esophageal varices? A. Vasopressin B. Neostigmine C. Lansoprazole D. Phytonadione

A. Vasopressin Vasopressin is a vasoconstrictor that can be used to control gastrointestinal bleeding. Neostigmine inhibits cholinesterase, permitting acetylcholine to function; it is used primarily for myasthenia gravis. Lansoprazole is a proton pump inhibitor that is used for the treatment of gastric and duodenal ulcers. Phytonadione is vitamin K; it promotes formation of prothrombin in the liver. Although this medication may be helpful, its effects take too long to be of value in an emergency situation.

The nurse is transcribing a new prescription for spironolactone (Aldactone) for a client who receives an angiotensin-converting enzyme (ACE) inhibitor. Which action should the nurse implement? A. Verify both prescriptions with the healthcare provider. B. Report the medication interactions to the nurse manager. C. Hold the ACE inhibitor and give the new prescription. D. Transcribe and send the prescription to the pharmacy.

A. Verify both prescriptions with the healthcare provider. The concomitant use of an angiotensin-converting enzyme (ACE) inhibitor and a potassium-sparing diuretic such as spironolactone, should be given with caution because the two drugs may interact to cause an elevation in serum potassium levels. Although the client is currently receiving an ACE inhibitor, verifying both prescriptions alerts the healthcare provider about the client's medication regimen and provides the safest action before administering the medication.

When the nurse is administering a course of aminoglycoside treatment to a client with Klebsiella infection, which adverse effects prompt the nurse to hold treatment and contact the health care provider? Select all that apply. One, some, or all responses may be correct. A. Vertigo B. Tinnitus C. Dizziness D. Heartburn E. Persistent headache

A. Vertigo B. Tinnitus C. Dizziness E. Persistent headache The nurse monitors the client administered aminoglycosides for signs of ototoxicity, which include vertigo, tinnitus, dizziness, and persistent headache. Any sign of ototoxicity should result in holding the treatment and contacting the health care provider. Heartburn is not associated with ototoxicity.

A health care provider prescribes digoxin for a client. The nurse teaches the client to be alert for which common early indication of acute digoxin toxicity? A. Vomiting B. Urticaria C. Photophobia D. Respiratory distress

A. Vomiting Nausea, vomiting, anorexia, and abdominal pain are early indications of acute toxicity in approximately 50% of clients who take a cardiac glycoside, such as digoxin. Urticaria is a rare, not common, manifestation of digoxin toxicity. Photophobia is a later, not early, manifestation of digoxin toxicity. Respiratory distress is not directly associated with digoxin toxicity.

Warfarin is prescribed for the client who takes phenytoin for a seizure disorder. Which medication interaction complicates seizure therapy? A. Warfarin inhibits the metabolism of phenytoin. B. Warfarin decreases phenytoin absorption. C. Phenytoin competes with warfarin for receptor occupation. D. Warfarin promotes excretion of phenytoin.

A. Warfarin inhibits the metabolism of phenytoin. Warfarin inhibits metabolism of phenytoin, which can result in phenytoin toxicity. Warfarin does not decrease phenytoin absorption. Phenytoin and warfarin act on different receptors. Excretion of phenytoin is not increased.

Which anticipatory guidance would the nurse include when teaching an adolescent about side effects of dactinomycin and doxorubicin therapy? A. Wear a baseball cap. B. Eat three meals daily. C. Avoid dairy products. D. Dress in light clothing.

A. Wear a baseball cap. Antineoplastic medications exert their effect on rapidly dividing tissues such as hair follicles, resulting in alopecia. Eating regular meals, avoiding dairy products, and wearing certain types of clothing are not related to the side effects of the antineoplastics that are being used.

A client is receiving furosemide to relieve edema. The nurse will monitor the client for which responses? Select all that apply. One, some, or all responses may be correct. A. Weight loss B. Negative nitrogen balance C. Increased urine specific gravity D. Excessive loss of potassium ions E. Pronounced retention of sodium ions

A. Weight loss D. Excessive loss of potassium ions Each liter of fluid weighs 2.2 pounds (1 kilogram). Assessing weight loss is an objective measure of the effectiveness of the medication. Furosemide is a potent diuretic that is used to provide rapid diuresis in clients with pulmonary edema; it acts in the loop of Henle and causes depletion of electrolytes, such as potassium and sodium. A negative nitrogen balance does not affect protein metabolism. With increased fluid loss, the specific gravity is likely to be lowered. Furosemide inhibits the reabsorption of sodium.

When would the nurse plan to administer pancrelipase to a child with cystic fibrosis? A. With meals and snacks B. In the morning and at bedtime C. On awakening and every 3 hours while the child is awake D. After each bowel movement and after postural drainage is performed

A. With meals and snacks Pancrelipase must be taken with food and snacks because it is essential for the digestion of nutrients. The enzyme is ineffective when taken without food; it is contraindicated at any other time.

The nurse is preparing the 0900 dose of losartan (Cozaar), an angiotensin II receptor blocker (ARB), for a client with hypertension and heart failure. The nurse reviews the client's laboratory results and notes that the client's serum potassium level is 5.9 mEq/L. Which action should the nurse take first? A. Withhold the scheduled dose. B. Check the client's apical pulse. C. Notify the healthcare provider. D. Repeat the serum potassium level.

A. Withhold the scheduled dose. The nurse should first withhold the scheduled dose of Cozaar because the client is hyperkalemic (normal range 3.5 to 5 mEq/l). Although hypokalemia is usually associated with diuretic therapy in heart failure, hyperkalemia is associated with several heart failure medications, including ARBs. Because hyperkalemia may lead to cardiac dysrhythmias, the nurse should check the apical pulse for rate and rhythm, and blood pressure.

A client is receiving clonidine for hypertension. Which side effect of clonidine will the nurse include when providing medication education? A. Xerostomia B. Diarrhea C. Euphoria D. Photosensitivity

A. Xerostomia Xerostomia (dry mouth) is one of the common side effects of this medication. The reaction usually diminishes over the first 2 to 4 weeks of therapy. This medication causes constipation, not diarrhea. This medication may cause depression, anxiety, fatigue, and drowsiness, not euphoria. Photosensitivity is not a side effect of this medication.

The nurse is caring for a client who is receiving isoniazid for tuberculosis (TB). Which assessment finding would indicate the client is having a possible adverse response to this medication? A. Yellowing of the sclera B. Tinnitus and decreased hearing C. Headache and sore throat D. Urinary frequency

A. Yellowing of the sclera Isoniazid is a first-line anti-tuberculosis drug that is used as part of the combination therapy for treatment of tuberculosis. These first-line medications may be used up to 2 years in clients who are being treated for tuberculosis. The use of long-term combination treatment increases the effectiveness and decreases the occurrence of resistant strands. Clients receiving this medication are at risk for drug-induced hepatitis. The appearance of jaundice (yellowing of the sclera) may indicate an elevation of the client's serum bilirubin levels and liver enzymes (AST and ALT). A small number of clients taking isoniazid develop severe hepatitis that may progress to liver failure and death, unless the medication is stopped immediately. Other common side effects include nausea and peripheral neuropathy. This medication is not ototoxic and can occasionally cause urinary retention, not frequency.

) A nurse is administering insulin glargine to a client with diabetes type I. The client asks the nurse why insulin is the only option for therapy. Which statement by the nurse is appropriate? A. Your body does not produce an adequate amount of insulin B. Insulin is better at controlling the disease than oral pills C. Your body has a resistance to insulin D. Oral pills take longer to produce therapeutic effects than insulin

A. Your body does not produce an adequate amount of insulin Rationale: Diabetes mellitus type 1 is characterized by the inability of the beta cells to produce insulin. The disease is managed by implementing an insulin regimen. Oral hypoglycemic medications are not effective in treating diabetes type 1. The body's resistance to insulin is characteristic of diabetes mellitus type 2. The onset of oral hypoglycemic medications is not relevant to a client with diabetes mellitus type 1.

A client diagnosed with gastroesophageal reflux disease (GERD) is being treated with antacid therapy. When teaching the client about the antacids, which information would the nurse reinforce? Antacids should be taken 1 hour before meals. These should be scheduled at 4-hour intervals. Antacid tablets are just as fast and effective as the liquid form. Antacids commonly interfere with the absorption of other medications.

Antacids commonly interfere with the absorption of other medications. Antacids interfere with absorption of medications such as anticholinergics, barbiturates, tetracycline, and digoxin. Liquid antacids are faster acting and more effective than antacid tablets. Antacids should be taken 1 or 2 hours after meals and at bedtime. Antacid tablets may be taken more frequently than every 4 hours.

A client is prescribed trimethoprim/sulfamethoxazole for recurrent urinary tract infections. Which information should the nurse include during client teaching? A. "A harmless skin rash may appear." B. "Drink at least eight large glasses of water a day." C. "Be sure to take the medication with food." D. "Stop the medication when your symptoms disappear."

B. "Drink at least eight large glasses of water a day." Trimethoprim/sulfamethoxazole (Bactrim) is a highly insoluble drug that can cause crystalluria and clients should drink plenty of fluids while taking this medication to lower the risk of developing kidney stones. Increased fluid intake is also recommended with a UTI to promote the "flushing out" of bacteria. The drug may be taken with or without food. Clients should take the medication for the prescribed length of time. Sulfonamide-containing products should be discontinued at the first appearance of skin rash. In rare instances, a skin rash may be followed by a more severe reaction, such as Stevens-Johnson syndrome or toxic epidermal necrolysis.

Which statement is important for the nurse to teach a client prescribed psyllium 3.5 g twice a day for constipation? A. "Urine may be discolored." B. "Each dose should be taken with a full glass of water." C. "Use only when necessary because it can cause dependence." D. "Daily use may inhibit the absorption of some fat-soluble vitamins."

B. "Each dose should be taken with a full glass of water." Because this medication has a strong affinity for fluids, it will swell in the intestine. The large bulk stimulates peristalsis. A full glass of fluid taken at the same time will help minimize the risk of esophageal obstruction or fecal impaction. Senna, a stimulant laxative, may discolor urine, not psyllium. Psyllium, a bulk-forming laxative, is among the safest laxatives on the market. It is useful with prolonged therapy because it is not systemically absorbed and is not potent in its action. Prolonged use of lubricant or saline/osmotic laxatives, not bulk-forming laxatives, can inhibit the absorption of some fat-soluble vitamins.

The nurse is providing information to a client about propranolol. Which statement by the client indicates the teaching has been effective? A. "I should expect to feel nervousness during the first few weeks." B. "I can have a heart attack if I stop this medication suddenly." C. "I could have an increase in my heart rate for a few weeks." D. "I may experience seizures if I stop the medication abruptly."

B. "I can have a heart attack if I stop this medication suddenly." Propranolol is commonly used to treat hypertension, abnormal heart rhythms, heart disease and certain types of tremors. It is in a class of medications called beta blockers. Suddenly discontinuing a beta blocker can cause angina, hypertension, dysrhythmias, or even a myocardial infarction (i.e., heart attack).

A client who has been taking the prescribed dose of zolpidem for 5 days returns to the clinic for a follow-up visit. Which statement by the client indicates the medication has been effective? A. "I have less pain." B. "I have been sleeping better." C. "My blood glucose is under control." D. "My blood pressure is coming down."

B. "I have been sleeping better." Zolpidem is a sedative-hypnotic that produces central nervous system depression in the limbic, thalamic, and hypothalamic areas of the brain. Zolpidem is not an analgesic, antidiabetic, or antihypertensive medication.

Which statement by a client prescribed ampicillin indicates that teaching by the nurse was effective? A. "I will miss eating grapefruit." B. "I must increase my fluid intake." C. "I can stop taking this medication any time." D. "I should take this medication just after eating."

B. "I must increase my fluid intake." The client should increase fluid intake when taking ampicillin to prevent nephrotoxicity; side effects include oliguria, hematuria, proteinuria, and glomerulonephritis. There are no restrictions on eating grapefruit when taking an antibiotic; this is contraindicated when taking some calcium channel blockers because grapefruit juice increases their serum level. An antibiotic should be continued until the entire prescription is completed; discontinuing before completion lowers its serum level, thereby decreasing its effectiveness. Ampicillin should be taken when the stomach is empty, either 1 to 2 hours before eating or 3 to 4 hours after eating.

A nurse is assessing a client with hyperthyroidism and is taking prescribed methimazole. Which client statement indicates a therapeutic response to the medication? A. "My intolerance to cold has improved." B. "I no longer feel heart palpitations." C. "I don't get constipated as easily." D. "I have lost a few pounds."

B. "I no longer feel heart palpitations." Rationale: Methimazole is an antithyroid medication used in the treatment of hyperthyroidism. Tachycardia and heart palpitations are signs of hyperthyroidism. The expected response of methimazole is a decrease in the severity of hyperthyroidism symptoms. Intolerance to cold, constipation, and weight gain are signs of hypothyroidism. Improvement in these symptoms do not evaluate the effectiveness of methimazole.

The nurse is providing teaching to a client who has been prescribed cyclophosphamide for breast cancer treatment. Which of the following statements made by the client would indicate that additional teaching is needed? A. "I will probably need to plan on using a wig to cover my hair loss." B. "I should limit the amount of fluids I drink while taking this medication." C. "I will need to stay away from children when my white blood cell count is low." D. "I may have trouble getting pregnant due to the damaging effects of the medication."

B. "I should limit the amount of fluids I drink while taking this medication." Cyclophosphamide is a chemotherapeutic medication. Some of the side effects of this medication include hair loss, low white cell count and infertility. The client is encouraged to drink about 2 to 3 liters of fluid per day to aid in eliminating the chemotherapy from the body.

The nurse is discharging a client on oral potassium replacement. Which of the following statements requires further teaching by the nurse? A. "I can still take my nonsteroidal anti-inflammatory medications occasionally for my arthritis pain." B. "I will continue to use salt substitutes to flavor my food." C. "I will take my furosemide first thing in the morning." D. "I will read the food labels for added potassium."

B. "I will continue to use salt substitutes to flavor my food." Salt substitutes are made using potassium. As the client is taking potassium supplements, they should avoid salt substitutes to prevent hyperkalemia from occurring. NSAIDS can be used occasionally. The furosemide should be taken in the morning. Some low-sodium prepared foods may contain potassium, so reading the labels is important.

Which response would a nurse give to a client who takes furosemide and digoxin and reports that everything looks yellow? A. "This is related to your heart problems, not to the medication." B. "I will hold the medication until I consult with your health care provider." C. "It is a medication that is necessary, and that side effect is only temporary." D. "Take this dose, and when I see your health care provider, I will ask about it."

B. "I will hold the medication until I consult with your health care provider." The response "I will hold the medication until I consult with your health care provider" is a safe practice because yellow vision indicates digitalis toxicity. The response "This is related to your heart problems, not to the medication" is incorrect; yellow vision is not a symptom of heart disease. The response "It is a medication that is necessary, and that side effect is only temporary" is incorrect; yellow vision is not a temporary side effect. The response "Take this dose, and when I see your health care provider, I will ask about it" is unsafe.

The nurse is reviewing discharge instructions with a client who has been prescribed ciprofloxacin following a minor burn injury. Which statement by the client requires additional teaching? A. "I will protect my skin from the sun with sunscreen and clothing." B. "I will not take ciprofloxacin prior to sun exposure." C. "After healing, I should have no scarring from this burn." D. "I can take ibuprofen for the pain related to this burn."

B. "I will not take ciprofloxacin prior to sun exposure." Ciprofloxacin is an antibiotic that is associated with causing photosensitivity. Clients should be instructed to protect their skin from sun exposure while taking this medication. Appropriate methods to protect the skin are to limit sun exposure and to wear sunscreen and protective clothing. For a superficial-thickness burn, no scarring will occur and healing should take 3 to 6 days. The client may take a nonsteroidal anti-inflammatory drug (NSAID), such as ibuprofen, to alleviate the pain associated with the burn. It is inappropriate for the client to stop taking their antibiotic. However, if the client cannot avoid sun exposure, the nurse may contact the health care provider and request that the antibiotic be changed to one that does not cause photosensitivity.

A client has received a prescription for nitrofurantoin to treat a UTI. Which of the following statements made by the client indicates the need for additional teaching about the medication? A. "I will be sure to finish taking the antibiotics, even if I start feeling better." B. "I will spend extra time in the sun to get plenty of vitamin D." C. "I'll call my primary health care provider immediately if I develop a rash after taking the medication." D. "I will take the medication with food."

B. "I will spend extra time in the sun to get plenty of vitamin D." Clients taking nitrofurantoin should avoid exposure to sunlight while taking the medication. Exposure to sunlight while taking this medication can lead to damage to the skin. A client planning to spend extra time in the sun while taking nitrofurantoin should be informed of the dangers of sun exposure and counseled to avoid sun exposure while taking the medication.

Which statement indicates that a client understands how to take antacids appropriately? A. "I will take this antacid at the onset of pain." B. "I will take this antacid 30 minutes after meals." C. "I will take this antacid every 4 hours around the clock." D. "I will take this antacid each time I have something to eat."

B. "I will take this antacid 30 minutes after meals." Antacids are most effective when taken after digestion has started but before the stomach begins to empty. Antacids should be taken before the onset of pain; pain indicates that gastric irritation has begun, and the aim of treatment is to protect the gastrointestinal mucosa. Antacids taken every 4 hours around the clock interfere with the absorption of nutrients. Antacids taken with food interfere with the absorption of nutrients.

Which response would a nurse give to a client diagnosed with type 1 diabetes who states "I hate shots. Why can't I take the insulin in tablet form?"? A. "Your diabetic condition is too serious for oral insulin." B. "Insulin is poorly absorbed orally, so it is not available in a tablet." C. "Insulin by mouth causes a high incidence of allergic and adverse reactions." D. "Once your diabetes is controlled, your primary health care provider might consider oral insulin."

B. "Insulin is poorly absorbed orally, so it is not available in a tablet." The chemical structure of insulin is altered by gastric secretions, rendering it ineffective. There is no such thing as oral insulin; this comment about the seriousness of the diabetic condition may increase anxiety. There are no data to support the statement regarding allergic or adverse reactions, and insulin is given parenterally, not orally. Insulin is not absorbed but is destroyed by gastric secretions; there is no insulin that is effective if taken by mouth.

The nurse is teaching a client with chronic renal failure about their medications. The client questions the purpose of taking aluminum hydroxide. How should the nurse respond? A. "It increases your urine output." B. "It decreases your blood's phosphate levels." C. "It is taken to control gastric acid secretions." D. "It will reduce your blood's calcium levels."

B. "It decreases your blood's phosphate levels." Aluminum binds to phosphates that tend to accumulate in the client with chronic renal failure due to decreased filtration capacity of the kidneys. Antacids such as aluminum hydroxide are commonly used in clients with chronic renal failure to decrease serum phosphate levels. Aluminum hydroxide will not increase urine production, control gastric acid secretions or lower serum calcium levels.

Which explanation will the nurse give as to why potassium iodide solution should be taken before a subtotal thyroidectomy? A. "The metabolic rate of the body will increase." B. "It will reduce the risk of hemorrhage during surgery." C. "It will maintain the functioning of the parathyroid glands." D. "The amount of thyroid hormones being secreted will decrease."

B. "It will reduce the risk of hemorrhage during surgery." Potassium iodide, which aids in decreasing vascularity of the thyroid gland, decreases the risk for hemorrhage. Thyroid hormone antagonists help decrease the body's metabolism. Potassium iodide does not regulate parathyroid function. Thyroid hormone antagonists help decrease the amount of thyroid hormones being secreted.

Which information would the nurse provide to a client with type 1 diabetes who requests information about the differences between penlike insulin delivery devices and syringes? A. "The penlike devices have a shorter injection time." B. "Penlike devices provide a more accurate dose delivery." C. "The penlike delivery system uses a smaller-gauge needle." D. "Penlike devices cost less by having reusable insulin cartridges."

B. "Penlike devices provide a more accurate dose delivery." Penlike insulin delivery devices are more accurate because they are easy to use; also, they promote adherence to insulin regimens because the medication can be administered discreetly. One disadvantage of the penlike insulin delivery device is that the injection time will be longer; the device must remain in place for several seconds after the insulin is injected to ensure that no insulin leaks out. The penlike insulin delivery device has a larger-gauge needle that has a smaller diameter. The insulin cartridges of a penlike insulin delivery device are single use and disposable.

A nurse is assessing a 9-year-old child after several days of treatment for a documented strep throat. Which statement is incorrect and suggests that further teaching is needed? A. "Sometimes I take my medicine with fruit juice." B. "Sometimes I take the pills in the morning and other times at night." C. "I am feeling much better than I did last week." D. "My mother makes me take my medicine right after school."

B. "Sometimes I take the pills in the morning and other times at night." Strep throat is a bacterial infection that is treated with antibiotics. It is important to take antibiotics on a regular schedule and at approximately the same time each day. Depending on the medication, it is OK to take it with food or juice. The client should be feeling better after several days of antibiotics —however should be cautioned to complete the prescribed amount.

A 65-year-old client is receiving amitriptyline. Which recommendation will the nurse make to the client concerning this medication? A. 'Obtain a complete cholesterol and lipid profile.' B. 'Have an eye examination to check for glaucoma.' C. 'Check your temperature daily for nighttime increases.' D. 'Watch for excessive sweating and possible weight loss.'

B. 'Have an eye examination to check for glaucoma.' In addition to baseline laboratory tests, an older adult should have an eye examination with glaucoma testing when taking amitriptyline. It causes dilation of the pupil (mydriasis), which interferes with drainage of aqueous humor through the canal of Schlemm. Interfering with the outflow of aqueous humor will increase intraocular pressure and may cause a progressive loss of vision in clients with glaucoma. Amitriptyline does not affect cholesterol production or temperature regulation. It does not cause excessive sweating or weight loss, but it can increase appetite especially for sweets, resulting in weight gain.

Pyridostigmine bromide is prescribed for a client with myasthenia gravis. The nurse evaluates that the medication regimen is understood when the client makes which statement? A. 'I will take the medication on an empty stomach.' B. 'I need to set an alarm so I take the medication on time.' C. 'It will be important to check my heart rate before taking the medication.' D. 'I should monitor for an increase in blood pressure after taking the medication.'

B. 'I need to set an alarm so I take the medication on time.' Pyridostigmine is a vital medication that must be taken on time; a missed or late dose can result in severe respiratory and neuromuscular consequences or even death. Pyridostigmine should be taken with a small amount of food to prevent gastric irritation. It is unnecessary to take the pulse rate before taking pyridostigmine. Pyridostigmine may cause hypotension, not hypertension, which is a sign of cholinergic crisis.

A registered nurse teaches a nursing student about caring for a client prescribed estradiol to treat low estrogen levels. Which statement by the student indicates to the nurse a need for additional learning? A. 'I should apply the emulsion once a day on the thighs.' B. 'I should avoid covering the medication with clothing after it is dried.' C. 'I should educate the client about the pharmacokinetic effects of estradiol.' D. 'I should advise the client to avoid applying sunscreen at the same time as the medication intake.'

B. 'I should avoid covering the medication with clothing after it is dried.' Covering the medication with clothing after it is dried helps prevent the transfer of the medication to other individuals. The nurse would instruct the client to apply the emulsion once a day on the thighs. The nurse would educate the client about the pharmacokinetic properties of the medication to ensure the medication's safe and effective administration. The nurse would advise the client to not apply sunscreen products at the same time because this action may reduce the absorption of estradiol.

The nurse teaches the mother prescribed nystatin for oral thrush, how to prevent aggravation. Which statements indicate the need for further teaching? Select all that apply. A. 'I should rinse the mouth with plain water after feeding.' B. 'I should boil the pacifier at least 20 minutes on alternate days.' C. 'apply the med at least 20 minutes before feeding.' D. 'apply the med to the infant's oral cavity four times a day.' E. 'boil the reusable nipples for at least 5 minutes after wash.'

B. 'I should boil the pacifier for at least 20 minutes on alternate days.' C. 'I should apply the medication at least 20 minutes before feeding.' E. 'I should boil the reusable nipples for at least 5 minutes after washing.' Nystatin is used to treat oral thrush in infants. Boiling the pacifier for 20 minutes on alternate days is inadequate because daily boiling of the pacifier is the best way to ensure efficient sterilization and killing of pathogens. Pacifiers should be boiled daily for at least 20 minutes. Nystatin should be administered to the child after feeding, not before feeding. Reusable nipples should be boiled at least 20 minutes after washing to remove spores, which are heat-resistant. Rinsing the infant's mouth after feeding with plain water reduces the risk of infection in the infant. Nystatin should be applied four times a day to the infant's oral cavity to ensure effective medication action.

The health care provider prescribes neostigmine for a client with myasthenia gravis. Which client statement indicates understanding regarding medication management plans? A. 'I must keep the medication in a container in the refrigerator.' B. 'I should take the medication at the exact time that is listed on the prescription.' C. 'I will plan to take the medication between meals.' D. 'I expect that the onset of the medication's action will occur several hours after I take it.'

B. 'I should take the medication at the exact time that is listed on the prescription.' Neostigmine should be taken as prescribed, usually before meals, to limit dysphagia and possible aspiration. Keeping neostigmine refrigerated is not necessary; it may be kept at room temperature. Neostigmine should be taken with milk to prevent gastrointestinal irritation; usually it is taken about 30 minutes before meals. The onset of the action of neostigmine occurs 45 to 75 minutes after administration; the duration of its action is 2.5 to 4 hours.

A client who takes furosemide and digoxin reports to the nurse that everything looks yellow. Which response by the nurse is most appropriate? A. 'This is related to your heart problems, not to the medication.' B. 'I will hold the medication until I consult with your health care provider.' C. 'It is a medication that is necessary, and that side effect is only temporary.' D. 'Take this dose, and when I see your health care provider, I will ask about it.'

B. 'I will hold the medication until I consult with your health care provider.' The response 'I will hold the medication until I consult with your health care provider' is a safe practice because yellow vision indicates digitalis toxicity. The response 'This is related to your heart problems, not to the medication' is incorrect; yellow vision is not a symptom of heart disease. The response 'It is a medication that is necessary, and that side effect is only temporary' is incorrect; yellow vision is not a temporary side effect. The response 'Take this dose, and when I see your health care provider, I will ask about it' is unsafe.

Trimethoprim/sulfamethoxazole is prescribed for a child with a urinary tract infection. Which statement by the parent indicates the nurse's instructions about administration have been understood? A. 'Mealtime is a good time to give the medication.' B. 'I'll make sure to give each pill with 6 to 8 oz of fluid.' C. 'It must be taken with orange juice to ensure acidity of urine.' D. 'The medication has to be taken every 4 hours to maintain a blood level.'

B. 'I'll make sure to give each pill with 6 to 8 oz of fluid.' This is a sulfa medication; water must be encouraged to prevent urine crystallization in the kidneys. This medication does not have to be given with meals; it is administered every 12 hours. Orange juice causes an alkaline urine; water is the best fluid to be administered with this medication. This medication maintains the blood level for 8 to 12 hours; it is an intermediate-acting medication.

A client with tuberculosis is started on rifampin. The nurse evaluates that the teaching about rifampin is effective when the client makes which statement? A. 'I need to drink a lot of fluid while I take this medication.' B. 'My sweat will turn orange from this medication.' C. 'I should have my hearing tested while I take this medication.' D. 'Most people who take this medication develop a rash.'

B. 'My sweat will turn orange from this medication.' Rifampin causes body fluids, such as sweat, tears, and urine, to turn orange. It is not necessary to drink large amounts of fluid with this medication; it is not nephrotoxic. Damage to the eighth cranial nerve is not a side effect of rifampin; it is a side effect of streptomycin sulfate, which is sometimes used to treat tuberculosis. A skin rash is not a side effect of rifampin.

A client with a seizure disorder will begin taking phenytoin. Which instructions will the nurse give to the client? A. 'Take the medication on an empty stomach.' B. 'Provide meticulous oral hygiene.' C. 'Taper off the medication if seizures are controlled for 3 months.' D. 'Stop taking the medication if you become pregnant.'

B. 'Provide meticulous oral hygiene.' Phenytoin can cause gingival overgrowth that increases the risk for periodontal disease. The medication should be taken with food or milk to decrease gastrointestinal side effects. The health care provider should oversee any dosage adjustment or discontinuation. Although it can affect the developing fetus, current evidence suggests that the effect of seizure activity on the developing fetus may cause even greater problems.

Which instruction would the nurse include in a teaching plan for nitroglycerin patches? A. 'Apply the patch on a distal extremity.' B. 'Remove a previous patch before applying the next one.' C. 'Massage the area gently after applying the patch to the skin.' D. 'Apply a warm compress to the site before attaching the patch.'

B. 'Remove a previous patch before applying the next one.' Removing the previous patch before applying the next patch ensures that the client receives just the prescribed dose. Ideally, a patch should be removed after 12 to 14 hours to avoid the development of tolerance. The patch should be rotated among hair-free and scar-free sites; acceptable sites include the chest, upper abdomen, proximal anterior thigh, or upper arm. The patch should be gently pressed against the skin to ensure adherence; it should not be massaged. Applying a warm compress to the site before attaching the patch is unnecessary and can result in excessive absorption of the medication.

When a female client with a new infant is prescribed amoxicillin for a urinary tract infection, which instruction would the nurse include when teaching about the use of this medication? A. 'Take this medication on an empty stomach.' B. 'Report signs of allergic reaction such as skin rash or itching.' C. 'Stop taking the medication as soon as you void without burning.' D. 'Breast-feeding should stop until you have finished with this medication.'

B. 'Report signs of allergic reaction such as skin rash or itching.' Penicillin class medications have a high incidence of allergic reaction, so the client should monitor for allergy and report symptoms of an allergic reaction. Amoxicillin may be taken with food. The entire course of treatment should be completed, not stopped when symptoms are absent. It is safe to breast-feed with amoxicillin.

A school-age child with a seizure disorder is prescribed divalproex/phenytoin. The nurse would include which instruction when teaching the parents about administering this medication? A. 'Crush the tablets and mix them with applesauce.' B. 'Take the child for regularly scheduled blood tests.' C. 'Stop the medication immediately if a rash develops.' D. 'Provide oral hygiene, especially gum massage and flossing.'

B. 'Take the child for regularly scheduled blood tests.' Adverse reactions to divalproex include thrombocytopenia, leukopenia, and lymphocytosis; blood studies must be performed on a regular basis. Tablets must be swallowed whole; they should not be broken, crushed, or chewed. If the medication is stopped suddenly, a seizure may result; a rash should be reported to the health care provider. Meticulous oral hygiene is more important for a child who is taking phenytoin.

Which response by the nurse is appropriate when a client asks what to expect when beginning treatment for tuberculosis? A. 'Therapy will last a few weeks.' B. 'Therapy will occur over two phases.' C. 'Therapy will involve one medication.' D. 'Therapy will require monitoring kidney function.'

B. 'Therapy will occur over two phases.' Therapy for tuberculosis occurs over two phases. The target of the induction phase is to achieve noninfectious sputum, and the target of the continuation phase is to eradicate the intracellular bacteria. Therapy for tuberculosis is prolonged, lasting 6 months to 2 years. Therapy for tuberculosis involves two to four medications. Therapy for tuberculosis requires monitoring liver, not kidney, function.

A health care provider prescribes oral antacids and intravenous ranitidine for a client with burns and crushing injuries caused by an accident. The client asks how these medications work. Which explanation would the nurse provide? A. 'These medications work together to decrease bowel irritability.' B. 'They limit acidity in the gastrointestinal tract.' C. 'They are very effective in clients with multiple trauma.' D. 'These medications decrease nausea and vomiting.'

B. 'They limit acidity in the gastrointestinal tract.' Increased acidity caused by the stress occurring with burns and crushing injuries contributes to the formation of Curling ulcer; ranitidine, an H 2 antagonist, decreases the formation of gastric acid, and an antacid neutralizes gastric acid once it is formed. These medications do not decrease irritability of the bowel; their purpose is to decrease gastrointestinal acidity. The response, 'They are very effective in clients with multiple trauma,' does not answer the client's question. Their purpose is not that of an antiemetic.

Which response by the nurse would be most appropriate to promote a sense of control in a 6-year-old child who is about to receive an injection? A. 'This won't hurt, so you shouldn't cry.' B. 'Which arm should I use to give you the medicine?' C. 'I know you're grown up. You won't cry, will you?' D. 'Close your eyes. You won't even know what's happening.'

B. 'Which arm should I use to give you the medicine?' Offering a choice and involving the child in decision-making gives the child a sense of control over the situation. Assuring the child that the injection will not hurt is not truthful and conveys an unrealistic expectation. Stating that the child is grown up and won't cry indicates an expectation that the child might not be able to meet. Instructing the child not to look negates the child's needs.

A client comes in for a pregnancy test. She tells the nurse that pregnancy may have occurred because she missed her contraceptive pills for 1 week when she had the flu. Which response by the nurse is appropriate? A. 'That's the trouble with using contraceptive pills. People frequently forget to take them.' B. 'You may be correct. The effect of contraceptive pills depends on them being taken on a regular schedule.' C. 'Let's find out whether you really are pregnant. If you are, you may want to consider having an abortion.' D. 'Contraceptive pills are unpredictable. You could have become pregnant even if you had taken them regularly.'

B. 'You may be correct. The effect of contraceptive pills depends on them being taken on a regular schedule.' An oral contraceptive program requires the client to take one tablet daily from the fifth day of the cycle and continue taking tablets for 20 or 21 days. Interrupting the monthly dosage program may permit release of luteinizing hormone, resulting in ovulation and possibly pregnancy. Stating that people often forget to take oral contraceptive pills is judgmental; contraceptive practice is the client's choice. It is premature to discuss abortion. Oral contraceptives that are taken on an exact schedule have a very high rate of success.

At the client's request, the nurse performs a fingerstick to test the client's blood glucose and the results are 322 mg/dL (17.9 mmol/L). Following the insulin sliding scale orders, the nurse administers 3 units of insulin lispro at 11:00 AM. When does the nurse anticipate the insulin lispro will begin to act? A. 3:00 pm B. 11:15 am C. 1:00 pm D. 12:00 PM

B. 11:15 am The onset of action for insulin lispro, which is a rapid acting insulin, is 10 to 15 minutes after administration. It was administered at 11:00 AM, so it will begin to act at 11:15 AM.

A depressed client has been prescribed a tricyclic antidepressant. Which time period indicates how long it usually takes before the client notices a significant change in the depression? A. 4 to 6 days B. 2 to 4 weeks C. 5 to 6 weeks D. 12 to 16 hours

B. 2 to 4 weeks It takes 2 to 4 weeks for the tricyclic antidepressant to reach a therapeutic blood level. Time spans of 4 to 6 days and 12 to 16 hours are both too short for a therapeutic blood level of the medication to be achieved. Improvement in depression should be demonstrated sooner than 5 to 6 weeks.

A client states, 'I keep my insulin in the refrigerator because that is where my parents kept it.' Which reason will the nurse include when explaining why insulin should be stored at room temperature? A. Its potency and effectiveness are maximized. B. Absorption is enhanced and local irritation is decreased. C. It is more convenient and drawing insulin into the syringe is facilitated. D. Adherence of insulin to the syringe and resistance upon injection are decreased.

B. Absorption is enhanced and local irritation is decreased. Insulin that is close to body temperature prevents vasoconstriction at the site and decreases irritation of tissues. Insulin can be stored at room temperature for up to 1 month but must be kept away from heat or sunlight. Inappropriate storage of insulin can decrease its stability and decrease, not increase, its therapeutic action. Although it may be more convenient to keep insulin in the refrigerator, this is not a valid rationale; temperature of the solution does not increase the viscosity of insulin. Neither adherence of insulin to the syringe nor decreased resistance upon injection occurs.

Pilocarpine is used as part of the diagnostic process of a toddler suspected of cystic fibrosis. The nurse knows that the pilocarpine would stimulate which process? A. Secretion of mucus B. Activity of sweat glands C. Excretion of pancreatic enzymes D. Release of bile from the gallbladder

B. Activity of sweat glands Pilocarpine is a cholinergic that is applied to the skin to stimulate sweat production; the sweat is then tested to confirm the diagnosis of cystic fibrosis. Pilocarpine does not stimulate the secretion of mucus, the excretion of pancreatic enzymes, or the release of bile from the gallbladder.

A client's phenytoin level is 16 mcg/L. Which action will the nurse take? A. Hold the medication and notify the health care provider. B. Administer the next dose of the medication as prescribed. C. Hold the next dose and then resume administration as prescribed. D. Call the health care provider to obtain a prescription with an increased dose.

B. Administer the next dose of the medication as prescribed. Administering the next dose of the medication as prescribed is within the therapeutic range of 10 to 20 mcg/L (40-80 mcmol/L); the nurse would administer the medication as prescribed. Holding the next dose and then resuming administration as prescribed is unsafe and will reduce the therapeutic blood level of the medication. Calling the health care provider to obtain a prescription with an increased dose is inappropriate because the blood level is within the therapeutic range.

A 9-month-old infant with iron-deficiency anemia has been getting supplements but shows no improvement. The nurse recognizes which action by the parents as the reason for the lack of improvement? A. Administering iron supplements through a straw B. Administering iron supplements with whole cow's milk C. Administering iron supplements along with orange juice D. Administering iron supplements at the back of the mouth

B. Administering iron supplements with whole cow's milk Whole cow's milk binds with free iron and reduces medication absorption. The infant has developed medication insufficiency for maximum therapeutic action. Administering iron supplements through a straw does not reduce medication absorption; it prevents the iron from staining the infant's teeth. Orange juice increases the absorption of iron supplements. Administering iron supplements at the back of the mouth does not reduce medication absorption; it prevents the iron from staining the infant's teeth.

The clinic nurse is planning care for a client with chlamydia. Which treatment would the nurse anticipate implementing? A. Administration of 250 mg of acyclovir orally in a single dose B. Administration of 1 g of azithromycin orally in a single dose C. Administration of 250 mg of ceftriaxone intramuscularly in a single dose D. Administration of 2.4 million units of benzathine penicillin G intramuscularly in a single dose

B. Administration of 1 g of azithromycin orally in a single dose The treatment of choice for chlamydial infection is 1 g of azithromycin orally in a single dose. The one-dose course is preferred because of its ease of completion. Acyclovir may be prescribed in a 7-day course for a genital herpes outbreak. Administering 250 mg of ceftriaxone intramuscularly in a single dose is the medication therapy recommended for gonorrhea. Benzathine penicillin G given intramuscularly as a single 2.4 million-unit dose is the treatment for primary, secondary, and early latent syphilis.

Which medication would the nurse anticipate developing a teaching plan for when a client reports becoming panicked and having an irrational fear of talking in public? A. Buspirone B. Alprazolam C. Diazepam D. Lorazepam

B. Alprazolam Alprazolam (a benzodiazepine) is a short-acting anxiolytic medication used to treat those clients with panic disorders and the irrational fear of talking openly in public (agoraphobia). Buspirone, an anxiolytic medication that is different both chemically and pharmacologically from the benzodiazepines, is always administered on a scheduled basis (not on an as-needed basis) for the treatment of anxiety. Diazepam is an anxiolytic medication commonly prescribed for the treatment of anxiety but has generally been replaced by short-acting benzodiazepines. Lorazepam is an intermediate-acting anxiolytic medication used in the treatment of acutely agitated clients.

The nurse is preparing to administer a prescribed dose of lactulose to a client who has cirrhosis. Which lab value will the nurse monitor to evaluate the therapeutic effect of the medication? A. Glucose B. Ammonia C. Potassium D. Bicarbonate

B. Ammonia Rationale: Hepatic encephalopathy is a manifestation of liver disease that has neurotoxic effects of ammonia. Lactulose acidifies feces in the intestines, which traps ammonia that can be then eliminated with defecation.

Which concern would prompt the nurse to contact the health care provider when a client with gastric ulcers receives a prescription for ranitidine 150 mg twice a day? A. Ranitidine can increase the risk for gastrointestinal bleeding. B. An administration route is not specified. C. Ranitidine is contraindicated for gastric ulcers. D. Ranitidine should be given with an adjuvant.

B. An administration route is not specified. It is necessary to clarify the route of administration because ranitidine can be given by mouth, intravenously, or intramuscularly; the health care provider's prescription is incomplete. Ranitidine does not increase the risk of gastrointestinal bleeding. Ranitidine is used to decrease gastric acid and is helpful for clients with a gastric ulcer. An adjuvant is not indicated.

Which medications would the nurse anticipate teaching the parents about for a preschooler newly diagnosed with cystic fibrosis? Select all that apply. One, some, or all responses may be correct. A. Steroids B. Antibiotics C. Antihistamines D. Pancreatic enzymes E. Fat-soluble vitamins

B. Antibiotics D. Pancreatic enzymes E. Fat-soluble vitamins Antibiotics are prescribed to treat recurrent respiratory tract infections. Thick secretions obstruct the pancreatic ducts, and essential pancreatic enzymes are blocked from reaching the duodenum; therefore pancreatic enzymes are administered with meals to aid digestion. Fat-soluble vitamins are necessary because of the decreased absorption of fat. Steroids are not indicated in the treatment of cystic fibrosis. Antihistamines are not used because of their drying effect on the already tenacious mucus.

When a client exhibits severe bradycardia, which type of medication will the nurse be prepared to administer? A. Nitrate B. Anticholinergic C. Antihypertensive D. Cardiac glycoside

B. Anticholinergic An anticholinergic medication will block parasympathetic effects, causing an increased heart rate. Nitrates will decrease preload, not increase the heart rate. Antihypertensive medications will lower the blood pressure and may decrease the heart rate. Cardiac glycoside will improve cardiac contractility but will decrease the heart rate.

The nurse is teaching a client with stable angina about their new prescription for nitroglycerin transdermal patch. Which instructions should the nurse include? Select all that apply. A. Remove the patch if ankle edema occurs B. Apply the patch to a hairless area of the body C. Notify your provider for persistent dizziness or any fainting episode D. Apply a second patch with chest pain E. Plan for patch-free time, usually overnight F. Rotate the application area

B. Apply the patch to a hairless area of the body C. Notify your provider for persistent dizziness or any fainting episode E. Plan for patch-free time, usually overnight F. Rotate the application area Nitroglycerin (NTG) acts directly on vascular smooth muscle to promote vasodilation. It decreases the pain of exertional angina primarily by decreasing cardiac oxygen demand. NTG comes in a variety of routes of administration. NTG patches contain a reservoir from which the drug is slowly released. Following release, the drug is absorbed through the skin and then into the blood. The rate of release is constant and, depending on the patch used, can range from 0.1 to 0.8 mg/ hr. Effects begin within 30 to 60 minutes and persist as long as the patch remains in place (up to 14 hours). Patches are applied once daily to a hairless area of skin. The site should be rotated to avoid local irritation. Tolerance develops if patches are used continuously (24 hours a day every day). Accordingly, a daily "patch-free" interval of 10 to 12 hours is recommended. This can be accomplished by applying a new patch each morning, leaving it in place for 12 to 14 hours, and then removing it in the evening. NTG can cause orthostatic hypotension and the client should let their provider know if dizziness and lightheadedness persist or the client has a fainting (syncopal) episode as these may indicate that the NTG dose needs to be adjusted/decreased. The other instructions are not appropriate for this medication.

Propylthiouracil and potassium iodide are prescribed for the client with hyperthyroidism. Which statement would the nurse include in the client's plan of care? A. Administer propylthiouracil and potassium iodide on an empty stomach. B. Assess the client for signs of infection and bleeding every shift. C. Stop the medications 2 weeks before thyroid surgery. D. Discontinue the medications if the heart rate is maintained within the expected range for 48 hours.

B. Assess the client for signs of infection and bleeding every shift. Propylthiouracil can cause depression of leukocytes and platelets. This creates an increased bleeding risk. Propylthiouracil and potassium iodide should be given with milk, juice, or food to prevent gastric irritation. Medication therapy decreases the risk of postoperative hemorrhage because this medication regimen decreases the size and vascularity of the thyroid gland. Medication therapy is continued for at least 6 to 8 weeks, even if the client's temperature and pulse return to the expected range.

The nurse is preparing to administer diltiazem to a client with heart disease. Which action should the nurse take first? A. Assess the client's lung sounds and monitor for wheezing B. Assess the client's blood pressure and apical pulse C. Assess the client's urine output and potassium level D. Auscultate the abdomen for bowel sounds

B. Assess the client's blood pressure and apical pulse Diltiazem is a calcium channel blocker that is used to treat hypertension, angina and tachyarrhythmias. The medication works by causing systemic vasodilation and lowering the client's heart rate. Common side effects of diltiazem include hypotension, orthostatic hypotension, bradycardia, edema and headaches. It is not necessary to auscultate the client's lung sounds prior to administering the medication. Wheezing is not considered a side effect of diltiazem. Because the medication can lead to hypotension and bradycardia, it is essential to assess the client's blood pressure and apical pulse prior to administration. It is not necessary to check the client's urine output or potassium level prior to administering the medication. Diltiazem does not affect a client's renal status or potassium level. It is not necessary to check the client's bowel sounds prior to administering the medication. Diltiazem does not affect a client's gastrointestinal system.

The healthcare provider prescribes digitalis (Digoxin) for a client diagnosed with heart failure. Which intervention should the nurse implement prior to administering the digoxin? A. Observe respiratory rate and depth. B. Assess the serum potassium level. C. Obtain the client's blood pressure. D. Monitor the serum glucose level.

B. Assess the serum potassium level. Hypokalemia (decreased serum potassium) will precipitate digitalis toxicity in persons receiving digoxin. The nurse should monitor the client's serum potassium levels. Blood pressure and respiratory rate will not inform the nurse about potential safety issues with digitalis.

) At 9 am, the nurse administers 10 units of insulin aspart subcutaneously to a client with a blood sugar of 322 mg/dL. At approximately what time should the nurse expect the insulin to peak? A. At 9:30 am B. At 10:00 am C. At noon D. This insulin does not peak because it acts over 24 hours.

B. At 10:00 am Insulin aspart is an analog of human insulin with a rapid onset (10 to 20 minutes), peak of 40 to 60 minutes and short duration (3 to 5 hours); therefore, the nurse should expect the insulin to peak at 10 am. The drug is structurally identical to human insulin. Insulin aspart (100 units/ mL) is supplied in 10 mL vials and 3 mL pre-filled pens and cartridges. Dosing is almost always done by subcutaneous injection or subcutaneous infusion with an insulin pump. Because insulin aspart acts rapidly, it is often used for sliding scale coverage and injections should be given 5 to 15 minutes before meals.

The nurse teaches an adolescent about administration of intermediate-acting insulin and regular insulin. Which response indicates the adolescent understands when to administer the second dose of NPH insulin? A. At lunch B. At dinnertime C. 1 hour after lunch D. 1 hour after dinner

B. At dinnertime The second dose of the intermediate-acting insulin should be given at dinnertime. NPH insulin peaks in 4 to 12 hours. A second dose is often prescribed approximately 10 to 12 hours after the first dose. A blood glucose reading at bedtime will determine the evening dose of regular insulin. A dose given at lunch is too early because it may precipitate a hypoglycemic reaction. A dose given 1 hour after lunch is also too early because it may precipitate a hypoglycemic reaction. A dose given 1 hour after dinner is too late.

Which ophthalmic solution is contraindicated for clients with glaucoma? A. Timolol B. Atropine C. Pilocarpine D. Epinephrine

B. Atropine Atropine, a mydriatic ophthalmic solution, is contraindicated for clients with glaucoma because it dilates the pupil, increasing intraocular pressure. Timolol, a beta blocker, decreases aqueous humor production; beta blockers are the preferred initial medications given to reduce intraocular pressure. Pilocarpine, a cholinergic, constricts the pupil, thereby increasing aqueous humor outflow. Epinephrine, an adrenergic agent, enhances aqueous humor outflow, thereby reducing intraocular pressure.

The nurse has administered fentanyl, atropine, cefazolin and benzocaine to a client for an endoscopic procedure. The nurse is monitoring the client and notes that the heart rate has increased from the pre-procedure baseline. The nurse knows that which of the following medications is most likely responsible for the client's increased heart rate? A. Fentanyl B. Atropine C. Cefazolin D. Benzocaine

B. Atropine Procedural sedation is used in endoscopic procedures as an effective way to provide an appropriate degree of pain and anxiety control; memory loss; and decreased awareness. The most commonly used medication regimen for gastrointestinal endoscopic procedure is still the combination of benzodiazepines, opioids, anticholinergics and topical anesthetics. Atropine is an anticholinergic drug that is used to dry secretions during the procedure. However, it can also increase the heart rate and dilate the pupils and is the most likely cause for the increased heart rate. Fentanyl is an opioid analgesic and short-term central nervous system (CNS) depressant and tends to slow breathing and lower heart rate and blood pressure. Benzocaine is a topical anesthetic and cefazolin is an antibiotic; neither should affect the heart rate.

A client is admitted to the hospital with a diagnosis of heart failure and acute pulmonary edema. The health care provider prescribes furosemide 40 mg intravenous (IV) stat to be repeated in 1 hour. Which nursing action will best evaluate the effectiveness of the furosemide in managing the client's condition? A. Performing daily weights B. Auscultating breath sounds C. Monitoring intake and output D. Assessing for dependent edema

B. Auscultating breath sounds Maintaining adequate gas exchange and minimizing hypoxia with pulmonary edema are critical; therefore assessing the effectiveness of furosemide therapy as it relates to the respiratory system is most important. Furosemide inhibits the reabsorption of sodium and chloride from the loop of Henle and distal renal tubule, causing diuresis; as diuresis occurs fluid moves out of the vascular compartment, thereby reducing pulmonary edema and the bilateral crackles. Although a liter of fluid weighs approximately 2.2 pounds (1 kg) and weight loss will reflect the amount of fluid lost, it will take time before a change in weight can be measured. Although identifying a greater output versus intake indicates the effectiveness of furosemide, it is the client's pulmonary status that is most important with acute pulmonary edema. Although the lessening of a client's dependent edema reflects effectiveness of furosemide therapy, it is the client's improving pulmonary status that is the best indicator of how furosemide improves the client's condition.

A primary health care provider prescribes atenolol 20 mg by mouth four times a day. Which information is important for the nurse to include in the discharge teaching plan for this client? A. Drink alcoholic beverages in moderation. B. Avoid abruptly discontinuing the medication. C. Increase the medication if chest pain develops. D. Report a pulse rate less than 70 beats/minute.

B. Avoid abruptly discontinuing the medication. An abrupt discontinuation of atenolol may cause an acute myocardial infarction. Alcohol is contraindicated for clients taking atenolol because it can cause additive hypotension. Clients should never increase medications without a health care provider's direction. The pulse rate can go much lower as long as the client feels well and is not dizzy.

Which instructions will the nurse include in the teaching plan for a client who will be taking simvastatin? Select all that apply. One, some, or all responses may be correct. A. Increase dietary intake of potassium. B. Avoid prolonged exposure to the sun. C. Schedule regular ophthalmic examinations. D. Take the medication at least half an hour before meals. E. Contact your health care provider if skin becomes gray-bronze.

B. Avoid prolonged exposure to the sun. C. Schedule regular ophthalmic examinations. E. Contact your health care provider if skin becomes gray-bronze. Simvastatin increases photosensitivity; the client should avoid sun exposure and use sunblock. The client should be monitored for the adverse effects of glaucoma and cataracts. Gray-bronze skin and unexplained muscle pain are signs of rhabdomyolysis. Rhabdomyolysis, a life-threatening response, is the disintegration of muscle associated with myoglobin in the urine. Simvastatin does not affect levels of potassium. The medication is most effective when taken at bedtime because cholesterol synthesis is highest at night.

The nurse is reviewing prescribed medications with a client. Which information should the nurse reinforce about captopril? A. Take the medication with meals. B. Avoid using salt substitutes. C. Restrict fluids to 1000 mL/day. D. Avoid green leafy vegetables.

B. Avoid using salt substitutes. Captopril is an angiotensin converting enzyme (ACE) inhibitor. It reduces aldosterone secretion, thereby reducing sodium and water retention. Captopril is used to treat hypertension and heart failure. Because it can cause an accumulation of serum potassium (i.e., hyperkalemia), clients should avoid the use of salt substitutes, which often contain potassium instead of sodium chloride. The other information does not apply to captopril.

The nurse understands which immunosuppressant medication interacts with allopurinol and may cause bone marrow suppression in children? A. Tacrolimus B. Azathioprine C. Cyclosporine D. Muromonab-DC3

B. Azathioprine When used with allopurinol, azathioprine may cause bone marrow suppression in children. Tacrolimus should be administered only after a careful assessment of a child's kidney functioning, history of past anaphylactic reactions, and availability of resuscitative equipment. The functioning level of various systems must be assessed before administering cyclosporine because this medication has a toxic effect on numerous organs. Baseline vital signs including weight should be assessed before administering muromonab-DC3 because of the potential risk for fluid retention.

Which B vitamin deficiency will result in Wernicke encephalopathy? A. B 3 (niacin) B. B 1 (thiamine) C. B 2 (riboflavin) D. B 6 (pyridoxine)

B. B 1 (thiamine) Severe deficiency of thiamine will result in Wernicke encephalopathy. Niacin deficiency causes pellagra. Riboflavin deficiency can result in cutaneous, oral, and corneal changes. Pyridoxine deficiency can progress to sideroblastic anemia, neurological disturbances, and xanthurenic aciduria, among other problems.

Daily regular insulin has been prescribed for a client with type 1 diabetes. The nurse administers the insulin at 8 AM. When will the nurse monitor the client for a potential hypoglycemic reaction? A. At breakfast B. Before lunch C. Before dinner D. In the early afternoon

B. Before lunch Regular insulin is short acting and peaks in 2 to 4 hours, which in this case will be at or before lunch. Breakfast is too soon; regular insulin peaks in 2 to 4 hours. Before dinner is too late; regular insulin peaks in 2 to 4 hours. The early afternoon is too late; regular insulin peaks in 2 to 4 hours.

When would the nurse have the laboratory obtain a blood sample to determine the peak level of an antibiotic administered by intravenous piggyback (IVPB)? A. Halfway between two doses of the medication B. Between 30 and 60 minutes after a dose C. Immediately before the medication is administered D. Anytime it is convenient for the client and the laboratory

B. Between 30 and 60 minutes after a dose Because the medication was administered by IV, the blood level of the medication will be at its highest shortly after administration. A medication blood level measured halfway between two doses will not obtain the peak level. Immediately before the medication is administered is done for a trough level, when the medication level is at its lowest. Anytime it is convenient for the client and the laboratory will produce inaccurate results; peak and trough levels are measured in relation to the time a medication is administered.

A client with myasthenia gravis has been receiving neostigmine and asks about its action. Which information would the nurse consider when formulating a response? A. Stimulates the cerebral cortex B. Blocks the action of cholinesterase C. Replaces deficient neurotransmitters D. Accelerates transmission along neural sheaths

B. Blocks the action of cholinesterase Neostigmine, an anticholinesterase, inhibits the breakdown of acetylcholine, thus prolonging neurotransmission. Neostigmine's action is at the myoneural junction, not the cerebral cortex. Neostigmine prevents neurotransmitter breakdown, but it is not a neurotransmitter. Neostigmine's action is at the myoneural junction, not the sheath.

A client with hypertensive heart disease who had an acute episode of heart failure is to be discharged on a regimen of metoprolol and digoxin. Which outcome would the nurse anticipate when metoprolol is administered with digoxin? A. Headaches B. Bradycardia C. Hypertension D. Junctional tachycardia

B. Bradycardia Metoprolol and digoxin both exert a negative chronotropic effect, resulting in a decreased heart rate. Metoprolol reduces, not produces, headaches. These medications may cause hypotension, not hypertension. These medications may depress nodal conduction; therefore junctional tachycardia would be less likely to occur.

The nurse is administering 40 mg of furosemide intravenously. Which sensation reported by the client would the nurse consider when determining that it is being administered too quickly? A. Full bladder B. Buzzing ears C. Fast heartbeat D. Numb arms and legs

B. Buzzing ears Rapid administration of furosemide can cause tinnitus (a perceived ringing or buzzing in the ears), loss of hearing, and ear pain. Furosemide has a diuretic effect, so a full bladder with a need to urinate is an anticipated response unrelated to speed of administration. Furosemide does not affect the heart rate. Furosemide does not cause peripheral neuropathy.

A client is prescribed cholestyramine for the treatment of type II hyperlipoproteinemia. Which vitamin would the nurse anticipate may become deficient because of this therapy? A. Niacin (vitamin B 3) B. Calciferol (vitamin D) C. Ascorbic acid (vitamin C) D. Cyanocobalamin (vitamin B 12)

B. Calciferol (vitamin D) Bile acid sequestrants (also known as bile acid-binding resins) bind with bile acids to form an insoluble compound that is then excreted in the feces. These medications decrease the absorption of fat-soluble vitamins (A, D, E, K). Vitamins B 3, C, and B 12 are water-soluble vitamins and are not affected by the administration of this medication.

Which vitamin would the nurse anticipate may become deficient in a client prescribed cholestyramine for the treatment of type II hyperlipoproteinemia? A. Niacin (vitamin B 3) B. Calciferol (vitamin D) C. Ascorbic acid (vitamin C) D. Cyanocobalamin (vitamin B 12)

B. Calciferol (vitamin D) Bile acid sequestrants (also known as bile acid-binding resins) bind with bile acids to form an insoluble compound that is then excreted in the feces. These medications decrease the absorption of fat-soluble vitamins (A, D, E, K). Vitamins B 3, C, and B 12 are water-soluble vitamins and are not affected by the administration of this medication.

Which mineral deficiency would a nurse suspect in a client who reports tingling in the fingers and around the mouth and exhibits carpopedal spasm and tremors after a surgical thyroidectomy ? A. Potassium B. Calcium C. Magnesium D. Sodium

B. Calcium The client is exhibiting signs and symptoms of hypocalcemia, which occurs with accidental removal of the parathyroid glands; calcium gluconate is administered to treat hypocalcemia. Deficits in potassium, magnesium, and sodium do not cause these classic manifestations.

Which education would the nurse provide the parents of an infant receiving the first diphtheria and tetanus toxoids and acellular pertussis vaccine (DTaP) at 2 months of age? A. Give the baby aspirin if there is pain. B. Call the clinic if marked drowsiness occurs. C. Apply ice to the injection site if there is swelling. D. Provide heat at the injection site if redness occurs.

B. Call the clinic if marked drowsiness occurs. Altered level of consciousness (such as marked drowiness) and seizures are rare but serious complications of the pertussis vaccine. Aspirin should not be given to infants and children because it is associated with Reye syndrome. Infants are sensitive to the application of ice. Heat will cause an extension of the inflammatory response and should be avoided.

The nurse is preparing to administer prescribed IV pantoprazole to the hospitalized client. The medication has been stocked in tablet form. Which action by the nurse is appropriate? A . Administer the medication to the client in oral form B. Call the pharmacy to stock the correct form of the medication C. Request that the healthcare provider change the order to tablets D. Ask the pharmacist if it is safe to give the client oral pantoprazole

B. Call the pharmacy to stock the correct form of the medication Rationale: Safety is of the utmost importance in preparing and administering medications. Suggested rights of administration vary from the classic five rights (listed first) through upward of eleven rights, including right medication, right patient, right dosage, right route, right time, right reason, right assessment data, right documentation, right response, right education, and right to refuse. In this case, the nurse has identified a potential issue with the right route of administration. The nurse should call the pharmacy and have the correct form of the medication provided for administration. Administering the medication by the incorrect route is considered a medication error. The healthcare provider's order is not incorrect, and the safety of administration via this route is not what is in question.

Which body system is affected by excessive use of amphetamine / dextroamphetamine? A. Renal B. Cardiac C. Musculoskeletal D. Gastrointestinal

B. Cardiac Norepinephrine is released when amphetamine/dextroamphetamine is taken. Excessive doses of amphetamine/dextroamphetamine can cause an increase in heart rate and dysrhythmias. Amphetamine/dextroamphetamine overuse does not directly affect the renal, musculoskeletal, or the gastrointestinal systems.

The nurse administers a parenteral preparation of potassium slowly to avoid which complication? A. Metabolic acidosis B. Cardiac arrest C. Seizure activity D. Respiratory depression

B. Cardiac arrest Too rapid an administration can cause hyperkalemia, which contributes to a long refractory period in the cardiac cycle, resulting in cardiac dysrhythmias and arrest. Although acidosis can cause hyperkalemia, hyperkalemia will not lead to acidosis. Hyperkalemia causes muscle flaccidity and weakness, not seizures. Respiratory depression can occur with too rapid intravenous (IV) magnesium administration, not potassium administration.

A client is receiving doxorubicin as part of a chemotherapy protocol. The nurse will assess the client for which system toxicity? A. Neurotoxicity B. Cardiotoxicity C. Ototoxicity D. Nephrotoxicity

B. Cardiotoxicity Heart failure and dysrhythmias secondary to cardiotoxicity are the primary life-threatening toxic effects unique to doxorubicin. Neurotoxicity, nephrotoxicity, and ototoxicity are not associated with this medication.

While taking a medical history, the client states, "I am allergic to penicillin." What related allergy to another type of antiinfective agent should the nurse ask the client about when taking the nursing history? A. Aminoglycosides. B. Cephalosporins. C. Sulfonamides. D. Tetracyclines.

B. Cephalosporins. According to research, there appears to be a cross sensitivity between penicillins and first generation cephalosporins; however, research shows there is no evidence of cross sensitivity between PCN and third or fourth generation cephalosporins.

Which action would the nurse take when a client arrives for an influenza vaccination and reports a low-grade fever with a cough? A. Administer aspirin with the vaccine. B. Check the temperature and current history. C. Hold the vaccine and notify the health care provider. D. Reschedule administration of the vaccine for the next month.

B. Check the temperature and current history. Vaccines may be administered during a mild febrile illness and upper respiratory infection, so the nurse would assess further. Administering aspirin is a dependent function of the nurse and requires a health care provider's prescription. Although holding the vaccine and administering it after the fever and cough are resolved is appropriate, notifying the health care provider is not necessary. Vaccinations should not be delayed unless the illness is moderate to severe.

A child with plumbism is prescribed edetate calcium disodium (calcium EDTA). Which assessment would be the most appropriate for the nurse to conduct before administering EDTA? A. Reviewing laboratory results for hypocalcemia B. Checking for protein in the urine C. Looking for signs of bone marrow depression D. Monitoring for increased intracranial pressure

B. Checking for protein in the urine Both lead toxicity and calcium EDTA damage the proximal renal tubules, resulting in increased excretion of protein and other substances. Hypocalcemia is attributable to only some chelating agents; however, it is not likely to occur with calcium EDTA, which replaces calcium. Bone marrow damage is caused by lead toxicity, not calcium EDTA. Lead encephalopathy, not calcium EDTA, causes an increase of intracranial pressure.

A nurse is reviewing new prescriptions for a client diagnosed with heart failure. The nurse notes captopril 25mg PO. Which action does the nurse perform next? A. Administer the medication before meals B. Clarify the prescription with the healthcare provider C. Take the client weight D. Check the client latest creatinine level

B. Clarify the prescription with the healthcare provider Rationale: The nurse should clarify the prescription with the healthcare provider. The prescription is missing a frequency, a necessary component of a medication prescription. Captopril should be administered before or after meals. However, the prescription does not have a frequency and should be clarified. Taking the client's weight and checking renal labs are important interventions after the prescription is clarified.

Which medication would the health care provider prescribe to treat the acute attack of gout and prevent future attacks? A. Ibuprofen B. Colchicine C. Probenecid D. Allopurinol

B. Colchicine High-dose colchicine decreases the inflammatory response to alleviate an acute gout attack; low-dose colchicine is used prophylactically to prevent future attacks. Ibuprofen is a nonsteroidal anti-inflammatory agent that may be used to treat an acute gout attack, but it does not possess activity to prevent future attacks. Probenecid increases uric acid excretion by the kidneys, making it useful for prophylaxis, but it does not possess the anti-inflammatory property needed to treat an acute gout attack. Allopurinol decreases uric acid formation, making it useful for prophylaxis, but it does not possess the anti-inflammatory properties needed to treat an acute gout attack.

A client is diagnosed at stage IV Hodgkin disease. Which therapy option is indicated? A. Radiation therapy B. Combination chemotherapy C. Radiation with chemotherapy D. Surgical removal of the affected nodes

B. Combination chemotherapy A protocol consisting of three or four chemotherapeutic agents that attack the dividing cells at various phases of development is the therapy of choice at this stage; alternating courses of different protocols generally are used. Radiation, alone or in combination with chemotherapy, is used in stages IA, IB, IIA, IIB, and IIIA. Radiation with chemotherapy is recommended for use in stage IIIA. Surgical removal of the affected nodes is not a therapy for Hodgkin disease at any stage. The nodes may be removed for biopsy or irradiated as part of therapy.

The nurse is preparing to administer prescribed digoxin to client with atrial fibrillation. The nurse notes the packaging for the medication is provided in a different route than prescribed. Which action should the nurse take? A. Administer the medication as ordered B. Consult the pharmacist regarding the error C. Alert the charge nurse to the medication error D. Alert the charge nurse to the medication error E .Contact the health care provider

B. Consult the pharmacist regarding the error Rationale: Careful consultation with a pharmacist regarding the error is the most appropriate action for the nurse to take if an error occurs when the pharmacy dispenses the medication. The medication as provided by the pharmacy is incorrect and cannot be administered. The charge nurse may be alerted, but the pharmacy can correct the error.

A client who is taking an oral hypoglycemic daily for type 2 diabetes develops an infection with anorexia. Which advice will the nurse provide to the client? Select all that apply. One, some, or all responses may be correct. A. Avoid solid food. B. Continue to take the oral medication. C. Drink fluids throughout the day. D. Monitor capillary glucose levels. E. Do not take medication until tolerating food.

B. Continue to take the oral medication. C. Drink fluids throughout the day. D. Monitor capillary glucose levels. Physiological stress increases gluconeogenesis, requiring continued pharmacological therapy despite an inability to eat; fluids prevent dehydration; monitoring of glucose levels permits early intervention if necessary. Skipping the oral hypoglycemic agent may precipitate hyperglycemia. Food intake will be attempted to prevent acidosis. Delaying an oral hypoglycemic agent may precipitate hyperglycemia.

Which client response must the nurse monitor to determine the effectiveness of amiodarone? A. Absence of ischemic chest pain B. Decrease in cardiac dysrhythmias C. Improvement in fasting lipid profile D. Maintenance of blood pressure control

B. Decrease in cardiac dysrhythmias. Amiodarone is a class III antidysrhythmic used for treating ventricular and supraventricular tachycardia and for conversion of atrial fibrillation. Results of fasting lipid profile are expected with antilipidemics. A degree of blood pressure control is expected with antihypertensives. Incidence of ischemic chest pain is expected with antianginal agents, such as nitrates.

The nurse administers lactulose to a client with cirrhosis of the liver. Which laboratory test change leads the nurse to determine that the lactulose is effective? A. Decreased amylase B. Decreased ammonia C. Increased potassium D. Increased hemoglobin

B. Decreased ammonia Lactulose destroys intestinal flora that break down protein and, in the process, give off ammonia. In clients with cirrhosis, ammonia is inadequately detoxified by the liver and can build to toxic levels. Amylase levels are associated with pancreatic problems. Increased potassium levels are associated with kidney failure. Hemoglobin is increased when the body needs more oxygen-carrying capacity, such as in smokers, or in high altitudes.

For which therapeutic effect will the nurse monitor the client who is prescribed alprazolam? A. Pain relief B. Decreased anxiety C. Reduction in dysrhythmias D. Reduced blood pressure

B. Decreased anxiety Alprazolam, an anxiolytic, promotes muscle relaxation, reduces anxiety, and facilitates rest. Possible adverse reactions to alprazolam are anger and hostility. Although drowsiness is a side effect of alprazolam caused by depression of central nervous system activity, it is not a hypnotic. Transient hypotension is a side effect of alprazolam, but this is not why it is given to an anxious client.

Which therapeutic effect would the nurse expect to identify when mannitol is administered to a client? A. Improved renal blood flow B. Decreased intracranial pressure C. Maintenance of circulatory volume D. Prevention of the development of thrombi

B. Decreased intracranial pressure As an osmotic diuretic, mannitol helps reduce cerebral edema. Although there may be a transient increase in blood volume as a result of an increased osmotic pressure, which increases renal perfusion, this is not the therapeutic effect. Prevention of the development of thrombi is not the reason for giving this medication.

A client diagnosed with multiple sclerosis is prescribed baclofen. Which assessment finding would indicate to the nurse that the client is experiencing a therapeutic response from the medication? A. Decreased Nausea B. Decreased muscle spasms C. Increased muscle tone and strength D. Increased range of motion to all extremities.

B. Decreased muscle spasms Rationale: Baclofen is a skeletal muscle relaxant and acts at the spinal cord level to decrease the frequency and amplitude of muscle spasms in clients with spinal cord injuries or diseases, or multiple sclerosis. None of the other options are related to the effects of this medications.

The nurse is evaluating the results of treatment with erythropoietin. Which assessment finding indicates an improvement in the underlying condition being treated? A. 2+ pedal pulses B. Decreased pallor C. Decreased jaundice D. 2+ deep tendon reflexes

B. Decreased pallor Erythropoietin stimulates red blood cell production, thereby decreasing the pallor that accompanies anemia. It would not have a role in alleviating jaundice. It would not have an appreciable effect on pulses or deep tendon reflexes.

A client has a tonic-clonic seizure. The nurse anticipates that the health care provider will prescribe the intravenous administration of which medication? A. Naloxone B. Diazepam C. Epinephrine HCl D. Atropine

B. Diazepam Parenterally administered diazepam is a benzodiazepine that has muscle relaxant and anticonvulsant effects that help limit massive muscular spasms. Naloxone does not limit seizures; it is an opioid antagonist and is used for opioid overdose. Epinephrine HCl does not limit seizures; it increases contractility of the heart. Atropine sulfate is not used for seizures. It is used for bradycardia resulting from vagal overstimulation.

The home care nurse is reviewing the medical record of a new client with a history of chronic obstructive pulmonary disease, atrial fibrillation and gout. After reviewing the client's medication list, for which medications should the nurse arrange to monitor blood levels? Select all that apply. A. Beclomethasone B. Digoxin C. Theophylline D. Allopurinol E. Montelukast

B. Digoxin C. Theophylline It is necessary to monitor blood levels for theophylline and digoxin to prevent toxicity. Both of those drugs can accumulate in the blood and reach toxic levels. The other medications are not known to accumulate and cause toxicity if taken as prescribed.

A client with cancer is receiving leucovorin as part of a chemotherapy protocol. Which purpose does leucovorin serve? A. Potentiating the effect of alkylating agents B. Diminishing toxicity of folic acid antagonists C. Limiting vomiting associated with chemotherapy D. Preventing alopecia

B. Diminishing toxicity of folic acid antagonists Leucovorin limits toxicity of folic acid antagonists, such as methotrexate sodium, by competing for transport into cells. Leucovorin does not potentiate the effect of alkylating agents. It does not have antiemetic properties. It will not prevent hair loss.

A client with heart failure is being discharged with a new prescription for the angiotensin-converting enzyme (ACE) inhibitor captopril (Capoten). The nurse's discharge instructions should include reporting which problem to the healthcare provider? A. Weight loss. B. Dizziness. C. Muscle cramps. D. Dry mucous membranes.

B. Dizziness. Angiotensin-converting enzyme (ACE) inhibitors are used in heart failure to reduce afterload by reversing vasoconstriction common in heart failure. This vasodilation can cause hypotension and resultant dizziness. Weight loss is desired if fluid overload is present, and may occur as the result of effective combination drug therapy such as diuretics with ACE inhibitors. It does not require reporting to the healthcare provider. Unlike ACE inhibitors, diuretics may result in hypokalemia and excessive diuretic administration may result in fluid volume deficit manifested by symptoms of dehydration.

The nurse observes bloody expectorant after a 4-year-old child with leukemia brushed his or her teeth. Which action should the nurse take next? A. Secure a smaller toothbrush for the child to use. B. Document and report the incident. C. Tell the child to be more careful when brushing the teeth. D. Rinse the child's mouth with half-strength hydrogen peroxide.

B. Document and report the incident. Because of the increased capillary fragility and decreased platelet count that accompany leukemia, even the slightest trauma can cause hemorrhage. Brushing the teeth has caused gingival bleeding, and the incident should be documented; this information may also help define the treatment plan. It is wiser to eliminate a toothbrush and use a sponge-type applicator. It cannot be assumed that a 4-year-old child will or can follow a direction to be more careful when brushing. Rinsing the child's mouth with half-strength hydrogen peroxide could irritate the gums, causing more trauma. If oral ulcers develop, the mouth should be rinsed with an isotonic solution such as normal saline.

A client with stage III Hodgkin's disease is started on a multiple-drug regimen of doxorubicin, bleomycin, vinblastine, and dacarbazine. Why are so many drugs necessary? A. Using smaller doses of several drugs reduces the likelihood of serious side effects. B. Each drug destroys the cancer cell at a different time in the cell cycle. C. Several drugs are used to destroy cells that are not susceptible to radiation therapy. D. Because there are stages of Hodgkin's disease, if one drug is ineffective, another will work.

B. Each drug destroys the cancer cell at a different time in the cell cycle. Cells are vulnerable to specific drugs through the stages of mitosis, and a combination bombards the malignant cells at various stages. The side effects of a drug are not ameliorated by a combination with others. Although the statement that several drugs are used to destroy cells that are not susceptible to radiation therapy is true, it is not the reason for using a combination of drugs. Although there is more than one stage of Hodgkin's disease, this is not the reason for using a combination of drugs.

Which intervention is indicated for treatment of a client admitted to the hospital with a diagnosis of acute salmonellosis? A. Cathartics B. Electrolytes C. Antidiarrheals D. Antispasmodics

B. Electrolytes Fluids of dextrose and normal saline and electrolytes are administered to prevent profound dehydration caused by excessive loss of water and electrolytes through diarrheal output. Cathartics are not necessary; salmonellosis is a condition that increases intestinal peristalsis, causing diarrhea. Antidiarrheals are not used when a bacterial infection is diagnosed because slowed peristalsis impedes the excretion of the microorganism. Antispasmodics are not used when bacterial infection is diagnosed because slowed peristalsis decreases excretion of the microorganism.

The nurse administers erythropoietin three times a week to a client receiving chemotherapy for cancer. Which client response demonstrates a therapeutic effect? A. Increase in band cells B. Elevated hematocrit C. Normalization of platelets D. Increase in the white blood cell (WBC) count

B. Elevated hematocrit Erythropoietin stimulates red blood cell production, thereby increasing the hematocrit and hemoglobin level. Erythropoietin increases red blood cells (RBCs), not WBCs, not platelets, and not immature neutrophils (band cells).

An infant is prescribed an antibiotic after cardiac surgery. Which instruction would the nurse emphasize to the parents regarding administration of the medication? A. Give the antibiotic between feedings. B. Ensure that the antibiotic is administered as prescribed. C. Shake the bottle thoroughly before giving the antibiotic. D. Keep the antibiotic in the refrigerator after the bottle has been opened.

B. Ensure that the antibiotic is administered as prescribed. Ensuring that the antibiotic is administered as prescribed is a priority because inadequate antibiotic therapy may predispose the infant to the development of bacterial endocarditis. Giving the antibiotic between feedings, shaking the bottle, and storing the medication in the refrigerator are not priority instructions because instructions often vary depending on the antibiotic.

Which medication would the nurse expect to administer to actively reverse the overdose sedative effects of benzodiazepines? A. Lithium B. Flumazenil C. Methadone D. Chlorpromazine

B. Flumazenil Flumazenil is the medication of choice in the management of overdose when a benzodiazepine is the only agent ingested by a client not at risk for seizure activity. Flumazenil competitively inhibits activity at benzodiazepine recognition sites on gamma-aminobutyric acid-benzodiazepine receptor complexes. Lithium is used in the treatment of mood disorders. Methadone is used for narcotic addiction withdrawal. Chlorpromazine is contraindicated in the presence of central nervous system depressants.

Which purpose is served by an evening snack of milk, crackers, and cheese for a client who is receiving NPH insulin? A. Encouragement to stay on the diet B. Food to counteract late insulin activity C. Added calories to promote weight gain D. High carbohydrates to provide nourishment for immediate use

B. Food to counteract late insulin activity The protein in milk and cheese is converted slowly to glucose (gluconeogenesis), providing the body with some glucose during sleep while the insulin is still acting. The purpose of an evening snack is to cover for insulin activity during sleep, not to encourage the client to stay on the diet. There are no data that indicate a need to gain weight. The foods chosen are rich in protein and are used slowly.

Hypertension develops in a school-age child with acute glomerulonephritis. Which medication would the nurse anticipate providing teaching for? A. Digoxin B. Furosemide C. Diazepam D. Phenytoin

B. Furosemide Furosemide inhibits the reabsorption of sodium and chloride from the loop of Henle and distal tubule, increasing urine output and thereby decreasing blood pressure. Digoxin increases the contractility and output of the heart; it is not an antihypertensive. Diazepam is inappropriate; it relaxes skeletal muscle, not the smooth muscle of the arterioles. Phenytoin is an anticonvulsant; it does not reduce blood pressure.

The nurse teaches the parents of a child with classic hemophilia how to administer plasma component factor VIII prescribed three times a week. Which instruction would the nurse give the parents about administration time? A. Whenever a bleed is suspected B. Give in the morning on scheduled days C. At bedtime while the child is lying quietly in bed D. On a regular schedule at the parents' convenience

B. Give in the morning on scheduled days Factor VIII has a short half-life; therefore prophylactic treatment involves administering the factor on the scheduled days in the morning, so the child will get the most benefit during the day, while he is most active. Prophylactic treatment is administered on a scheduled basis to prevent bleeds from occurring. Administering the medication at bedtime will limit its effectiveness because bleeds are more common when the child is active. Administering the medicine on a regular schedule at the parents' convenience does not take into consideration the properties of the medication.

A client prescribed atenolol has a blood pressure of 120/68 mmHg, displaying a sinus bradycardia with a rate of 58 beats/minute, and a P-R interval of 0.24. Which action should the nurse take? A. Lower the head of the bed and assess the client for orthostatic vital sign changes. B. Give the medication as prescribed and continue to monitor the client. C. Prepare to administer atropine sulfate IV push. D. Hold the prescribed dose and contact the healthcare provider.

B. Give the medication as prescribed and continue to monitor the client. Since the client's blood pressure is within normal limits, and the pulse is above 50 beats/min with a first degree block, the medication can be administered. Atenolol is a beta-blocker that slows the heart rate and lowers the blood pressure; this drug is generally held if the heart rate is less than 50 beats/min or the client exhibits dizziness related to hypotension.

Which food would the nurse instruct a client taking diltiazem to avoid? Select all that apply. One, some, or all responses may be correct. A. Alcohol B. Grapefruit juice C. Cheddar cheese D. Summer sausage E. Dark green vegetables

B. Grapefruit juice Clients taking calcium-channel blockers such as diltiazem would be instructed to avoid drinking grapefruit juice or eating grapefruit because it can interfere with metabolism of the medication. Clients taking acetaminophen would be instructed to avoid alcohol. Aged cheese and meat, such as sausage, should be avoided in clients taking monoamine oxidase inhibitors (MAOIs). Clients taking anticoagulants, such as warfarin, should avoid dark green vegetables.

The nurse is assisting a client who is taking amlodipine with meal planning. Which fluid selected by the client would require follow up by the nurse? A. Black coffee B. Grapefruit juice C. Green tea D. Chocolate Milk

B. Grapefruit juice Rationale: Grapefruit juice affects the metabolism of certain medications, such as amlodipine, and may cause toxicity if taken together. Clients who are taking antibiotics, such as tetracycline, should avoid consuming milk products. Clients who are taking warfarin should avoid consuming green tea. Clients who are taking stimulants should avoid consuming black coffee.

The nurse is caring for a client with Parkinson's disease. Which finding indicates that the client might be experiencing an adverse side effect from the dopamine-enhancing drugs? A. Urinary retention B. Hallucinations C. Kidney failure D. Hypertensive urgency

B. Hallucinations Carbidopa-levodopa-entacapone is the treatment of choice for clients with Parkinson's disease. Common side effects include dyskinesia, confusion and dizziness. Serious side effects include hallucinations, paranoia and agitation. Hallucinations may be relieved by decreasing the dose of levodopa, but this may decrease the effect of the drug on the motor symptoms of Parkinson's disease.

The nurse would instruct a client to stop taking an oral contraceptive and notify the health care provider immediately for the presence of which clinical findings? Select all that apply. One, some, or all responses may be correct. One, some, or all responses may be correct. A. Nausea B. Headaches C. Weight loss D. Visual disturbances E. Increased menstrual flow

B. Headaches D. Visual disturbances Headaches, either sudden or persistent, may indicate hypertension or a cardiovascular event. Visual disorders, such as partial or complete loss of vision or double vision, may indicate neuro-ocular lesions, which are associated with the use of some oral contraceptives. Nausea is a side effect that is not life threatening; the dose may need to be adjusted or the product changed to an alternative. Weight gain, not loss, may occur because of fluid retention. Menorrhagia is less likely to occur.

A client receives doxorubicin as part of a chemotherapy protocol. The nurse would assess the client for signs and symptoms of which adverse effect? A. Toxic epidermal necrolysis B. Heart failure C. Pulmonary fibrosis D. Ototoxicity

B. Heart failure Heart failure and dysrhythmias are life-threatening toxic effects unique to doxorubicin. It is a vesicant that can cause severe tissue damage if the medication infiltrates; however, this is different from the tissue destruction associated with toxic epidermal necrolysis. Pulmonary fibrosis and ototoxicity are not adverse effects of doxorubicin.

The anticholinesterase medication pyridostigmine is prescribed for the client with myasthenia gravis. When providing medication teaching, the nurse explains that the client should expect a decrease in which function? A. Bowel function B. Heart rate C. Skeletal muscle contraction D. Urinary frequency

B. Heart rate Anticholinesterase medications inactivate cholinesterase, the enzyme that mediates breakdown of acetylcholine. As a result of increased cholinergic activity, the heart rate will typically decrease. As a result of the increased cholinergic activity, the client can anticipate increased (not decreased) bowel movements and increased (not decreased) urinary frequency. Skeletal muscle contractions are increased (not decreased); this is the reason the medication is administered for myasthenia gravis.

A nurse is providing education to a client about newly prescribed diltiazem. Which statement will the nurse include in the teaching? A. Skip the dose if your systolic blood pressure is less than 120 mmHg B. Hold the dose if your heart rate is less than 50 beats/min C. Call your healthcare provider if you experience any fever D. Notify your healthcare provider if you notice any weight loss

B. Hold the dose if your heart rate is less than 50 beats/min Rationale: Diltiazem is a calcium channel blocker medication used in the treatment of hypertension and cardiac arrhythmias such as atrial flutter and fibrillation. Diltiazem can cause bradycardia. The nurse should instruct the client how to take their pulse and hold the dose if less than 50 beats/min. Diltiazem should be held if the systolic blood pressure is below 90 mmHg. Fever and weight loss are not effects associated with the use of diltiazem.

A hospitalized 8-month-old infant is receiving digoxin to treat Tetralogy of Fallot. Prior to administering the next dose of the medication, the parent reports that the baby vomited one time, just after breakfast. The infant's heart rate is 92 bpm. What action should the nurse take? A. Give the scheduled dose after the client is done eating lunch. B. Hold the medication and notify the primary health care provider. C. Reduce the next dose by half and then resume the normal medication schedule. D. Double the next dose to make up for the medication lost from vomiting.

B. Hold the medication and notify the primary health care provider. Toxic side effects of digoxin include bradycardia, dysrhythmia, nausea, vomiting, anorexia, dizziness, headache, weakness and fatigue. It isn't typically necessary to hold the medication for infants and children if there is only one episode of vomiting. However, it is appropriate to hold the medication and notify the primary health care provider (HCP) of the vomiting episode and the lower than normal heart rate. A digoxin level may need to be drawn. The normal resting heart rate for infants 1 to 11 months old is 100 to 160 bpm.

Valsartan, an angiotensin II receptor antagonist, is prescribed for a client. The nurse will monitor the client for which adverse effect? A. Constipation B. Hyperkalemia C. Hypertension D. Change in visual acuity

B. Hyperkalemia Hyperkalemia may occur with valsartan. Angiotensin II receptor antagonists, such as valsartan, block vasoconstrictor and aldosterone-producing effects of angiotensin II at receptor sites to decrease blood pressure. Hypotension, not hypertension, may occur. Diarrhea, not constipation, may occur with valsartan. Valsartan does not cause altered visual acuity.

A client is experiencing both tingling of the extremities and tetany. The nurse will review the client's laboratory report to check for which electrolyte abnormality? A. Hypokalemia B. Hypocalcemia C. Hyponatremia D. Hypochloremia

B. Hypocalcemia Paresthesias (tingling of the extremities) and tetany are signs of hypocalcemia. These are not expected findings for hypokalemia, hyponatremia, or hypochloremia.

A client with cirrhosis of the liver has been taking chlorothiazide. The provider adds spironolactone to the client's medication regimen to prevent which condition? A. Hyponatremia B. Hypokalemia C. Ascites D. Peripheral neuropathy

B. Hypokalemia Spironolactone is a potassium-sparing diuretic often used in conjunction with thiazide diuretics. The provider was prompted to add spironolactone to the chlorothiazide to prevent potassium loss. It stimulates sodium excretion so will not prevent hyponatremia. Spironolactone is a relatively weak diuretic that will not have a significant effect on ascites. Peripheral neuropathy is not a concern in this scenario and spironolactone would not have an effect on it if it was a concern.

An older adult client has been prescribed zolpidem for insomnia. The nurse should monitor the client for which side/adverse effect of this medication? A. Tachypnea B. Hypotension C. Tachycardia D. Constipation

B. Hypotension Rationale: Zolpidem is classified as a non-benzodiazepine and acts as a GABA agonist which can cause central nervous system depression, including drowsiness and lightheadedness. Nurses should be aware of the sedative effects and assess for hypotension, bradycardia, and bradypnea. Diarrhea, not constipation, is more commonly associated with the administration of this medication.

The nurse is caring for a client who has just received epidural anesthesia. The nurse would monitor for which adverse effects? A. Uterine atony B. Hypotension C. Decreased urine production D. Precipitous second stage of labor

B. Hypotension Regional anesthesia lowers the blood pressure, which puts both mother and fetus in jeopardy. The client may not have the sensation to void, but the amount of urine manufactured does not decrease because a regional block does not affect the kidneys. Epidural anesthesia does not shorten the second stage of labor and does not cause uterine atony.

A client presents to the emergency department with chest pain. A myocardial infarction is suspected, and 500 mL of 5% dextrose in water (D 5W) with 50 mg of nitroglycerin intravenously (IV) has been prescribed. The nurse will monitor the client for which common side effect of nitroglycerin? A. Bradycardia B. Hypotension C. Nausea and vomiting D. Leg cramps

B. Hypotension The major action of intravenous nitroglycerin is venous and then arterial dilation, leading to a decrease in blood pressure; orthostatic hypotension can occur. Bradycardia is not an anticipated response. Nausea and vomiting may occur but are not the most common side effects of IV nitroglycerin. Leg cramps are not a side effect of this medication.

A nurse is providing education on the use of pregabalin to a client with a seizure disorder. Which client statement indicates further teaching is required? A. I will record the number of seizures I experience B. I will hold the dose if my seizures are controlled C. I will notify my healthcare provider if I have significant mood changes D. I will report any weight gain to my healthcare provider

B. I will hold the dose if my seizures are controlled Rationale: Pregabalin is an anticonvulsant medication used to manage seizure disorders. The client should take the medication as prescribed as abrupt discontinuation can lead to seizure activity. Recording the number of seizures helps to evaluate the effectiveness of the medication. Pregabalin can cause suicidal thoughts and behaviors. The client should promptly report significant mood changes. Pregabalin can cause weight gain and peripheral edema. These side effects should be reported to the healthcare provider.

An 11-year-old client reports having bedwetting issues (enuresis). Which medication would the nurse anticipate when developing a teaching plan ? A. Alprazolam B. Imipramine C. Lithium salts d. Clomipramine

B. Imipramine Certain conditions of pediatric clients necessitate the usage of tricyclic antidepressant medications as an adjuvant. Childhood enuresis is one such condition that necessitates the administration of imipramine. Alprazolam is the medication of choice for treating anxiety disorders. Lithium salt is prescribed to treat bipolar disorders. Clomipramine is a tricyclic antidepressant medication prescribed for treating obsessive-compulsive disorder.

A peak and trough level is prescribed for a client receiving antibiotic therapy. When should the nurse should obtain the trough level? A. Sixty minutes after the antibiotic dose is administered. B. Immediately before the next antibiotic dose is given. C. Upon completion of the prescribed antibiotic regime. D. An hour before the next antibiotic dose is given.

B. Immediately before the next antibiotic dose is given. Trough levels are drawn when the blood level is at its lowest, which is typically just before the next dose is given.

A nurse is preparing to administer doxorubicin to a client with bladder carcinoma. How will the nurse prepare this medication? A. While wearing sterile gloves B. In a biological safety cabinet C. Inside a temperature-controlled room D. By withdrawing into a syringe undiluted

B. In a biological safety cabinet Rationale: Doxorubicin should be prepared in a biological safety cabinet. Doxorubicin is a high-risk medication whose fumes may cause health hazards. A biosafety cabinet controls the airflow while preparing this medication. Sterile gloves are not required. Standard gloves, a gown, and a mask should be worn when preparing this medication. The temperature of the room is not a specified guideline for preparing this medication. Doxorubicin should be diluted with normal saline before administration.

) A child with type 1 diabetes is receiving 15 units of regular insulin and 20 units of NPH insulin at 7:00 AM each day. Which time would the nurse anticipate a hypoglycemic reaction from the NPH insulin to occur? A. Before noon B. In the afternoon C. Within 30 minutes D. During the evening

B. In the afternoon NPH insulin is an intermediate-acting insulin that peaks approximately 6 to 8 hours after administration. It was administered at 7:00 AM, so between 1:00 PM and 3:00 PM is when the nurse would anticipate that a hypoglycemic reaction would occur. Noon is when a reaction from a short-acting insulin is expected. Short-acting insulin peaks 2 to 4 hours after administration. Within 30 minutes of administration is when a reaction from a rapid-acting insulin is expected. Rapid-acting insulin peaks 30 to 60 minutes after administration. During the evening or nighttime is when a reaction from a long-acting insulin is expected. Long-acting insulin has a small peak 10 to 16 hours after administration.

A nurse is providing care to a client in cardiogenic shock. The client is on a prescribed dopamine infusion at 10 mcg/kg/min with orders to titrate as needed. The latest blood pressure is 75/40 mmHg. Which action does the nurse perform next? A. Recheck the client blood pressure B. Increase the infusion rate to 12 mcg/kg/min C. Report the findings to the healthcare provider D. Decrease the infusion rate to 8 mcg/kg/min

B. Increase the infusion rate to 12 mcg/kg/min Rationale: The nurse should increase the dose of dopamine. The therapeutic goal of dopamine is to increase blood pressure and improve cardiac output. A blood pressure of 75/40 mmHg indicates the current rate is not effective. Rechecking the client's blood pressure is not necessary. The client is in cardiogenic shock and hypotension is an expected finding. The nurse can recheck the blood pressure after titrating the dose. Reporting the findings to the healthcare provider is important. However, the nurse should first titrate the dose to ensure the client's blood pressure is maintained. Decreasing the infusion rate will cause further hypotension.

A client presents to the clinic for a follow-up appointment after starting pyridostigmine for management of myasthenia gravis. Which new client problems are adverse effects of pyridostigmine? Select all that apply. One, some, or all responses may be correct. A. Respiratory depression B. Increased urinary frequency C. Diplopia D. Muscle twitching E. Diarrhea

B. Increased urinary frequency D. Muscle twitching E. Diarrhea Pyridostigmine is an acetylcholinesterase inhibitor. By inhibiting the enzyme that breaks down acetylcholine, it increases cholinergic activity. The increased cholinergic activity is responsible for the most common adverse effects of pyridostigmine such as diarrhea and increased urinary frequency. Another adverse effect is muscle twitching that occurs as pyridostigmine affects the muscle weakness that is characteristic of myasthenia gravis. Respiratory depression is a clinical manifestation of myasthenia gravis that can occur due to respiratory muscle weakness. Weakness of eye muscles results in ptosis or diplopia in about half of clients with myasthenia gravis.

Which topic will the nurse include in the discharge teaching of a client who has had a total gastrectomy? A. Daily use of a stool softener B. Injections of vitamin B 12 for life C. Monthly injections of iron dextran D. Replacement of pancreatic enzymes

B. Injections of vitamin B 12 for life Intrinsic factor is lost with removal of the stomach, and vitamin B 12 is needed to maintain the hemoglobin level and prevent pernicious anemia. Adequate diet, fluid intake, and exercise should prevent constipation. Iron-deficiency anemia is not expected. Secretion of pancreatic enzymes should not be affected because this surgery does not alter this function.

Which intervention would the nurse include in the plan of care for a client with breast cancer who received doxorubicin and cyclophosphamide 12 days ago and now has a white blood cell (WBC) count of 1.4 cells/mm 3 and reports shortness of breath and activity intolerance? Select all that apply. One, some, or all responses may be correct. A. Use an electric razor when shaving. B. Institute neutropenic precautions. C. Place client on airborne precautions. D. Transfuse two units of packed red blood cells (RBCs). E. Instruct nursing staff to wear a dosimeter badge.

B. Institute neutropenic precautions. Doxorubicin and cyclophosphamide can lower the client's blood cell counts. Clients with low WBC counts need interventions to prevent infection, which include instituting neutropenic precautions. The nurse would instruct the client to use an electric razor if the platelet count was less than 50,000 cells/µL. Airborne precautions would be indicated if the client was ill with an infectious disease. The nurse would transfuse RBCs for a client with anemia (if prescribed by the health care provider). Nursing staff would wear dosimeter badges when caring for a client receiving internal radiation (brachytherapy).

A client with diabetes mellitus is scheduled to receive an intravenous (IV) administration of 25 units of insulin in 250 mL normal saline. Which type of insulin would the nurse recognize as compatible with IV solutions? A. NPH insulin B. Insulin lispro C. Insulin detemir D. Insulin glargine

B. Insulin lispro Insulin lispro is compatible with IV solutions; it is a rapid-acting insulin. Insulin glargine is not compatible with IV solutions; it is a long-acting insulin. NPH insulin is not compatible with IV solutions; it is an intermediate-acting insulin.

Which insulin would the nurse conclude has the fastest onset of action? A. NPH insulin B. Insulin lispro C. Regular insulin D. Insulin glargine

B. Insulin lispro3 Insulin lispro has an onset of 0.25 hours, a peak action of 0.5 to 1.5 hours, and a duration of 3 to 4 hours. Neutral protamine Hagedorn (NPH) or intermediate-acting insulin has an onset of 1.5 hours, a peak action of 4 to 12 hours, and a duration of 18 to 24 hours. Regular insulin has an onset of 0.5 hours, a peak action of 1 to 5 hours, and a duration of 6 to 10 hours. Insulin glargine has an onset of 1 to 1.5 hours, no peak action, and a duration of 20 to 24 hours.

Which parameter would the nurse monitor to evaluate the effectiveness of desmopressin acetate (DDAVP) administered to a client with diabetes insipidus? A. Arterial blood pH B. Intake and output C. Fasting serum glucose D. Pulse and respiratory rates

B. Intake and output DDAVP replaces antidiuretic hormone, facilitating the reabsorption of water and the consequent return of balanced fluid intake and urinary output. The mechanisms that regulate pH are not affected. DDAVP does not alter serum glucose levels; diabetes mellitus, not diabetes insipidus, results in hyperglycemia. Although the correction of tachycardia is consistent with the correction of dehydration, the client is not dehydrated if the fluid intake is adequate; respirations are unaffected.

Which birth control agent requires administration once every 3 months? A. Progestin-only pills B. Intramuscular progestin C. Combination biphasic forms D. Combination monophasic forms

B. Intramuscular progestin Intramuscular progestin requires the administration of a single injection every 3 months. Progestin-only oral contraceptive pills should be taken on a daily basis because of a higher incidence of ovulatory cycles. Combination biphasic and monophasic forms are administered as either a 21- or 28-day course.

A client with upper gastrointestinal (GI) bleeding develops mild anemia. Which agent is indicated for treatment of this condition? A. Dextran B. Iron salts C. Vitamin B 12 D. Erythropoietin

B. Iron salts Iron salts are needed in the formation of hemoglobin, so iron that is lost through bleeding must be replaced. Erythropoietin increases red blood cell (RBC) production, but the client's anemia is caused by GI bleeding, not impaired RBC production. Dextran is a plasma volume expander; it does not affect erythrocyte production. Vitamin B 12 is a water-soluble vitamin that must be used as a supplement when an individual has pernicious anemia.

Which mechanism of action explains how propylthiouracil (PTU) manages hyperthyroidism? A. It binds previously formed thyroid hormones. B. It decreases production of thyroid hormones. C. Vascularity of the thyroid gland is decreased. D. The need for thyroid iodine supplements is reduced.

B. It decreases production of thyroid hormones. PTU is a thyroid hormone antagonist that inhibits thyroid hormone synthesis by decreasing the use of iodine in the manufacture of these hormones. PTU does not affect the vascularity of the thyroid gland. Iodine-containing agents are given for severe hyperthyroidism and before a thyroidectomy. PTU does not affect the amount of already formed thyroid hormones.

The client wants to know why midazolam will be administered preoperatively. Which reason would the nurse provide? A. It reduces pain. B. It induces sedation. C. It prevents respiratory depression. D. It limits oral secretions.

B. It induces sedation. Midazolam, a benzodiazepine, reduces anxiety and induces sedation. Analgesics are given to reduce pain. It does not prevent respiratory depression and has been known to promote apnea. Atropine, an anticholinergic, is given to decrease oral and respiratory secretions.

The health care provider has prescribed tetracycline for a 28-year-old female client with severe acne. When teaching the client about this medication, which information is important for the nurse to include? A. It may cause staining of the teeth. B. It may decrease the effectiveness of oral contraceptives. C. It should be taken with food or milk. D. It may cause hearing loss.

B. It may decrease the effectiveness of oral contraceptives. Tetracycline, a broad-spectrum antibiotic, can decrease the effectiveness of oral contraceptives; therefore, it is important to recommend use of an additional form of contraception such as a condom when taking this medication. Tetracycline should be taken on an empty stomach and never with milk. It is not given to children younger than 8 years old because it can stain developing teeth. Tetracycline is not known to cause hearing loss.

A nurse is administering vincristine to a client with cancer. The client asks the nurse how the medication works. Which statement by the nurse is appropriate? A. It stops the synthesis of proteins in cancer cells B. It prevents cell division of cancer cells C. It interrupts the S-phase of cancer cell reproduction D. It alters the DNA structure of cancer cells

B. It prevents cell division of cancer cells Rationale: Antimitotics, such as vincristine, kill cancerous cells by inhibiting cell division and mitosis. Stopping the synthesis of proteins in cancer cells is the expected action of antitumor antibiotics. Interruption of the S-phase of cell reproduction is the expected action of antimetabolites. Altering the DNA structure of cancer cells is the expected action of alkylating agents.

The nurse administers a tube of glucose gel to a client who is hypoglycemic. Which explanation would the nurse share regarding the reversal of hypoglycemia? A. It liberates glucose from hepatic stores of glycogen. B. It provides a glucose source that is rapidly absorbed. C. Insulin action is blocked as it competes for tissue sites. D. Glycogen is supplied to the brain as well as other vital organs.

B. It provides a glucose source that is rapidly absorbed. The glucose gel provides a simple sugar for rapid use by the body. Liberating glucose from hepatic stores of glycogen is related to the action of glucagon. It is a medication that mobilizes glycogen storage in the liver, leading to an increased blood glucose level. Glucose does not compete with insulin. Glucose gel does not supply glycogen to the brain and other vital organs.

The nurse is providing instructions to a client with a new prescription for levothyroxine 50 mcg daily to treat hypothyroidism. Which of the following is important for the nurse to include in the discharge instructions? A. It can be taken with an antacid if stomach upset occurs. B. It should be taken in the morning. C. It must be stored in a dark container. D. It may decrease the client's energy level.

B. It should be taken in the morning. A thyroid supplement should be taken in the morning on an empty stomach with 8 ounces of water to maximize effects. Also, the client should avoid foods high in fiber, iron or soybeans within four hours of taking this medication because they may interfere with this drug's absorption. The medication should not be given in the evening or prior to bedtime because it may cause insomnia. It is not necessary to keep in a dark container. As the medication replaces thyroid hormone the client's energy level should be improved not decreased.

Chemotherapy via regional perfusion is the treatment of choice for a client's malignant sarcoma of the liver. Which reason would the nurse provide to explain to the client why this method of medication administration probably was selected? A. Medication therapy can be continued at home with little difficulty. B. Larger doses of medications can be delivered to the actual site of the tumor. C. Toxic effects of the chemotherapeutic medications are confined to the area of the tumor. D. Combinations of medications are used to attack neoplastic cells at various stages of the cell cycle.

B. Larger doses of medications can be delivered to the actual site of the tumor. Regional perfusion therapy permits relative isolation of the tumor area and saturation with the medication(s) selected. This method of medication administration requires medical and nursing supervision and cannot be continued at home. Although toxic effects are confined mainly to the treated area, some migration may still occur. Combinations of chemotherapeutic medications are administered via intravenous or oral routes, not via regional perfusion.

A client with Parkinson's disease is taking carbidopa-levodopa (Sinemet). Which observation by the nurse would indicate that the desired outcome of the medication is being achieved? A. Decreased blood pressure. B. Lessening of tremors. C. Increased salivation. D. Increased attention span.

B. Lessening of tremors.. Sinemet increases the amount of levodopa to the CNS (dopamine to the brain). Increased amounts of dopamine improve the symptoms of Parkinson's, such as involuntary movements, resting tremors, shuffling gait, etc. Decreased drooling would be a desired effect, not increased salivation.

A client with multiple myeloma who is receiving the alkylating agent melphalan returns to the oncology clinic for a follow-up visit. For which adverse effect will the nurse monitor the client? A. Hirsutism B. Leukopenia C. Constipation D. Photosensitivity

B. Leukopenia Melphalan depresses the bone marrow, causing a reduction in white blood cells (leukopenia), red blood cells (anemia), and thrombocytes (thrombocytopenia); leukopenia increases the risk of infection. Hirsutism occurs with the administration of androgens to women. Diarrhea, not constipation, occurs with melphalan. Photosensitivity occurs with 5-fluorouracil, floxuridine, and methotrexate, not with melphalan.

A client with follicular non-Hodgkin's lymphoma is to be treated with rituximab, a targeted monoclonal antibody. The nurse will monitor the client for which common side effect of rituximab? A. Polyphagia B. Leukopenia C. Constipation D. Hypertension

B. Leukopenia Rituximab targets the CD 20 antigen, which regulates cell cycle differentiation and is found on malignant B lymphocytes; as a result, rituximab therapy can cause leukopenia and neutropenia. Anorexia, not polyphagia, may occur with rituximab therapy. Frequent stools and diarrhea, not constipation, may occur with rituximab therapy. Hypotension, not hypertension, may occur as a fatal infusion reaction to rituximab therapy.

A nurse is providing care to a client diagnosed with a myocardial infarction. The client has a history of hypothyroidism and hypertension. Which prescribed medication will the nurse clarify before administering it to the client? A. Morphine B. Levothyroxine C. Aspirin D. Labetalol

B. Levothyroxine Rationale: Levothyroxine is a synthetic thyroid hormone used in the treatment of hypothyroidism. Levothyroxine can induce cardiac stimulant effects and is contraindicated in clients with a recent myocardial infarction (MI). Morphine and aspirin are commonly administered after a cardiac event. Morphine relieves pain associated with cardiac ischemia and aspirin decreases platelet aggregation that leads to blood clotting. Labetalol is a beta-blocker used in the treatment of hypertension. There is no known contraindication for the use of labetalol after an MI.

The nurse is caring for a client with diabetes mellitus. The client reports feeling hungry and thirsty. The client's most recent blood glucose level was 175 mg/dL. Which type of insulin should the nurse anticipate being prescribed for this client? A. Glucagon B. Lispro C. Exenatide D. Sitagliptin

B. Lispro The inpatient client with an elevated blood sugar is usually prescribed a short-acting insulin such as lispro, aspart or regular (Humulin-R) insulin. Glucagon is a medication used to treat hypoglycemia, not hyperglycemia. Exenatide and sitagliptin are not insulins.

The health care provider prescribes lidocaine to treat a ventricular dysrhythmia in a client with cirrhosis of the liver. Which alterations in the usual lidocaine dosage would the nurse anticipate for this client? A. Higher than usual dosage to compensate for the impaired liver function B. Lower than usual dosage because the medication is metabolized at a diminished rate C. Reduced dosage because other organs will compensate for the sluggish liver D. Equal dosage to that needed for other clients but used over a shorter duration

B. Lower than usual dosage because the medication is metabolized at a diminished rate Less than the usual adult dose will be prescribed because the liver will not be able to break down lidocaine as effectively as necessary. A dose higher to compensate for the impaired liver function increases the concentration of lidocaine in the blood, leading to toxicity. Lidocaine is metabolized by the liver; other organs cannot assist in the process. This may be life threatening because the client cannot metabolize lidocaine at the required rate, and toxicity may result.

Which action would the nurse implement for a client who has overdosed on barbiturates? Select all that apply. One, some, or all responses may be correct. A. Apply ice packs. B. Maintain airway. C. Administer naloxone. D. Give activated charcoal. E. Give intravenous fluids.

B. Maintain airway. D. Give activated charcoal. E. Give intravenous fluids. Due to the adverse effect of respiratory depression, the nurse will administer oxygen and maintain the airway. Any unabsorbed medication can be removed by giving the client activated charcoal. To eliminate any barbiturate that has been absorbed, the nurse would administer intravenous fluids to alkalinize the urine. Naloxone is given to reverse the effects of opioid overdoses. The nurse will provide warm blankets, not ice packs, to clients who have overdosed.

Which parent education would the nurse give about why the MMR vaccine is administered at 12 to 15 months of age? A. There is an increased risk of side effects in infants. B. Maternal antibodies provide immunity for about 1 year. C. It interferes with the effectiveness of vaccines given during infancy. D. There are rare instances of these infections occurring during the first year of life.

B. Maternal antibodies provide immunity for about 1 year. Maternal antibodies to measles, mumps, and rubella infection persist in the infant until approximately 15 months of age. Side effects are no more common among infants than in toddlers. The measles vaccination does not interfere with the effectiveness of other vaccines. Although the measles, mumps, and rubella do occasionally occur after the administration of the MMR vaccine during the first year of life, the vaccine is not given during this time because of the presence of maternal antibodies.

The client diagnosed with heart failure is prescribed oral digoxin. What is the priority nursing assessment for this medication? A. Monitor serum electrolytes and creatinine B. Measure apical pulse prior to administration C. Maintain accurate intake and output ratios D. Monitor blood pressure every 4 hours

B. Measure apical pulse prior to administration Digoxin is an antiarrhythmic and an inotropic drug. It works to increase cardiac output and slow the heart rate. The priority assessment is to measure the apical pulse for one minute prior to administering the drug. The nurse will withhold the dose and notify the healthcare provider if the apical rate is less than 60 beats per minute. Intake and output ratios and daily weights should be monitored for a client in heart failure, but this is not the priority assessment. Impaired renal function may contribute to drug toxicity, which is why the nurse will monitor serum electrolytes, creatinine and BUN; the nurse should also monitor serum digoxin levels.

A client receiving chemotherapy develops bone marrow suppression. The nurse will monitor for which thrombocytopenic effect? Select all that apply. One, some, or all responses may be correct. A. Deep vein thrombosis B. Melena C. Purpura D. Emboli E. Hematuria

B. Melena C. Purpura E. Hematuria Black, tarry feces caused by the action of intestinal secretions on blood are associated with bleeding in the gastrointestinal tract; bleeding is related to a reduced number of thrombocytes, which are part of the coagulation process. Hemorrhages into the skin and mucous membranes (purpura) may occur with reduced numbers of thrombocytes, which are part of the coagulation process. Blood in the urine (hematuria) may occur with a reduced number of thrombocytes, which are part of the coagulation process. Deep vein thrombosis and emboli are effects of thrombocytosis.

The nurse teaches a client about the dangers of using sodium bicarbonate regularly. Which effect of sodium bicarbonate is the nurse trying to prevent? A. Gastric distention B. Metabolic alkalosis C. Chronic constipation D. Cardiac dysrhythmias

B. Metabolic alkalosis Prolonged use of sodium bicarbonate may cause systemic alkalosis, as well as retention of sodium and water. Gastric distention is not an effect of sodium bicarbonate. Chronic constipation is not an effect of sodium bicarbonate. Cardiac dysrhythmias are not an effect of sodium bicarbonate.

A client develops hemolytic anemia. Which client medication can cause this adverse effect? A. Famotidine B. Methyldopa C. Levothyroxine D. Ferrous sulfate

B. Methyldopa Methyldopa is associated with acquired hemolytic anemia and should be discontinued to prevent progression and complications. Famotidine will not cause these symptoms; it decreases gastric acid secretion, which will decrease the risk of gastrointestinal bleeding. Ferrous sulfate is an iron supplement to correct, not cause, anemia. Levothyroxine is not associated with red blood cell destruction.

Which medications would the nurse identify as commonly used as an adjunct during conscious sedation for minor surgeries? A. Diazepam B. Midazolam C. Lorazepam D. Clonazepam

B. Midazolam When used in conjunction with an opioid analgesic, midazolam causes conscious sedation, which is a semiconscious state suitable for minor surgeries and endoscopic procedures. Diazepam and lorazepam are used for anesthetic induction, preanesthetic medication, and the production of conscious sedation when used in conjunction with opioid analgesics. Clonazepam is used to treat seizures and anxiety.

Which medication for treatment of gastroesophageal reflux disease would be contraindicated in the pregnant client? A. Ranitidine B. Misoprostol C. Esomeprazole D. Calcium carbonate

B. Misoprostol Misoprostol is contraindicated in pregnancy because it can cause uterine contractions, expelling the developing fetus. Ranitidine, esomeprazole, and calcium carbonate are not contraindicated during pregnancy.

Which medication is indicated to treat shift-work sleep disorder (SWSD)? A. Caffeine B. Modafinil C. Atomoxetine D. Methylphenidate

B. Modafinil Modafinil is a unique nonamphetamine stimulant used to treat SWSD. This medication promotes wakefulness in clients suffering from excessive sleepiness associated with SWSD. Caffeine is a central nervous stimulant used to promote wakefulness, but this medication is not as effective in the treatment of SWSD. Atomoxetine is a nonstimulant used to treat attention-deficit/hyperactivity disorder (ADHD). Methylphenidate is considered a first-choice medication for the treatment of ADHD.

The nurse is planning care for a pediatric client with a new prescription for adenosine to treat symptomatic supraventricular tachycardia (SVT). Which action should the nurse include in the plan of care? A. Monitor for ventricular dysrhythmias B. Monitor for shortness of breath C. Monitor for hypertension D. Monitor for nausea.

B. Monitor for shortness of breath Rationale: After giving adenosine, the nurse would monitor for shortness of breath, dyspnea, and a worsening of asthma, as they are expected effects/outcomes with this medication. Monitoring for ventricular dysrhythmias is necessary when giving dobutamine, dopamine, and epinephrine but not adenosine. Vomiting is not an expected outcome of adenosine. The nurse should include monitoring for hypotension, not hypertension, in the plan of care after administration of adenosine and instruct parents to change positions slowly to minimize orthostatic hypotension.

The oncology nurse is preparing to administer the initial dose of vincristine to a child diagnosed with acute lymphocytic leukemia. Which interventions should the nurse include in the plan of care? Select all that apply. A. Apply pressure to the injection site if extravasation occurs. B. Monitor liver function tests regularly. C. Monitor for numbness or tingling in the fingers and toes. D. Select the appropriate catheter for intrathecal administration. E. Verify blood return before, during and after intravenous administration.

B. Monitor liver function tests regularly. C. Monitor for numbness or tingling in the fingers and toes. E. Verify blood return before, during and after intravenous administration. Acute lymphocytic leukemia (ALL) is the most common type of cancer in children and treatment protocols include vincristine, an anticancer drug. Vincristine is for intravenous use only; intrathecal (i.e., spinal) administration can be fatal. Vincristine is a vesicant that can cause significant local damage if extravasation occurs; treatment includes subcutaneous injection of an antidote and warm compresses (topical cooling may worsen the effect). Peripheral neuropathy is a major side effect associated with vincristine. The nurse should monitor for decreased hepatic functioning because vincristine is metabolized in the liver.

When a client with type 1 diabetes develops heart failure, digoxin is prescribed. Which nursing action is important to include when planning care? A. Administer the digoxin 1 hour after the client's morning insulin. B. Monitor the client for cardiac dysrhythmias. C. Monitor for increased risk of hyperglycemia. D. Increase digoxin dosage if insulin requirements are increased.

B. Monitor the client for cardiac dysrhythmias. The speed of conduction is decreased when digoxin is given, and this can result in a variety of cardiac dysrhythmias. The risk for hyperglycemia is not increased. Administration times for insulin and digoxin do not have to be coordinated. Dosage of digoxin is not dependent on insulin dosage.

A client with a diagnosis of anemia is receiving packed red blood cells. Which nursing action is important when administering the transfusion? A. Assessing the client for fluid overload B. Monitoring the client's response, particularly within the first 10 minutes C. Assuring that the transfusion flows at a consistent rate during the procedure D. Having the client tested for human immunodeficiency virus (HIV) before administering the blood transfusion

B. Monitoring the client's response, particularly within the first 10 minutes Transfusion reactions usually occur early during the administration of a blood transfusion (first 30 mL of blood); early detection of a transfusion reaction will permit a quick termination of the infusion. The risk of fluid overload is unlikely, and this information can be frightening. The donor's, not the recipient's, blood is tested for HIV. The flow rate will be slower during the first 10 to 15 minutes of the infusion to limit the amount of blood infused; this allows time to assess the client's response for signs and symptoms of a transfusion reaction before too much of the blood is infused.

Which nursing action is the priority when administering chelation therapy for a toddler? A. Assessing vital signs B. Monitoring urine output C. Conducting a behavioral assessment D. Providing education to reduce lead exposure

B. Monitoring urine output Adequate urinary output must be ensured with administration of calcium EDTA, the medication used for chelation therapy. Children receiving the medication intramuscularly must be able to maintain adequate oral intake of fluids. Monitoring vital signs, conducting a behavioral assessment, and providing education to reduce lead exposure are not priority nursing actions when administering chelation therapy.

The nurse is preparing to administer metoprolol to a client with a history of hypertension. Which of the following data is the priority for the nurse to review prior to administration? A. Potassium level B. Most recent heart rate C. Creatinine level D. Respiratory rate

B. Most recent heart rate Rationale: Beta-blockers, such as metoprolol, can decrease heart rate and blood pressure, so the nurse should review these specific vital signs prior to administering the medication. Most prescriptions will state to hold the medication if the heart rate or blood pressure is less than a designated value. Potassium and creatinine levels are monitored with clients who are taking lisinopril, an ACE inhibitor. Respiratory rate is an important part of assessment but is not the priority for the administration of a beta-blocker.

The client is discharged from the hospital with a new prescription for furosemide. During a follow-up visit one week later, the nurse notes the following findings. Which finding is most important to report to the health care provider? A. Constipation B. Muscle cramps C. Occasional lightheadedness D. Increased urine production

B. Muscle cramps Furosemide is a loop (potassium-wasting) diuretic. It can cause dehydration and hypokalemia, which can result in muscle cramps. This is the most important finding. Dizziness or lightheadedness may occur as the body adjusts to the medication. The nurse should reinforce to the client that they should get up slowly when rising from a sitting or lying position. The client should tell the HCP if these findings persist or become worse. Increased urine production is an expected action of the medication. Some people experience constipation when taking this medication, but it is not as important to report that finding as the possibility of hypokalemia.

The nurse is reviewing medication instructions with a client who is taking digoxin. The nurse should reinforce to the client to report which of the following side effects? A. Rash, dyspnea, edema B. Nausea, vomiting, fatigue C. Hunger, dizziness, diaphoresis D. Polyuria, thirst, dry skin

B. Nausea, vomiting, fatigue Digoxin is considered an antidysrhythmic and inotrope, that is used to treat atrial dysrhythmias and congestive heart failure. The medication produces a positive inotropic effect, prolongs the refractory period and slows conduction through the sinoatrial (SA) and atrioventricular (AV) nodes. Overall, digoxin increases cardiac output and slows the heart rate. The effects of digoxin produce many side effects and clients who take digoxin are at risk for digoxin toxicity. Because digoxin improves cardiac output, side effects of the medication would not include dyspnea or edema. Rashes are also not considered a side effect of digoxin. Common manifestations of digoxin toxicity include nausea, vomiting and fatigue. Hunger, dizziness and diaphoresis, together, are not considered side effects of digoxin. Although dizziness could occur with another side effect of digoxin, such as bradycardia. Polyuria, thirst and dry skin are not considered side effects of digoxin.

The nurse is caring for a child who is receiving vincristine. Which body systems are most important for the nurse to assess after medication administration? Select all that apply. One, some, or all responses may be correct. A. Respiratory B. Neurological C. Reproductive D. Hematologic E. Gastrointestinal

B. Neurological D. Hematologic E. Gastrointestinal Vincristine is neurotoxic; therefore the child should be monitored for paresthesias, seizures, footdrop, bowel and bladder problems, and alterations in the function of cranial nerves. Hematologic problems such as anemia, thrombocytopenia, and leukopenia occur, although they are not as severe as with other chemotherapeutic agents, such as cyclophosphamide. Gastrointestinal adverse effects include severe constipation, intestinal necrosis, intestinal perforation, and paralytic ileus, in addition to nausea and vomiting. Respiratory problems are not associated with vincristine therapy. The reproductive system is not affected by vincristine therapy.

Which information will the nurse share about alopecia characteristics to a client who is to receive chemotherapy after surgery for cancer? A. Usually rare B. Not permanent C. Frequently prolonged D. Usually preventable

B. Not permanent Once the medications that interfere with cell division are stopped, the hair will grow back; sometimes the hair will be a different color or texture. Alopecia is a common side effect of chemotherapy. Hair loss persists while the medications are being received; once the medications are withdrawn, the hair grows back. Although ice caps on the head and rubber bands around the scalp have been used to try to limit alopecia, they have not been particularly effective.

The nurse is caring for a client with osteoporosis who has been prescribed alendronate. When providing care, which intervention would be a priority? A. Administer the alendronate 30 to 60 minutes before the client eats. B. Notify the health care provider if the client reports jaw pain. C. Encourage the client to increase their intake of vitamin D. D. Monitor the client's serum calcium levels.

B. Notify the health care provider if the client reports jaw pain. Alendronate is a bisphosphonate that helps slow down bone resorption, decreasing osteoporosis. Osteonecrosis of the jaw is a rare, adverse reaction to alendronate, and jaw pain can be a symptom of this. Therefore, notifying the health care provider of the jaw pain is the priority. The other interventions are also correct for a client with osteoporosis, but are not as important as reporting the potential adverse drug effect.

A 5-year-old child is receiving dactinomycin and doxorubicin therapy after nephrectomy for Wilms tumor. Which intervention would the nurse include when planning care? A. Adding citrus juices to meals B. Offering warm saline mouthwash C. Scheduling booster immunizations D. Reporting red-orange colored urine

B. Offering warm saline mouthwash The use of warm saline mouthwash will minimize oral discomfort; ulceration of the oral mucosa occurs as a result of the antineoplastic effect on the rapidly dividing gastrointestinal epithelium. Oral anesthetics may be prescribed by the health care provider. Adding citrus juices to meals is contraindicated because it will aggravate the stomatitis that is a common side effect of both chemotherapeutic agents. Immunizations must be postponed because of the immunosuppressant effects of chemotherapy. Urine and other body fluids may become red-orange during the first 48 hours after doxorubicin is started; this is an expected response that need not be reported.

Clients who take rifampin should not take medications from which class? A. Loop diuretics B. Oral contraceptives C. Proton pump inhibitor D. Intermediate-acting insulin

B. Oral contraceptives Rifampin increases metabolism of oral contraceptives, which may result in an unplanned pregnancy. Rifampin does not interact with a loop diuretic, a proton pump inhibitor, or intermediate-acting insulin.

Which class is contraindicated in clients who take rifampin? A. Loop diuretics B. Oral contraceptives C. Proton pump inhibitor D. Intermediate-acting insulin

B. Oral contraceptives Rifampin increases metabolism of oral contraceptives, which may result in an unplanned pregnancy. Rifampin does not interact with a loop diuretic, a proton pump inhibitor, or intermediate-acting insulin.

A nurse has administered sublingual nitroglycerin to a client in the emergency department. Which clinical finding indicates an adverse response to the medication? A. Persistent chest pain B. Orthostatic hypotension C. Decreased heart rate D. Labored breathing

B. Orthostatic hypotension Rationale: Decreased blood pressure when changing positions is an unexpected response to nitroglycerin. The nurse should instruct the client to lay down and elevate the feet to promote venous return. Persistent chest pain is not an unexpected response. Additional doses may be required to alleviate angina. A side effect of nitroglycerin is tachycardia, not a decreased heart rate. Nitroglycerin is not associated with respiratory effects.

A child is prescribed intravenous mannitol. The nurse understands mannitol belongs to which classification of diuretics? A. Loop B. Osmotic C. Potassium sparing D. Carbonic anhydrase inhibitor

B. Osmotic Osmotic diuretics, such as mannitol, increase the osmotic pressure of glomerular filtrate and thus decrease absorption of sodium; they are used to treat cerebral edema and increased intraocular pressure. Loop diuretics, such as furosemide, inhibit resorption of sodium and potassium in the loop of Henle; they are used for heart failure and pulmonary edema. Potassium-sparing diuretics, such as spironolactone, interfere with sodium resorption in the distal tubules, thus decreasing potassium excretion; they are used to treat cirrhotic ascites and pulmonary edema. Carbonic anhydrase inhibitors, such as acetazolamide, increase sodium excretion by decreasing sodium-hydrogen ion exchange. They are used to treat seizure disorders and open-angle glaucoma.

The hospice nurse is visiting a client diagnosed with end-stage lung cancer and metastases to the bone. What should the nurse keep in mind when planning for effective pain management? A. High doses of opioid analgesics will be required. B. Pain therapy is based on the client's report of pain. C. Relief of pain will be achieved quickly. D. The client will most likely become addicted.

B. Pain therapy is based on the client's report of pain. Every person's pain experience is unique and should be treated based on the individual's goals for pain management. Therefore, the amount of medication needed is dependent on the client's needs and reports of pain relief. The nurse should not assume that high doses of analgesics will be needed to alleviate the client's pain. Immediate or quick pain relief might be difficult to achieve, especially in light of the client's type of cancer and bone metastases. Addiction is a psychological condition and not a concern for this client. However, the client may develop a physical dependence and tolerance to pain medications that may require an increase in dosage to manage pain effectively.

The alkylating agent cyclophosphamide is prescribed for a school-age child with cancer. Which clinical manifestation would the nurse be alert for while the child is receiving this medication? A. Irritability B. Pain with urination C. Unpredictable nausea D. Hyperplasia of the gums

B. Pain with urination Cystitis is a potentially serious adverse reaction to cyclophosphamide; it sometimes can be prevented by increasing hydration because the fluid flushes the bladder. Irritability may be present but is not a result of cyclophosphamide administration. Unpredictable nausea is an expected but manageable side effect of cyclophosphamide. Hyperplasia of the gums is unrelated to cyclophosphamide administration; it may occur with prolonged phenytoin therapy.

Which immunizations would the nurse determine are safe for a child who is receiving prednisone? Select all that apply. One, some, or all responses may be correct. A. Rubeola B. Pertussis C. Varicella D. Inactivated poliovirus E. Tetanus immune globulin

B. Pertussis D. Inactivated poliovirus E. Tetanus immune globulin Inactivated vaccines are safe for a child receiving prednisone. The pertussis (whooping cough) vaccine is made from inactivated toxins. It is safe to give the child the inactivated poliovirus vaccine; it is not a live attenuated virus vaccine. Tetanus immune globulin is an antitoxin that provides transient passive immunity; tetanus toxoid is contraindicated. Live attenuated virus vaccines are contraindicated for a child on prednisone. Both the rubeola (measles) and the varicella (chickenpox) vaccines are made live attenuated viruses.

A client has a prescription for a sublingual nitroglycerin tablet. Which technique will the nurse teach the client to use? A. Place the pill inside the cheek and let it dissolve. B. Place the pill under the tongue and let it dissolve. C. Chew the pill thoroughly and then swallow it. D. Swallow the pill with a full glass of water.

B. Place the pill under the tongue and let it dissolve. Sublingual medication is placed under the tongue and is quickly absorbed through the mucous membranes into blood. The buccal route requires placing medication between the cheek and gums. Chewing the pill and then swallowing it may be done for oral administration of some large pills, but not with the sublingual route of administration. Taking the pill with water is required with the oral route of administration of medication, but not with sublingual. In addition, a full glass of water may be an excessive amount of fluid to swallow one pill.

The nurse is planning discharge instructions for a client prescribed cyclosporine following a liver transplant. Which adverse reactions should the nurse instruct the client to report to the healthcare provider? A. Changes in urine color. B. Presence of hand tremors. C. Increasing body hirsutism. D. Nausea and vomiting.

B. Presence of hand tremors. Neurological complications, such as hand tremors, occur in about 50% of clients taking cyclosporine and should be reported. Although this drug can be nephrotoxic, changes in urine color typically does not occur. Nausea is a common side effects, but is not usually severe.

Which nursing care will be included for a client who is receiving doxorubicin for acute myelogenous leukemia? A. Increasing citrus foods B. Providing frequent oral hygiene C. Encouraging activity D. Administering medications parenterally

B. Providing frequent oral hygiene Stomatitis and hyperuricemia are possible complications of therapy; therefore oral care and hydration are important. A cidic foods such as citrus foods and fluids will cause pain for clients with stomatitis. Rest, not increased activity, is important for increased fatigability. Abnormal bleeding is a common problem; thus injections (administering medications parenterally) are contraindicated.

A nurse is administering lidocaine to a client with a myocardial infarction. Which assessment finding requires the nurse's immediate action? A. Respiratory rate of 22 B. Pulse rate of 48 beats per minute C. Central venous pressure reading of 9 mm Hg D. Blood pressure of 144/92

B. Pulse rate of 48 beats per minute Lidocaine can cause significant bradycardia and hypotension. A pulse of 48 beats per minute needs immediate attention and is often treated with atropine. At this time the respiratory rate of 22 and blood pressure of 144/92 should be monitored. A normal central venous pressure ranges from 4 to 12 mm Hg. A central venous pressure above 12 my indicate hypervolemia or cardiac failure.

When the nurse is administering intravenous potassium to a client with hypokalemia, which finding is most important to communicate to the health care provider? A. U waves on cardiac monitor B. QRS duration of 0.28 seconds C. Decreased bowel sounds D. Weakened grip strength

B. QRS duration of 0.28 seconds When administering intravenous potassium supplements, it is important to evaluate for clinical manifestations of hyperkalemia. Widening of the Q waves is a potentially fatal manifestation of hyperkalemia (because it may lead to cardiac arrest) and would be communicated rapidly to the health care provider so that the infusion can be stopped and the potassium level can be rechecked. The other findings would be reported to the health care provider but are expected with hypokalemia and are not an indication for a change in treatment. U waves are an expected manifestation of hypokalemia because of changes in ventricular repolarization. Decreased bowel sounds may occur because of decreased peristalsis caused by low potassium levels but should improve with potassium administration. Weakened grips may occur with hypokalemia because normal extracellular potassium levels are needed for skeletal muscle contraction.

Which information would the nurse include when teaching a client about the administration of ranitidine? A. Ranitidine increases gastrointestinal peristalsis. B. Ranitidine reduces gastric acidity in the stomach. C. Ranitidine neutralizes the acid that is present in the stomach. D. Ranitidine stops the production of hydrochloric acid in the stomach.

B. Ranitidine reduces gastric acidity in the stomach. Ranitidine inhibits histamine at H 2-receptor sites in the stomach, resulting in reduced gastric acid secretion. Ranitidine does not increase gastrointestinal peristalsis, and it does not completely stop the production of hydrochloric acid in the stomach. Ranitidine reduces, rather than neutralizes, gastric acidity.

A client diagnosed with tuberculosis is taking isoniazid. To prevent a food and medication interaction, the nurse will advise the client to avoid which food item? A. Hot dogs B. Red wine C. Sour cream D. Grapefruit juice

B. Red wine Clients taking isoniazid should avoid foods containing tyramine such as red wine, tuna fish, and hard cheese. Hot dogs, sour cream, and grapefruit juice do not contain tyramine and are not contraindicated. Grapefruit juice slows metabolism of many medications, but isoniazid is not one of them.

A client with heart failure is prescribed spironolactone (Aldactone). Which information is most important for the nurse to provide to the client about diet modifications? A. Do not add salt to foods during preparation. B. Refrain for eating foods high in potassium. C. Restrict fluid intake to 1000 ml per day. D. Increase intake of milk and milk products.

B. Refrain for eating foods high in potassium. Spironolactone (Aldactone), an aldosterone antagonist, is a potassium-sparing diuretic, so a diet high in potassium should be avoided, along with table salt substitutes, which generally contain potassium chloride that can lead to hyperkalemia.

The nurse is preparing to administer trimethoprim and sulfamethoxazole (TMP-SMX) to a client. When assessing client allergies, the client reports that they are allergic to glipizide. What action by the nurse is most appropriate? A. Prepare to administer the medication B. Report the allergies to the healthcare provider C. Review the health record to see if the client is on glipizide D. Assess the client blood sugar

B. Report the allergies to the healthcare provider Rationale: While administering a sulfonamide with a sulfonylurea may increase the risk of a hypoglycemic reaction, the real concern is the potential allergy to TMP-SMX. It may be safe to administer the medication, but the healthcare provider should be notified first.

Which condition would the nurse monitor for in the client on aminoglycoside therapy and skeletal muscle relaxants? A. Stroke B. Respiratory arrest C. Myocardial infarction D. Abdominal discomfort

B. Respiratory arrest Aminoglycosides can intensify the effect of skeletal muscle relaxants, placing the client at risk for respiratory arrest. Aminoglycoside therapy with muscle relaxants does not increase the risk of stroke, myocardial infarction, or abdominal discomfort.

Which advice will the nurse give the client to avoid lipodystrophy when self-administering insulin therapy? A. Exercise regularly. B. Rotate injection sites. C. Use the Z-track technique. D. Vigorously massage the injection site.

B. Rotate injection sites. Fibrous scar tissue can result from the trauma of repeated injections at the same site. Exercise reduces blood glucose but is unrelated to lipodystrophy. Insulin is given subcutaneously; the Z-track technique is used with some intramuscular injections. Gentle pressure applied over the injection site after insulin administration promotes absorption; it should not be vigorously massaged.

A client abruptly stops taking a barbiturate. Which withdrawal complication would the nurse anticipate that the client may experience? A. Ataxia B. Seizures C. Diarrhea D. Urticaria

B. Seizures Seizures are a serious side effect that may occur with abrupt withdrawal from barbiturates. Ataxia, diarrhea, and urticaria are not associated with barbiturate withdrawal.

A hospitalized infant is receiving gentamicin. While monitoring for drug toxicity, the nurse should focus on which laboratory result? A. Platelet counts B. Serum creatinine C. Thyroxin levels D. Growth hormone levels

B. Serum creatinine Toxicity to the aminoglycoside antibiotic gentamicin is seen in increased BUN and serum creatinine levels. Kidney damage may be reversible if the drug is stopped at the first sign of toxicity. In addition to nephrotoxicity, this medication has a Black Box warning for neurotoxicity and ototoxicity.

The nurse is preparing to administer digoxin to a client with recurring atrial fibrillation. Which laboratory value should be of highest concern for the nurse? A. Hemoglobin 9.4 g/dL B. Serum potassium 3.1 mEq/L C. Serum creatinine 1.9 mg/dL D. B-type natriuretic peptide 140 pg/mL

B. Serum potassium 3.1 mEq/L Digoxin is a cardiac glycoside used to treat atrial dysrhythmias and heart failure. Because digoxin competes with potassium ions, digoxin should not be given when the client's potassium level is below normal range. Giving digoxin to a client with hypokalemia can cause digoxin toxicity and life-threatening cardiac dysrhythmias. Although all of the lab values are outside of normal range, the low potassium level (normal range 3.5-5.0 mEq/L) should be of highest concern for the client at this time. The nurse should hold the digoxin and notify the health care provider.

The nurse is providing discharge education to a client who will be starting daily atenolol for the treatment of hypertension. Which side effect is most important for the client to notify their health care provider about? A. Decreased libido B. Slow, irregular heart rate C. Dizziness in the morning D. Decreased exercise tolerance

B. Slow, irregular heart rate Atenolol is a Beta-1 selective adrenergic blocking agent or a "beta blocker." These medications are commonly used to treat hypertension or chronic angina. Due to their selectivity, they are the preferred medications for clients who have the comorbidities of Chronic Obstructive Pulmonary Disease (COPD). Common adverse effects often relate to the therapeutic action of the drug and include impotence, decreased libido, dizziness, decreased exercise tolerance, slowed heart rate, arrhythmias and heart failure. The client should be taught to assess their heart rate and to notify the health care provider of any changes to the heart rate or rhythm.

A client is receiving chemotherapy with doxorubicin. Which development will the nurse teach the client to report immediately? A. Nausea B. Sore throat C. Loss of hair D. Constipation

B. Sore throat A sore throat is indicative of a respiratory tract infection, which may be the first clinical sign of bone marrow suppression, which can be life-threatening. Nausea is an expected side effect of doxorubicin, but it is not life-threatening. Hair loss is not a side effect of doxorubicin but, regardless, is not life-threatening. Constipation is an expected side effect of doxorubicin, but it is not life-threatening.

A client with Hodgkin's disease is started on chemotherapy. The nurse teaches the client to notify the health care provider for which adverse response to chemotherapy? A. Hair loss B. Sores in the mouth C. Moderate diarrhea after treatment D. Nausea for 6 hours after treatment

B. Sores in the mouth Stomatitis is a common response to chemotherapy and should be brought to the health care provider's attention because a swish-and-swallow anesthetic solution can be prescribed to make the client more comfortable. Hair loss is also anticipated with some chemotherapeutic medications; the effects are temporary and reversible. Moderate diarrhea is expected and is not a cause for concern unless dehydration results. Nausea is expected but should be reported if it lasts more than 24 hours.

Which drug category applies to cocaine? A. Opioids B. Stimulants C. Barbiturates D. Hallucinogens

B. Stimulants Cocaine is classified as a stimulant. It is inhaled in its powdered form or smoked as crack; its use creates experiences similar to but more intense than those experienced with amphetamines, and its withdrawal results in a deeper crash. Opioids and barbiturates are central nervous system depressants. Hallucinogens produce cerebral excitation that can yield a state similar to psychosis.

A nurse is preparing to administer reconstituted doxorubicin (Myocet) to a client with thyroid carcinoma. Nuclear medicine calls for the client, and the nurse is unable to administer the medication. Which action should the nurse perform with the medication? A. Save the medication in a syringe with an aluminum needle B. Store the medication in the refrigerator inside the syringe C. Discard the medication in the hazardous waste container D. Add the medication to the intravenous fluids in the client room

B. Store the medication in the refrigerator inside the syringe Rationale: Doxorubicin that is stored in a refrigerator is stable for up to 48 hours. The medication remains stable at room temperature for up to 24 hours. Saving the medication with an aluminum needle will cause discoloration of the solution and form a dark precipitate. Discarding the medication is not necessary. The medication can be stored for 24-48 hours. Doxorubicin should not be added to intravenous fluids. The medication should be dissolved completely with a diluent.

A client with midsternal pain presents to the emergency department. Vital signs are stable. Which form of nitroglycerin would the nurse anticipate giving initially? A. Oral capsule B. Sublingual spray C. Intravenous solution D. Transdermal patch

B. Sublingual spray Nitroglycerin spray provides prompt relief of symptoms. The nurse administers one to two sprays, up to a maximum of three sprays, onto or under the tongue every 5 minutes until pain is relieved. If unrelieved after three sprays, intravenous (IV) nitroglycerin may be considered. Both the transdermal and oral forms of nitroglycerin are used for prophylactic purposes, not management of acute pain.

Which information would be included in the teaching plan for the older adult client with peptic ulcer disease who is taking an antacid and sucralfate? A. Antacids should be taken 30 minutes before a meal. B. Sucralfate should be taken on an empty stomach 1 hour before meals. C. Sucralfate is prescribed for the long-term maintenance of peptic ulcer disease. D. Sodium bicarbonate is an inexpensive over-the-counter antacid with few adverse effects.

B. Sucralfate should be taken on an empty stomach 1 hour before meals. Sucralfate works best in a low pH environment; therefore it should be given on an empty stomach either 1 hour before or 2 hours after meals. Sucralfate also should be administered no sooner than 30 minutes before or after an antacid. The acid-neutralizing effects of antacids last approximately 30 minutes when taken on an empty stomach and 3 to 4 hours when taken after meals. When sucralfate and an antacid are both prescribed, they are each most effective when the sucralfate is scheduled an hour before meals and the antacid is scheduled after meals. Sucralfate is prescribed for the short-term treatment of peptic ulcers. Its use is limited to 4 to 8 weeks. The client should follow the recommendations of the primary health care provider with regard to antacid selection. Sodium bicarbonate can produce acid-base imbalances, which could be harmful, especially in older adult clients.

A 43-year-old female client is prescribed thyroid replacement hormone following a thyroidectomy. Which adverse effects should the nurse instruct the client to report immediately to the healthcare provider? A. Tinnitus and dizziness. B. Tachycardia and chest pain. C. Dry skin and intolerance to cold. D. Weight gain and increased appetite.

B. Tachycardia and chest pain. Thyroid replacement hormone increases the metabolic rate of all tissues. Common signs and symptoms of toxicity include tachycardia and chest pain and should be reported to the healthcare provider immediately.

Which information will the nurse include when teaching a client about potassium chloride effervescent tablets? A. Chew the tablet completely. B. Take the medication with food. C. Take the medication at bedtime. D. Use warm water to dissolve the tablet.

B. Take the medication with food. Eating food when taking the medication will decrease gastrointestinal irritation. Side effects of this medication include abdominal cramps, diarrhea, and ulceration of the small intestine. Chewing the tablet completely will cause oral mucosal irritation and is not the way the medication should be administered. Taking the medication at bedtime increases the possibility of mucosal irritation because the gastrointestinal tract is empty during the night. The tablet should be dissolved in cold water or juice to make it more palatable.

The nurse in an ambulatory clinic is speaking with the parents of a 2-year-old child diagnosed with acute otitis media. Which information is most important for the nurse to include in the instructions to the parents? A. The child may be given acetaminophen or ibuprofen drops for pain. B. The child must complete the entire course of the prescribed antibiotic. C. The child should return to the clinic to evaluate effectiveness of the treatment. D.The child may be given a decongestant to relieve pre

B. The child must complete the entire course of the prescribed antibiotic. Acute otitis media (AOM) is an inflammation of the middle ear space with a rapid onset of the signs and symptoms of acute infection, namely, fever and otalgia (ear pain). It is one of the most prevalent early childhood illnesses. Treatment for AOM is one of the most common reasons for antibiotic use in the ambulatory setting. When antibiotics are necessary, it is most important to complete the entire course to prevent antibiotic resistance. The child should be seen after antibiotic therapy is complete to ensure that the infection has resolved. Supportive care of AOM includes treating the fever and pain. Decongestants or antihistamines are not recommended for children with ear infections.

A client has been robbed, beaten, and sexually assaulted. The primary health care provider prescribes 0.25 mg of alprazolam for agitation. Which event would alert the nurse to administer this medication? A. The client's crying increases. B. The client requests something to calm her. C. The nurse determines a need to reduce her anxiety. D.The primary health care provider is getting ready to perform a vaginal

B. The client requests something to calm her. Because a sexual assault is a threat to the sense of control over one's life, some control should be given back to the client as soon as possible. Crying is a typical way to express emotions; the client should be told that medication is available if desired. The nurse determining a need to reduce the client's anxiety or administering the medication when the primary health care provider is getting ready to do a vaginal examination takes control away from the client. She may view these actions as an additional assault on the body, which increases feelings of vulnerability and anxiety and does not restore control.

A client with myasthenia gravis begins taking pyridostigmine. Two days later, the client develops loose stools and increased salivation. Which conclusion would the nurse make about these new developments? A. The client is experiencing a myasthenic crisis. B. The medication is causing cholinergic side effects. C. The medication is triggering a paradoxical reaction. D. The client is exhibiting toxic effects of the medication.

B. The medication is causing cholinergic side effects. Because this medication inhibits the destruction of acetylcholine, parasympathetic activity may increase, resulting in cholinergic side effects such as diarrhea and increased salivation. The signs do not indicate a myasthenic crisis. Myasthenic crisis is characterized by difficulty breathing or speaking, morning headaches, feeling tired during the daytime, waking up frequently at night, not sleeping well, a weak cough with increased secretions (mucus or saliva), an inability to clear secretions, a weak tongue, trouble swallowing or chewing, and weight loss. Side effects are not temporary and not paradoxical; they continue as long as the medication is continued. The dosage may be adjusted or an anticholinergic may be given to limit side effects. Toxicity or cholinergic crisis is manifested by increased muscle weakness, including muscles of respiration.

Which conclusion would the nurse make about the development of loose stools and increased salivation two days after a client with myasthenia gravis begins taking pyridostigmine? A. The client is experiencing a myasthenic crisis. B. The medication is causing cholinergic side effects. C. The medication is triggering a paradoxical reaction. D. The client is exhibiting toxic effects of the medication. Rationale

B. The medication is causing cholinergic side effects. Because this medication inhibits the destruction of acetylcholine, parasympathetic activity may increase, resulting in cholinergic side effects such as diarrhea and increased salivation. The signs do not indicate a myasthenic crisis. Myasthenic crisis is characterized by difficulty breathing or speaking, morning headaches, feeling tired during the daytime, waking up frequently at night, not sleeping well, a weak cough with increased secretions (mucus or saliva), an inability to clear secretions, a weak tongue, trouble swallowing or chewing, and weight loss. Side effects are not temporary and not paradoxical; they continue as long as the medication is continued. The dosage may be adjusted or an anticholinergic may be given to limit side effects. Toxicity or cholinergic crisis is manifested by increased muscle weakness, including muscles of respiration.

) The nurse is reinforcing teaching about levothyroxine for a client newly-diagnosed with hypothyroidism. Which information should the nurse make sure to reinforce about this medication? A. The medication must be stored in a dark container. B. The medication should be taken in the morning. C. The medication will decrease the client's heart rate. D. The medication may decrease the client's energy level.

B. The medication should be taken in the morning. A thyroid supplement, such as levothyroxine, should be taken on an empty stomach in the morning. Morning dosing minimizes the side effect of insomnia and an empty stomach facilitates absorption. The medication does not need to be stored in a dark container. Levothyroxine will cause an increase in the client's energy level and heart rate.

The nurse is providing education to the parents of a 10-year-old child who is diagnosed with diabetes insipidus (DI) and has been prescribed vasopressin. What important information should the nurse include regarding this medication? A. The child will need intravenous therapy for several weeks. B. The parents must closely monitor the child's fluid intake. C. The child may experience brief episodes of chest pain. D. The child should be observed for signs of dehydration.

B. The parents must closely monitor the child's fluid intake. Diabetes insipidus is characterized by a decreased secretion of antidiuretic hormone (ADH), resulting in excess diuresis and fluid volume deficit. Vasopressin, a synthetic form of ADH, is the drug of choice to treat DI. Vasopressin therapy can lead to excessive water retention and water intoxication. A major cause of intoxication is failure to reduce water intake once ADH therapy has begun. Because treatment prevents continued fluid loss, failure to decrease fluid intake will result in water buildup. Hence, at the onset of treatment, clients should be instructed to reduce their accustomed intake of fluid.

Which criterion is an indicator that the nitroglycerin sublingual tablets have lost their potency? A. Sublingual tingling is experienced. B. The tablets are more than 3 months old. C. The headache is less severe. D. Onset of relief is delayed.

B. The tablets are more than 3 months old. Nitroglycerin tablets are affected by light, heat, and moisture. Loss of potency can occur after 3 months, reducing the medication's effectiveness in relieving pain. A new supply should be obtained routinely. Experiencing sublingual tingling indicates the tablets have retained their potency. Headaches may decrease over time; this is not an indicator of medication potency. A delay in relief reflects the ischemia, not the medication.

Which statement about benzodiazepines requires correction? A. They are indicated for ethanol withdrawal. B. These medications increase the activity of gamma-aminobutyric acid. C. Benzodiazepines are the first-line medications used in chronic anxiety disorders. D. These medications depress activity in the brainstem.

B. These medications increase the activity of gamma-aminobutyric acid. Benzodiazepines act by decreasing the activity of gamma-aminobutyric acid, which is an inhibitory neurotransmitter. Apart from their indication in the treatment of depression, benzodiazepines are also prescribed for ethanol withdrawal, insomnia, and muscle spasms. Benzodiazepines are the first-line medications of choice in acute and chronic anxiety disorders. Benzodiazepines act by depressing activity in the brainstem and the limbic system.

The nurse is discharging a client with a new prescription for tiotropium to help manage the symptoms of chronic obstructive pulmonary disease. What information should the nurse include in the discharge teaching? A. It may be a few days before you feel the full effects of tiotropium. B. This medication cannot be used to relieve sudden breathing problems. C. Be sure to swallow the capsules with a full glass of water. D. A common side effect is nausea and loose stools.

B. This medication cannot be used to relieve sudden breathing problems. Tiotropium is a long-acting anticholinergic bronchodilator. The medication comes as a capsule to use with a specially designed inhaler - clients should never swallow the capsules. For new prescriptions, it's important to tell the client that they may start breathing better with the full dose but it may take a few weeks to feel the full effects. It cannot be used as a fast-acting inhaler. Due to its anticholinergic properties, it may cause constipation (not loose stools).

A client with thyroid cancer is scheduled for a thyroidectomy. Which information will the nurse teach the client? A. The dietary intake of carbohydrates must be restricted. B. Thyroxine replacement therapy will be required indefinitely. C. Chemotherapy will be used in conjunction with the surgery. D. A tracheostomy is required for clients having this procedure.

B. Thyroxine replacement therapy will be required indefinitely. Thyroxine is given postoperatively to suppress thyroid-stimulating hormone (TSH) and prevent hypothyroidism. Increased intake of carbohydrates and proteins is needed because of the increased metabolic activity associated with hyperthyroidism. Chemotherapy is uncommon; radiation may be used to eradicate remaining tissue. A tracheostomy is not planned; it is needed only in an emergency related to respiratory distress.

Intravenous furosemide has been prescribed for a client with severe edema and hypertension. Which subjective clinical manifestations lead the nurse to suspect that the furosemide is infusing too rapidly? Select all that apply. One, some, or all responses may be correct. A. Hunger B. Tinnitus C. Weakness D. Leg cramps E. Excess salivation

B. Tinnitus C. Weakness D. Leg cramps Tinnitus is a central nervous system side effect of furosemide. Weakness and leg cramps result from hypokalemia caused by an overload of furosemide. Nausea and anorexia, not hunger, are side effects of dehydration that may occur with an overload of furosemide. Dry mouth, not salivation, results from dehydration caused by an overload of furosemide.

For which purpose would a nurse advise a client with chloroquine-resistant malaria to take oral quinine immediately after meals? A. To delay its absorption B. To minimize gastric irritation C. To reduce its antidysrhythmic action D. To decrease stimulation of the appetite

B. To minimize gastric irritation Quinine administered orally can cause gastric irritation, resulting in nausea and vomiting. Administration of the medication immediately after meals minimizes its irritating effect. Absorption of the medication is not significantly affected by administration after meals. Quinidine, not quinine, is given for its antidysrhythmic effect. The appetite is not affected by this medication as long as gastric irritation is avoided.

The nurse receives an order to administer intravenous (IV) iron sucrose to a client with anemia. Which statement best describes the purpose of administering this medication using the IV route? A. To ensure that the entire dose of medication is given B. To prevent the drug from causing tissue irritation C. To provide more even distribution of the drug D. To enhance absorption of the medication

B. To prevent the drug from causing tissue irritation Iron sucrose is an iron supplement used to treat iron deficiency anemia. If given subcutaneously or intramuscularly, the tissue can become irritated and may result in bleeding into the muscle; therefore, the best route for this medication is intravenous (IV). The rate for administration will vary on the dosage but is typically at a slower rate due to the risk of adverse reactions. The other statements do not accurately describe the purpose for the IV route.

A client is to receive metoclopramide intravenously 30 minutes before initiating chemotherapy for cancer of the colon. The nurse explains that metoclopramide is given for which purpose? A. To stimulate production of gastrointestinal (GI) secretions B. To stimulate peristalsis of the upper gastrointestinal (GI) tract C. To prolong excretion of the chemotherapeutic medication D. To increase absorption of the chemotherapeutic medication

B. To stimulate peristalsis of the upper gastrointestinal (GI) tract Prokinetic medications such as metoclopramide stimulate peristalsis in the GI tract. This enhances the emptying of stomach contents into the duodenum to decrease gastroesophageal reflux and vomiting, all of which are precipitated by chemotherapeutic agents. Metoclopramide does not stimulate the production of gastrointestinal secretions. Metoclopramide has no effect on the excretion of chemotherapeutic medications. Metoclopramide has no effect on the absorption of chemotherapeutic medications.

A client with burns develops a wound infection. The nurse plans to teach the client that local wound infections are primarily treated with which antibiotic formulation? A. Oral B. Topical C. Intravenous D. Intramuscular

B. Topical Topical antibiotics are applied directly to the wound and are effective against many gram-positive and gram-negative organisms found on the skin. Although oral, intravenous, and intramuscular antibiotics may be administered, they are most effective for systemic rather than local infections; the vasculature in and around a burn is impaired, and the medication may not reach the organisms in the wound.

Which issue related to antibiotic use is an increased risk for the older adult? A. Allergy B. Toxicity C. Resistance D. Superinfection

B. Toxicity The older adult is at increased risk for toxicity related to antibiotic use because of reduced metabolism and excretion of medications. Allergy, resistance, and superinfection are a risk for all antibiotic recipients but not an increased risk in the older adult population.

Which clinical indicator would the nurse monitor to determine if the client's simvastatin is effective? A. Heart rate B. Triglycerides C. Blood pressure D. International normalized ratio (INR)

B. Triglycerides Therapeutic effects of simvastatin include decreased levels of serum triglycerides, low-density lipoprotein (LDL), and cholesterol. INR is not related to simvastatin; it is a measure used to evaluate blood coagulation. Heart rate and blood pressure are not related to simvastatin.

In evaluating the effects of lactulose (Cephulac), which outcome would indicate that the drug is performing as intended? A. An increase in urine output. B. Two or three soft stools per day. C. Watery, diarrhea stools. D. Increased serum bilirubin.

B. Two or three soft stools per day. The medication lactulose can be administered for either chronic constipation or for portal-systemic encephalopathy in clients with hepatic disease. Two to three stools a day indicate that lactulose is performing as intended for chronic constipation. This would also indicate it should be effective for the clients with encephalopathy because the lactulose's action prevents absorption of ammonia in the colon as it increases water absorption and softens the stool. The efficacy of the use for ammonia absorption would have to be verified by a serum ammonia level and observation of clearing of the client's mental status.

The health care provider prescribes a cholinergic medication to treat a client's urinary problem. Which effect would the nurse anticipate? A. Urinary frequency decreases. B. Urinary retention is prevented. C. Pain is controlled. D. Urinary urgency decreases.

B. Urinary retention is prevented. Cholinergics intensify and prolong the action of acetylcholine, which increases tone in the genitourinary tract, preventing urinary retention. Cholinergic antagonists are prescribed for frequency and urgency associated with a spastic bladder; therefore cholinergics would worsen rather than decrease these problems. Cholinergic do not have analgesic effects.

Which assessment would the nurse perform specific to the safe administration of intravenous mannitol? A. Body weight daily B. Urine output hourly C. Vital signs every 2 hours D. Level of consciousness every 8 hours

B. Urine output hourly Mannitol, an osmotic diuretic, increases the intravascular volume that must be excreted by the kidneys. The client's urine output should be monitored hourly to determine the client's response to therapy. Although mannitol results in an increase in urinary excretion that is reflected in a decrease in body weight (1 L of fluid is equal to 2.2 pounds [1 kg]), a daily assessment of the client's weight is too infrequent to assess the client's response to therapy. Urine output can be monitored hourly and is a more frequent, accurate, and efficient assessment than is a daily weight. Vital signs should be monitored every hour considering the severity of the client's injury and the administration of mannitol. Although the level of consciousness should be monitored with a head injury, conducting assessments every 8 hours is too infrequent to monitor the client's response to therapy.

Cholinergic agonists are prescribed for which type of urinary condition? A. Kidney stones B. Urine retention C. Spastic bladder D. Urinary tract infections

B. Urine retention Cholinergics intensify and prolong the action of acetylcholine, which increases the tone in the genitourinary tract, preventing urinary retention. Cholinergics will not prevent renal calculi. Anticholinergics, not cholinergic agonists, are prescribed for the frequency and urgency associated with a spastic bladder. Preventing urinary tract infections is a secondary gain because cholinergics help prevent urinary retention that can lead to a urinary tract infection, but this is not the purpose for administering these medications.

The nurse is preparing to teach a client with type 2 diabetes mellitus about their newly prescribed exenatide pen. Which instructions should the nurse include? Select all that apply. A. Take the exenatide immediately after meals. B. You may experience some weight loss. C. Take any oral medications 1 hour before the exenatide. D. Inject yourself in the abdominal or thigh area. E. After use, store the injector pen in the refrigerator.

B. You may experience some weight loss. C. Take any oral medications 1 hour before the exenatide. D. Inject yourself in the abdominal or thigh area. Exenatide (Byetta) is a non-insulin, incretin mimetic used for the treatment of diabetes. It works by lowering blood glucose by slowing gastric emptying, stimulating glucose-dependent insulin release, suppressing postprandial glucagon release, and reducing appetite. Some initial, minor weight loss is common. Exenatide comes in pre-filled, injector pens. Injections are made subcutaneously into the thigh, abdomen, or upper arm. Exenatide should be administered 0 to 60 minutes before the morning and evening meals — never after the meal. Exenatide delays gastric emptying and hence can slow the absorption of oral drugs; this is of particular concern with oral contraceptives and antibiotics. To minimize this interaction, the client should take oral drugs at least 1 hour before exenatide. Common side effects include nausea and vomiting. The pen should be stored at room temperature after first use.

The nurse is teaching the client with bacterial vaginosis who has been prescribed metronidazole tablets. What statement is appropriate? A. You may continue to experience symptoms after you stop the medication B. You should avoid drinking alcohol while taking this medication C. Call your healthcare provider if you experience diarrhea D. Your sexual partner will need to be treated as well

B. You should avoid drinking alcohol while taking this medication Rationale: Alcohol should be avoided while on metronidazole to reduce the risk of a disulfiram reaction. Routine treatment of male sexual partners is not needed and does not affect re-infection rates. If the client experiences continued symptoms, this may indicate treatment failure and the need for follow-up may be required. Diarrhea is a common side effect of metronidazole and should subside once treatment ends.

The nurse is educating a client with seizure disorder about newly prescribed phenytoin. Which statement should the nurse include in the teaching? A. Blood work will be required if you have a seizure while taking this medication B. You will need to have routine visits with a dentist when taking this medication C. It is normal to have a change in your gait when you first start this medication D. Avoid grapefruit juice when taking this medication

B. You will need to have routine visits with a dentist when taking this medication Rationale: Phenytoin is the first-line medication for the treatment of seizures. Clients should be instructed that they will need routine lab work to ensure that they are at a therapeutic level with the medication, even if they have been seizure-free. This medication can cause gingival hyperplasia, which will require routine dental visits. The client does not need to avoid grapefruit juice with this medication. Difficulty with hand and gait coordination could indicate toxicity and should be reported to the healthcare provider.

A client is prescribed alendronate. Which instruction should the nurse emphasize when teaching about this medication? A. "Take the medication with a full glass of milk two hours after meals." B. "It is recommended that you take this medication with calcium and a glass of juice." C. "Be sure to take this medication on an empty stomach." D. "You may take this medication after any meal, at the same time every day."

C. "Be sure to take this medication on an empty stomach." Alendronate (Fosamax) is used to treat and prevent osteoporosis. It should be taken first thing in the morning with 6 to 8 ounces of plain water at least 30 minutes before other medication or food. Food and fluids (other than water) greatly decrease the absorption of this medication. The client must also be instructed to remain in the upright position for 30 minutes following the dose to facilitate passage into the stomach and minimize irritation of the esophagus.

Which statement is important for the nurse to include in the teaching plan of a client with irritable bowel syndrome who has instructions to take psyllium for constipation? A. "Urine may be discolored." B. "Stop taking the laxative once a bowel movement occurs." C. "Each dose should be taken with a full glass of water or juice." D. "Daily use may inhibit the absorption of some fat-soluble vitamins."

C. "Each dose should be taken with a full glass of water or juice." This bulk-forming laxative works by absorbing water into the intestine, which increases bulk and distends the bowel to initiate reflex bowel activity, thus promoting a bowel movement. A full glass of fluid taken at the same time will help minimize the risk of esophageal obstruction or fecal impaction. Senna, a stimulant laxative, may cause urine discoloration. Bulk-forming laxatives, such as psyllium, are the only laxatives that are recommended for long-term use and in cases of irritable bowel syndrome; they are used to prevent constipation and should not be stopped once a bowel movement occurs. Prolonged use of lubricant laxatives, such as mineral oil, can inhibit the absorption of some fat-soluble vitamins.

) The nurse understands that the prescribed levothyroxine is effective when the client with hypothyroidism makes which statement? A. "I still feel lethargic and fatigued." B. "I have to change my sheets in the morning because I sweat a lot at night." C. "I have been having daily, formed bowel movements." D. "I was reprimanded at work after becoming angry with my boss."

C. "I have been having daily, formed bowel movements." Levothyroxine sodium is utilized to treat hypothyroidism. The nurse must first understand signs and symptoms of hypothyroidism, such as fatigue, lethargy, constipation, hypotension, anorexia and weight gain. In doing so, the nurse can identify that reports of having regular bowel movements is indicative of the levothyroxine working as intended, as constipation is a common symptom of hypothyroidism.

The nurse in the primary care office is following up with a client who has been experiencing frequent constipation. Which statement by the client about using psyllium indicates that additional teaching is needed? A. "I will take it in the morning with lots of water." B. "I will only take it until my constipation is relieved." C. "I will take it together with my other medications." D. "I may notice some bloating while I am taking it."

C. "I will take it together with my other medications." When using psyllium to manage constipation, it needs to be taken with at least 240 mL of water. The client may experience abdominal discomfort or bloating while taking psyllium. When using psyllium to treat constipation, it should not be used long-term and be discontinued when the constipation has resolved. Bulk laxatives can interfere with the absorption of some medications, so it should be taken two hours before or two hours after other medications. The client's statement about taking the psyllium with other medications requires additional teaching.

A client is being discharged with a prescription for an iron supplement. Which client statement indicates the need for further teaching by the nurse? A. "I will have greenish-black stools from the medication." B. "I will not take antacids with my iron supplement." C. "I will take the iron supplement with a full glass of milk." D. "I will take vitamin C along with the iron supplement."

C. "I will take the iron supplement with a full glass of milk." Iron supplements should be taken along with Vitamin C, such as orange juice, because this increases the absorption. Conversely, antacids, milk, caffeinated beverages, and calcium supplements can decrease the absorption of iron. Iron should be taken one hour before or two hours after meals to enhance absorption, although clients who report gastrointestinal intolerance may take it with food. Iron will cause stool to turn greenish-black and tarry.

The nurse is reviewing discharge instructions with the parent of a 3-year-old client who was admitted for poisoning after ingesting cherry-flavored acetaminophen. Which statement by the parent would require follow up by the nurse? A. "I should use non-flavored medications." B. "I will reach out to the poison control center if this happens again." C. "I will use ipecac syrup to induce vomiting." D. "I will have all medications in a locked cabinet."

C. "I will use ipecac syrup to induce vomiting." Rationale: Accidental ingestions (poisoning) are the most frequent accident in toddlers. Therefore, it is imperative to focus on keeping all poisonous substances, drugs, and small objects securely out of the reach of children and medications in a locked cabinet. Parents should be instructed to call the poison control center in case of accidental ingestion and to have the number listed on their cell phone. Since 2003, the American Academy of Pediatrics has discouraged the use of syrup of ipecac to induce vomiting after accidental ingestion. Instead, families should call the poison control center immediately. Using non-flavored medications will decrease the likelihood the child will consume a large amount; it does not prevent the ingestion of the medication or determine what should be done after accidental ingestion.

A 42-year-old male client diagnosed with hypertension tells the nurse he no longer wants to take the prescribed propranolol. Which client statement best explains the reason why he does not want to take this medication? A. "I have difficulty falling asleep." B. "I'm having problems with my stomach." C. "I'm experiencing decreased sex drive." D. "I feel so tired all the time."

C. "I'm experiencing decreased sex drive." Propranolol is a beta-blocker used to treat many conditions, such as essential tremors, angina, hypertension and heart rhythm disorders. Common side effects of this drug include nausea, diarrhea, constipation, stomach cramps, rash, tiredness, dizziness, sleep problems and vision changes. Additionally, propranolol may cause decreased sex drive, impotence or difficulty having an orgasm in men. The clients can be switched to an alternative antihypertensive, such as an angiotensin-converting enzyme (ACE) inhibitor or a calcium channel blocker.

Which statement by a client who had a laminectomy and is receiving a skeletal muscle relaxant that will be continued after discharge indicates that teaching was effective? A. "I'm going to take the medication between meals." B. "If the medication makes me sleepy, I'll stop taking it." C. "If the medication upsets my stomach, I'll take it with milk." D. "I'll take an extra dose of the medication before I do anything active."

C. "If the medication upsets my stomach, I'll take it with milk." These medications tend to irritate the gastric mucosa and should be taken with milk or food to limit gastrointestinal irritation. Drowsiness is an expected side effect; safety precautions are indicated, but the medication should not be discontinued. Taking an extra dose of the medication before activity can result in toxicity if the extra dose, in addition to the prescribed dose, exceeds the therapeutic range; the dosage prescribed by the health care provider should be followed.

Which explanation would the nurse provide to a client with tuberculosis who asks why vitamin B 6 (pyridoxine) is given with isoniazid? A. "It will improve your immunologic defenses." B. "The tuberculostatic effect of isoniazid is enhanced." C. "Isoniazid interferes with the synthesis of this vitamin." D. "Destruction of the tuberculosis organisms is accelerated."

C. "Isoniazid interferes with the synthesis of this vitamin." Isoniazid often leads to vitamin B 6 (pyridoxine) deficiency because it competes with the vitamin for the same enzyme; this deficiency most often is manifested by peripheral neuritis, which can be controlled by the regular administration of vitamin B 6. Vitamin B 6 does not improve immune status. Pyridoxine does not enhance the effects of isoniazid. Pyridoxine does not destroy organisms.

Which information will the nurse include when teaching about tetanus immune globulin prescribed to a client with a puncture wound? A. "It will take about a week to become effective." B. "Immune globulin provides lifelong passive immunity." C. "It provides immediate, passive, short-term immunity." D. "Immune globulins stimulate the production of antibodies."

C. "It provides immediate, passive, short-term immunity." Tetanus immune globulin contains ready-made antibodies and provides immediate, short-term, passive immunity. Passive immunity lasts a short time, not throughout life. Immune globulins confer passive artificial immunity, not long-lasting active immunity. Immune globulins are antibodies; they do not stimulate the production of antibodies.

The daughter of a client with Alzheimer's disease asks the nurse, "Will the medication my mother is taking cure her dementia?" What is the best response by the nurse? A. "It will help your mother live independently again." B. "It is used to halt the progression of Alzheimer's disease." C. "It will not improve dementia but can help control emotional responses." D. "It will provide a steady improvement in memory."

C. "It will not improve dementia but can help control emotional responses." Drug therapy for Alzheimer's disease such as memantine and donepezil produce modest improvements in cognition, behavior, and function, and slightly delayed disease progression. They do not reverse the dementia or halt the progression of Alzheimer's disease. At best, drugs currently in use may slow loss of memory and improve cognitive functions (e.g., memory, thought, reasoning) and emotional lability. However, these improvements are modest and last a short time and for many clients, even these modest goals are elusive.

The nurse has given discharge instructions to a client who suffers from sensory neuropathy due to diabetes. The client was prescribed gabapentin. Which of the following statements indicates that the client understands the nurse's instructions regarding the medication? A. "The medication might cause me to have insomnia." B. "I can stop taking the medication at any time." C. "My doctor prescribed it for the pain in my legs." Correct answer D. "It is safe to take extra doses if my pain becomes worse."

C. "My doctor prescribed it for the pain in my legs." Correct answer

A client asks the nurse if glipizide (Glucotrol) is an oral insulin. Which response should the nurse provide? A. "Yes, it is an oral insulin and has the same actions and properties as intermediate insulin." B. "Yes, it is an oral insulin and is distributed, metabolized, and excreted in the same manner as insulin." C. "No, it is not an oral insulin and can be used only when some beta cell function is present." D. "No, it is not an oral insulin, but it is effective for those who are resistant to injectable insulins."

C. "No, it is not an oral insulin and can be used only when some beta cell function is present." An effective oral form of insulin has not yet been developed because when insulin is taken orally, it is destroyed by digestive enzymes. Glipizide (Glucotrol) is an oral hypoglycemic agent that enhances pancreatic production of insulin.

Which rationale will the nurse give for the need to take penicillin G and probenecid for syphilis? A. "Each medication attacks the organism during different stages of cell multiplication." B. "The penicillin treats the syphilis, and the probenecid relieves the severe urethritis." C. "Probenecid delays excretion of penicillin, thus maintaining blood levels for longer periods." D. "Probenecid decreases the potential for an allergic reaction to penicillin, which treats the syphilis."

C. "Probenecid delays excretion of penicillin, thus maintaining blood levels for longer periods." Probenecid results in better use of penicillin by delaying the excretion of penicillin through the kidneys. Penicillin destroys Treponema pallidum during all stages of its development; probenecid delays the excretion of penicillin. Probenecid does not treat urethritis. Probenecid does not prevent allergic reactions.

A nurse is teaching a client with stable angina about newly prescribed SL nitroglycerin. Which statement should the nurse include in the teaching? A. "Take this medication after each meal and at bedtime." B. "Take one tablet 30 minutes before any physical activity." C. "Take one tablet immediately when you experience chest pain." D. "Take this medication with 8 ounces of water."

C. "Take one tablet immediately when you experience chest pain." Rationale: Nitroglycerin is a vasodilator used to treat angina or ischemic chest pain. When teaching a client about SL nitroglycerin, the nurse should instruct the client to take one tab and place it under their tongue immediately when experiencing chest pain. The client only takes this medication when experiencing chest pain. The client should not eat or drink when taking this medication.

Which information is most important for the nurse to teach a client prescribed an antihypertensive medication to be taken once in the morning and a 2-gram sodium diet? A. "Avoid adding salt to cooked foods." B. "Use less salt when preparing foods." C. "Take your medicine exactly as prescribed." D. "Measure your blood pressure every morning."

C. "Take your medicine exactly as prescribed." The most effective way to lower the blood pressure is to take the prescribed medication daily. Restricting salt in the diet will help limit fluid retention and thus reduce the blood pressure, but it is not as effective as an antihypertensive. Salt should not be added during food preparation. The natural sodium content of foods should be calculated in a 2-gram sodium diet. It is not necessary to take daily blood pressure measurements unless specifically prescribed to do so by the primary health care provider.

Which is the priority short-term goal when teaching a client with type 1 diabetes who is placed on an insulin pump to control the diabetes? A. "The client will adhere to the medical regimen." B. "The client will remain normoglycemic for 3 weeks." C. "The client will demonstrate correct use of the insulin pump." D. "The client will list three self-care activities that are necessary to control the diabetes."

C. "The client will demonstrate correct use of the insulin pump." Demonstrating the correct use of the administration equipment is a short-term, client-oriented goal that is necessary for the client to control the diabetes and is measurable when the client performs a return demonstration for the nurse. Adhering to the medical regimen is not a short-term goal. Remaining normoglycemic for 3 weeks is measurable, but it is a long-term goal. Although listing three self-care activities that are necessary to control the diabetes is measurable and a short-term goal, it is not the one with the greatest priority when a client has an insulin pump that must be mastered before discharge.

Which response would the nurse give to a client receiving chemotherapy who develops sores in the mouth and asks the nurse why this happened? A. "The sores occur because of the direct irritating effects of the medication." B. "These tissues are poorly nourished because you have a decreased appetite." C. "The frequently dividing cells of the gastrointestinal tract are damaged by the medication." D. "This side effect occurs because it targets the cells of the gastrointestinal system."

C. "The frequently dividing cells of the gastrointestinal tract are damaged by the medication." Many chemotherapeutic agents function by interfering with the DNA replication associated with cellular reproduction (mitosis). Frequent cellular mitosis of the stratified squamous epithelium of the mouth and anus results in these areas being powerfully affected by the medications. The response "The sores occur because of the direct irritating effects of the medication" is inaccurate; most agents are administered parenterally. A decreased appetite (anorexia) does not cause stomatitis. Chemotherapeutic agents affect the most rapidly proliferating cells, which include not only the cells of the gastrointestinal epithelium but also those of the bone marrow and hair follicles.

The nurse is talking with a client who was admitted with an acute myocardial infarction due to coronary artery disease. The clients asks what the purpose for the prescribed carvedilol is. How should the nurse respond? A. "A beta blocker will prevent postural hypotension." B. "Most people develop hypertension after a heart attack." C. "This drug will decrease the workload on your heart." D. "Beta blockers will help to increase your heart rate."

C. "This drug will decrease the workload on your heart." One action of beta blockers is to decrease systemic vascular resistance by dilation of the arterioles. This is useful for clients with coronary artery disease and will reduce the risk of another MI or a sudden cardiac event. Some of the more commonly prescribed beta blockers include metoprolol and carvedilol (Coreg). The other responses are incorrect.

A nurse is educating a client with diabetes type 2 about newly prescribed glipizide. Which statement by the nurse best describes the action of glipizide? A. "This medication absorbs the excess carbohydrates from your intestinal tract." B. "This medication will inhibit the release of glucose stored in the liver." C. "This medication will stimulate your pancreas to release insulin." D. "This medication works by increasing the ability of the cells to uptake glucose."

C. "This medication will stimulate your pancreas to release insulin." Rationale: The action of sulfonylureas, such as glyburide, is to stimulate the pancreas to release insulin. Biguanides, such as metformin, work by decreasing the release of glucose from the liver and increasing the uptake of glucose into the cells. The action of a-glucosidase inhibitors is to decrease the absorption of carbohydrates in the gastrointestinal tract.

A client diagnosed with tuberculosis is prescribed rifampin and isoniazid. Which information should the nurse include when reinforcing information about these medications? A. "You can take the medication with food." B. "You may experience an increase in appetite." C. "You may notice an orange-red color to your urine." D. "You may have occasional problems sleeping."

C. "You may notice an orange-red color to your urine." Rifampin can cause reddish-orange discoloration of the urine and other body fluids, including tears and sweat. This is harmless, but the client needs to be made aware of it. The nurse should caution the client not to wear soft contacts while taking this medication because they can become discolored. The other information does not apply to those two medications.

Which instructions about the use of nitroglycerin to prevent angina will the nurse provide to a client? A. 'At the point when pain first occurs, place two tablets under the tongue.' B. 'Place one tablet under the tongue before activity, and swallow another if pain occurs.' C. 'Before physical activity, place one tablet under the tongue, and repeat the dose in 5 minutes if pain occurs.' D. 'Place one tablet under the tongue when pain occurs and use an additional tablet after the attack to prevent recurrence.'

C. 'Before physical activity, place one tablet under the tongue, and repeat the dose in 5 minutes if pain occurs.' Anginal pain, which can be anticipated during certain activities, may be prevented by dilating the coronary arteries immediately before engaging in the activity. Generally, one tablet is administered at a time; doubling the dosage may produce severe hypotension and headache. The sublingual form of nitroglycerin is absorbed directly through the mucous membranes and should not be swallowed. When the pain is relieved, rest generally will prevent its recurrence by reducing oxygen consumption of the myocardium.

Which instruction would the nurse include when teaching the client about sublingual nitroglycerin? A. 'Once the tablet is dissolved, spit out the saliva.' B. 'Take tablets 3 minutes apart up to a maximum of five tablets.' C. 'Common side effects include headache and low blood pressure.' D. 'Once opened, the tablets should be refrigerated to prevent deterioration.'

C. 'Common side effects include headache and low blood pressure.' The primary side effects of nitroglycerin are headache and hypotension. It is not necessary to spit out saliva into which nitroglycerin has dissolved. For pain that is not relieved, additional tablets may be taken every 5 minutes up to a total of three tablets. It should be stored at room temperature.

The nurse is teaching a 12-year-old child about the action of insulin injections. Which statement indicates the child understands how insulin works in the body? A. 'Glucose is released as fats break down.' B. 'It keeps glucose from being stored in the liver.' C. 'Glucose is carried into cells where it is used for energy.' D. 'It stops the wasting of blood glucose by converting it to glycogen.'

C. 'Glucose is carried into cells where it is used for energy.' Specialized insulin receptors on insulin-sensitive cells transport glucose through cell membranes, making it available for use. Insulin does not break down fats to release glucose, prevent glucose from being stored in the liver, or convert glucose into glycogen.

A 6-month-old infant is to receive scheduled immunizations. The parents ask why two influenza vaccines are given: Haemophilus influenzae type B (Hib) and pneumococcal conjugate vaccine (PCV). Which response by the nurse is appropriate? A. 'PCV prevents influenza.' B. 'Hib is given to prevent pneumonia.' C. 'Hib and PCV prevent different bacterial diseases.' D.'They are given together to protect against viral and bacterial diseases.

C. 'Hib and PCV prevent different bacterial diseases.' Both vaccines protect against bacterial infections. The PCV protects against bacterial pneumonia. The Hib vaccine protects against bacterial infections caused by Haemophilus influenzae type B; these include otitis media, meningitis, epiglottitis, septic arthritis, and sepsis. The PCV conjugate vaccine protects against infections caused by the Streptococcus pneumoniae bacterium (pneumococcal pneumonia).

The nurse provides teaching about ampicillin. Which client statement indicates that additional teaching is needed? A. 'I should take this on an empty stomach with a full glass of water.' B. 'This medicine will work best if I space the time out evenly.' C. 'I can stop this medication after I am symptom-free for 48 hours.' D. 'If I get worse, I will notify my primary health care provider.'

C. 'I can stop this medication after I am symptom-free for 48 hours.' It is most important for the client to complete the full antibiotic prescription to prevent the development of antibiotic-resistant bacteria. Ampicillin should be taken on an empty stomach with a full glass of water. If the medication is spaced out evenly, the ability to maintain a steady therapeutic serum medication level is increased. Because the client has an infection, it is important to report worsening because this may indicate antibiotic failure requiring alternative treatment.

The nurse teaches a teenage client about the administration of levofloxacin to treat a sinus infection. The nurse concludes the teaching is effective when the client makes which statement? A. 'I should take the medication at mealtime.' B. 'I should take the medication just before a meal.' C. 'I should take the medication 1 hour before a meal.' D. 'I should take the medication 30 minutes after a meal.'

C. 'I should take the medication 1 hour before a meal.' Absorption of the oral solution levofloxacin is enhanced when the stomach is empty, and it should be taken 1 hour before meals or 2 hours after meals. Tablets can be taken without regard for food. Food in the stomach will interfere with absorption. If the medication is taken just before a meal, food in the stomach shortly afterward will interfere with absorption. If the medication is taken 30 minutes after a meal, food remaining in the stomach will interfere with absorption.

Isoniazid (INH) is prescribed as a prophylactic measure for a client whose spouse has active (TB). Which statements made by the client indicate that there is a need for further teaching? A. 'I plan to start taking vitamin B 6 with breakfast.' B. 'I'll still be taking this medication 6 months from now.' C. 'I sometimes allow our children to sleep in our bed at night.' D. 'I know I also have tuberculosis because the skin test was positive.' E. 'I plan to attend a wine tasting event this evening.

C. 'I sometimes allow our children to sleep in our bed at night.' D. 'I know I also have tuberculosis because the skin test was positive.' E. 'I plan to attend a wine tasting event this evening.' The children are at an increased risk because the client's spouse has TB; the children should be screened as members of the household. The positive skin test indicates that the client has been exposed to the bacilli and developed antibodies, not necessarily the disease itself; further diagnostic studies are indicated. Wine contains tyramine and histamine, which when taken concurrently with INH can cause headache, flushing, and a drop in blood pressure; these should be avoided when taking INH. Pyridoxine (vitamin B 6) should be taken to prevent neuritis, which is associated with INH. The prophylactic medication therapy will be continued for 6 to 12 months.

A pregnant client with an infection tells the nurse that she has taken tetracycline for infections in the past and prefers to take it now. Which response would the nurse give regarding the avoidance of tetracycline administration during pregnancy? A. 'It affects breast-feeding adversely.' B. 'Tetracycline causes fetal allergies.' C. 'It alters the development of fetal teeth buds.' D. 'It increases fetal tolerance to the medication.'

C. 'It alters the development of fetal teeth buds.' Tetracycline has an affinity for calcium; if used during tooth bud development it may cause discoloration of teeth. Tetracycline does not adversely affect breast-feeding, cause fetal allergies to the medication, or increase fetal tolerance of the medication.

The health care provider prescribes isosorbide dinitrate 10 mg for a client with chronic angina pectoris. The client asks the nurse why the isosorbide dinitrate is prescribed. How will the nurse respond? A. 'It prevents excessive blood clotting.' B. 'It suppresses irritability in the ventricles.' C. 'It decreases cardiac oxygen demand.' D. 'The inotropic action increases the force of contraction of the heart.'

C. 'It decreases cardiac oxygen demand.' Isosorbide dinitrate dilates peripheral veins and arteries thus decreasing preload and decreasing oxygen demand. Preventing blood from clotting is the action of anticoagulants. Suppressing irritability in the ventricles is the action of antidysrhythmics. Increasing the force of contraction of the heart is the action of cardiac glycosides.

Neomycin is prescribed preoperatively for a client with colon cancer. The client asks why this is necessary. Which response would the nurse provide? A. 'It kills cancer cells that may be missed during surgery.' B. 'This medication is helpful in decreasing the inflammatory response associated with surgical procedures.' C.'It kills intestinal bacteria to decrease the

C. 'It kills intestinal bacteria to decrease the risk for infection.' Neomycin is an aminoglycoside antibacterial medication that provides preoperative intestinal antisepsis. Neomycin is not a cancer chemotherapeutic medication; therefore, it does not kill cancer cells. It is not an anti-inflammatory medication; therefore it is not given for that purpose. Antibiotic alteration of body flora increases the risk for superinfections, rather than preventing them.

Which information would the nurse include in the teaching plan on ampicillin? A. 'Take the ampicillin with meals.' B. 'Store the ampicillin in a light-resistant container.' C. 'Notify the health care provider if diarrhea develops.' D. 'Continue the medication until a negative culture is obtained.'

C. 'Notify the health care provider if diarrhea develops.' Diarrhea is a possible side effect that can be related to superinfection or to the destruction of bacterial flora in the intestine; it can lead to fluid and electrolyte imbalance. Ampicillin is absorbed best when taken with water on an empty stomach. Although storage in an airtight container is necessary, protection from light is not. A culture generally is not repeated unless the client's condition indicates that the medication was ineffective.

The clinic nurse receives a call from the mother of an infant prescribed digoxin. The mother reports she forgot whether she gave the morning dose of digoxin. Which response by the nurse is most appropriate? A. 'Give the next dose immediately.' B. 'Wait 2 hours before giving the medication.' C. 'Skip this dose and give it at the next prescribed time.' D. 'Take the baby's pulse and give the medication if it's more than 90 beats/min.'

C. 'Skip this dose and give it at the next prescribed time.' An additional dose may cause overdosage, leading to toxicity; it is better to skip the dose. Giving the dose without waiting may cause an overdose, which could result in toxicity. Even waiting 2 hours may cause an overdose, leading to toxicity. Taking the pulse is not a reliable method for determining a missed dose; 90 to 110 beats/min is within the expected range for this age.

A client with hypertension is prescribed an angiotensin II receptor blocker (ARB). Which instructions will the nurse provide about this medication? Select all that apply. One, some, or all responses may be correct. A. 'Monitor the blood pressure daily.' B. 'Stop treatment if a cough develops.' C. 'Stop the medication if swelling of the mouth, lips, or face develops.' D. 'Have blood drawn for potassium levels 2 weeks after starting the medication.'

C. 'Stop the medication if swelling of the mouth, lips, or face develops.' D. 'Have blood drawn for potassium levels 2 weeks after starting the medication.' The medication should be stopped if angioedema occurs, and the health care provider should be notified. Electrolyte levels of potassium, sodium, and chloride should be obtained 2 weeks after the start of therapy and then periodically thereafter. Daily monitoring of blood pressure is not indicated. A dry cough may occur during treatment with ARBs; however, it is not necessary to discontinue the medication because the cough usually resolves.

A client with a seizure disorder is receiving phenytoin and phenobarbital. Which client statement indicates that the instructions regarding the medications are understood? A. 'I will not have any seizures with these medications.' B. 'These medicines must be continued to prevent falls and injury.' C. 'Stopping the medications can cause continuous seizures and I may die.' D. 'By my staying on the medicines I will prevent postseizure confusion.'

C. 'Stopping the medications can cause continuous seizures and I may die.' Combination therapy suggests that this client has seizures that are difficult to control. Sudden withdrawal of any antiepileptic medication can cause onset of frequent seizures or even status epilepticus. Death can occur if seizures are continuous due to lack of adequate oxygenation and cardiac irregularities. It is important to take medication as prescribed to lessen the frequency of seizures; there is no guarantee that seizures will stop. Medication may or may not eliminate the seizures; stress may precipitate a seizure. Antiepileptic medications are not prescribed to prevent falls and injury and the added central nervous system (CNS) depression increases fall risk. Although seizures may occur while the client is taking the medications, the medications do not stop postseizure confusion.

Which instruction would the nurse give to clients prescribed psychotropic medications who are experiencing anticholinergic-like side effects? A. 'Restrict fluid intake.' B. 'Eat a diet high in carbohydrates.' C. 'Suck on sugar-free hard candies.' D. 'Avoid products that contain aspirin.'

C. 'Suck on sugar-free hard candies.' Hard candy may produce salivation, which helps alleviate the anticholinergic-like side effect of dry mouth that is experienced with some psychotropics. Dry mouth increases the risk for cavities; candy with sugar adds to this risk. Fluids should be encouraged, not discouraged; fluids may alleviate dry mouth. Eating a diet high in carbohydrates and avoiding aspirin are unnecessary.

A child receiving methotrexate and undergoing cranial radiation is very weak. The mother asks the nurse if she should give her child vitamins. Which response by the nurse is most appropriate? A. 'That's an excellent idea. I'll try to get a prescription for her.' B. 'Unfortunately, vitamins won't make her feel any better now.' C. 'That won't be possible. Vitamins interfere with the action of methotrexate.' D. 'After we receive the laboratory reports, your daughter will be getting vitamins.'

C. 'That won't be possible. Vitamins interfere with the action of methotrexate.' Many vitamin supplements contain folic acid, which negates the action of methotrexate, a folic acid antagonist. Vitamin therapy is contraindicated, so the nurse would not try to obtain a prescription, and vitamins will not be prescribed after the laboratory reports have come back. Although vitamins contribute to well-being, stating that the client won't feel any better does not answer the question.

The mother of an infant recently prescribed phenobarbital for seizures calls the pediatric clinic and states that the infant is lethargic and sleeps for long periods. Which response by the nurse is most appropriate? A. 'There's a medication that will prevent this problem.' B. 'This means that your baby's dosage needs to be adjusted.' C. 'This is a temporary response to the medication; it usually stops after a few weeks.' D. 'Many infants experience the same problem, but your baby needs the medication.'

C. 'This is a temporary response to the medication; it usually stops after a few weeks.' Drowsiness is frequently a side effect of barbiturate therapy because it depresses the central nervous system; the infant will adapt to this over time. Stimulants are not routinely administered because they counteract the desired effect of seizure reduction. The dosage does not need adjustment; this response demonstrates little understanding of barbiturate therapy. The mother's concern is with her own baby; the medication's side effects should be explained.

A client prescribed omeprazole for gastroesophageal reflux disease reports a new occurrence of significant diarrhea. Which response by the nurse is most appropriate? A. 'Stop taking your omeprazole.' B. 'This is a normal side effect of omeprazole.' C. 'We are going to collect a stool sample for testing.' D. 'Antidiarrheal medication can be used to decrease this.'

C. 'We are going to collect a stool sample for testing.' Omeprazole has been linked to an increased risk of diarrhea because of Clostridium difficile, so the stool should be tested. The nurse would not instruct the client to stop any medications without consulting the prescribing health care provider. Significant diarrhea in the setting of omeprazole is not a normal finding and should be investigated for Clostridium difficile. Antidiarrheal medication should not be recommended until Clostridium difficile infection is investigated and ruled out.

A client has type 2 diabetes controlled with oral antidiabetic medications. When admitted for elective surgery, the health care provider prescribes regular insulin. Which information would the nurse include when teaching the client about the addition of insulin? A. 'You will need a higher serum glucose level while on bed rest.' B. 'The stress of surgery may cause hypoglycemia.' C. 'With insulin, dosage can be adjusted to your changing needs during recovery from surgery.' D. 'The possibility of surgical complications is greater when a client takes oral hypoglycemics.'

C. 'With insulin, dosage can be adjusted to your changing needs during recovery from surgery.' There is better control of blood glucose levels with short-acting (regular) insulin. The level of glucose must be maintained as close to normal as possible; elevated glucose levels are not desirable for clients on bed rest. The stress of surgery will precipitate hyperglycemia (not hypoglycemia), which is best controlled with exogenous insulin. Oral hypoglycemics do not increase surgical complications.

The client asks the nurse how long she will have to take tamoxifen for breast cancer treatment. Which response by the nurse is appropriate? A. 'You'll have to take it for the rest of your life.' B. 'You'll need to take it for 10 days, like an antibiotic.' C. 'You'll need to take it for 5 years, after which it will be discontinued.' D. 'You'll need to take it for several months, until the bone pain subsides.'

C. 'You'll need to take it for 5 years, after which it will be discontinued.' Tamoxifen is an estrogen antagonist antineoplastic medication that has been found to be effective in 50% to 60% of women with estrogen receptor-positive cancer of the breast. After 5 years of administration there is an increased risk of complications, and the medication is discontinued. Tamoxifen usually is prescribed for 5 years after initiation of therapy, not for the rest of the client's life; this duration will not produce positive effects for the client. Tamoxifen usually is prescribed for 5 years after initiation of therapy, not just for 10 days. Tamoxifen may cause the adverse effect of bone pain, which indicates the medication's effectiveness. Medication is given to manage the pain and the tamoxifen is continued.

Pyridoxine (vitamin B 6) and isoniazid (INH) are prescribed as part of the medication protocol for a client with tuberculosis. Which response indicates that vitamin B 6 is effective? A. Weight gain B. Improvement of stomatitis C. Absence of paresthesias D. Absence of night sweats

C. Absence of paresthesias One of the most common side effects of INH is peripheral neuritis due to vitamin B 6 deficiency, and vitamin B 6 will counteract this problem. Weight gain is not a therapeutic effect of this vitamin. Vitamin B 6 does not affect stomatitis and does not prevent night sweats.

Which effect would the nurse assess for in a client with Alzheimer disease who has been taking galantamine and is newly prescribed paroxetine for depression? A. Allergic B. Dystonic C. Additive D. Extrapyramidal

C. Additive Paroxetine and galantamine potentiate each other's actions. Giving these medications concurrently will not precipitate an allergic reaction. Dystonic effects are associated with antipsychotic medications. Extrapyramidal effects occur with antipsychotic medications.

A client with severe iron-deficiency anemia is prescribed a parenteral form of iron. Which intervention does the nurse prepare to implement before administering the medication? A. Obtain the client's vital signs. B. Use the Z-track administration method. C. Administer a small test dose. D. Obtain informed consent.

C. Administer a small test dose. The most serious adverse effect of iron dextran is an anaphylactic reaction. Although anaphylactic reactions are rare, their possibility demands that iron dextran be used only when clearly required. To reduce this risk, each dose must be preceded by a small test dose and the client must be closely monitored while receiving the test dose. The nurse should be aware that even the test dose can trigger anaphylactic and other hypersensitivity reactions. In addition, even when the test dose is uneventful, patients can still experience anaphylaxis. The medication does not require informed consent and obtaining the client's vital signs does not prevent an anaphylactic reaction. If the medication is ordered to be administered intramuscularly, the Z-track technique should be used to minimize discomfort, leakage and surface discoloration.

A client's dose of isosorbide dinitrate (Imdur) is increased from 40 mg to 60 mg PO daily. When the client reports the onset of a headache prior to the next scheduled dose, which action should the nurse implement? A. Hold the next scheduled dose of Imdur 60 mg and administer a PRN dose of acetaminophen (Tylenol). B. Administer the 40 mg of Imdur and then contact the healthcare provider. C. Administer the 60 mg dose of Imdur and a PRN dose of acetaminophen (Tylenol). D. Do not administer the next dose of Imdur or any acetaminophen until notifying the healthcare provider.

C. Administer the 60 mg dose of Imdur and a PRN dose of acetaminophen (Tylenol). Imdur is a nitrate which causes vasodilation. This vasodilation can result in headaches, which can generally be controlled with acetaminophen until the client develops a tolerance to this adverse effect.

Which nursing intervention would prevent stimulation of the pancreas in a client with acute pancreatitis? A. Maintain the gastric pH at a level of less than 3.5. B. Encourage the resumption of activities of daily living. C. Administer the histamine H 2-receptor antagonist as prescribed. D. Ensure that the nasogastric tube remains in the fundus of the stomach.

C. Administer the histamine H 2-receptor antagonist as prescribed. The histamine H 2-receptor antagonist medication inhibits histamine at H 2-receptor sites in parietal cells, thus decreasing gastric secretion and preventing pancreatic stimulation. A lower pH will stimulate pancreatic secretion, which contains bicarbonate ions that neutralize the acid. The client should rest to decrease stimulation of the pancreas. The tube should be positioned nearer the pylorus for the removal of gastric contents.

A client with type 1 diabetes mellitus has a finger-stick glucose level of 258 mg/dL (14.3 mmol/L) at bedtime. A prescription for sliding-scale regular insulin exists. Which would the nurse do? A. Call the health care provider. B. Encourage intake of fluids. C. Administer the insulin as prescribed. D. Give the client 4 ounces of orange juice.

C. Administer the insulin as prescribed. A value of 258 mg/dL (14.3 mmol/L) is above the expected range of 70 to 100 mg/dL (3.6-5.6 mmol/L); the nurse would administer the regular insulin as prescribed. Calling the health care provider is unnecessary; a prescription for insulin exists and should be implemented. Encouraging the intake of fluids is insufficient to lower a glucose level this high. Giving the client orange juice is contraindicated, because this will increase the glucose level further; orange juice, a complex carbohydrate, and a protein should be given if the glucose level is too low.

Which action would the nurse take when administering tetracycline? A. Administer the medication with meals or a snack. B. Provide orange or other citrus fruit juice with the medication. C. Administer the medication at least an hour before ingestion of milk products. D.Offer antacids 30 minutes after administration if gastrointestinal side effects

C. Administer the medication at least an hour before ingestion of milk products. Any product containing aluminum, magnesium, or calcium ions should not be taken in the hour before or after an oral dose of tetracyclines (with the exception of doxycycline) because it decreases absorption by as much as 25% to 50%. Food interferes with absorption; it should be given 1 hour before or 2 hours after meals. Citrus juice does not improve absorption. Antacids will interfere with absorption.

A nurse is providing education on the use of subcutaneous octreotide to a client who will be administering the medication at home. What will the nurse include in the teaching? A. Store any remaining medication at room temperature B. Inject the medication into the gluteal area C. Administer the medication between meals D. Use the medication immediately after removing it from the refrigerator

C. Administer the medication between meals Rationale: Side effects of octreotide include abdominal pain, ileus, and diarrhea. The client should be instructed to administer the medication between meals to decrease gastrointestinal effects. Unused medication should be discarded. The prescription is for a subcutaneous injection. The gluteal area is indicated for intramuscular injections. The medication should be allowed to reach room temperature before administration to decrease skin reactions at the injection site.

A client with advanced cancer of the bladder is scheduled for a cystectomy and ileal conduit. Which intervention would the nurse anticipate the health care provider will prescribe to prepare the client for surgery? A. Intravesical chemotherapy B. Instillation of a urinary antiseptic C. Administration of an antibiotic D. Placement of an indwelling catheter

C. Administration of an antibiotic Intestinal antibiotics and a complete cleansing of the bowel with enemas until returns are clear are necessary to reduce the possibility of fecal contamination when the bowel is resected to construct the ileal conduit. Intravesical chemotherapy is unnecessary because the urinary bladder is removed with this surgery. Instillation of a urinary antiseptic is not necessary. There is no evidence of a urinary tract infection. The urinary bladder will be removed, so there is no need for an indwelling urinary catheter. No data indicate that the client is experiencing urinary retention before surgery.

The nurse in a urology office is developing a plan of care for a client newly diagnosed with urge urinary incontinence due to an overactive bladder. Which interventions should the nurse include? A. Administer ant-seizure medications B. Administration of cholinergic drugs C. Administration of anticholinergic drugs D. Administration of loop diuretics

C. Administration of anticholinergic drugs Incontinence is an involuntary loss of urine severe enough to cause social or hygienic problems. It is not a normal consequence of aging or childbirth and can have several possible causes and can be either temporary or chronic. Urge incontinence is often referred to as overactive bladder (OAB). Nocturnal frequency and incontinence are common with OAB. Interventions include treatment with drugs that relax the smooth muscle and increase the bladder's capacity including anticholinergics, not anti-seizure, loop diuretics or cholinergic drugs.

A client with type 1 diabetes experiences tremors, pallor, and diaphoresis. These signs and symptoms are manifestations of which cause? A. Overeating B. Viral infection C. Aerobic exercise D. Missed insulin dose

C. Aerobic exercise These responses are indicative of hypoglycemia, which can be caused by increased activity in clients who take insulin. Overeating causes hyperglycemia. Infections cause hyperglycemia because of the release of stress-related hormones. Missing an insulin dose causes hyperglycemia.

The nurse is preparing to administer ceftriaxone to a client. Which of the following findings from the client's medical record should cause the nurse to question this prescription? A. White blood cells in the urine B. History of hypertension C. Allergy to cephalexin D. Current tobacco smoker

C. Allergy to cephalexin Rationale: Ceftriaxone and cephalexin are both cephalosporins; therefore, an allergy to cephalexin should cause the nurse to question any prescription for a cephalosporin. Hypertension and tobacco use do not affect the ability to take ceftriaxone. Elevated white blood counts (WBCs) in the urine indicate a possible infection and may be why antibiotics were prescribed, but this finding should not cause the nurse to be concerned about the medication.

A client with lymphosarcoma is receiving allopurinol and methotrexate. The nurse can help the client prevent complications related to uric acid nephropathy by administering which medication in relation to which changes in fluid intake? A. Allopurinol and restricting the fluid intake B. Methotrexate and restricting fluid intake C. Allopurinol and encouraging increased fluid intake D. Methotrexate and encouraging increased fluid intake

C. Allopurinol and encouraging increased fluid intake Allopurinol decreases serum uric acid levels before and during chemotherapy; increased fluid intake aids in the increased excretion of uric acid. Allopurinol and increased fluids help prevent renal tubular impairment and kidney failure because of hyperuricemia. The client should be encouraged to follow a diet that promotes urine alkalinity. If the oral route is used, administering the methotrexate after providing an antacid will limit gastric irritation, not uric acid nephropathy. Fluid intake should be increased to 2 to 3 liters per day to prevent urate deposits and calculus formation.

Which sedative-hypnotics would the nurse identify as being used to treat insomnia associated with a panic disorder? Select all that apply. One, some, or all responses may be correct. A. Phenelzine B. Paroxetine C. Alprazolam D. Imipramine E. Clonazepam

C. Alprazolam E. Clonazepam Alprazolam and clonazepam are examples of benzodiazepines, a class of sedative-hypnotics used to treat clients with insomnia associated with panic disorders. Phenelzine is a monoamine oxidase inhibitor used to treat panic disorders and promote sleep. Paroxetine is a selective serotonin reuptake inhibitor used to treat panic disorders and promote sleep. Imipramine is a tricyclic antidepressant used to treat panic disorders and promote sleep.

Which primary anxiolytic medication would the nurse anticipate developing a teaching plan for when a client with social anxiety disorder has a history of exhibiting an intense, irrational fear of being scrutinized by others? Select all that apply. One, some, or all responses may be correct. A. Sertraline B. Paroxetine C. Alprazolam D. Venlafaxine E. Clonazepam

C. Alprazolam E. Clonazepam Manifestations of social anxiety disorder include stuttering, sweating, palpitations, dry throat, and muscle tension. Clients with this disorder exhibit an intense, irrational fear of being scrutinized by others. Alprazolam and clonazepam are benzodiazepines that are well tolerated in clients, and the benefits are immediate. Sertraline and paroxetine are selective serotonin reuptake inhibitors that are also used in the treatment of social anxiety disorder, but they do not act quickly. Venlafaxine is used to treat posttraumatic stress disorder.

A client with adenocarcinoma receives doxorubicin intravenously (IV) to reduce the tumor mass. Which clinical finding indicates that doxorubicin toxicity may have occurred? A. Fever B. Blue tinge to the urine C. Alteration in cardiac rhythm D. Increasing anxiety

C. Alteration in cardiac rhythm Doxorubicin is cardiotoxic and causes dysrhythmias. It increases the risk for infections secondary to myelosuppression, which may result in fever; however, this is not the result of toxicity. Blue-tinged urine is a side effect of doxorubicin, not a toxic effect. Feelings of nervousness are a side effect of doxorubicin, not a toxic effect.

Which clinical finding indicates that doxorubicin toxicity may have occurred? A. Fever B. Blue tinge to the urine C. Alteration in cardiac rhythm D. Increasing anxiety

C. Alteration in cardiac rhythm Doxorubicin is cardiotoxic and causes dysrhythmias. It increases the risk for infections secondary to myelosuppression, which may result in fever; however, this is not the result of toxicity. Blue-tinged urine is a side effect of doxorubicin, not a toxic effect. Feelings of nervousness are a side effect of doxorubicin, not a toxic effect.

A client who had a myocardial infarction has runs of ventricular tachycardia. Which medication will the nurse prepare to administer? A. Digoxin B. Furosemide C. Amiodarone D. Norepinephrine

C. Amiodarone Amiodarone decreases the irritability of the ventricles by prolonging the duration of the action potential and refractory period. It is used in the treatment of ventricular dysrhythmias such as ventricular tachycardia. Digoxin slows and strengthens ventricular contractions; it will not rapidly correct ectopic beats. Furosemide, a diuretic, does not affect ectopic foci. Norepinephrine is a sympathomimetic and is not the medication of choice for ventricular irritability.

The cardiac monitor reveals several runs of ventricular tachycardia. Which medication is used to treat this dysrhythmia? A. Atropine B. Epinephrine C. Amiodarone D. Sodium bicarbonate

C. Amiodarone Amiodarone suppresses ventricular activity; therefore it is used for treatment of premature ventricular complexes (PVCs). It works directly on the heart tissue and slows the nerve impulses in the heart. Atropine blocks vagal stimulation; it increases the heart rate and is used for bradycardia, not PVCs. Epinephrine increases myocardial contractility and heart rate; therefore it is contraindicated in the treatment of PVCs. Sodium bicarbonate increases the serum pH level; therefore it combats metabolic acidosis.

A teenager with a deep laceration of his leg does not remember the date of the last tetanus immunization received. The nurse explains that tetanus immunoglobulin and tetanus toxoid are required. Which explanation underlies the nurse's statement? A. Neither medication is effective alone. B. Both eliminate the need for additional medications. C. Antibodies provide protection, whereas the toxoid stimulates a response. D. Tetanus toxoid minimizes the risks related to the tetanus immunoglobulin.

C. Antibodies provide protection, whereas the toxoid stimulates a response. Tetanus immunoglobulin provides immediate protection, whereas the tetanus toxoid initiates an active immune response. Each is effective alone, but the combination is preferred. They do not confer lifelong immunity. After the initial routine immunizations and boosters, it is recommended that the tetanus toxoid be administered every 10 years. Tetanus immunoglobulin does not carry major side effects because it is derived from human serum.

A client has an anaphylactic reaction after receiving intravenous penicillin. Which would the nurse conclude is the cause of this reaction? A. An acquired atopic sensitization occurred. B. There was passive immunity to the penicillin allergen. C. Antibodies to penicillin developed after a previous exposure. D. Genes encoded for allergies cause a reaction on an initial penicillin exposure.

C. Antibodies to penicillin developed after a previous exposure. Hypersensitivity results from the production of antibodies in response to exposure to certain foreign substances (allergens). Earlier exposure is necessary for the development of these antibodies. Hay fever and asthma, not penicillin allergy, are atopic conditions. The reaction is an active, not passive, immune response. Antibodies developed when there was a previous, not first, exposure to penicillin.

Which instructions would the nurse include when teaching parents how to administer liquid iron to their child? Select all that apply. One, some, or all responses may be correct. A. Protect the child from sunlight. B. Administer the medication with food. C. Anticipate that stools tend to be blackish-green. D. Give the medication with a glass of orange juice. E. Have the child drink it through a straw.

C. Anticipate that stools tend to be blackish-green. D. Give the medication with a glass of orange juice. E. Have the child drink it through a straw. Iron thickens the consistency of stools and may turn stools a blackish-green color. Citrus juices contain vitamin C and are acidic, meaning that they increase the absorption of iron. Direct contact with iron stains the teeth. Use of a needleless syringe permits accurate dosing and limited exposure of the teeth to the medication when given to very young children, but adolescents can drink the medication through a straw, which deposits the medication behind the teeth. The child will not experience photosensitivity when undergoing iron therapy. The medication should be taken between meals because it is best absorbed in an environment that has a low pH.

Which action is most important for the nurse to implement prior to the administration of the antiarrhythmic drug adenosine (Adenocard)? A. Assess pupillary response to light. B. Instruct the client that facial flushing may occur. C. Apply continuous cardiac monitoring. D. Request that family members leave the room.

C. Apply continuous cardiac monitoring. Adenosine (Adenocard) is an antiarrhythmic drug used to restore a normal sinus rhythm in clients with rapid supraventricular tachycardia. The client's heart rate should be monitored continuously for the onset of additional arrhythmias while receiving adenosine.

The inpatient hospital nurse is caring for a client with hypokalemia. The health care provider prescribed a potassium intravenous (IV) infusion of 40 mEq potassium chloride in 250 mL normal saline to be infused over 4 hours. The nurse receives the infusion from the pharmacy. Which action should the nurse take next? A. Confirm patency of the peripheral venous access device and start the infusion B. Notify the health care provider of the inappropriate dose of the prescribed IV potassium C. Ask another nurse to verify the prescription, IV solution and serum potassium level D. Ask another nurse to witness the addition of the prescribed potassium to the IV solution

C. Ask another nurse to verify the prescription, IV solution and serum potassium level Since potassium chloride is considered a high alert medication, especially when given IV, having two nurses verify the order and IV bag is recommended. The nurses should compare the supplied IV bag to the prescriber's order. If potassium IV is infused too rapidly or in too high a dose, it can cause dysrhythmias and cardiac arrest. In addition, the second nurse should also verify the client's most recent serum potassium level to ensure that the prescription is appropriate. The prescribed dose and amount of IV solution is within normal range for IV potassium replacement therapy. Potassium should never be added by a nurse to an IV bag.

The nurse is caring for a client with diabetes type I who received a prescribed dose of regular insulin 30 minutes prior to the meal. The client reports nausea and vomiting. Which action should the nurse take? A. Administer another dose of regular insulin B. Encourage the client to eat a small amount of carbohydrates C. Assess blood glucose level D. Notify the healthcare provider

C. Assess blood glucose level Rationale: When a client who has been administered a regular insulin injection vomits, the nurse should monitor blood glucose and frequently assess for signs of hypoglycemia. After 30 minutes, most of the medication would have been absorbed. Any food ingested may be lost, and repeating the dose would further lower glucose levels. Giving intravenous insulin would also lower glucose levels, causing further hypoglycemia. Before the nurse notifies the healthcare provider, the nurse should assess the client's blood glucose level.

A client with an intravenous (IV) infusion containing 40 mEq of potassium reports a stinging pain at the IV site. Which actions will the nurse take? Select all that apply. One, some, or all responses may be correct. A. Restart the IV in a different vein. B. Assist the client through guided imagery. C. Assess the IV site. D. Ask the health care provider for pain medication. E. Verify that the potassium is adequately diluted and not infusing too rapidly.

C. Assess the IV site. E. Verify that the potassium is adequately diluted and not infusing too rapidly. It is important to first make sure that the IV catheter is patent and that there is no infiltration. The potassium dosage is large and can be very irritating to veins if it isn't sufficient diluted or if it infuses too rapidly. A 40-mEq dose should be diluted in at least 1 L of IV solution. Although imagery may help distract the client from discomfort, this response provides no information as to why the stinging sensation is occurring. Asking the provider for an analgesic doesn't address the underlying problem.

Which administration instruction would the nurse give a client prescribed ranitidine 150 mg daily to treat peptic ulcer disease (PUD)? A. As needed B. With meals C. At bedtime D. Before meals

C. At bedtime Ranitidine is typically administered in a single dose at bedtime. This medication is used for 4 to 6 weeks in combination with other therapy; it is not used as needed, with meals, or when indigestion occurs.

A client is admitted to the emergency department with multiple fractures and potential internal injuries. The client's history reveals abuse of multiple medications for the past 8 months.Which medication when withdrawn will cause the most serious life-threatening responses? A. Heroin B. Methadone C. Barbiturates D. Amphetamines

C. Barbiturates Withdrawal from central nervous system depressants, such as barbiturates, is associated with more severe morbidity and mortality. Symptoms begin with anxiety, shakiness, and insomnia; within 24 hours convulsions, delirium, tachycardia, and death may occur. Withdrawal from heroin or methadone is rarely life threatening, but it does cause severe discomfort, including abdominal cramping and diarrhea. Withdrawal from amphetamines is rarely life threatening, but it causes severe exhaustion and depression.

The nurse is reinforcing teaching to a 24-year-old woman receiving acyclovir for a Herpes Simplex Virus type 2 infection. Which instructions should the nurse provide the client with? A. Continue to take prophylactic doses for at least five years after the diagnosis B. Complete the entire course of the medication for an effective cure C. Begin treatment with acyclovir at the onset of symptoms of recurrence D. Stop treatment if she thinks she may be pregnant

C. Begin treatment with acyclovir at the onset of symptoms of recurrence When the client is aware of early symptoms, such as pain, itching or tingling, treatment is very effective. Medications for herpes simplex do not cure the disease. They simply decrease the intensity of the symptoms. Acyclovir (Zovirax) is not known to have an impact on the fetus. Acyclovir should not be taken for preventive purposes, regardless of the date of diagnosis.

A client is scheduled for discharge after surgery. The medical record indicates that the client has not had a bowel movement since before surgery, which was 4 days ago. Which prescribed medication will the nurse administer to ensure a bowel movement before discharge? A. Lactulose B. Docusate sodium C. Bisacodyl suppository D. Psyllium

C. Bisacodyl suppository A bisacodyl suppository should produce results before the client leaves the facility; it usually takes effect in 15 to 60 minutes. Lactulose takes about 24 hours to take effect; docusate sodium takes 1 to 3 days; psyllium takes 12 to 24 hours.

Which antidiarrheal agent should be used with caution in clients taking high dosages of aspirin for arthritis? A. Loperamide (Imodium). B. Probanthine (Propantheline). C. Bismuth subsalicylate (Pepto Bismol). D. Diphenoxylate hydrochloride with atropine (Lomotil).

C. Bismuth subsalicylate (Pepto Bismol). Bismuth subsalicylate (Pepto Bismol) contains a subsalicylate that increases the potential for salicylate toxicity when used concurrently with aspirin (acetylsalicylic acid, another salicylate preparation).

A client who is receiving atenolol for hypertension frequently reports feeling dizzy. Which effect of atenolol is responsible for this response? A. Depleting acetylcholine B. Stimulating histamine release C. Blocking the adrenergic response D. Decreasing adrenal release of epinephrine

C. Blocking the adrenergic response The beta-adrenergic blocking effect of atenolol decreases the heart's rate and contractility; it may result in orthostatic hypotension and decreased cerebral perfusion, causing dizziness. Depleting acetylcholine is not an action of atenolol. Stimulating histamine release is not an action of atenolol. Decreasing adrenal release of epinephrine is not an action of atenolol.

A nurse is assessing a client who started taking prescribed olmesartan 2 weeks ago. Which finding indicates an expected response to the medication? A. Heart rate of 85 beats/min B. Urinary output of 45 ml/hr C. Blood pressure of 125/79 mmHg D. Respiratory rate of 20 breaths/min

C. Blood pressure of 125/79 mmHg Rationale: Olmesartan is an angiotensin II receptor antagonist used in the treatment of hypertension. The expected outcome is to maintain the blood pressure within normal limits. Although within normal limits, the heart rate, urinary output, and respiratory rate are not used to evaluate the efficacy of olmesartan.

A client is prescribed digoxin 0.25 mg by mouth daily. The health care provider has written a new order to give metoprolol tartrate 25 mg twice a day by mouth. In assessing the client prior to administering the medications, which finding should the nurse report to the health care provider? A. Urine output of 50 mL/hour B. Respiratory rate of 16 C. Blood pressure of 94/60 D. Heart rate of 76 BPM

C. Blood pressure of 94/60 Both medications decrease the heart rate. Metoprolol (Lopressor)affects blood pressure. Therefore, the heart rate and blood pressure must be within normal range (HR 60 to 100 BPM and systolic BP greater than 100 mm Hg) in order to safely administer both medications.

The nurse administers a dose of metoprolol for a client. Which assessment is most important for the nurse to obtain? A. Temperature. B. Lung sounds. C. Blood pressure. D. Urinary output.

C. Blood pressure. It is most important to monitor the blood pressure of clients taking this medication because metoprolol is an antianginal, antiarrhythmic, antihypertensive agent.

Donepezil is prescribed for a client who has mild dementia of the Alzheimer type. Which information would the nurse include when discussing this medication with the client and family? A. Fluids should be limited to 4 large glasses per day. B. Constipation should be reported to the primary health care provider immediately. C. Blood tests that reflect liver function will be performed routinely. D. The client's medication dosage may be self-adjusted according to the client's response.

C. Blood tests that reflect liver function will be performed routinely. Donepezil may affect the liver because alanine aminotransferase (ALT) is found predominantly in the liver; most ALT increases indicate hepatocellular disease. Clients taking this medication should have regular liver function tests and report light stools and jaundice to the primary health care provider. Fluids should not be limited, because a side effect of donepezil is constipation. A side effect of constipation is expected; therefore, fluids, high-fiber foods, and exercise should be recommended to help keep the stools soft. The client should not increase or decrease the dosage abruptly; donepezil should be taken exactly as prescribed.

Which laboratory test result would alert the nurse that fluid intake would need to be increased in a child receiving vincristine? A. Urine pH of 6 B. Urine specific gravity of 1.020 C. Blood uric acid level of 7.5 mg/dL D. Blood urea nitrogen level of 15 mg/dL

C. Blood uric acid level of 7.5 mg/dL The normal blood uric acid level for children ranges from 2.5 to 5.5 mg/dL. An increase in the uric acid level caused by the destruction of cells may lead to renal problems; increased fluid intake helps dilute the urine. A urine pH of 6 is within the expected range of 4.5 to 8. A urine specific gravity of 1.020 is within the expected range of 1.005 to 1.030 (usually 1.010-1.025). A blood urea nitrogen level of 15 mg/dL is within the expected range of 5 to 18 mg/dL.

A client receiving cancer chemotherapy asks the nurse why an antibiotic was prescribed. Which tissue affected by chemotherapy will the nurse consider when formulating a response? A. Liver B. Blood C. Bone marrow D. Lymph nodes

C. Bone marrow Prolonged chemotherapy may slow production of leukocytes in bone marrow, thus suppressing the immune system. Antibiotics may be required to help counter infections that the body can no longer handle easily. The liver does not produce leukocytes. Although leukocytes are in both blood and lymph nodes, these cells are more mature than those found in the bone marrow and thus are more resistant to the effects of chemotherapy.

The nurse is monitoring a 4-month-old infant who is prescribed digoxin. The infant's blood pressure is 92/78 mm Hg; resting pulse is 78 beats per minute; respirations are 28 breaths per minute; and serum potassium level is 4.8 mEq/L. The infant is irritable and has vomited twice since receiving the morning dose of digoxin. Which finding is most indicative of digoxin toxicity? A. Irritability B. Vomiting C. Bradycardia D. Dyspnea

C. Bradycardia The most common sign of digoxin toxicity in children is bradycardia which is a heart rate below 100 beats per minute in an infant. Normal resting heart rate for infants 1-11 months-old is 100-160 beats per minute.

Which medication may be useful in managing hypertension in a child with acute glomerulonephritis? A. Digoxin B. Diazepam C. Captopril D. Phenytoin

C. Captopril Captopril, an angiotensin-converting enzyme inhibitor antihypertensive, blocks the conversion of angiotensin I to the constrictor angiotensin II. Digoxin is not an antihypertensive; it increases the contractility and output of the heart. Diazepam is not an antihypertensive; it relaxes skeletal muscle. Phenytoin is not an antihypertensive; it is an anticonvulsant.

Which angiotensin-converting enzyme inhibitors (ACE inhibitors) are appropriate for a client with liver dysfunction? Select all that apply. One, some, or all responses may be correct. A. Ramipril B. Enalapril C. Quinapril C. Captopril D. Lisinopril

C. Captopril D. Lisinopril Captopril and Lisinopril are the best choices for someone with liver dysfunction because they are the only two ACE inhibitors that are not inactive in the administered form and then are metabolized to the active form once they are in the body, usually by the liver. Ramipril, Enalapril, Quinapril, and Benazepril are not good choices for the client with liver dysfunction.

A client diagnosed with breast cancer is prescribed doxorubicin. Which assessment finding would the nurse recognize as a toxic effect of this medication? A. Paralytic ileus B. Red-tinged urine C. Cardiac dysrhythmias D. Increased serum magnesium

C. Cardiac dysrhythmias Doxorubicin has the potential for cardiac toxicity, including changes in heart rhythm. Paralytic ileus is a toxic effect of vincristine, not doxorubicin. Red-tinged urine is a benign side effect, the result of metabolism of the doxorubicin. The magnesium level is not influenced by doxorubicin.

Which clinical finding indicates that a client taking digoxin may have developed digoxin toxicity? A. Constipation B. Decreased urination C. Cardiac dysrhythmias D. Metallic taste in the mouth

C. Cardiac dysrhythmias The development of cardiac dysrhythmias is often a sign of digoxin toxicity. Constipation is not a sign of toxicity; gastrointestinal signs and symptoms of toxicity include anorexia, nausea, vomiting, and diarrhea. Decreased urination is not a sign of toxicity. Digoxin does not cause a metallic taste in the mouth.

Which nursing intervention is a priority for a school-age child with lead poisoning undergoing chelation therapy? A. Scrupulous skin care B. Provision of a high-protein diet C. Careful monitoring of intake and output D. Daily blood sampling for liver function tests

C. Careful monitoring of intake and output Kidney function must be adequate to excrete the lead; if it is not adequate, nephrotoxicity or kidney damage may result. Skin breakdown is not associated with chelation therapy. A high-protein diet is not necessary. Liver damage does not occur with chelation therapy.

The nurse is teaching a client who has a new prescription for sublingual nitroglycerin. Which point should the nurse emphasize? A. Take the medication at the same time each day B. Rest in bed for an hour after taking medication C. Carry the nitroglycerine with you at all times D. Keep the medication bottle in the refrigerator

C. Carry the nitroglycerine with you at all times The medication should be kept in its original dark-colored glass container. Nitroglycerin should be carried by the client at all times so it can be used when anginal pain occurs. When needed, the client should sit and place tablet under his or her tongue. Sitting is safe because the drug can cause lightheadedness or dizziness, but it's not necessary to rest in bed. The client should never pack this and any other medications in a checked a bag when traveling.

Which concept is important to teach a client in relation to why medication cocktails are more effective than a single medication in cancer therapy? A. Medication resistance B. Tumor doubling time C. Cellular growth cycle D. Retained radioactive particles

C. Cellular growth cycle Different medications destroy cells at different stages of their replication; rapidly dividing cells not destroyed by one medication may be destroyed by another medication during a different stage of cell replication. Although certain tumors are medication resistant, it is not the reason for multiple chemotherapeutic medications; medication-resistant tumors may be treated with surgery, radiation therapy, or other methods. The doubling time of the tumor is a factor that influences the length of time chemotherapy will be given, but it is not the reason multiple medications are given. Retained radioactive particles can occur with internal radiation therapy, not chemotherapy.

A client is prescribed alprazolam. Which action must the nurse include in the client assessment during the initiation of therapy? A. Measure the client's urine output. B. Examine the client's pupils daily. C. Check the client's blood pressure. D. Assess the abdomen for distention.

C. Check the client's blood pressure. Orthostatic hypotension is a common side effect of alprazolam that occurs early in therapy, so checking the client's blood pressure is the appropriate action. An alteration in urine output is not a common side effect, but it may occur after prolonged use. Central nervous system depression is not an early side effect, but it may occur after prolonged use; examining the client's pupils daily is not appropriate at this time. Distention is not a common side effect, but distention from constipation may occur after prolonged use.

The health care provider prescribes an oral hypoglycemic medication for the client with type 2 diabetes. Which statement will the nurse need to consider when developing the teaching plan? A. Oral hypoglycemics work by decreasing absorption of carbohydrates. B. Oral hypoglycemics work by stimulating the pancreas to produce insulin. C. Clients taking oral hypoglycemics may subconsciously relax dietary rules to gain a sense of control. D. Serious adverse effects are not a problem for oral hypoglycemics.

C. Clients taking oral hypoglycemics may subconsciously relax dietary rules to gain a sense of control. Taking a tablet may give the client a false sense that the disease is under control, and this can lead to dietary indiscretions. Some oral hypoglycemics work by stimulating the pancreas to produce insulin, others work by decreasing carbohydrate absorption, and others work in a variety of other ways; therefore teaching should be specific to the medication prescribed. Oral hypoglycemic medications can have serious adverse effects.

se of which medication would the nurse identify as a potential risk for hearing impairment in a child? A. Amoxicillin B. Gentamicin C. Clindamycin D. Ciprofloxacin

C. Clindamycin Gentamicin can be ototoxic because of its effects on the eighth cranial nerve. Reactions to amoxicillin are usually allergic in nature. Impaired hearing does not occur with ciprofloxacin or with clindamycin.

Which laboratory values are important for the nurse to monitor in the client who takes zidovudine? A. Cardiac enzymes B. Serum electrolytes C. Complete blood counts (CBCs) D. Urinalysis

C. Complete blood counts (CBCs) Zidovudine can cause anemia, leukopenia, and granulocytopenia; these blood dyscrasias can be life threatening, and the CBC should be monitored. Cardiac enzymes are not affected directly by zidovudine. Serum electrolytes are not affected directly by zidovudine. Monitoring urinalysis findings is not routinely indicated.

Which common side effect will the nurse address in the care plan of a client with cancer receiving the plant alkaloid vincristine? A. Color-blindness B. Anuria C. Constipation D. Hyperphosphatemia

C. Constipation Although most chemotherapy causes diarrhea, vincristine can cause severe constipation, impaction, or paralytic ileus. Visual changes may occur, but color-blindness is not one of them. Polyuria, not anuria, is common. Hyperuricemia may occur, but hyperphosphatemia is not associated with this medication.

The nurse is caring for a client who is receiving procainamide intravenously. It is most important that the nurse monitors which parameter? A. Serum potassium levels B. Hourly urinary output C. Continuous ECG readings D. Neurological signs

C. Continuous ECG readings Procainamide is used to suppress cardiac arrhythmias. When administered intravenously, it must be accompanied by continuous cardiac monitoring.

The nurse is assessing a client who is taking rifampin for the treatment of tuberculosis. Which finding reported by the client should the nurse immediately report to the healthcare provider? A. Blurred vision B. Orange-tinged tears C. Dark amber urine D. Diarrhea

C. Dark amber urine Rationale: Rifampin causes a temporary yellow-orange discoloration of body fluids. Soft contact lenses may be permanently stained. Dark amber urine is an indication of liver dysfunction and should be reported. A major adverse effect of ethambutol, not rifampin, is optic neuritis. Diarrhea is a common side effect of antibiotics and is not the priority in this case.

A client who had a myocardial infarction receives a prescription for a nitroglycerin patch. Which statement would the nurse identify as the purpose of the nitroglycerin patch? A. Decreased heart rate lowers cardiac output. B. Increased cardiac output increases oxygen demand. C. Decreased cardiac preload reduces cardiac workload. D. Peripheral venous and arterial constriction increases peripheral resistance.

C. Decreased cardiac preload reduces cardiac workload. Nitroglycerin reduces cardiac workload by decreasing the preload of the heart by its vasodilating effect. It decreases blood pressure, not heart rate (which may increase to compensate for the decreased blood pressure). It decreases, not increases, oxygen demand. Nitroglycerin dilates, not constricts, peripheral veins and arteries.

A beta blocker is prescribed for the client with persistent ventricular tachycardia. Which response indicates that the beta blocker is working effectively? A. Decreased anxiety B. Reduced chest pain C. Decreased heart rate D. Increased blood pressure

C. Decreased heart rate A decreased heart rate is the expected response to a beta blocker. Beta blockers inhibit the activity of the sympathetic nervous system and of adrenergic hormones, decreasing the heart rate, conduction velocity, and workload of the heart. A beta blocker is not an anxiolytic and does not reduce anxiety. A beta blocker is not an analgesic and does not reduce chest pain. Beta blockers reduce blood pressure.

A client with diabetes asks how exercise will affect insulin and dietary needs. Which effects of exercise would the nurse share? A. Increases the amount of insulin needed and increases the need for carbohydrates B. Increases the amount of insulin needed and decreases the need for carbohydrates C. Decreases the amount of insulin needed and increases the need for carbohydrates D. Decreases the amount of insulin needed and decreases the need for carbohydrates

C. Decreases the amount of insulin needed and increases the need for carbohydrates Exercise increases the uptake of glucose by active muscle cells; carbohydrates are needed to supply energy for the increased metabolic rate associated with exercise. The need for insulin is decreased.

Which outcome does allopurinol produce to prevent acute gouty attacks? A. Promotes uric acid excretion B. Decreases synovial swelling C. Decreases uric acid production D. Prevents crystallization of uric acid

C. Decreases uric acid production Allopurinol interferes with the final steps in uric acid formation by inhibiting the production of xanthine oxidase. This medication does not promote uric acid excretion. It does not affect bone density. Allopurinol has no effect on swelling of the synovial membranes. This medication prevents the synthesis of uric acid, not its crystallization.

Which condition is contraindicated for St. John's wort herbal therapy? A. Anxiety B. Seizures C. Dementia D. Cardiac disease

C. Dementia St. John's wort is contraindicated for dementia; this herbal therapy is used to treat anxiety. Bupropion therapy is contraindicated for seizures. Valerian (Valeriana officinalis) is contraindicated for cardiac disease.

A client is treated with lorazepam for status epilepticus. Which effect of lorazepam is the reason it is given? A. Decreases anxiety associated with seizures B. Promotes rest after the seizure episode C. Depresses the central nervous system (CNS) D. Provides amnesia for the convulsive episode

C. Depresses the central nervous system (CNS) Lorazepam is used to treat status epilepticus because it depresses the CNS. It also functions as an anxiolytic and sedative and can cause anterograde amnesia; however, these are not the reasons it is prescribed for status epilepticus.

) The nurse is planning to teach an adolescent about diabetes and self-administration of insulin. Which would the nurse complete first? A. Establish realistic goals. B. Assess the adolescent's intellectual ability. C. Determine what the adolescent knows about diabetes. D. Gather the equipment that will be needed for the demonstration.

C. Determine what the adolescent knows about diabetes. Before developing and instituting a teaching plan, the nurse must assess the adolescent's attitudes, experience, knowledge, and understanding of the health problem. Before realistic goals can be set there must be an assessment. The adolescent's intellectual ability is only one aspect of the information the nurse must collect and can be assessed as the nurse is determining what the adolescent knows about diabetes. Performing a demonstration is premature until readiness for learning has been established.

Amlodipine is prescribed for a client with hypertension. Which response to the medication will the nurse instruct the client to report to the health care provider? A. Blurred vision B. Dizziness on rising C. Difficulty breathing D. Excessive urination

C. Difficulty breathing Dyspnea may indicate development of pulmonary edema, which is a life-threatening condition. Blurred vision may occur in some people, but it is not life-threatening. Dizziness on rising and excessive urination are common side effects of this medication that are not life-threatening.

A nurse is preparing to administer morning medications to a client with heart failure. The morning lab values are: sodium 142 mEq/L (142 mmol/L), potassium 2.9 mEq/L (2.9 mmol/L), digoxin level 1.4 ng/mL. Which of the following medications should the nurse not administer until after speaking with the health care provider? A. Spironolactone B. Carvedilol (Coreg) C. Digoxin (Lanoxin) D. Ferrous sulfate

C. Digoxin (Lanoxin) Because the potassium levels are low (normal is 3.5 to 5 mEq/L or 3.5 to 5 mmol/L), the nurse should not give the digoxin; hypokalemia can predispose a person to digoxin toxicity. The other medications can be administered. Although carvedilol can increase plasma digoxin concentration, the digoxin level is normal. Spironolactone is a potassium-sparing diuretic and because the potassium level is low, this too can be given. Ferrous sulfate does not affect the given lab values.

A health care provider prescribes mannitol for a client with a head injury. Which mechanism of action is responsible for therapeutic effects of this medication? A. Decreasing the production of cerebrospinal fluid B. Limiting the metabolic requirements of the brain C. Drawing fluid from brain cells into the bloodstream D. Preventing uncontrolled electrical discharges in the brain

C. Drawing fluid from brain cells into the bloodstream Mannitol, an osmotic diuretic, pulls fluid from the brain to relieve cerebral edema. Mannitol's diuretic action does not decrease the production of cerebrospinal fluid. Mannitol does not affect brain metabolism; rest and lowered body temperature reduce brain metabolism. Preventing uncontrolled electrical discharges in the brain is the action of phenytoin sodium, not mannitol.

A nurse is monitoring a pt receiving baclofen (Lioresal) for side effects related to the medication. Which of the following would indicate that the pt is experiencing a side effect? A. Polyuria B. Diarrhea C. Drowsiness D. Muscular excitability

C. Drowsiness Rationale: Baclofen slows the CNS system thus causing drowsiness. The other answers are not side effects of baclofen

Which test would the nurse identify as specific for the diagnosis of myasthenia gravis? A. Electromyography B. Pyridostigmine test C. Edrophonium chloride test D. History of physical deterioration

C. Edrophonium chloride test The edrophonium chloride test uses a medication that is a cholinergic and an anticholinesterase; it blocks the action of cholinesterase at the myoneural junction and inhibits the destruction of acetylcholine. Its action of increasing muscle strength is immediate for a short time. The results of electromyography will be added to the database, but they are nonspecific. Pyridostigmine is a slower-acting anticholinesterase medication that is commonly prescribed to treat myasthenia gravis; edrophonium chloride is used instead of pyridostigmine to diagnose myasthenia gravis because, when injected intravenously, it immediately increases muscle strength for a short time. The results of a history and physical are added to the database, but the data collected are not as definitive as another specific test for the diagnosis of myasthenia gravis.

Why would lactulose be prescribed for a client with a history of cirrhosis of the liver? A. The desire to drink alcohol is decreased. B. Diarrhea is controlled and prevented. C. Elevated ammonia levels are lowered. D. Abdominal distension secondary to ascites is decreased.

C. Elevated ammonia levels are lowered. Lactulose is an ammonia detoxicant. It decreases serum ammonia concentration by preventing reabsorption of ammonia. Lactulose has been used to lower blood ammonia content in clients with portal hypertension and hepatic encephalopathy secondary to chronic liver disease. Lactulose has no effect on the craving for alcohol or reduction of ascites or abdominal distension. Lactulose is also used as a hyperosmotic laxative; therefore it will not relieve diarrhea.

Which effect of povidone-iodine would the nurse consider when using it on the client's skin before obtaining a specimen for a blood culture? A. Avoids drying the skin B. Preferred to alcohol swabs because it doesn't create false-positive blood alcohol results C. Eliminates surface bacteria that may contaminate the culture D. Provides a cooling agent to diminish the feeling from the puncture wound

C. Eliminates surface bacteria that may contaminate the culture Povidone-iodine exerts bactericidal action that helps eliminate surface bacteria that will contaminate culture results. Whether it affects blood alcohol results is irrelevant when used to obtain blood for cultures. It does dry the skin. Although povidone-iodine may provide a cool feeling, this is not a reason for its use.

Which intervention would the nurse take to improve nutrition after identifying that a client receiving chemotherapy has lost weight? Select all that apply. One, some, or all responses may be correct. A. Provide low-carbohydrate meals. B. Decrease fluid intake at mealtime. C. Encourage the intake of preferred foods. D. Promote the intake of small, frequent meals D. Administer prescribed antiemetics before meals

C. Encourage the intake of preferred foods. D. Promote the intake of small, frequent meals D. Administer prescribed antiemetics before meals Selecting preferred foods increases the likelihood of the client eating the food. Small, frequent feedings are better tolerated than large meals. Antiemetics should be administered prophylactically to decrease nausea and enhance appetite. The diet should provide maximum protein and carbohydrates to meet demands related to restoration of body cells and energy. Decreasing fluid intake may have deleterious effects.

A client is experiencing anaphylaxis from an insect sting. Which medication should the nurse administer? A. Dopamine. B. Ephedrine. C. Epinephrine. D. Diphenhydramine.

C. Epinephrine. Epinephrine is an adrenergic agent that stimulates beta receptors to increase cardiac automaticity in cardiac arrest and relax bronchospasms in anaphylaxis. Epinephrine is the medication of choice in treating anaphylaxis.

A client has surgery for the insertion of an implanted infusion port for chemotherapy. How often will the port need to be flushed when not in use? A. Every day B. Once a week C. Every month D. Twice a year

C. Every month Once-a-month flushes usually are adequate to keep an implanted infusion port from clotting. Every day or once a week is unnecessary. Twice a year may jeopardize the viability of the port.

A client with Hodgkin's disease adds doxorubicin to current therapy. Which advice will the nurse provide about this medication? A. Cease taking any medication that contains vitamin D. B. Keep the doxorubicin in a dark place protected from light. C. Expect urine to turn red for a few days after taking this medication. D. Take the doxorubicin on an empty stomach with large amounts of fluids.

C. Expect urine to turn red for a few days after taking this medication. Doxorubicin causes the urine to turn red for a few days; the client should be informed of this expectation so as not to become alarmed when it occurs. Discontinuing the intake of vitamin D is true for plicamycin, not the medications in this protocol. It is unnecessary to keep doxorubicin in a dark area, protected from light. Doxorubicin is not given orally, only via the intravenous route.

A 12-year-old child with cystic fibrosis is prescribed four pancrelipase capsules five times a day. The nurse explains to the child they would take the medication with meals and snacks to accomplish which goal? A. Enhance oxygenation B. Limit excretion of fats C. Facilitate nutrient utilization D. Prevent iron-deficiency anemia

C. Facilitate nutrient utilization Pancreatic enzyme replacement is needed because children with cystic fibrosis cannot manufacture pancreatic enzymes that promote the digestion of food. This results in large amounts of fat in the stool, which can cause bloating and abdominal cramping. Increased oxygenation is not the effect of pancrelipase; pancrelipase contains enzymes to break down fats, proteins, and carbohydrates. Pancrelipase promotes the body's ability to metabolize and absorb fat rather than limit its excretion. The purpose of pancrelipase is not the prevention of anemia.

Which medication would the nurse expect to administer to control bleeding in a child with hemophilia A? A. Albumin B. Fresh frozen plasma C. Factor VIII concentrate D. Factors II, VII, IX, X complex

C. Factor VIII concentrate Factor VIII is the missing plasma component necessary to control bleeding in a child with hemophilia A. Factor VIII is not provided by albumin. Although fresh frozen plasma does contain factor VIII, there is an insufficient amount in a plasma transfusion; a higher volume is required. A complex of factors II, VII, IX, and X is not useful in this situation.

Which vaccine is used to prevent human papilloma virus infection? A. Varivax B. RotaTeq C. Gardasil D. Hepatitis A vaccine

C. Gardasil Gardasil is a quadrivalent vaccine used to prevent genital cancers and warts caused by human papilloma virus. Varivax is associated with protection from the varicella virus; this vaccine is sometimes given in combination with the MMR vaccine. The RotaTeq vaccine is used to vaccinate against a rotavirus infection. The hepatitis A vaccine is used to protect against the hepatitis A virus.

A nurse is teaching parents of a child recently prescribed the medication phenytoin for seizure control. Which side effect will the nurse include? A. Hypertension B. Insomnia C. Gingival hyperplasia D. Increased appetite

C. Gingival hyperplasia Gingival hyperplasia (overgrowth of the gums) is a common side effect of phenytoin. Other common side effects include ataxia, central nervous system depression, drowsiness, headache, hypotension, mental confusion, nausea, vomiting, rash and nystagmus.

A nurse is teaching a parent how to administer oral iron supplements to a 2-year-old child. Which intervention should be included in the teaching? A. Stop the medication if the stools become tarry green B. Add the medicine to a bottle of formula C. Give the medicine with orange juice and through a straw D. Administer the iron with your child's meals

C. Give the medicine with orange juice and through a straw Absorption of iron is facilitated in an environment rich in vitamin C. Because liquid iron preparation will stain teeth, a straw should be used. Parents should be informed that dark, tarry stools are expected outcomes of taking iron supplements. Iron is best absorbed on an empty stomach (but it may be given after meals if the child experiences an upset stomach).

The nurse is reviewing the medical record of a client who received a new prescription for benztropine. For which condition in the client's record should the nurse clarify the prescription with the health care provider? A. Cataracts B. Schizophrenia C. Glaucoma D. Parkinson's disease

C. Glaucoma Benztropine is an anticholinergic medication used to treat extrapyramidal disorders caused by antipsychotic medications or Parkinson's disease. Use of benztropine or other anticholinergics is contraindicated for individuals diagnosed with glaucoma, ileus and prostatic hypertrophy. Adverse effects include tachycardia, urinary retention and increased intraocular pressure.

The nurse is caring for a client who was recently prescribed atropine as a treatment for symptomatic bradycardia. Which condition should the nurse question as a contraindication when taking this medication? A. Urinary incontinence B. Right-sided heart failure C. Glaucoma D. Increased intracranial pressure

C. Glaucoma The nurse should question the use of atropine with a client who has glaucoma. Atropine is contraindicated in clients with angle-closure glaucoma because it can cause pupillary dilation with an increase in aqueous humor. This can lead to an increase in optic pressure causing blurred vision and ocular pain.

A child with iron-deficiency anemia is prescribed oral iron therapy. Anticipatory guidance regarding which side effect would the nurse provide? A. Bloody stool B. Orange urine C. Greenish-black stool D. Staining of the mouth

C. Greenish-black stool Iron is excreted in the feces, and the change in color results from the insoluble iron compound excreted in the stool. Blood in the stool is associated with lower intestinal bleeding, not supplemental iron ingestion. Orange urine is not associated with supplemental iron ingestion; it occurs with phenazopyridine hydrochloride or rifampin administration. Staining of the mucous membranes of the mouth should not occur with oral administration of iron if a straw is used and the teeth are brushed immediately after administration. The teeth, not the mucous membranes, may become stained if these precautions are not taken.

The healthcare provider prescribes a beta-1 agonist medication to be administered. The nurse should anticipate the medication to be prescribed for a client diagnosed with which condition? A. Glaucoma. B. Hypertension. C. Heart failure. D. Asthma.

C. Heart failure. Beta-1 agonists improve cardiac output by increasing the heart rate and blood pressure. They are indicated in heart failure, shock, atrioventricular block dysrhythmias, and cardiac arrest.

A client with supraventricular tachycardia (SVT) has a heart rate of 170 beats/minute. After treatment with diltiazem, which assessment indicates to the nurse that the diltiazem is effective? A. Increased urine output B. Blood pressure of 90/60 mm Hg C. Heart rate of 98 beats/minute D. No longer complaining of heart palpations

C. Heart rate of 98 beats/minute Diltiazem hydrochloride's purpose is to slow down the heart rate. SVT has a heart rate of 150 to 250 beats/minute. A heart rate of 110 beats/minute indicates that the diltiazem hydrochloride is having the desired effect. Hypotension is a side effect of diltiazem hydrochloride, not a desired effect. Heart palpations are experienced by some with various dysrhythmias. A decreased sensation of heart palpations is a positive finding but is not present in all clients. Increased urine output may occur over a period of time because of the increased ventricular filling time but would not occur until after the heart rate had stabilized.

A client with cirrhosis of the liver develops ascites, and the health care provider prescribes spironolactone. The nurse will monitor the client for which adverse medication effect? A. Bruising B. Tachycardia C. Hyperkalemia D. Hypoglycemia

C. Hyperkalemia Spironolactone is a potassium-sparing diuretic that is used to treat clients with ascites; therefore the nurse would monitor the client for signs and symptoms of hyperkalemia. Bruising and purpura are associated with cirrhosis, not with the administration of spironolactone. Spironolactone does not cause tachycardia. Spironolactone does not cause hypoglycemia.

The nurse is counseling a 34-year-old client who has requested a prescription for oral contraceptives. Which condition would warrant additional discussion? A. Anemia B. Depression C. Hypertension D. Dysmenorrhea

C. Hypertension One of the side effects of oral contraceptives is hypertension; therefore they are contraindicated for any woman who already has hypertension, particularly at the client's age or older. Anemia is not a contraindication for women who want to take oral contraceptives because oral contraceptives may help this condition by decreasing bleeding. Depression is not a contraindication for women who want to take oral contraceptives. Oral contraceptives may be prescribed for women with menstrual difficulties such as dysmenorrhea.

A client with human immunodeficiency virus (HIV)-associated Pneumocystis jiroveci pneumonia is to receive pentamidine isethionate intravenously (IV) once daily. The nurse will monitor the client for which adverse effect? A. Hypertension B. Hypokalemia C. Hypoglycemia D. Hypercalcemia

C. Hypoglycemia Pentamidine isethionate can cause either hypoglycemia or hyperglycemia even after therapy is discontinued, and blood glucose levels should be monitored. Hypotension, not hypertension, occurs with pentamidine isethionate. Hyperkalemia, not hypokalemia, occurs with pentamidine isethionate. Hypocalcemia, not hypercalcemia, occurs with pentamidine isethionate.

A client with diabetes presents to the emergency department with a 3-hour history of profound weakness and nervousness. According to the spouse, the client became confused shortly after self-administering the morning dose of 10 units of regular insulin and 25 units of NPH insulin. The client had a light breakfast and no additional intake since that time. Which condition would the nurse identify as the likely cause of the client's signs and symptoms? A. Hyperglycemia B. Hyperinsulinemia C. Hypoglycemia D. Hypoinsulinemia

C. Hypoglycemia Severe hypoglycemia is a finding in diabetic clients who take insulin and miss a meal. Signs and symptoms of hypoglycemia are nervousness, weakness, confusion, and disorientation. Hyperglycemia is rare in clients who are on insulin therapy and decrease their intake. Hyperinsulinemia is a condition where an excess of insulin is produced by the pancreas in response to conditions such as insulin resistance or insulinomas. Hypoinsulinemia refers to abnormally low levels of insulin in the blood.

A client with a diagnosis of acquired immunodeficiency syndrome (AIDS) receives pentamidine for a protozoal infection. The nurse will monitor the client for which common side effects? Select all that apply. One, some, or all responses may be correct. A. Leukocytosis B. Hypokalemia C. Hypoglycemia D. Increased serum calcium E. Decreased blood pressure

C. Hypoglycemia E. Decreased blood pressure Hypoglycemia is a side effect of pentamidine. Hypotension (decreased blood pressure) and dysrhythmias are common side effects of this medication. Neutropenia, not leukocytosis, is associated with this medication. Hyperkalemia, not hypokalemia, may occur. Hypocalcemia, not hypercalcemia, may occur.

A client is receiving furosemide to relieve edema. The nurse will monitor the client for which adverse effect? A. Hypernatremia B. Elevated blood urea nitrogen C. Hypokalemia D. Increase in the urine specific gravity

C. Hypokalemia Furosemide is a potent diuretic used to provide rapid diuresis; it acts in the loop of Henle and causes depletion of electrolytes, such as potassium and sodium. Furosemide inhibits the reabsorption, not retention, of sodium. Furosemide does not affect protein metabolism and will not elevate blood urea nitrogen. Because furosemide increases water excretion relative to solutes, the specific gravity of the fluid more likely will be low.

The nurse is caring for a client with osteomyelitis who is receiving IV infusion of prescribed vancomycin. Which statement by the client would be a priority for the nurse to report to the healthcare provider? A. I fell some burning at the catheter site B. I feel a little nauseous C. I have a ringing in my ears D. I have a headache

C. I have a ringing in my ears Rationale: The nurse who is caring for a client with osteomyelitis who is receiving IV infusion of vancomycin should assess the client for toxicity. The client who reports ringing in the ear could be experiencing ototoxicity, which is an adverse effect of vancomycin and should be reported to the healthcare provider. Headache, nausea, and burning at the IV site are side effects of the medication but not a priority for the nurse to report to the healthcare provider.

The nurse is caring for a client diagnosed with heart failure who will begin treatment with digoxin. Which therapeutic effect would the nurse expect to find after administering this medication? A. Decreased chest pain with decreased blood pressure B. Increased heart rate with increased respirations C. Improved respiratory status with increased urinary output D. Diaphoresis with decreased urinary output

C. Improved respiratory status with increased urinary output Digoxin (Lanoxin), a cardiac glycoside, is used in clients with heart failure to slow and strengthen the heartbeat. As cardiac output is improved, renal perfusion is improved and urinary output increases. The other findings are related to adverse, not therapeutic, effects related to digoxin or are not typically seen at all with digoxin.

The health care provider prescribes cisplatin for a client with metastatic cancer. Which action will the nurse take to prevent toxic effects? A. Ask the client's health care provider about prescribing leucovorin. B. Encourage regular vigorous oral care. C. Increase hydration to promote diuresis. D. Assist the client in selecting foods appropriate for a high-protein, low-residue diet.

C. Increase hydration to promote diuresis. Cisplatin is nephrotoxic and can cause kidney damage unless the client is adequately hydrated. Leucovorin, a form of folic acid, is used to combat toxic effects of methotrexate; cisplatin does not interfere with folic acid metabolism. Gentle, not vigorous, oral care is needed to cleanse the mouth without further aggravating the expected stomatitis. A low-residue diet is unnecessary. Prolonged gastrointestinal irritation is not the major concern.

Digoxin is prescribed for a client. Which therapeutic effect of digoxin would the nurse expect? A. Decreased cardiac output B. Decreased stroke volume of the heart C. Increased contractile force of the myocardium D. Increased electrical conduction through the atrioventricular (AV) node

C. Increased contractile force of the myocardium Digoxin produces a positive inotropic effect that increases the strength of myocardial contractions and thus cardiac output. The positive inotropic effect of digoxin increases, not decreases, cardiac output. Digoxin increases the strength of myocardial contractions (positive inotropic effect) and slows the heart rate (negative chronotropic effect); these effects increase the stroke volume of the heart. Digoxin decreases the refractory period of the AV node and decreases conduction through the sinoatrial (SA) and AV nodes.

A client with a history of methamphetamine use is admitted to the medical unit. Which clinical manifestation would the nurse expect when assessing the client who is under the influence on admission? A. Constricted pupils B. Intractable diarrhea C. Increased heart rate D. Decreased respirations

C. Increased heart rate Methamphetamine is a stimulant that causes the release of adrenaline, which activates the sympathetic nervous system and causes increased heart rate. The pupils will dilate, not constrict, because the sympathetic nervous system is activated. Clients withdrawing from opioids, not methamphetamine, experience diarrhea. The respirations will increase, not decrease, because of the activation of the sympathetic nervous system.

The nurse is preparing to administer a client's prescribed NPH and regular insulins. Which action should the nurse take first when mixing the insulins in one syringe? A. Draw up the NPH B. Draw up the regular insulin C. Inject air into the NPH D. Inject air into the regular insulin

C. Inject air into the NPH Rationale: When mixing insulins in the same vial, the process should be to inject air into the long-acting insulin, inject air into the short-acting insulin, draw up the short-acting insulin, and then draw up the long-acting insulin.

Which nursing intervention is most important when caring for a client receiving the antimetabolite cytosine arabinoside (Arc-C) for chemotherapy? A. Hydrate the client with IV fluids before and after infusion. B. Assess the client for numbness and tingling of extremities. C. Inspect the client's oral mucosa for ulcerations. D. Monitor the client's urine pH for increased acidity.

C. Inspect the client's oral mucosa for ulcerations. Cytosine arabinoside (Arc-C) affects the rapidly growing cells of the body, therefore stomatitis and mucosal ulcerations are key signs of antimetabolite toxicity.

Which insulin will the nurse prepare for the emergency treatment of ketoacidosis? A. Glargine B. NPH insulin C. Insulin aspart D. Insulin detemir

C. Insulin aspart Insulin aspart is a rapid-acting insulin (within 10-20 minutes) and is used to meet a client's immediate insulin needs. Glargine is a long-acting insulin, which has an onset of 1.5 hours; for diabetic acidosis, the individual needs rapid-acting insulin. NPH insulin is an intermediate-acting insulin, which has an onset of 1 to 2 hours; for diabetic acidosis, the individual needs rapid-acting insulin. Insulin detemir is a long-acting insulin; for diabetic acidosis, the individual needs rapid-acting insulin.

Which complication of diabetes would the nurse suspect when a health care provider prescribes one tube of glucose gel for a client with type 1 diabetes? A. Diabetic acidosis B. Hyperinsulin secretion C. Insulin-induced hypoglycemia D. Idiosyncratic reactions to insulin

C. Insulin-induced hypoglycemia Glucose gel delivers a measured amount of simple sugars to provide glucose to the blood for rapid action. Acidosis occurs when there is an increased serum glucose level; therefore glucose gel is not indicated. Diabetes mellitus involves a decreased insulin production. Glucose gel is not indicated in idiosyncratic reactions to insulin.

The nurse is to administer a muscle relaxant, succinylcholine, just before a client undergoes electroconvulsive therapy. Which muscles does this medication inhibit? A. Biceps and triceps B. Facial and thoracic C. Intercostal and diaphragmatic D. Sternocleidomastoid and abdominal

C. Intercostal and diaphragmatic Succinylcholine causes paralysis of muscles including the intercostals and diaphragm, so artificial support of respiration is required to sustain life during the procedure. Inhibition of the biceps, triceps, facial, thoracic, sternocleidomastoid, and abdominal muscles is the purpose of the medication, not a disadvantage of it.

The health care provider prescribes propylthiouracil (PTU) for a client with hyperthyroidism. Which action of PTU will the nurse include in teaching? A. Increases the uptake of iodine B. Causes the thyroid gland to atrophy C. Interferes with the synthesis of thyroid hormone E. Decreases the secretion of thyroid-stimulating hormone (TSH)

C. Interferes with the synthesis of thyroid hormone PTU, used in the treatment of hyperthyroidism, blocks the synthesis of thyroid hormones by preventing iodination of tyrosine. Propylthiouracil does not increase the uptake of iodine. Iodine solutions reduce the size and vascularity of the thyroid gland. TSH, secreted by the anterior pituitary, is not affected by propylthiouracil.

The nurse is administering an osmotic diuretic to a client with a traumatic brain injury. Which finding best indicates that the medication was effective? A. 250 mL clear, yellow urine output over four hours B. Clear bilateral lung sounds to posterior auscultation C. Intracranial pressure reading of 14 mmHg D. Bilateral ovoid pupils that are slow to constrict

C. Intracranial pressure reading of 14 mmHg Osmotic diuretics, such as mannitol, are used to reduce intracranial or intraocular pressure. Intracranial pressure (ICP) for a client with a head injury should be less than 20 mmHg and the osmotic diuretic may be administered to reduce a high ICP. The osmotic diuretic will reduce the amount of water normally reabsorbed by the renal tubules and loop of Henle, so urinary output is increased, which is an expected occurrence, but does not indicate effectiveness of the medication. Ovoid pupils may indicate the presence of cerebral hypertension. An osmotic diuretic is not intended to reduce pulmonary edema, thus clear lung sounds are not an indicator for effectiveness of the diuretic for this particular client.

Which medication therapy is indicated for management of Wernicke encephalopathy associated with Korsakoff syndrome? A. Traditional phenothiazines B. Judicious use of antipsychotics C. Intramuscular injections of thiamine D. Oral administration of chlorpromazine

C. Intramuscular injections of thiamine Thiamine is a coenzyme necessary for the production of energy from glucose. If thiamine is not present in adequate amounts, nerve activity is diminished and damage or degeneration of myelin sheaths occurs. A traditional phenothiazine is a neuroleptic antipsychotic that should not be prescribed because it is hepatotoxic. Antipsychotics must be avoided; their use has a higher risk of toxic side effects in older or debilitated persons. Chlorpromazine, a neuroleptic, cannot be used because it is severely toxic to the liver.

When discussing the therapeutic regimen of vitamin B 12 for pernicious anemia with a client, which teaching would the nurse provide? A. Weekly Z-track injections provide needed control. B. Daily intramuscular injections are required for control. C. Intramuscular injections once a month will maintain control. D. Oral vitamin B 12 tablets taken daily will provide symptom control.

C. Intramuscular injections once a month will maintain control. Intramuscular injections bypass the vitamin B 12 absorption defect (lack of intrinsic factor, the transport carrier component of gastric juices). A monthly dose usually is sufficient because it is stored in active body tissues, such as the liver, kidney, heart, muscles, blood, and bone marrow. The Z-track method need not be used. Because it is stored and only slowly depleted, daily injections are unnecessary. Vitamin B 12 is ineffective if taken by mouth because clients with pernicious anemia lack intrinsic factor.

Which medication is considered first-line therapy for an infant with congenital syphilis? A. Vidarabine B. Pyrimethamine C. Intravenous (IV) penicillin D. Trimethoprim-sulfamethoxazole

C. Intravenous (IV) penicillin IV penicillin destroys the cell wall of Treponema pallidum, the causative organism of syphilis. Vidarabine is an antiviral medication; it does not treat congenital syphilis in an infant. Pyrimethamine and trimethoprim-sulfamethoxazole are ineffective in the treatment of syphilis.

A client is prescribed aluminum hydroxide for peptic ulcer disease. Which statement by the client demonstrates an understanding of the action of the medication? A. It decreases the production of gastric secretions. B. It produces an adherent barrier over the ulcer. C. It helps maintain a gastric pH of 3.5 or above. D. It slows down the gastric motor activity.

C. It helps maintain a gastric pH of 3.5 or above. The objective of antacids is to neutralize gastric acids and keep a gastric pH of 3.5 or above, which is necessary for pepsinogen inactivity.

The health care provider prescribes metformin as monotherapy for the client with type 2 diabetes. The nurse will teach the client to monitor for which adverse effect? A. Weight gain B. Constipation C. Lactic acidosis D. Hypoglycemia

C. Lactic acidosis Metformin carries a black box warning regarding the possibility of lactic acidosis; clients must know how to monitor for this condition. An advantage of metformin over some other antidiabetic medications is that it does not cause weight gain and may actually result in weight loss for some clients. Constipation is not a problem, but many clients will develop diarrhea initially. Metformin does not increase pancreatic production of insulin and, when used without other antidiabetic medications, will not cause hypoglycemia.

A client with advanced liver disease has been taking rifaximin. Which assessment finding would indicate that the medication is being effective? A. Less jaundice B. Increased appetite C. Less confusion D. Less edema

C. Less confusion Clients with advanced liver disease experience elevated serum ammonia levels which typically lead to hepatic encephalopathy. Signs and symptoms of hepatic encephalopathy include personality changes, confusion, restlessness and forgetfulness. Rifaximin is an antibiotic that helps reduce ammonia levels and hepatic encephalopathy by stopping the growth of bacteria and production of ammonia in the GI tract. Lessening confusion would indicate that the medication is being effective.

A client is receiving combination chemotherapy for the treatment of metastatic carcinoma. For which systemic side effect would the nurse monitor the client? A. Ascites B. Nystagmus C. Leukopenia D. Polycythemia

C. Leukopenia Leukopenia, a reduction in white blood cells, is a systemic effect of chemotherapy as a result of myelosuppression. Ascites is not a side effect of chemotherapy. Chemotherapy does not affect the eyes; nystagmus is an involuntary, rapid rhythmic movement of the eyeballs. Also, nystagmus is a local, not a systemic, response. The red blood cells will be decreased, not increased.

Which systemic side effect would the nurse monitor for in a client receiving combination chemotherapy for the treatment of metastatic carcinoma? A. Ascites B. Nystagmus C. Leukopenia D. Polycythemia

C. Leukopenia Leukopenia, a reduction in white blood cells, is a systemic effect of chemotherapy as a result of myelosuppression. Ascites is not a side effect of chemotherapy. Chemotherapy does not affect the eyes; nystagmus is an involuntary, rapid rhythmic movement of the eyeballs. Also, nystagmus is a local, not a systemic, response. The red blood cells will be decreased, not increased.

A client has been diagnosed with hypothyroidism. Which medication should the nurse administer to treat the client's bradycardia? A. Epinephrine B. Adenosine C. Levothyroxine D. Atropine

C. Levothyroxine The treatment for bradycardia from hypothyroidism is to treat the hypothyroidism using levothyroxine sodium, a T4 replacement hormone. If the heart rate were so slow that it causes hemodynamic instability, then atropine or epinephrine might be an option for short-term management. Adenosine slows atrioventricular (AV) conduction in the heart and would be contraindicated for a client with bradycardia.

Which antidiarrheal medication would the nurse anticipate administering to a client with severe diarrhea who is prescribed intravenous fluids, sodium bicarbonate, and an antidiarrheal medication? A. Psyllium B. Bisacodyl C. Loperamide D. Docusate sodium

C. Loperamide Loperamide inhibits peristalsis and prolongs transit time by its effect on the nerves in the muscle wall of the intestines. Psyllium is not an antidiarrheal; it is a bulk laxative that promotes easier expulsion of feces. Bisacodyl is a laxative, not an antidiarrheal; it increases gastrointestinal motility. Docusate sodium corrects constipation, not diarrhea; water and fat are increased in the intestine, permitting easier expulsion of feces.

The nurse is caring for a client prescribed furosemide and digoxin for the treatment of heart failure. The client reports seeing halos and bright lights. Which laboratory result would be anticipated? A. Low sodium level B. Low digitalis level C. Low potassium level D. Low serum osmolality

C. Low potassium level Rationale: Digitalis toxicity is an accumulation of digitalis (digoxin) in the body that leads to nausea, vomiting, visual disturbances, atrial or ventricular tachydysrhythmias, ventricular fibrillation, sinoatrial block, and atrioventricular block. Clients with heart failure who take digoxin are commonly given diuretics. Hypokalemia can increase the risk of digitalis toxicity. Digitalis toxicity may also develop in the presence of hypomagnesemia. Clients with dig toxicity would have elevated digoxin levels. Sodium would likely be normal. The serum osmolality would likely be normal or high in a client on a diuretic.

A client who was prescribed atorvastatin (Lipitor) one month ago calls the triage nurse at the clinic complaining of muscle pain and weakness in his legs. Which statement reflects the correct drug-specific teaching the nurse should provide to this client? A. Increase consumption of potassium-rich foods since low potassium levels can cause muscle spasms. B. Have serum electrolytes checked at the next scheduled appointment to assess hyponatremia, a cause of cramping. C. Make an appointment to see the healthcare provider, because muscle pain may be an indication of a serious side effect. D. Be sure to consume a low-cholesterol diet while taking the drug to enhance the effectiveness of the drug.

C. Make an appointment to see the healthcare provider, because muscle pain may be an indication of a serious side effect. Myopathy, suggested by the leg pain and weakness, is a serious and potentially life-threatening complication of Lipitor, and should be evaluated immediately by the healthcare provider.

Which vaccine is contraindicated for a child undergoing chemotherapy? A. Influenza (Hib) B. Hepatitis B (Hep B) C. Measles, mumps, rubella (MMR) D. Diphtheria, tetanus, acellular pertussis (DTaP)

C. Measles, mumps, rubella (MMR) The MMR vaccine contains an attenuated live virus and should not be administered to a child undergoing chemotherapy because of the compromise to the child's immune system. There are no contraindications to administering the Hib, Hep B, or DTaP vaccines to a child who is immunosuppressed.

A client who is a smoker expresses a desire to postpone her first pregnancy for at least 5 years and declines to use a barrier method. Which method would the nurse anticipate providing education for? A. A birth control patch B. A vaginal ring C. Medroxyprogesterone D. Combined oral contraceptive pills

C. Medroxyprogesterone Medroxyprogesterone is a long-acting progestin-only contraceptive that is less likely to cause cardiovascular problems in women who smoke than contraceptives containing estrogen might. Vaginal rings, combined oral contraceptive pills, and the birth control patch all contain estrogen and are not recommended for women who smoke.

A client who was admitted with a diagnosis of acute lymphoblastic leukemia is receiving chemotherapy. Which assessment findings would alert the nurse to the possible development of thrombocytopenia? Select all that apply. One, some, or all responses may be correct. A. Fever B. Diarrhea C. Melena D. Hematuria E. Ecchymosis

C. Melena D. Hematuria E. Ecchymosis Thrombocytopenia is a condition characterized by abnormally low levels of thrombocytes, also known as platelets, in the blood. This reduction in platelet activity impairs blood clotting, so any assessment finding associated with potentially abnormal bleeding would alert the nurse to the possibility of thrombocytopenia. This includes melena (digested blood in feces), hematuria (bleeding within the renal system), and ecchymosis (bleeding into skeletal soft tissue). Fever and diarrhea are common side effects of chemotherapy but are not findings specifically attributed to thrombocytopenia.

A client who has a long history of medication and alcohol abuse mentions taking ginkgo biloba. Which condition is this client taking ginkgo biloba to treat? A. Insomnia B. Depression C. Memory impairment D. Anxiety and nervousness

C. Memory impairment Ginkgo biloba is an herb used to treat age-related memory impairment and dementia. It has not been shown to be effective in treating insomnia, depression, or anxiety.

Which medications are immunosuppressives prescribed to prevent kidney rejection? Select all that apply. One, some, or all responses may be correct. A. Cyclosporine B. Methotrexate C. Methylprednisolone D. Tacrolimus E. Mycophenolate mofetil

C. Methylprednisolone D. Tacrolimus E. Mycophenolate mofetil Standard triple therapy includes a corticosteroid prednisone (methylprednisolone), an antimetabolite (mycophenolate), and a calcineurin inhibitor (tacrolimus or cyclosporine). Methotrexate is a folic acid antagonist used in cancer chemotherapy and to manage severe rheumatoid arthritis.

The nurse is caring for a client who recently received an allogeneic bone marrow transplant for the treatment of leukemia. Which nursing intervention is a priority for this client? A. Provide education on infection prevention in the community B. Assist the client with ambulation every 2 hours C. Monitor the client for signs of infection D. Introduce the client to another bone marrow recipient

C. Monitor the client for signs of infection Leukemia is cancer that results in the uncontrolled production of immature WBCs (" blast" cells) in the bone marrow. Hematopoietic stem cell transplantation (HSCT), also called bone marrow transplantation (BMT), is standard treatment for the patient with leukemia who has a closely matched donor, e.g., a sibling (allogeneic). The client has an impaired immune system due to the diagnosis of leukemia and the treatment related to the bone marrow transplant, placing the client at an increased risk for infections. The priority if for the nurse to monitor for signs of infection including a temperature above 100.5 °F (38 °C), chills and cough. The other options are also appropriate for this client; however, they are not the priority immediately following a bone marrow transplant.

The nurse incorrectly administers carvedilol (Coreg) to a client with an order for benztropine (Cogentin). What is the priority nursing intervention after making this medication error? A. Complete an incident report B. Notify the nurse manager C. Monitor the client's blood pressure D. Notify the health care provider

C. Monitor the client's blood pressure Because the nurse mistakenly administered a beta blocker medication, the priority intervention is to monitor the client for any adverse physiological response to the given drug. Carvedilol blocks alpha1 and beta receptors in blood vessels, causing dilation and a decrease in blood pressure.

A health care provider prescribes enalapril for a client. Which nursing action is important? A. Assess the client for hypokalemia. B. Monitor for adverse effects on renal function. C. Monitor the client's blood pressure during therapy. D. Assess the client for hypoglycemia.

C. Monitor the client's blood pressure during therapy. Enalapril is an antihypertensive. A lowering of the client's blood pressure reflects a therapeutic response and needs to be monitored regularly. The client may be at risk for hyperkalemia, not hypokalemia. Enalapril has renal protective effects rather than adverse renal effects. Although an antihypertensive of choice for some clients with diabetes, it does not affect glucose levels.

A client has a tonic-clonic seizure caused by an overdose of aspirin. Which action would the nurse take next? A. Check reflexes every 2 hours. B. Insert a urinary retention catheter. C. Monitor vital signs every 15 minutes. D. Prepare a setup for a central venous pressure (CVP) line.

C. Monitor vital signs every 15 minutes. Because of the lethal toxicity of an aspirin overdose, hypotensive crisis and cardiac irregularities can occur. The central nervous system is not involved at the reflex level at this time. Inserting a urinary retention catheter is not the priority at this time. CVP readings are not indicated in this situation.

The nurse is caring for a client who is being treated for heart failure. After completing the medication reconciliation process, the nurse notes that the prescriber has added lisinopril 5mg orally bid. Which medication from the list below should the nurse question due to possible drug-to-drug interaction with lisinopril? A. Metoprolol B. Glipizide C. Naproxen D. Enoxaparin

C. Naproxen Nonsteroidal anti-inflammatory (NSAIDs) drugs, such as naproxen, reduce the antihypertensive effects of angiotensin converting enzyme (ACE) inhibitors such as lisinopril. The use of NSAIDs and ACE inhibitors may also predispose patients to develop acute renal failure. Additionally, naproxen increases the risk of heart attack or stroke with heart disease. The nurse should clarify the naproxen with the health care provider. The other medications are not known to interact with lisinopril.

) A client with type 1 diabetes self-administers neutral protamine Hagedorn (NPH) insulin every morning at 8:00 AM. The nurse evaluates that the client understands the action of the insulin when the client identifies which time range as the highest risk for hypoglycemia? A. 9:00 AM to 10:00 AM B. 10:00 AM to 11:00 AM C. Noon to 8:00 PM D. 8:00 PM to midnight

C. Noon to 8:00 PM The time of greatest risk for hypoglycemia occurs when the insulin is at its peak. The action of intermediate-acting insulin peaks in 4 to 12 hours. Nine to 10:00 AM and 10:00 AM to 11:00 AM are too soon for NPH to produce a hypoglycemic response. NPH insulin will have produced a hypoglycemic response before 8:00 PM and after noon. A hypoglycemic response that occurs in 45 to 60 minutes after administration is associated with rapid-acting insulins.

An 80-year-old client with depression requires the prescription of antidepressant medication. Which tricyclic antidepressant medication causes fewer complications in older clients? A. Doxepin B. Amoxapine C. Nortriptyline D. Trimipramine

C. Nortriptyline Nortriptyline and desipramine are preferred for use in older adults because these antidepressant medications have less anticholinergic activity. Doxepin, amoxapine, and trimipramine have more cholinergic activity than nortriptyline and are not the preferred medications for older adult clients.

A client hospitalized for uncontrolled hypertension and chest pain was started on a daily diuretic 2 days ago upon admission, with prescriptions for a daily basic metabolic panel. The client's potassium level this morning is 2.7 mEq/L (2.7 mmol/L). Which action will the nurse take next? A. Send another blood sample to the laboratory to retest the serum potassium level. B. Notify the health care provider that the potassium level is above normal. C. Notify the health care provider that the potassium level is below normal. D. No action is required because the potassium level is within normal limits.

C. Notify the health care provider that the potassium level is below normal. The health care provider should be notified immediately because the client's potassium is below normal. The normal potassium level range is 3.5 mEq/L to 5.0 mEq/L (3.5-5.0 mmol/L). Clients on diuretic therapy require close monitoring of their electrolytes because some can cause hypokalemia, whereas others spare potassium, which can cause hyperkalemia. Retesting the serum potassium level is unnecessary and will delay the treatment required by the client.

A nurse is providing discharge education on the use of sustained-release procainamide to a client with newly diagnosed atrial flutter. What will the nurse include in the teaching? A. You will need to have laboratory blood tests performed every 3 months B. Hold the medication if your heart rate is below 70 beats/min C. Notify your healthcare provider if you begin experiencing joint pain D. Crush your medication and mix it with food to mask the taste

C. Notify your healthcare provider if you begin experiencing joint pain Rationale: Procainamide is an antiarrhythmic medication used in the management of atrial flutter. One of the adverse effects of procainamide is systemic lupus syndrome characterized by fever and painful joints. The client should be instructed to notify the healthcare provider of any adverse symptoms. Lab tests need to be conducted frequently (every week) at the start of therapy to monitor complete blood counts and procainamide blood levels. Procainamide does not have a direct effect on heart rate. Sustained-release medications should not be crushed or chewed.

After completing a week of antibiotic therapy, an infant develops oral thrush. Which medication is indicated for treatment of this condition? A. Acyclovir B. Vidarabine C. Nystatin D. Fluconazole

C. Nystatin White, adherent patches on the tongue, palate, and inner aspects of the infant's cheeks indicate oral candidiasis (thrush). Oral candidiasis is caused by a fungus called Candida albicans. Nystatin is an antifungal agent prescribed to treat oral thrush in an infant. Acyclovir and vidarabine are antiviral agents and are not used to treat oral candidiasis in the infant. Fluconazole can effectively treat oral thrush, but its use in infants is not approved by the US Food and Drug Administration.

Which drink would the nurse instruct a client with iron deficiency anemia to choose to drink with the supplement for efficient absorption? A. Water B. Skim milk C. Orange juice D. A strawberry milkshake

C. Orange juice Iron should be taken before breakfast on an empty stomach to permit maximal absorption; the ascorbic acid in orange juice enhances the absorption of iron. Water does not provide the ascorbic acid necessary for absorption of iron. Iron should not be taken with milk or other dairy products, which may interfere with its absorption.

A client with insomnia takes a new medication to promote sleep. Two hours later, the client is wide awake and feeling more energized than before the medication was taken. Which type of response is the client demonstrating? A. Allergic B. Synergistic C. Paradoxical D. Idiosyncratic

C. Paradoxical A paradoxical response to a medication is directly opposite to the desired therapeutic response. An allergic alternatives that you believe are definitely incorrect and reread the information given to make sure you understand the intent of the question. This approach increases response is an antigen-antibody reaction. A synergistic response involves medication combinations that enhance each other. Idiosyncratic responses to a medication are unpredictable and unrelated to the medication purpose. They are commonly attributed to immune responses or genetic factors.

The nurse is administering spironolactone for a client diagnosed with cirrhosis of the liver and ascites. Which electrolyte should the nurse anticipate to be spared when giving this medication? A. Sodium B. Phosphate C. Potassium D. Albumin

C. Potassium Spironolactone is a potassium-sparing diuretic. Indications for this medication include edema associated with heart failure, cirrhosis, and nephrotic syndrome. The nurse should anticipate that potassium is spared and should watch for signs of heart arrhythmias if the potassium is too elevated. This type of diuretic inhibits the action of aldosterone on the kidneys, which does not allow the body to reabsorb sodium. An adverse effect could be hyponatremia. This medication has no effects on phosphate and albumin is not an electrolyte.

Which client would benefit most from the administration of prophylactic antibiotics? Select all that apply. One, some, or all responses may be correct. A. Chickenpox infection B. Fever of unknown origin C. Preoperative hip replacement D. Congenital bicuspid aortic valve E. Current chemotherapy treatment

C. Preoperative hip replacement D. Congenital bicuspid aortic valve E. Current chemotherapy treatment Prophylactic antibiotics are indicated in the preoperative hip replacement client because this decreases the occurrence of infection postoperatively. Prophylactic antibiotics are indicated for the client with congenital bicuspid aortic valve disease because this decreases the risk of endocarditis with an invasive procedure. Prophylactic antibiotics are indicated for the current chemotherapy treatment client because this decreases the risk of infection due to neutropenia. A client with the chickenpox infection has a viral infection for which antibiotics are ineffective. A fever of unknown origin should not be treated with antibiotics because that may eliminate the ability to discover the causative organism, and a virus could be the cause of the fever.

The nurse adds 20 mEq of potassium chloride to the intravenous solution of a client with diabetic ketoacidosis. Which purpose would this medication serve? A. Treats hyperpnea B. Prevents flaccid paralysis C. Prevents hypokalemia D. Treats cardiac dysrhythmias

C. Prevents hypokalemia Once treatment with insulin for diabetic ketoacidosis is begun, potassium ions reenter the cell, causing hypokalemia; therefore potassium, along with replacement fluids, is needed to prevent hypokalemia. Potassium will not correct hyperpnea. Flaccid paralysis does not occur in diabetic ketoacidosis. There is no mention of dysrhythmias in the scenario; they are not a universal finding in diabetic ketoacidosis (and are commonly absent) and hypokalemia does not always cause these to occur.

The nurse is admitting a client to the hospital with findings of liver failure and ascites. A health care provider (HCP) orders spironolactone. The nurse understands that the pharmacological effects of the medication, are which of the following? A. Combines safely with antihypertensives B. Depletes potassium reserves C. Promotes sodium and chloride excretion D. Increases aldosterone levels

C. Promotes sodium and chloride excretion Spironolactone is considered a diuretic, that is indicated for individuals with hypertension, edema, congestive heart failure and potassium loss. Spironolactone promotes sodium and chloride excretion while sparing potassium and decreasing aldosterone levels. Spironolactone is often combined with other diuretics and anti-hypertensive agents. Kidney function and electrolytes should be monitored more closely when spironolactone is used in combination with other medications. The medication is considered a potassium-sparing diuretic, because as aldosterone levels decrease and sodium and water is excreted, potassium is spared. A major side effect of spironolactone is hyperkalemia.

The nurse is assessing a client who is receiving antibiotic therapy for an infection. Which finding should indicate to the nurse that the client may be experiencing an allergic reaction to a medication? A. Xerostomia B. Hypertension C. Pruritus D. Lymphadenopathy

C. Pruritus Rationale: If the client experiences pruritus, the nurse should be concerned about the possibility of an allergic reaction. Xerostomia, or dry mouth, and lymphadenopathy are not signs of a hypersensitivity reaction. A client experiencing an allergic reaction will experience hypotension.

Which effect may be experienced by a client who reports frequently taking calcium carbonate? A. Diarrhea B. Water retention C. Rebound hyperacidity D. Bone demineralization

C. Rebound hyperacidity The antacid action of calcium carbonate adds alkalinity, neutralizing gastric pH; this in turn stimulates renewed secretion of acid by the gastric mucosa. This medication causes constipation, not diarrhea. Calcium carbonate does not contain sodium, as do some antacids; thus it does not promote fluid retention. This antacid provides a source of calcium, which helps prevent bone demineralization.

A child with Wilms tumor is prescribed doxorubicin hydrochloride. Which common side effect unique to doxorubicin would the nurse expect to observe in the child? A. Hair loss B. Vomiting C. Red urine D. Stomatitis

C. Red urine Red urine is a common side effect of doxorubicin administration. The medication is not metabolized and is excreted in the urine. The genitourinary responses to vincristine are nocturia, oliguria, urine retention, and gonadal suppression. Hair loss, vomiting, and stomatitis occur with both medications.

A nurse is providing care to a client with diabetes insipidus. The client is on a prescribed vasopressin infusion with orders to titrate as needed. The nurse decreases the dose of vasopressin based on which clinical finding? A. Increased blood pressure B. Decreased urine osmolarity C. Reduced volume of urine output D. Elevated heart rate

C. Reduced volume of urine output Rationale: Diabetes insipidus is an endocrine disorder that causes the excretion of large quantities of diluted urine. Vasopressin decreases urine output by allowing the reabsorption of water in the kidneys. A reduction in the volume of urine output indicates the medication is delivering the intended effect, and the dose can be decreased. Vasopressin can increase blood pressure and heart rate. However, these are not the intended effects of vasopressin for a client with diabetes insipidus. A decrease in the urine osmolarity indicates dilution is still present.

The nurse is administering docusate sodium to a postpartum client. Which of the following should the nurse include in the medication teaching? A. This medication will help with your uterine cramping B. Breastfeeding is contraindicated while taking this medication C. Report to the healthcare provider if you experience diarrhea D. This medication lowers your risk of hemorrhage

C. Report to the healthcare provider if you experience diarrhea Rationale: Docusate sodium is a stool softener/laxative and should not be administered to a client who is experiencing diarrhea. This medication does not directly affect breastfeeding or the risk of hemorrhage. Docusate sodium does not affect uterine cramping.

Which mechanism of action would the nurse identify for levodopa therapy prescribed to a client diagnosed with Parkinson disease? A. Blocks the effects of acetylcholine B. Increases the production of dopamine C. Restores the dopamine levels in the brain D. Promotes the production of acetylcholine

C. Restores the dopamine levels in the brain Levodopa is a precursor of dopamine, a catecholamine neurotransmitter; it increases dopamine levels in the brain that are depleted in Parkinson disease. Blocking the effects of acetylcholine is accomplished by anticholinergic medications. Increasing the production of dopamine is ineffective because it is believed that the cells that produce dopamine have degenerated in Parkinson disease. Levodopa does not affect acetylcholine production.

A nurse is providing instructions to a client receiving baclofen. Which of the following would be included in the teaching plan? A. Limit Fluid Intake B. Hold the medication if diarrhea occurs. C. Restrict alcohol intake. D. Notify the Physician if weakness occurs.

C. Restrict alcohol intake. Rationale: Due to the depressive effects on the CNS, alcohol should be limited.

An adolescent is prescribed an antineoplastic agent. Which instruction would the nurse give to the parents before discharge? A. Limit contact with all peers and family members. B. Withhold medications when nausea occurs to prevent vomiting. C. Schedule laboratory blood tests to evaluate response to the medication. D. Return weekly for a bone marrow aspiration to monitor effectiveness of therapy.

C. Schedule laboratory blood tests to evaluate response to the medication. Blood tests indicate response to therapy; if the white blood cell count drops precipitously, therapy may be halted temporarily. Children undergo therapy for extended periods, and prolonged separation from their peers may lead to social isolation. Contact with children who have active infections should be avoided. Although nausea commonly occurs with this therapy, antiemetic measures are instituted; the medication is not withdrawn for this reason. A bone marrow aspiration is a painful procedure and is performed selectively (e.g., to confirm the diagnosis), not routinely.

The nurse is preparing to administer the next dose of prescribed vancomycin to the client being treated for sepsis. Which of the following laboratory results would be the priority for the nurse to review? A. Peak serum drug level B. Serum potassium level C. Serum creatinine level D. White blood cell count

C. Serum creatinine level Rationale: Vancomycin can lead to interstitial nephritis and therefore, serum creatinine should be monitored. Prior to a dose, a trough level would be drawn to help assess minimum inhibitory concentration; however, peak levels are not needed for this purpose and are drawn after administration. Do not hold the next vancomycin doses while waiting for the results of vancomycin levels unless there is a concern about renal function. Therefore, the priority is serum creatinine. While the treatment of infection is the goal, assessing white blood count (WBC) prior to administration is not necessary.

The nurse is providing discharge medication teaching to a client who will be taking furosemide and digoxin after discharge from the hospital. Which information is important for the nurse to include in the teaching plan? A. Maintenance of a low-potassium diet B. Avoidance of foods high in cholesterol C. Signs and symptoms of digoxin toxicity D. Importance of monitoring output

C. Signs and symptoms of digoxin toxicity The risk of digoxin toxicity increases when the client is receiving digoxin and furosemide, a loop diuretic; loop diuretics can cause hypokalemia, which potentiates the effects of digoxin, leading to toxicity. Digoxin toxicity can result in dysrhythmias and death. When a client is receiving a loop diuretic, the diet should be high in potassium. Although teaching the need to avoid foods high in cholesterol may be included in the teaching plan, it is not the priority. It is not necessary to monitor output.

Which responses would the nurse offer a client who asks whether she should take soy supplements to decrease menopausal symptoms and improve bone density? Select all that apply. One, some, or all responses may be correct. A. These supplements can cause high cholesterol. B. Estrasorb is applied like a lotion but can't transfer to men. C. Soy supplements might interfere with thyroid hormone absorption. D. These products have fewer contraindications than other estrogen products. E. Some women have reported nausea, bloating, diarrhea, and abdominal pain.

C. Soy supplements might interfere with thyroid hormone absorption. E. Some women have reported nausea, bloating, diarrhea, and abdominal pain. It is true that soy supplements might affect thyroid function and that some women have reported mild stomach and gastrointestinal side effects such as constipation, bloating, nausea, diarrhea, and abdominal pain. It is not accurate to say that soy supplements can raise cholesterol levels; in fact, they are commonly used to reduce blood lipids. Estrasorb is applied like a lotion but can transfer to men and increase their estradiol levels. It is a common misconception that 'natural' products such as soy have fewer contraindications than other products; in fact, they have many of the same contraindications and are not necessarily a safer alternative to traditional medications.

Which medication requires the nurse to monitor the client for signs of hyperkalemia? A. Furosemide B. Metolazone C. Spironolactone D. Hydrochlorothiazide

C. Spironolactone Spironolactone is a potassium-sparing diuretic; hyperkalemia is an adverse effect. Furosemide, metolazone, and hydrochlorothiazide generally cause hypokalemia.

A client has had a recent brain attack (cerebrovascular accident/stroke). Which preventative would the nurse anticipate will be prescribed daily to avoid straining due to constipation? A. Stimulant laxatives such as bisacodyl B. Tap-water enemas C. Stool softener D. Saline laxatives such as magnesium citrate

C. Stool softener A stool softener can soften stool and promote defecation, thus avoiding the Valsalva maneuver. Enemas may precipitate a forcible exhalation against a closed glottis (Valsalva maneuver) during evacuation. Elevated intraabdominal and intrathoracic pressures associated with the Valsalva maneuver increase intracranial pressure and should be avoided. Also, daily enemas promote dependence. Stimulant laxatives are not recommended for daily use because laxative dependency has occurred in some clients. Saline laxatives can cause hypermagnesemia if given this frequently.

e nursing is preparing to administer phenytoin IV push to a client. The client has dextrose 5% in water infusing continuously. Which action is appropriate? A. Pinch the line above the infusion port during the administration B. Hold the medication and collaborate with the provider prior to administration C. Stop the infusion and flush the port with normal saline prior to administration D. Ask the pharmacy to mix the medication into an IV piggyback (IVPB) infusion

C. Stop the infusion and flush the port with normal saline prior to administration Rationale: If giving phenytoin as an infusion, it cannot be administered with D5W because it will precipitate. The D5W should be disconnected, the port flushed with normal saline solution (NSS), medication administered, and the port flushed again with NSS before the D5W is reconnected. The provider does not need to be contacted as this is best practice and aligns with hospital protocol. Administering the medication via IVPB does not reduce the risk for precipitation.

A client receiving intravenous vancomycin reports ringing in both ears. Which initial action would the nurse take? A. Notify the primary health care provider. B. Consult an audiologist. C. Stop the infusion. D. Document the finding and continue to monitor the client.

C. Stop the infusion. The first action the nurse would take is to stop the infusion immediately. Vancomycin can cause temporary or permanent hearing loss. The nurse would stop the medication infusion and then notify the health care provider at once if a client reports any hearing problems or ringing in the ears. An audiologist may need to be consulted at a later date, but this is not the best first action. The nurse would document the findings; however, this is not the initial action.

A client with pulmonary tuberculosis develops tinnitus and vertigo. Which antitubercular medication would the nurse suspect is causing these symptoms? A. Isoniazid B. Rifampin C. Streptomycin D. Ethambutol

C. Streptomycin Streptomycin is ototoxic and can cause damage to the eighth cranial nerve, resulting in deafness. Assessment for ringing or roaring in the ears, vertigo, and hearing acuity should be made before, during, and after treatment. Isoniazid does not affect the ear; however, blurred vision and optic neuritis, as well as peripheral neuropathy, may occur. Rifampin does not affect hearing; however, visual disturbances may occur. Ethambutol does not affect hearing; however, visual disturbances may occur.

The nurse is administering a histamine H 2 antagonist to a client who has extensive burns. Which complication will it prevent? A. Colitis B. Gastritis C. Stress ulcer D. Metabolic acidosis

C. Stress ulcer An ulcer of the upper gastrointestinal tract is related to excessive secretion of stress-related hormones, which increases hydrochloric acid production. Histamine H 2 antagonists decrease acid secretion. Colitis is not a complication of burns. Gastritis is not a complication of burns. Metabolic acidosis is not a complication of burns unless hypermetabolism or renal failure occurs; metabolic acidosis is not treated with H 2 antagonists.

A health care provider in the emergency department identifies that a client is in cardiogenic shock. Which type of medication is indicated for management of this condition? A. Loop diuretic B. Cardiac glycoside C. Sympathomimetic D. Alpha-adrenergic blocker

C. Sympathomimetic Sympathomimetics are vasopressors that induce arterial constriction, which increases venous return and cardiac output. Diuretics promote excretion of fluid, which is not indicated. Cardiac glycosides slow and strengthen the heartbeat; they do not increase the blood pressure and may decrease it. Alpha-adrenergic blockers decrease peripheral resistance, resulting in a decreased blood pressure.

An older adult client is to receive intravenous (IV) gentamicin for urosepsis. Before administering the medication, for which finding should the nurse notify the health care provider (HCP)? A. The client has a history of acid reflux disease. B. The client has a history of retinopathy. C. The client has a history of chronic kidney disease. D. The client has a history of urinary retention.

C. The client has a history of chronic kidney disease. Gentamicin is an aminoglycoside antibiotic. Aminoglycosides are used to treat severe infections, such as septicemia, and are only given for a short period of time due to their toxic effects. They are not metabolized by the liver. Instead they are excreted by glomerular filtration. Aminoglycosides are nephrotoxic and requires close monitoring of renal function. A client with chronic kidney disease should not receive this medication. The other conditions do not represent a contraindication to gentamicin.

The nurse is monitoring a client who is taking prescribed nitroglycerin for angina. Which finding indicates the medication has a therapeutic effect? A. The client blood pressure is 150/80 mm/Hg. B. The client heart rate is 110. C. The client reports a decrease in chest pressure. D. The client reports a headache.

C. The client reports a decrease in chest pressure. Rationale: Nitroglycerin acts to decrease myocardial oxygen consumption. Dilatation of the veins reduces the amount of blood returning to the heart (preload), so the chambers have a smaller volume to pump resulting in decreased oxygen needs. Decreased oxygen demand reduces pain caused by dilating coronary blood flow. While blood pressure may decrease slightly due to the vasodilatory effects of nitroglycerin, it is a secondary effect and not the desired therapeutic effect of this drug. Increased blood pressure and increased preload would mean the heart would work harder, increasing oxygen demand and thus angina. Decreased heart rate is not an effect of nitroglycerin.

A client is prescribed the benzodiazepine alprazolam for the management of panic attacks. Which action by the patient makes the nurse confident that the medication information discussed has been understood? A. The client removes the pepperoni from a pizza. B. The client asks for an extra bottle of flavored water to drink with dinner. C. The client requests a prescription for oral contraceptives before being discharged. D. The client states that chewable antacids may be taken to relieve heartburn.

C. The client requests a prescription for oral contraceptives before being discharged. Benzodiazepines increase the risk of congenital anomalies and so should not be taken by pregnant women. Refraining from eating pepperoni is appropriate for people taking monoamine oxidase inhibitors because tyramine needs to be strictly avoided. Appropriate hydration is critical for those taking lithium. Antacids can affect both absorption and metabolism of benzodiazepines and should be avoided.

Which would the nurse include when teaching a client with Parkinson disease about carbidopa-levodopa? A. Multivitamins should be taken daily. B. A high-protein diet should be followed. C. The medication should be taken with meals. D. Alcohol consumption should be in moderation.

C. The medication should be taken with meals. Carbidopa-levodopa should be taken with meals to reduce the nausea and vomiting that commonly are caused by this medication. Multivitamins are contraindicated; vitamins may contain pyridoxine (vitamin B 6), which diminishes the effects of levodopa. A high-protein diet is contraindicated. Sinemet contains levodopa, an amino acid that may increase blood urea nitrogen levels. Also, some proteins contain pyridoxine, which increases the peripheral metabolism of levodopa, decreasing the amount of levodopa crossing the blood-brain barrier.

A charge nurse is observing a staff nurse prepare 1 ml of intravenous digoxin for a client with heart failure. After the staff nurse prepares the medication, the nurse notices precipitate in the syringe. Which action by the staff nurse likely caused this reaction? A. D5W was used as the diluent. B. The medication was not allowed to reach room temperature. C. The medication was added to 1 mL of diluent. D. Air was not inserted into the vial.

C. The medication was added to 1 mL of diluent. Rationale: When administering digoxin, 1 milliliter of digoxin should be mixed into at least 4 milliliters of diluent. Using a smaller amount of diluent will cause precipitation of the medication. Dextrose 5% in water (D5W) is compatible with digoxin and can be used to dilute the medication. Digoxin is not a temperature-controlled medication. Precipitation occurs as a result of incompatibilities or improper mixing. The insertion of air into a vial facilitates the withdrawal of the medication. Omission of this does not cause medication precipitation.

Status epilepticus develops in an adolescent with a seizure disorder who is taking antiseizure medication. Which reason would the nurse identify as the most common reason for the development of status epilepticus? A. The provider failed to account for a growth spurt. B. The amount prescribed is insufficient to cover activities. C. The prescribed antiseizure medication probably is not taken consistently. D. The client is prescribed a medication that is ineffective in preventing seizures.

C. The prescribed antiseizure medication probably is not taken consistently. Skipping doses of the medication is a form of denial that an adolescent client may engage in once the seizures are controlled; also, adolescents tend to feel invincible. The dosage is based not on activity but on the type of seizure. Medications are prescribed according to the type of seizure and are effective if taken consistently. The dosage of antiepileptic medications is based on many factors, including age, type of medication, and presence of infection, as well as on changes in height and weight.

The nurse is teaching a school-age child how to use an insulin pump. Which instruction by the nurse is most important for the child to understand? A. The needle must be changed every day. B. A blood glucose check is necessary once a day. C. The pump is an attempt to mimic the way a healthy pancreas works. D. Subcutaneous pockets near the abdomen are used to implant the pump.

C. The pump is an attempt to mimic the way a healthy pancreas works. The basal infusion rate mimics the low rate of insulin secretion during fasting, and the bolus before meals mimics the high output after meals. The subcutaneous needle and tubing may be left in place for as long as 3 days. Blood glucose monitoring is performed at least four times a day. Most insulin pumps are battery-driven syringes external to the body.

Which information would the nurse include when teaching parents about the side effects of iron supplements? A. The urine may turn red. B. The skin will turn yellow. C. The teeth may become stained. D. The stools will take on a clay color.

C. The teeth may become stained. Liquid oral iron supplements may stain the teeth; brushing the teeth after administration may limit the discoloration. There should be no change in the color of the urine. Yellowing of the skin is a sign of jaundice; it is not a side effect of an iron supplement. The stools will become black-green; clay-colored stools are a sign of biliary obstruction.

The nurse is teaching a client diagnosed with depression about a new prescription of nortriptyline. What information would be essential for the nurse to emphasize about this medication? A. Episodes of diarrhea can be expected B. The medication must be stored in the refrigerator C. The use of alcohol should be avoided D. Symptom relief occurs in a few days

C. The use of alcohol should be avoided Nortriptyline is a tricyclic antidepressant used to manage chronic neurogenic pain and depression. Adverse reactions include central nervous system (CNS) side effects such as suicidal thoughts, drowsiness, fatigue, lethargy, and confusion. Clients who are prescribed this medication should be educated to avoid the use of alcohol consumption or other CNS depressant drugs as this can worsen the adverse reactions of the medication and cause injury.

The nurse is educating a client about newly prescribed alprazolam. Which information should the nurse include in the teaching? A. Tardive dyskinesia is common early in treatment. B. Administration of paroxetine may be needed to prevent adverse effects. C. The use of grapefruit juice should be avoided. D. Hyperactivity is seen with long-term use.

C. The use of grapefruit juice should be avoided. Rationale: Grapefruit or grapefruit juice is a known food-drug interaction and may increase drug levels of alprazolam to potentially toxic concentrations. Paroxetine when given with alprazolam will increase the incidence of adverse side effects. Alprazolam is used to treat tardive dyskinesia and is not an adverse side effect of this medication. In general, side effects of benzodiazepines with long-term use include drowsiness, lethargy, and weight gain but not hyperactivity.

Which advantage does aluminum and magnesium hydroxide have over baking soda (sodium bicarbonate) for the treatment of heartburn? A. They can be used for short-term relief. B. Absorption by the stomach mucosa is markedly enhanced. C. There is no direct effect on the systemic acid-base balance when taken as directed. D. Fewer side effects, such as diarrhea or constipation, are experienced when they are used properly.

C. There is no direct effect on the systemic acid-base balance when taken as directed. Nonsystemic antacids are not readily absorbed, so they do not alter the acid-base balance. Sodium bicarbonate is absorbed and can alter the acid-base balance. For this reason, sodium bicarbonate should not be taken long-term. Both nonsystemic antacids and sodium bicarbonate can be used for short-term use. Nonsystemic antacids are insoluble and not readily absorbed. Diarrhea and constipation are side effects of nonsystemic antacids.

A health care provider prescribes a diuretic for a client with hypertension. Which mechanism of action explains how diuretics reduce blood pressure? A. They facilitate vasodilation. B. They promotes smooth muscle relaxation. C. They reduce the circulating blood volume. D. They block the sympathetic nervous system.

C. They reduce the circulating blood volume. Diuretics decrease blood volume by blocking sodium reabsorption in the renal tubules, thus promoting fluid loss and reducing arterial pressure. Direct relaxation of arteriolar smooth muscle is accomplished by vasodilators, not diuretics. Vasodilators, not diuretics, act on vascular smooth muscle. Medications that act on the nervous system, not diuretics, inhibit sympathetic vasoconstriction.

The nurse is providing medication teaching for a client prescribed famotidine for the treatment of gastroesophageal reflux disease (GERD). Which statement by the client indicates an understanding of the teaching? A. I will take this medication once a day in the morning B. I will no longer have discomfort at night once I begin this medication C. This medication will both prevent and treat heartburn D. My treatment will be done in one week

C. This medication will both prevent and treat heartburn Rationale: H2 receptor blockers (antagonists) are used to prevent and treat conditions caused by too much acid being produced in the stomach. These conditions include gastric ulcers, duodenal ulcers, and GERD.

A client has primary open-angle glaucoma. Which ophthalmic preparation is indicated to manage this condition? A. Tetracaine B. Fluorescein C. Timolol maleate D. Atropine sulfate

C. Timolol maleate Timolol maleate is a beta-adrenergic antagonist that decreases aqueous humor production and increases outflow, thereby reducing intraocular pressure. Tetracaine is a topical anesthetic; it will not reduce the increased intraocular pressure associated with glaucoma. Fluorescein is a dye used to identify corneal abrasions and foreign bodies. Atropine sulfate, a mydriatic, is contraindicated because it dilates the pupil, obstructing drainage and increasing intraocular pressure.

In addition to hydration, parenteral lorazepam is prescribed for a client during alcohol withdrawal delirium. Which primary purpose accurately explains why this medication is given during detoxification? A. To prevent injury when seizures occur B. To enable the client to sleep better during periods of agitation C. To reduce the anxiety tremor state and prevent more serious withdrawal symptoms D. To calm the client and promote acceptance of the treatment plan

C. To reduce the anxiety tremor state and prevent more serious withdrawal symptoms Lorazepam potentiates the actions of gamma-aminobutyric acid, which reduces anxiety and irritability that are common during withdrawal. This medication helps reduce the risk of seizures but does not prevent physical injury if a seizure occurs. Although the medication may enable the client to sleep better during periods of agitation, this is not its primary objective. The ability of the client to accept treatment depends on readiness to accept the reality of the problem.

Which explanation would the nurse provide to a client with gastric ulcer disease who asks the nurse why the health care provider has prescribed metronidazole? A. To augment the immune response B. To potentiate the effect of antacids C. To treat Helicobacter pylori infection D. To reduce hydrochloric acid secretion

C. To treat Helicobacter pylori infection Approximately two-thirds of clients with peptic ulcer disease are found to have Helicobacter pylori infecting the mucosa and interfering with its protective function. Antibiotics do not augment the immune response, potentiate the effect of antacids, or reduce hydrochloric acid secretion.

A client recently diagnosed with heart failure has been prescribed digoxin and furosemide. Which of the following foods should the nurse teach the client to eat at least one serving a day? A. Blueberries B. Wheat cereal C. Tomato juice D. Pear nectar

C. Tomato juice Digoxin, an antiarrhythmic, and furosemide, a diuretic, are commonly prescribed for clients with heart failure. A common side effect for furosemide is depletion of potassium. Of the food choices, tomato juice is the highest in potassium. To reduce the risk of potassium depletion, the client should be encouraged to drink at least 1/2 cup of tomato juice every day which is about 400 mg of potassium. The other choices are low in potassium which would be recommended for clients diagnosed with chronic renal failure.

Which symptom of levodopa toxicity will a client taking levodopa be taught as a reason to contact the primary health care provider? A. Nausea B. Dizziness C. Twitching D. Constipation

C. Twitching Abnormal involuntary movements (dyskinesias), such as muscle twitching, rapid eye blinking, facial grimacing, head bobbing, and an exaggerated protrusion of the tongue, are signs of toxicity; these probably result from the body's failure to readjust properly to the reduction of dopamine. Nausea is a side effect of therapy, not toxicity. Dizziness is a side effect of therapy, not toxicity. Constipation is unrelated to levodopa toxicity.

Which assessment would be brought to the health care provider's attention before administration of intravenous potassium chloride? A. Progressively worsening muscle weakness B. Poor tissue turgor with tenting C. Urinary output of 200 mL during the previous 8 hours D. Oral fluid intake of 300 mL during the previous 12 hours

C. Urinary output of 200 mL during the previous 8 hours Decreased urinary output may result in the retention of potassium, causing hyperkalemia. Progressively worsening muscle weakness is a manifestation of hypokalemia, which is the reason for prescribing the potassium. Reporting poor tissue turgor with tenting is unnecessary; this may indicate dehydration, which is probably the rationale for the fluid prescribed. Reporting an oral fluid intake of 300 mL during the previous 12 hours is unnecessary; this can precipitate dehydration or can compound an existing dehydration, which can be treated with appropriate hydration.

A nurse is preparing to administer insulin to a client with diabetes mellitus type 1. The client has regular insulin and insulin glargine prescribed. How will the nurse prepare these medications? A. Draw up the glargine insulin before the regular insulin B. Mix the insulins in a larger syringe C. Use a separate syringe for each insulin D. Draw up the regular insulin before the glargine insulin

C. Use a separate syringe for each insulin Rationale: Insulin glargine is a clear, long-acting insulin that should not be mixed with other insulins. Mixing insulin glargine with other medications can cause precipitate formation. The insulins should be drawn up in separate syringes. Short-acting insulins should be drawn up before long-acting insulins. However, insulin glargine should not be combined with any other medication. A larger syringe does not address incompatibility issues.

A client develops severe bone marrow suppression related to cancer treatment. Which instruction is important for the nurse to include in the client's teaching? A. Be prepared to experience alopecia. B. Increase fluids to at least 3 liters per day. C. Use a soft toothbrush for oral hygiene. D. Monitor your intake and output of fluids.

C. Use a soft toothbrush for oral hygiene. Thrombocytopenia occurs with several cancer treatment programs; using a soft toothbrush helps prevent bleeding gums. Although alopecia does occur, it is not related to bone marrow suppression. Increasing fluids will neither reverse bone marrow suppression nor stimulate hematopoiesis. Monitoring intake and output of fluids is not related to bone marrow suppression.

Which action would the nurse take when administering iron dextran? A. Use a transdermal needle. B. Massage the injection site. C. Use the Z-track method. D. Apply a local anesthetic first.

C. Use the Z-track method. The Z-track injection method prevents seepage of iron dextran through the needle track, thereby limiting irritation of subcutaneous tissue and staining of the skin. The length of a transdermal needle is too short to reach a muscle; a 1.5-inch (3.8 cm) needle is required. Massage will force iron dextran into the subcutaneous tissue, causing irritation and staining. Although an injection may be uncomfortable, a local anesthetic is unnecessary.

The nurse understands which antiepileptic medication would be used as the first-line treatment for absence seizures? A. Phenytoin B. Diazepam C. Valproic acid D. Acetazolamide

C. Valproic acid Valproic acid is used as the first-line treatment for absence seizures. Phenytoin is used to treat partial, secondary, and generalized tonic-clonic seizures. Diazepam is used to treat status epilepticus. Acetazolamide is used as an adjunct medication for the treatment of absence seizures.

A school-age child diagnosed with acute lymphocytic leukemia (ALL) becomes constipated after receiving induction therapy with prednisone, vincristine, and asparaginase. Which would the nurse suspect as the cause? A. Diet, which lacks bulk B. Inactivity, which results from illness C. Vincristine, which decreases peristalsis D. Prednisone, which causes gastric irritability

C. Vincristine, which decreases peristalsis Constipation, which may progress to paralytic ileus, is a side effect of vincristine. Lack of bulk and inactivity each may contribute to constipation, but neither is the primary cause of this child's constipation. Prednisone may cause nausea and vomiting, but it does not cause constipation.

A thin 24-year-old woman who runs 10 miles each week asks the nurse for advice about preventing osteoporosis. Which vitamin would the nurse recommend? A. Vitamin E B. Vitamin B C. Vitamin D D. Vitamin C

C. Vitamin D All women, except those who are pregnant or lactating, should ingest between 1000 and 1300 mg of calcium daily; if the client is unable to ingest enough calcium in food, supplements of calcium and vitamin D are recommended. Vitamin C helps maintain cartilage and connective tissue integrity but does not help prevent osteoporosis. Vitamins E and B do not help prevent osteoporosis.

The nurse is reviewing medication instructions with parents of an infant receiving digoxin and spironolactone. Which parental response indicates instructions have been understood? A. Activity should be restricted. B. Orange juice should be given daily. C. Vomiting should be reported to the health care provider. D. Anti-inflammatory medications should be avoided.

C. Vomiting should be reported to the health care provider. Vomiting is a classic sign of digoxin toxicity, and the health care provider must be notified. Infants regulate their own activity according to their energy level. Orange juice is rarely needed because spironolactone spares potassium. There is no restriction on anti-inflammatory medications with spironolactone.

Which item will the nurse teach a client who has had an excision of a thrombosed external hemorrhoid to use when cleaning the anus after a bowel movement? A. Betadine pads B. Soft facial tissue C. Witch hazel-moistened pads D Sterile 4 × 4-inch (10 × 10 cm) pads

C. Witch hazel-moistened pads Witch hazel-moistened pads are not irritating and are soothing to the anal mucosa. Betadine may cause excessive drying and irritation. The rectum always is contaminated; external cleansing with Betadine will not appreciably affect the bacteria present. Dry facial tissue is irritating and can cause trauma. Sterile gauze pads are unnecessary; the rectal area is considered contaminated.

During the admission process, the client reports heavy alcohol use for at least one year. What effect does the nurse anticipate the hospitalized client will experience when alcohol consumption stops? A. Bradycardia B. Somnolence C. Withdrawal D. Tachypnea

C. Withdrawal The findings of alcohol withdrawal develop within 24 to 48 hours after people either stop or significantly reduce their alcohol consumption. Findings of withdrawal can range from "mild" (shaking or sweating, or perhaps nausea, headache, anxiety, tachycardia or hypertension) to severe (delirium tremens or DTs), which are characterized by rapid heartbeat, fever, hallucinations or seizures.

A client is prescribed trimethoprim/sulfamethoxazole for recurrent urinary tract infections. Which statement by the nurse about this medication is correct? A. "You can stop the medication after five days." B. "Be sure to take the medication with food." C. "It is safe to take with oral contraceptives." D. "Drink at least eight glasses of water a day."

D. "Drink at least eight glasses of water a day." Trimethoprim/sulfamethoxazole is a highly insoluble medication and should be taken with a large volume of fluid. This medication can be taken with or without food. The full prescribed amount should be taken at evenly-spaced intervals until the medication is finished. Unlike many other antibiotics, trimethoprim/sulfamethoxazole does not seem to affect hormonal birth control such as the pill, the patch or ring.

A client with angina has been instructed about the use of sublingual nitroglycerin. Which statement by the client indicates the need for additional teaching? A. "I'll call the health care provider if pain continues after three tablets five minutes apart." B. "I will rest briefly right after taking one tablet." C. "I understand that the medication should be kept in the dark bottle." D. "I can swallow two or three tablets at once if I have severe pain."

D. "I can swallow two or three tablets at once if I have severe pain." Clients must understand that just one sublingual tablet should be taken at a time. Clients must also understand that they should rest when experiencing angina. Two or three tablets should not be used at once, even in the setting of severe pain, as this can lead to significant hypotension. The client should notify their primary healthcare provider should they not have a relief of symptoms with nitroglycerin use.

The nurse in the urgent-care clinic is reviewing discharge instructions with a client who is prescribed doxycycline. Which statement by the client indicates understanding of the instructions? A. "I will not wear my contact lenses while taking this medication." B. "I will carry glucose tablets with me in case I experience low blood sugar." C. "I will take this medication with an antacid to prevent an upset stomach." D. "I will apply sunscreen when outside to prevent a sunburn."

D. "I will apply sunscreen when outside to prevent a sunburn." Doxycycline is a tetracycline antibiotic. All tetracyclines can increase the sensitivity of the skin to ultraviolet light. The most common result is a sunburn. Clients on these types of medications should prevent sunburn by avoiding prolonged exposure to sunlight, wearing protective clothing and applying sunscreen to exposed skin while outdoors. This drug should be taken two hours before or after antacids, not with them. Hypoglycemia is not a common side effect of doxycycline. Wearing contact lenses is not contraindicated with this medication.

Which statement made by a client prescribed metformin extended release to control type 2 diabetes mellitus indicates the need for further education? A. "I will take the medication with food." B. "I must swallow my medication whole and not crush or chew it." C. "I will notify my doctor if I develop muscular or abdominal discomfort." D. "I will stop taking metformin for 24 hours before and after having a test involving dye."

D. "I will stop taking metformin for 24 hours before and after having a test involving dye." Metformin must be withheld for 48 hours before the use of iodinated contrast materials to prevent lactic acidosis. Metformin is restarted when kidney function has returned to normal. Metformin is taken with food to avoid adverse gastrointestinal effects. If crushed or chewed, metformin XL will be released too rapidly and may lead to hypoglycemia. Muscular and abdominal discomfort is a potential sign of lactic acidosis and must be reported to the health care provider.

Which instruction about medications would the nurse include when teaching a client with type 1 diabetes who tells the nurse, "I take guaifenesin cough syrup when I have a cold."? A. "Substitute an elixir for the cough syrup." B. "Increase fluid intake and use a humidifier to control the cough." C. "The small amounts of sugar in medications are not a concern with diabetes." D. "Include the glucose in the cough syrup when calculating daily carbohydrate allowance."

D. "Include the glucose in the cough syrup when calculating daily carbohydrate allowance." Cough syrup contains a glucose base; the client should use a glucose-free product or account for the glucose. Elixirs contain natural sweeteners. Although increasing fluid intake and using a room humidifier will also loosen secretions, this does not include important information about the medication. Additional sweeteners, even the amounts in medications, may increase serum glucose levels beyond the desired range; once control is achieved, it is unwise to alter dietary intake or medications without supervision.

Which instruction would the nurse include when teaching about hydrochlorothiazide given to a client diagnosed with a transient ischemic attack (TIA) related to hypertension? A. "Resume regular eating habits." B. "Drink a protein supplement daily." C. "Avoid eating foods high in insoluble fiber." D. "Increase the intake of potassium-rich foods."

D. "Increase the intake of potassium-rich foods." The client must increase the dietary intake of potassium because of potassium loss associated with hydrochlorothiazide. The client should be taught about medication-induced deficiencies, which may necessitate a change in diet, and not just return to regular eating habits once home. Protein supplements are not necessary, and protein may be obtained from meat, fish, and dairy products in the diet or complementary vegetable and grain proteins. Foods high in insoluble fiber are part of the food pyramid and should be included in the diet.

A client with cirrhosis of the liver asks the nurse about the purpose of taking lactulose. How should the nurse respond? A. "It is used to control portal hypertension." B. "It adds dietary fiber to your diet." C. "It helps to regenerate your liver." D. "It helps to reduce ammonia levels in your blood."

D. "It helps to reduce ammonia levels in your blood." Lactulose is a synthetic disaccharide that can be given orally or rectally. It blocks the absorption and production of ammonia from the gastrointestinal tract, reducing serum ammonia levels, and is used to treat hepatic encephalopathy. The other answers are incorrect.

Which instruction will the nurse provide to a client with type 2 diabetes who develops gout when teaching about the administration of allopurinol? A. "Allopurinol masks symptoms of hypoglycemia." B. "Increase your oral hypoglycemic medications." C. "Increase attention to diabetic foot care." D. "Monitor blood glucose levels more frequently."

D. "Monitor blood glucose levels more frequently." Allopurinol can potentiate the effect of oral hypoglycemics, causing hypoglycemia; the blood glucose level should be monitored more frequently. Allopurinol does not mask symptoms of hypoglycemia. Medications should not be increased; this is a determination made by the health care provider. Allopurinol does not increase the risk for problems related to foot care.

Which response would a nurse give to a client taking an oral hypoglycemic tablet daily who asks if an extra tablet should be taken before exercise? A. "You will need to decrease your exercise." B. "An extra tablet will help your body use glucose correctly." C. "When taking medicine, your diet will not be affected by exercise." D. "No, but you should observe for signs of hypoglycemia while exercising."

D. "No, but you should observe for signs of hypoglycemia while exercising." Exercise improves glucose metabolism; with exercise, there is a risk of developing hypoglycemia, not hyperglycemia. Exercise should not be decreased because it improves glucose metabolism. An extra tablet probably will result in hypoglycemia because exercise alone improves glucose metabolism. Control of glucose metabolism is achieved through a balance of diet, exercise, and pharmacologic therapy.

Which statement by a client who has a gastric ulcer and asks what to do if epigastric pain occurs indicates that teaching was effective? A. "Eliminating fluids with meals will prevent pain." B. "I will increase my food intake to avoid an empty stomach." C. "Taking an aspirin with milk will relieve my pain and coat my ulcer." D. "Taking an antacid preparation will decrease pain due to gastric acid."

D. "Taking an antacid preparation will decrease pain due to gastric acid." Over-the-counter antacid preparations neutralize gastric acid and relieve pain. Reduction of fluids with meals does not affect pain. Although eating food initially prevents gastric acid from irritating the gastric walls, it can precipitate acid production. Aspirin is contraindicated because it irritates gastric mucosa and promotes bleeding by preventing platelet aggregation.

Which statement by a client prescribed ampicillin 250 mg by mouth every 6 hours indicates to the nurse that teaching has been effective? A. "I should drink a glass of milk with each pill." B. "I should drink at least six glasses of water every day." C. "The medicine should be taken with meals and at bedtime." D. "The medicine should be taken 1 hour before or 2 hours after meals."

D. "The medicine should be taken 1 hour before or 2 hours after meals." Ampicillin is a form of penicillin that should be given on an empty stomach; food delays absorption. The response "I should drink a glass of milk with each pill" is incorrect; opaque liquids, such as milk, delay the absorption of this medication. It is not necessary to drink at least six glasses of water every day; however, it is appropriate to prevent crystalluria when the client is prescribed sulfonamides. The response "The medicine should be taken with meals and at bedtime" is incorrect; food delays the absorption of this medication.

Which response will be given by a nurse caring for a client with chronic hepatitis B who asks "Are there any medications to help me get rid of this problem?"? A. "Sedatives can be given to help you relax." B. "We can give you immune serum globulin." C. "Vitamin supplements are frequently helpful and hasten recovery." D. "There are medications to help reduce viral load and liver inflammation."

D. "There are medications to help reduce viral load and liver inflammation." Medications are available to help reduce the viral load (antivirals), including lamivudine, ribavirin, and adefovir dipivoxil. Although sedatives can be given to help the client relax, sedatives are given only as needed and do not treat the hepatitis. The response "We can give you immune serum globulin" would be used only during the incubation period. Vitamins are used as adjunctive therapy and will not eliminate the hepatitis.

Which explanation would the nurse include when teaching a client scheduled for a bowel resection about the purpose of preoperative antibiotics? A. "They prevent incisional infection." B. "Antibiotics prevent postoperative pneumonia." C. "These medications limit the risk of a urinary tract infection." D. "They are given to eliminate bacteria from the gastrointestinal (GI) tract."

D. "They are given to eliminate bacteria from the gastrointestinal (GI) tract." The GI tract contains numerous bacteria; antibiotics are given to decrease the number of microorganisms in the bowel before surgery. Preventing incisional infection is a potential complication prevented by the use of sterile technique when changing the dressing. Avoiding postoperative pneumonia is a potential complication prevented by coughing, deep breathing, and early ambulation postoperatively. Limiting the risk of a urinary tract infection is a potential complication prevented by hygiene, meatal care, and increased hydration postoperatively.

Which information would the nurse include when teaching a client about antacid tablets? A. "Take them at 4-hour intervals." B. "Take them 1 hour before meals." C. "They are as effective as the liquid forms." D. "They interfere with the absorption of other medications."

D. "They interfere with the absorption of other medications." Antacids interfere with absorption of drugs such as anticholinergics, barbiturates, some antibiotics, and cardiac medications. They may be taken as frequently as every 1 to 2 hours without adverse effects. Antacids should be given 1 or 2 hours after meals and at bedtime. Liquid antacids have a faster onset of action than tablets.

A client with benign prostatic hypertrophy has been prescribed tamsulosin. Which statement by the nurse correctly describes how this medication works? A. "This medication will shrink your enlarged prostate gland." B. "This medication will eliminate your nocturia." C. "Your sexual desire will increase with this medication." D. "This medication will improve the flow of urine."

D. "This medication will improve the flow of urine." Tamsulosin is an alpha-adrenergic blocker that is prescribed to promote bladder and prostate gland relaxation for clients with benign prostatic hypertrophy or hyperplasia (BPH). Tamsulosin will relax the smooth muscle of the bladder neck and prostate, allowing urine to flow more easily. Tamsulosin does not shrink the prostate, nor does it increase sexual desire. Although tamsulosin may reduce episodes of having to void during the night (nocturia), it might not eliminate them.

A client begins treatment with rifampin for suspected pulmonary tuberculosis. Which information should the nurse include when teaching the client about this drug? A. "It is important to stay upright for 30 minutes after taking this drug." B. "Check your radial pulse before taking the drug." C. "Avoid prolonged exposure to the sun while taking this drug." D. "You may notice an orange-red color to your urine."

D. "You may notice an orange-red color to your urine." Rifampin can cause a harmless reddish-orange discoloration of urine, feces, saliva, sweat, tears, and skin, even contact lenses. This effect can be very alarming for the client who may interpret it as some sort of bleeding. Understanding that this is a normal effect will promote adherence. The other instructions are not indicated when taking rifampin.

A client is diagnosed with pulmonary tuberculosis, and the health care provider prescribes a combination of rifampin and isoniazid. The nurse evaluates that the teaching regarding the medications is effective when the client reports which action as most important? A. 'Report any changes in vision.' B. 'Take the medicine with my meals.' C. 'Call my doctor if my urine or tears turn red-orange.' D. 'Continue taking the medicine even after I feel better.'

D. 'Continue taking the medicine even after I feel better.' The medication should be taken for the full course of therapy; most regimens last from 6 to 9 months, depending on the state of the disease. Visual changes are not side effects of this medication. The medication should be taken 1 hour before meals or 2 hours after meals for better absorption. Urine or tears turning red-orange is a side effect of rifampin; although this should be reported, it is not an adverse side effect.

A client with type 2 diabetes takes one glyburide tablet daily. The client asks whether an extra tablet should be taken before exercise. Which response will the nurse provide? A. 'You will need to decrease how much you are exercising.' B. 'An extra pill will help your body use glucose when exercising.' C. 'The amount of medication you need to take is not related to exercising.' D. 'Do not take an extra pill because you may become hypoglycemic when exercising.'

D. 'Do not take an extra pill because you may become hypoglycemic when exercising.' Exercise improves glucose metabolism. Exercise is associated with a risk for hypoglycemia, not hyperglycemia; an additional antidiabetic agent is contraindicated. Exercise should not be decreased because it improves glucose metabolism. Also, this response does not answer the client's question. An extra tablet probably will result in hypoglycemia because exercise alone improves glucose metabolism. Control of glucose metabolism is achieved through balanced diet, exercise, and pharmacological therapy.

A 3-year-old child is prescribed a liquid iron preparation. The nurse would include which intervention when teaching the parent about the administration of this medication? A. 'Monitor the stools for diarrhea.' B. 'Administer with meals to improve absorption.' C. 'Avoid giving the child orange juice with the iron preparation.' D. 'Have the child drink the diluted iron preparation through a straw.'

D. 'Have the child drink the diluted iron preparation through a straw.' A liquid iron preparation may stain tooth enamel; therefore it should be diluted and administered through a straw. Constipation, rather than loose stools, often results from the administration of iron. Iron absorption is improved when taken on an empty stomach. The exception is acidic foods, such as citrus juices, which improve absorption.

The nurse is preparing a client for discharge from the emergency department. Which client statement provides evidence that the client understands medication teaching for high-dose ampicillin? A. 'I should take this medication with meals.' B. 'This medicine may cause constipation.' C. 'I must avoid dairy products while taking this medicine.' D. 'I must increase my intake of fluids while taking this medication.'

D. 'I must increase my intake of fluids while taking this medication.' Because penicillin in high doses is nephrotoxic, keeping hydrated maintains adequate renal perfusion for medication excretion. It should be taken on an empty stomach for best absorption. Dietary restrictions are not imposed while this medication is taken. It may cause diarrhea, but not constipation.

Which client statements indicate that the teaching about furosemide is understood? Select all that apply. One, some, or all responses may be correct. A. 'It may take 2 or 3 days for this medication to take effect.' B. 'I should wear dark glasses when outdoors during the day.' C. 'I should avoid lying flat in bed.' D. 'I need to change my position slowly.' E. 'I should eat more food that is high in potassium.'

D. 'I need to change my position slowly.' E. 'I should eat more food that is high in potassium.' Furosemide may cause hypovolemia, which can result in orthostatic hypotension with sudden changes in position. With loop diuretics, such as furosemide, there is an increase in potassium excretion, so increased potassium intake is needed. The response to furosemide is rapid with diuresis beginning within an hour of administration. Furosemide does not cause photophobia, so dark glasses are not necessary. Lying flat in bed is unnecessary.

Sublingual nitroglycerin is prescribed for a client with a history of a myocardial infarction and atrial tachycardia. The nurse instructs the client about the prophylactic use of these tablets. Which statement by the client indicates the teaching was effective? A. 'I should take the medicine three times a day.' B. 'I will be sure to take my pulse after I have exercised.' C. 'It will be important to avoid activities that can cause angina.' D. 'I should take one tablet before attempting activity that has caused angina.'

D. 'I should take one tablet before attempting activity that has caused angina.' The response about taking one tablet before activity that has caused angina indicates that the client understands the nurse's teaching. Taking a nitroglycerin tablet before such an activity probably will prevent an episode of angina, which is an example of prophylactic use of a medication. Taking the medicine three times a day is an example of scheduled administration of a nitrate medication for prophylaxis, but the client is being prescribed sublingual nitrate. The statement to avoid activities that can cause angina indicates avoidance of activity rather than taking medication to prevent angina during the activity. Blood pressure, not pulse, is the parameter most affected by nitroglycerin.

After teaching a client about sulfonamide use for a urinary tract infection, which client statement would the nurse review for correction? A. 'I will avoid the sunlight.' B. 'I will increase my fluid intake.' C. 'I will let my doctor know if I develop a rash.' D. 'I will stop taking the medication when my symptoms subside.'

D. 'I will stop taking the medication when my symptoms subside.' The nurse instructs the client to complete the entire course of treatment, not stop when symptoms subside. The client on sulfonamide therapy should avoid prolonged exposure to sun, increase fluid intake to support the kidneys, and report a rash to investigate possible hypersensitivity.

A client is admitted to the hospital for a subtotal thyroidectomy. When discussing postoperative medication therapy with the client, which advice will the nurse include in the teaching? A. 'You will be taking iodine daily to increase the formation of thyroid hormone.' B. 'After your body adjusts to postsurgical status, you will be weaned off this medication.' C. 'The propylthiouracil that is prescribed will stimulate the secretion of thyroid-stimulating hormone.' D. 'If you develop palpitations, nervousness, or tremors, the dose of thyroid hormone may need to be decreased.'

D. 'If you develop palpitations, nervousness, or tremors, the dose of thyroid hormone may need to be decreased.' Excessive thyroid hormone replacement may lead to signs and symptoms of hyperthyroidism. Iodine may be administered before, not after, surgery. Thyroid hormone replacement is required for life. Propylthiouracil blocks thyroid hormone synthesis; this often is administered before, not after, surgery.

A 20-year-old woman visiting the clinic says that she wishes to begin using depot medroxyprogesterone acetate as a form of birth control. Which important information would the nurse include when teaching the client about this medication? A. 'Medroxyprogesterone offers protection against the herpes simplex virus.' B. 'You will need a repeat injection every 6 months.' C. 'Increase your intake of iron-rich foods to prevent anemia from increased blood loss during menstruation.' D. 'Increase your calcium intake and exercise because loss of bone mineral density may occur.'

D. 'Increase your calcium intake and exercise because loss of bone mineral density may occur.' Loss of bone mineral density is a significant side effect of depot medroxyprogesterone acetate, and increased calcium intake and exercise should be encouraged. Medroxyprogesterone should be administered every 11 to 13 weeks; 6 months is too long before the next dose. Menstrual periods usually lighten or disappear over time. Medroxyprogesterone confers no protection against herpes simplex virus.

The nurse is teaching a 10-year-old child with type 1 diabetes about insulin requirements. Which statement by the nurse correctly identifies when insulin needs decrease? A. 'Insulin needs often decrease when puberty is reached.' B. 'When there is an infection is present, the body requires less insulin.' C. 'Emotional stress can cause insulin needs to decrease.' D. 'Increased muscle activity such as exercise, cause insulin needs to decrease.'

D. 'Increased muscle activity such as exercise, cause insulin needs to decrease.' Exercise reduces the body's need for insulin. Increased muscle activity accelerates transport of glucose into muscle cells, thus producing an insulin like effect. With increased growth and associated dietary intake, the need for insulin increases during puberty. An infectious process may require increased insulin. Emotional stress increases the need for insulin.

The nurse provides medication discharge instructions to a client who received a prescription for digoxin. Which statement by the client leads the nurse to conclude that the teaching was effective? A. 'I will avoid foods high in potassium.' B. 'I must increase my intake of vitamin K.' C. 'I should adjust the dosage according to my activities.' D. 'It will be important to check my pulse rate daily.'

D. 'It will be important to check my pulse rate daily.' Checking the pulse rate daily is necessary for monitoring cardiac function; digoxin slows and strengthens the heart rate. Digoxin should be withheld, and the health care provider notified, if the pulse rate falls below a predetermined rate (e.g., 60 beats per minute). Hypokalemia increases the potential for digoxin toxicity; potassium intake may need to be increased, not decreased. An increase in the intake of foods rich in vitamin K is unnecessary; digoxin does not affect vitamin K or vitamin K clotting factors. Adjusting the dosage according to activities is not an appropriate decision for the client; the health care provider should make this decision.

The parent of a newborn asks the nurse why, except for hepatitis B vaccine, the immunization does not start until the infant is 2 months. Which response would the nurse provide? A. 'A newborn's spleen can't produce efficient antibodies.' B. 'Infants younger than 2 months are rarely exposed to infectious disease.' C. 'The immunization will attack the infant's immature immune system and cause the disease.' D. 'Maternal antibodies interfere with the development of active antibodies by the inf

D. 'Maternal antibodies interfere with the development of active antibodies by the infant when immunized.' Passive antibodies received from the mother will be diminished by age 8 weeks and will no longer interfere with the development of active immunity to most communicable diseases. The spleen does not produce antibodies. Young infants often are exposed to infectious diseases. The viruses in immunizations are inactivated or attenuated; they may cause irritability and fever but will not cause the related disease.

The parents of a child with cystic fibrosis tell the nurse they have switched to natural pancreatic enzymes because of financial issues. Which response by the nurse is most appropriate? A. 'You don't need to give the enzymes now that your child is in school.' B. 'Natural enzymes don't have any side effects and can be taken without regard to meals.' C. 'If you are using generic enzymes, you will need to give twice as many to achieve the required effect.' D. 'Natural enzymes are not as effective as the brand-name product. This is something you need to discuss with your health care provider.'

D. 'Natural enzymes are not as effective as the brand-name product. This is something you need to discuss with your health care provider.' Natural pancreatic enzymes are not considered adequate in children with cystic fibrosis because of the bioavailability of the enzymes. Pancreatic enzyme supplementation is a lifelong treatment for cystic fibrosis. All medications have side effects, and pancreatic enzymes should be taken with meals. Giving twice as many natural enzymes does not constitute accurate dosing.

A client asks the nurse what she should do if she forgets to take her contraceptive pill 1 day. Which response by the nurse is appropriate? A. 'Take your pills as instructed.' B. 'Call your primary health care provider immediately.' C. 'Continue as usual, and there shouldn't be a problem.' D. 'On the next day take 1 pill in the morning and 1 pill before bedtime.'

D. 'On the next day take 1 pill in the morning and 1 pill before bedtime.' The client should make up for the missed pill by taking 2 pills the next day; taking 1 pill in the morning and 1 pill in the evening decreases the chance of the client becoming nauseated. Telling the client to take her pills as instructed does not explain what is to be done if a pill is missed; missing 1 pill can alter hormone levels and predispose the client to becoming pregnant. It is unnecessary to call the primary health care provider unless other problems are identified. Telling the client that there should be no problem if she continues as usual is incorrect advice; again, missing 1 pill can alter hormone levels and predispose a woman to pregnancy.

Which statement made by a client recently diagnosed with type 1 diabetes indicates that further education is necessary regarding the teaching plan? A. 'I will need to have my eyes and vision examined once a year.' B. 'I will need to check my blood sugar at home to evaluate my response to my treatment plan.' C. 'I can improve metabolic and cardiac risk factors of this disease if I follow a healthy diet and exercise routine.' D. 'Once I get my glucose levels under control, there is a good chance that I will be able to switch from insulin to an oral medication.'

D. 'Once I get my glucose levels under control, there is a good chance that I will be able to switch from insulin to an oral medication.' Type 1 diabetes mellitus (DM) is an autoimmune disorder in which beta cells are destroyed. No insulin or very little insulin is produced. A person with type 1 DM will need lifelong insulin injections to control blood sugar. Early detection of changes in the eye permits treatment plan adjustments that can slow or halt progression of retinopathy. Blood glucose monitoring should be done at home to evaluate the treatment plan. Disease risk factors can be improved with a healthy diet and exercise routine.

An antianxiety medication is prescribed for an extremely anxious client. The client states, 'I'm afraid to take this medication because I heard it's addictive.' Which response by the nurse is most appropriate? A. 'This medication rarely causes dependence when the dosage is controlled.' B. 'You may require increases in your dosage; however, it rarely causes dependence.' C. 'It usually results in psychological but not physiological dependence.' D.'The medication has the potential for physiological and psychological

D. 'The medication has the potential for physiological and psychological dependence.' Antianxiety medications have the potential for physiological and psychological dependence; the nurse would teach the client about both the advantages and disadvantages of taking this medication. Physiological or psychological dependence may develop even when the dosage is controlled. Tolerance does develop and can lead to dependence.

A client has been receiving digoxin. The client calls the clinic and complains of 'yellow vision.' Which response would the nurse provide? A. 'This is related to your illness rather than to your medication.' B. 'This is an expected side effect; you will become accustomed to it over time.' C. 'This side effect is only temporary. You should continue the medication.' D. 'The medication may need to be discontinued. Come to the clinic this afternoon.'

D. 'The medication may need to be discontinued. Come to the clinic this afternoon.' Yellow vision indicates digoxin toxicity; the medication should be withheld until the health care provider can assess the client and check the digoxin blood level. Yellow vision is related to digoxin therapy, not the client's underlying medical condition. Yellow vision is a sign of digoxin toxicity; taking more digoxin will escalate the digoxin toxicity.

A client with cancer experiences severe nausea and vomiting from chemotherapy. The client wants to know if it is true that smoking marijuana will help. How will the nurse respond? A. 'Nurses are not allowed to discuss illegal substances with clients.' B. 'Marijuana is effective for nausea and vomiting if it is injected.' C. 'Marijuana is not proven to be effective in preventing chemotherapy-induced nausea and vomiting.' D. 'There are some tetrahydrocannabinol (THC)-based medications that contain marijuana control chemotherapy-induced nausea and vomiting in some people.'

D. 'There are some tetrahydrocannabinol (THC)-based medications that contain marijuana control chemotherapy-induced nausea and vomiting in some people.' THC, an ingredient in marijuana, acts as an antiemetic in some people and can be absorbed through the gastrointestinal tract or inhaled. THC-based medications, dronabinol and nabilone, are available by prescription to control nausea and vomiting resulting from cancer chemotherapy. Nurses are not forbidden to talk about illegal issues and marijuana is prescribed legally in some states. Marijuana is not injected. THC is an effective antiemetic for some clients.

How would the nurse reply when a client prescribed a tetracycline class medication asks why milk and antacids should be avoided before and after dosing? A. 'Taking these together can lead to kidney impairment.' B. 'The pairing of these substances leads to tooth staining.' C. 'Severe diarrhea can occur when taking these substances together.' D. 'This can lead to decreased absorption of the medication you need.'

D. 'This can lead to decreased absorption of the medication you need.' Tetracyclines chelate with calcium, iron, and magnesium, so substances containing these minerals are avoided to optimize absorption of the antimicrobial.

When a female client becomes hypothyroid, levothyroxine is prescribed. The client asks whether she can become pregnant while taking levothyroxine. How will the nurse respond? A. 'If you become pregnant, thyroid abnormalities will develop in the fetus.' B. 'Yes, but you will have a high-risk pregnancy.' C. 'This medication causes infertility for the length of time that it is taken.' D. 'This medicine will not interfere with your ability to become pregnant.'

D. 'This medicine will not interfere with your ability to become pregnant.' Hormone replacement should stabilize the metabolic rate and should not interfere with the client's becoming pregnant. If thyroid function remains controlled, there is no reason why the client should not become pregnant. Because thyroid function will be normalized, the fetus will not be negatively affected, and pregnancy risk will not be increased.

The nurse teaches the parents of a child prescribed long-term phenytoin therapy about care. Which statement indicates the teaching has been effective? A. 'We give the medication between meals.' B. 'We'll call the clinic if her urine turns pink.' C. 'She's eating high-calorie foods, and we encourage fluids, too.' D. 'We'll have her massage her gums and floss her teeth frequently.'

D. 'We'll have her massage her gums and floss her teeth frequently.' A common side effect of phenytoin is gingival hyperplasia. Meticulous oral hygiene may reduce the risk of this side effect. Phenytoin is strongly alkaline and should be administered with meals to help prevent gastric irritation. Pink urine may be observed during medication excretion; it is expected and does not require treatment. Avoidance of overeating and overhydration may result in better seizure control.

Allopurinol is prescribed for a child undergoing chemotherapy for cancer of the bone. When given the medication, the child asks, 'Why do I have to take this pill?' Which response by the nurse is most appropriate? A. 'It protects your body from getting new problems after your treatment is over.' B. 'It stops your sick white cells from going to other parts of your body.' C. 'The health care provider wouldn't prescribe anything for you unless it was important.' D. 'With the other medicines, it helps you get rid of the things that are making you sick.'

D. 'With the other medicines, it helps you get rid of the things that are making you sick.' Telling the child it helps get rid of the things making the child sick is the most accurate and age-appropriate response to the child's question. Telling the child that the medicine protects the body from new problems is inaccurate, and not being truthful will interfere with the development of the child's trust in the nurse. Telling the child that it stops sick white cells from spreading is inaccurate and may instill more fear. Telling the child that it is needed because the health care provider says so is insensitive to the question and does not provide an explanation.

Carbidopa-levodopa is prescribed for a client with Parkinson's disease. Which instruction will the nurse include when teaching the client about this medication? A. 'Take this medication between meals.' B. 'Blood levels of the medication should be monitored weekly.' C. 'It can cause happy feelings followed by feelings of depression.' D. 'You may experience dizziness when moving from sitting to standing.'

D. 'You may experience dizziness when moving from sitting to standing.' Carbidopa-levodopa is a metabolic precursor of dopamine; it reduces sympathetic outflow by limiting vasoconstriction, which may result in orthostatic hypotension. Carbidopa-levodopa should be administered with food to minimize gastric irritation. Although periodic tests to evaluate hepatic, renal, and cardiovascular status are required for prolonged therapy, these tests aren't required on a weekly basis. Carbidopa-levodopa may produce either happiness or depression, but no established pattern of such responses exists.

A breast-feeding mother asks the nurse if the use of herbal medicines will increase breast milk supply. Which nursing response is most appropriate? A. 'It may be safe if taken with lots of water.' B. 'It does not effectively increase breast milk supply.' C. 'It may cause iron deficiency anemia in the infant.' D. 'You should speak to your health care provider about this.'

D. 'You should speak to your health care provider about this.' The use of herbs may increase breast milk supply, but research is limited, so the mother should consult with her health care provider. The herbs are safe for the mother with or without water. However, the priority in this case is to inform the parent of the adverse effects that can result in the infant. Early introduction of solids may increase the risk for iron deficiency anemia in the infant. The herbs increase breast milk supply.

Which period of time would a nurse recognize as the greatest risk of hypoglycemia when caring for a client who receives regular insulin daily at 8:00 AM? A. 8:30 AM to 9:30 AM B. 8:00 PM to 12:00 AM C. 1:00 PM to 8:00 PM D. 10:00 AM to 1:00 PM

D. 10:00 AM to 1:00 PM Regular insulin peaks in 2 to 5 hours; therefore the greatest risk is between 10:00 AM and 1:00 PM. Although the onset of action occurs earlier, during the period from 8:30 AM to 9:30 AM, the level is not yet at its highest, so the risk of hypoglycemia is not at its greatest. NPH insulin's peak action is 4 to 12 hours; if hypoglycemia occurs, it will most likely happen between 12:00 PM and 8:00 PM.

Nortriptyline is prescribed for a depressed client. Which time period identifies when the nurse would expect a therapeutic response? A. 1 to 3 days B. 12 to 24 hours C. 30 minutes to 2 hours D. 2 to 3 weeks

D. 2 to 3 weeks As with other tricyclics, optimal therapeutic effects take 2 to 3 weeks to occur. One to 3 days, 12 to 24 hours, and 30 minutes to 2 hours are all too soon to expect a response to nortriptyline.

The nurse teaches an adolescent with type 1 diabetes about peak action of NPH insulin and the risk for hypoglycemia. The nurse determines teaching has been effective when the adolescent identifies insulin peak action within which time frame? A. 1 to 2 hours B. 2 to 4 hours C. 5 to 10 hours D. 4 to 12 hours

D. 4 to 12 hours NPH insulin onset is 1.5 to 4 hours, peaks in 4 to 12 hours, and has a duration of 12 to 18 hours.

The nurse gave a client the prescribed sodium polystyrene sulfonate. Which assessment finding indicates that the medication has been effective? A. Control of diarrhea B. An increase in serum sodium level C. An increase in serum calcium level D. A decrease in serum potassium level

D. A decrease in serum potassium level Sodium polystyrene sulfonate is given to treat hyperkalemia. The effectiveness of the medication is determined by a decreasing serum potassium level. Sodium polystyrene sulfonate binds with the potassium in the gastrointestinal system and often causes diarrhea. Sodium retention and hypernatremia may occur as an adverse effect; this does not indicate effectiveness. Sodium polystyrene sulfonate decreases serum calcium levels in a small number of clients but does not increase calcium.

A nurse is assessing a client with heart failure who is taking prescribed torsemide. Which clinical finding indicates effectiveness of the medication? A. Symmetrical pulses bilaterally B. Full strength to bilateral extremities C. Intact whisper test D. Absence of peripheral edema

D. Absence of peripheral edema Rationale: Torsemide is a loop diuretic used in the treatment of hypertension and fluid overload. The expected therapeutic response of torsemide is a decrease in fluid retention evidenced by the absence of peripheral edema. Symmetrical pulses bilaterally and full strength to bilateral extremities do not evaluate the effectiveness of torsemide. An intact whisper test indicates the absence of ototoxicity, an adverse effect of torsemide. However, this does not evaluate medication effectiveness.

Which action is likely to reduce the pancreatic and gastric secretions of a client with pancreatitis? A. Encourage clear liquids. B. Obtain a prescription for morphine. C. Assist the client into a semi-Fowler position. D. Administer prescribed anticholinergic medication.

D. Administer prescribed anticholinergic medication. Anticholinergic medications block the neural impulses that stimulate pancreatic and gastric secretions. Oral fluids stimulate pancreatic secretion. Morphine sulfate is an analgesic and does not decrease gastric secretions. The semi-Fowler position decreases pressure against the diaphragm to help relieve discomfort, but it does not decrease pancreatic secretions.

A sulfonamide preparation is prescribed for a child with a urinary tract infection. Which nursing responsibility is a priority when administering this medication? A. Weighing the child daily B. Giving the medication with milk C. Taking the child's temperature frequently D. Administering the medication at the prescribed times

D. Administering the medication at the prescribed times For the desired blood level to be maintained, the medication must be administered in the exact amount at the times directed. If the blood level of the medication falls, the microorganisms have an opportunity to build resistance to the medication. Weighing is important with medications that affect fluid balance, such as diuretics. Sulfa medications should be given on an empty stomach to promote absorption. Monitoring the temperature is important with antipyretic medications.

A client with the diagnosis of primary hypertension is started on a regimen of hydrochlorothiazide. Which information will the nurse include when providing instructions regarding this medication? A. A common side effect is decreased sexual libido. B. One dose should be omitted if dizziness occurs when standing up. C. The client should adjust the dosage daily based on the client's blood pressure. D. An antihypertensive medication will likely be required for the remainder of life.

D. An antihypertensive medication will likely be required for the remainder of life. If medication is necessary to control primary hypertension, usually it is a lifetime requirement. The client will not adjust the dosage without the health care provider's direction. Impotence may occur with some antihypertensive medications but not with hydrochlorothiazide. The medication will not be stopped; orthostatic hypotension can be controlled by a slow change of body position.

A client taking multiple medications for hypertension develops a persistent, hacking cough. Which antihypertensive medication class would the nurse identify as the likely cause of the cough? A. Thiazide diuretics B. Calcium channel blockers C. Direct renin inhibitors D. Angiotensin-converting enzyme (ACE) inhibitors

D. Angiotensin-converting enzyme (ACE) inhibitors The ACE breaks down kinins. When ACE is inhibited, the increase of kinins in the lung can cause bronchial irritation, leading to the common adverse effect sometimes referred to as an ACE cough. A cough is not a side effect of thiazide diuretics, calcium channel blockers, or direct renin inhibitors.

A client develops leukopenia 3 weeks after having a renal transplant. Which factor would the nurse conclude is the cause of the leukopenia? A. Bacterial infection B. High creatinine levels C. Rejection of the kidney D. Antirejection medications

D. Antirejection medications Antirejection medications alter the immune response by causing bone marrow suppression. The white blood cell (WBC) count drops precipitously. Leukocytosis, not leukopenia, occurs with an infection. High creatinine levels are related to kidney failure, but do not cause leukopenia. The WBC count is increased, not decreased, with kidney rejection.

The nurse is preparing to administer digoxin to a client admitted for acute decompensated heart failure. Which action is the priority before giving this drug? A. Monitor oxygen saturation on room air B. Assess the client's weight and compare to the baseline C. Auscultate the lungs for crackles in the bases D. Assess the apical pulse for a full minute

D. Assess the apical pulse for a full minute Digoxin, a cardiac glycoside, is used to slow the heart rate and increase the force of contraction. The priority for the nurse is to count the client's apical pulse for one full minute, even if the heart rhythm is regular. Typically, when the pulse is less than 60, digoxin should not be given. The other actions are also appropriate assessments for a client with heart failure. However, they are not the priority when administering digoxin.

Which action will the nurse take after stopping the antibiotic infusion of a client who becomes restless and flushed, and begins to wheeze during the administration of an antibiotic? A. Check the client's temperature. B. Take the client's blood pressure. C. Obtain the client's pulse oximetry. D. Assess the client's respiratory status.

D. Assess the client's respiratory status. The client is experiencing an allergic reaction that may progress to anaphylaxis. Anaphylactic shock can lead to respiratory distress as a result of laryngeal edema or severe bronchospasm. Assessing and maintaining the client's airway is the priority. Checking the client's temperature and taking the client's blood pressure are not the priority; vital signs should be obtained after airway patency is ensured and maintained. Pulse oximetry is only one portion of the needed respiratory status assessment.

The nurse is preparing to administer an intravenous piggyback antibiotic that has been newly prescribed. Shortly after initiation, the client becomes restless and flushed and begins to wheeze. After stopping the infusion, which priority action will the nurse take? A. Notify the primary health care provider immediately about the client's condition. B. Take the client's blood pressure. C. Obtain the client's pulse oximetry. D. Assess the client's respiratory status.

D. Assess the client's respiratory status. The client is experiencing an allergic reaction. Severe allergic reactions commonly cause respiratory distress as a result of laryngeal edema or severe bronchospasm. Assessing and maintaining the client's airway is the priority. The nurse must determine the client's status before notifying the primary health care provider. Vital signs, including blood pressure and pulse oximetry, are obtained after airway patency is ensured and maintained.

A nurse is providing care to a client who takes phenytoin for seizure prevention. The latest laboratory report shows a phenytoin level of 32 mcg/mL. Which action does the nurse take next? A. Examine the oral cavity B. Percuss the abdomen C. Check the skin turgor D. Assess the pupillary response

D. Assess the pupillary response Rationale: A phenytoin level of 32 mcg/mL is not an expected response to therapy. The therapeutic range of phenytoin is 10 to 20 mcg/mL. Signs of phenytoin toxicity include nystagmus, ataxia, and confusion. The pupillary response will assess for symmetrical movements of the eye. Examining the oral cavity, percussing the abdomen, and checking for skin turgor do not evaluate symptoms of phenytoin toxicity.

During a procedure, the client's heart rate drops to 38 beats/min. Which medication is indicated to treat bradycardia? A. Digoxin B. Lidocaine C. Amiodarone D. Atropine sulfate

D. Atropine sulfate Atropine blocks vagal stimulation of the sinoatrial (SA) node, resulting in an increased heart rate. Digoxin slows the heart rate; hence, it would not be indicated in this situation. Lidocaine decreases myocardial sensitivity and will not increase the heart rate. Amiodarone is an antidysrhythmic medication used for ventricular tachycardia; it will not stimulate the heart rate.

A health care provider prescribes tolterodine for a client with an overactive bladder. Which action is important to include in client teaching? A. Maintain a strict record of fluid intake and urinary output. B. Chew the extended-release capsule thoroughly before swallowing. C. Report episodes of diarrhea or any increase in respiratory secretions. D. Avoid activities requiring alertness until the response to medication is known.

D. Avoid activities requiring alertness until the response to medication is known. Tolterodine, a urinary tract antispasmodic, may cause dizziness and blurred vision, placing the client at risk for injury. Although it is important to know if the client is experiencing anuria or overflow incontinence, which may indicate urinary retention, a detailed intake and output record is unnecessary. An extended-release capsule should be swallowed whole and should not be opened or chewed. If chewed or opened, the client will receive a surge of action and the long-term action of the medication is gone. Tolterodine is classified as an anticholinergic, and adverse reactions include constipation and dry mouth; diarrhea and an increase in respiratory secretions are associated with medications classified as cholinergics.

A nurse is providing education on the use of carbidopa/levodopa to a client with Parkinson's disease. What will the nurse include in the teaching? A. This medication will stop the progression of your condition B. Notify your healthcare provider if your urine appears dark C. Eat plenty of whole-grain foods when taking this medication D. Avoid eating meals that are high in protein

D. Avoid eating meals that are high in protein Rationale: Carbidopa/levodopa is a combination medication used in the management of Parkinson's disease. Consuming high-protein meals can impair the effects of levodopa. The nurse should instruct the client to eat protein in small portions. Carbidopa/levodopa does not halt the progression of Parkinson's disease. The medication is intended to reduce the symptoms associated with the condition. Darkening of bodily fluids can occur when taking the medication. However, the client should be informed this is not a harmful side effect. Whole grains contain pyridoxine, a vitamin that interferes with the effects of levodopa.

A client was prescribed furosemide. The nurse would instruct the client to include which food in the diet? A. Liver B. Apples C. Cabbage D. Bananas

D. Bananas Furosemide is a loop diuretic that increases potassium excretion by preventing renal absorption. Bananas have a significant amount of potassium. Bananas: 450 mg; cabbage: 243 mg; liver: 73.6 mg; apples: 100 to 120 mg.

The nurse would counsel a pregnant client to take her iron supplement at which time of the day for efficient absorption? A. Bedtime B. After lunch C. Dinnertime D. Before breakfast

D. Before breakfast Iron should be taken before breakfast on an empty stomach to permit maximal absorption. Iron should not be taken at bedtime or with meals or after meals.

An adolescent with leukemia is to be given a chemotherapeutic agent. Which time is best for the nurse to administer the prescribed antiemetic? A. As nausea occurs B. An hour before meals C. Just before each meal is eaten D. Before each dose of chemotherapy

D. Before each dose of chemotherapy The purpose of an antiemetic before chemotherapy is to prevent the child from experiencing nausea during and after the administration of the medication. Waiting until nausea has occurred is too late; the medication should be given before nausea occurs. The meals are not causing the nausea; the nausea is caused by the chemotherapy, and if nausea is not prevented, the child will not eat.

Which class of medication would the nurse anticipate preparing to administer to a client with alcohol dependence admitted to a detoxification unit? A. Opiate B. Antipsychotic C. Antidepressant D. Benzodiazepine

D. Benzodiazepine Benzodiazepines are indicated for the treatment of alcohol withdrawal because they help prevent seizures and calm the vital signs. Opiates are not indicated for the treatment of alcohol detoxification. Antipsychotics and antidepressants are contraindicated because they can lower the seizure threshold.

A client receiving fluphenazine decanoate develops dystonia/extrapyramidal side effect early in therapy. Which medication would the nurse anticipate administering to reverse this side effect? A. Nafarelin B. Fluoxetine C. Trandolapril D. Benztropine

D. Benztropine Dystonia is an extrapyramidal side effect (EPS) of fluphenazine decanoate. The anticholinergic benztropine is used to reverse the signs and symptoms (e.g., oculogyric crisis, torticollis, retrocollis) of dystonia. Nafarelin is a gonadotropin that stimulates the release of luteinizing hormone and follicle-stimulating hormone. Fluoxetine is a selective serotonin reuptake inhibitor antidepressant. Trandolapril is an angiotensin-converting enzyme inhibitor antihypertensive.

A client develops extrapyramidal effects after taking a neuroleptic medication, and the nurse notes extrapyramidal effects. Which medication can limit these side effects? A. Zolpidem B. Hydroxyzine C. Dantrolene D. Benztropine mesylate

D. Benztropine mesylate Benztropine mesylate, an anticholinergic, helps balance neurotransmitter activity in the central nervous system (CNS) and helps control extrapyramidal tract symptoms. Zolpidem is a sedative-hypnotic medication used for short-term insomnia. Hydroxyzine is a sedative that depresses activity in the subcortical areas in the CNS; it is used to reduce anxiety. Dantrolene, a muscle relaxant, has a direct effect on skeletal muscle by acting on the excitation-contraction coupling of muscle fibers and not at the level of the CNS, unlike most other muscle relaxation medications.

The nurse working in an intensive care unit is caring for a client diagnosed with acute angina. The client is receiving an intravenous infusion of nitroglycerin. What is the priority assessment for this client? A. Heart rate B. Neurologic status C. Urine output D. Blood pressure

D. Blood pressure Nitroglycerin (NTG) is a vasodilator used to promote myocardial tissue perfusion and relieve chest pain associated with coronary artery occlusion. The systemic vasodilation that occurs as a result of this medication can cause profound hypotension. The client's blood pressure should be evaluated every 15 minutes until stable, and then every 30 minutes to every hour thereafter. Clients receiving IV nitroglycerin should also be placed on continuous ECG monitoring. NTG is not known to affect neurologic status, urine output or heart rate.

The nurse on a cardiac unit is caring for a client who is receiving nitroglycerin intravenously for unstable angina. During administration of the medication, which assessment is the priority? A. Respiratory rate B. Cardiac enzymes C. Cardiac rhythm D. Blood pressure

D. Blood pressure Nitroglycerin is a drug that is used to provide relief from myocardial chest pain and treat hypertensive emergencies. Nitroglycerin causes vasodilation. Common adverse effects of nitroglycerin include hypotension, headache and dizziness; therefore, monitoring the client's blood pressure is the priority. Nitroglycerin does not affect respirations, cardiac enzyme levels or heart rhythm.

A 31-year-old client is seeking contraceptive information. While obtaining the client's history, which factor indicates to the nurse that oral contraceptives are contraindicated? A. Older than 30 years B. Current hypothyroidism C. Two multiple pregnancies D. Blood pressure 162/110

D. Blood pressure 162/110 Oral contraceptives may cause or exacerbate hypertension; even borderline hypertension places the client at risk for a brain attack. Oral contraceptives are not contraindicated for women older than 30 years of age if there are no known risk factors. There is no relationship between oral contraceptives and multiple births. Contraceptives are not contraindicated in clients who have hypothyroidism.

Which client response indicates to the nurse that a vasodilator medication is effective? A. Absence of adventitious breath sounds B. Increase in the daily amount of urine produced C. Pulse rate decreases from 110 to 75 beats/minute D. Blood pressure changes from 154/90 to 126/72 mm Hg

D. Blood pressure changes from 154/90 to 126/72 mm Hg Vasodilation will lower the blood pressure. The pulse rate is not decreased and may increase. Breath sounds are not directly affected by vasodilation, although vasodilator medications can decrease preload and afterload, which could indirectly affect breath sounds in heart failure. The urine output is not affected immediately, although control of blood pressure can help preserve renal function over time.

A client is receiving clonidine 0.1 mg/24 hr via transdermal patch. Which assessment finding indicates the desired effect of the medication has been achieved? A. Absence of nausea and vomiting. B. Change in peripheral edema from +3 to +1. C. Denial of anginal pain and shortness of breath. D. Blood pressure from 180/120 mmHg to 140/70 mmHg.

D. Blood pressure from 180/120 mmHg to 140/70 mmHg. Clonidine acts as a centrally-acting analgesic and antihypertensive agent. A reduction of the blood pressure reading of 180/120 mmHg to 140/70 mmHg indicates a reduction in hypertension.

Which change in data indicates to the nurse that the desired effect of the angiotensin II receptor antagonist valsartan has been achieved? A. Dependent edema reduced from +3 to +1. B. Serum HDL increased from 35 to 55 mg/dL. C. Pulse rate reduced from 150 to 90 beats/minute. D. Blood pressure reduced from 160/90 mmHg to 130/80 mmHg.

D. Blood pressure reduced from 160/90 mmHg to 130/80 mmHg. Valsartan is an angiotensin receptor blocker, prescribed for the treatment of hypertension. The desired effect is a decrease in blood pressure.

A client with an infection is receiving vancomycin. Which laboratory blood test result would the nurse report? A. Hematocrit: 45% B. Calcium: 9.0 mg/dL (2.25 mmol/L) C. White blood cells (WBC): 10,000 mm 3 (10 × 10 9/L) D. Blood urea nitrogen (BUN): 30 mg/dL (10.2 mmol/L)

D. Blood urea nitrogen (BUN): 30 mg/dL (10.2 mmol/L) Vancomycin is a nephrotoxic medication. An elevated BUN can be an early sign of toxicity. The BUN of a healthy adult is 10 to 20 mg/dL (3.6-7.1 mmol/L). This hematocrit is expected in a healthy adult; the range is from 40 to 52. The expected range of the white blood cell (WBC) count is 5000 to 10,000 mm 3 (5 to 10 × 10 9/L) for a healthy adult. This calcium level is within the expected range of 9.0 to 10.5 (2.25-2.75 mmol/L) for a healthy adult.

A client is prescribed an oral contraceptive. As part of teaching, the nurse plans to inform the client of the possibility of which adverse effect? A. Cervicitis B. Ovarian cysts C. Fibrocystic breasts D. Breakthrough bleeding

D. Breakthrough bleeding Breakthrough bleeding, or midcycle bleeding, commonly occurs when women start using oral contraceptives. If it persists, the dosage should be changed. There is no evidence that cervicitis, ovarian cysts, or fibrocystic breasts are related to the use of oral contraceptives.

A client with a diagnosis of dementia of the Alzheimer type has been taking donepezil 10 mg/day for 3 months. The client's partner calls the clinic and reports that the client has increasing restlessness and agitation accompanied by nausea. Which advice would the nurse give the partner? A. Give the medication with food. B. Administer the medication at bedtime. C. Omit 1 dose today and start with a lower dose tomorrow. D. Bring the partner to the clinic for testing and a physical examination.

D. Bring the partner to the clinic for testing and a physical examination.. Many people with dementia experience physical problems such as urinary tract infections but cannot adequately verbalize what is happening. They may just become more restless and agitated. Because the client has been taking this dose for 3 months, the problems probably are not being caused by the medication. The client should be brought in for an evaluation. Taking the medication with meals is recommended to decrease gastrointestinal side effects, but this client is experiencing more than this. Donepezil can cause insomnia. The client is already restless and agitated. Taking the medication at bedtime will not help. The nurse would not advise a modification of the dosage without consulting the health care provider.

Which clinical finding would indicate to the nurse that an infant is experiencing life-threatening zidovudine toxicity? A. Fatigue and lethargy B. Increased urine output C. Progressive weight loss D. Bruises on the limbs and trunk

D. Bruises on the limbs and trunk Zidovudine can cause life-threatening blood dyscrasias, including thrombocytopenia. With zidovudine toxicity the infant will demonstrate agitation, restlessness, and insomnia, not fatigue and lethargy. Urine output is unrelated to zidovudine toxicity; decreased urine output may be related to decreased fluid intake, vomiting, and diaphoresis associated with the illness. Weight loss is usually a response to the illness rather than to the therapy.

A client is diagnosed with trigeminal neuralgia. Which nonanalgesic can decrease the client's pain? A. Echinacea B. Leucovorin C. Allopurinol D. Carbamazepine

D. Carbamazepine Carbamazepine is an anticonvulsant that can be used to control neurological pain such as trigeminal neuralgia. Echinacea is an herbal medication used by some to treat respiratory infections, but not neuralgic pain. Leucovorin is used as rescue medication with methotrexate to prevent cellular damage; it does not relieve neurological pain. Allopurinol is used in the treatment of gout but does not relieve pain.

A client is prescribed furosemide and digoxin for heart failure. The nurse should monitor the client for which potential adverse drug effect? A. Pulmonary hypertension B. Acute arterial occlusion C. Acute kidney injury D. Cardiac dysrhythmias

D. Cardiac dysrhythmias Digoxin is a cardiac glycoside, or positive inotrope that increases myocardial contractility. By increasing contractile force, digoxin can increase cardiac output in clients with heart failure (HF). Furosemide is a potassium-wasting (loop) diuretic, prescribed to prevent fluid overload in clients with HF. Clients who take furosemide are at risk for developing hypokalemia. Potassium ions compete with digoxin and a low potassium level can cause digoxin toxicity, leading to lethal cardiac dysrhythmias. Therefore, it is imperative that potassium levels be kept within normal range (3.5 to 5 mEq/L) while taking digoxin.

A client receives a cardiac glycoside, a diuretic, an angiotensin-converting enzyme (ACE) inhibitor, and a vasodilator. The client's apical pulse rate is 44 beats/minute. The nurse concludes that the decreased heart rate is caused by which medication? A. Diuretic/furosemide B. Vasodilator/nitroglycerin C. ACE inhibitor/ "ace" to -pril D. Cardiac glycoside/digoxin

D. Cardiac glycoside/digoxin A cardiac glycoside such as digoxin decreases the conduction speed within the myocardium and slows the heart rate. The primary effect of a diuretic is on the kidneys, not the heart; it may reduce the blood pressure, not the heart rate. A vasodilator can cause tachycardia, not bradycardia, which is an adverse effect. ACE inhibitors act on the renin-angiotensin system and are not associated with decreased heart rates.

A client receives doxorubicin infusions for treatment of acute lymphocytic leukemia. Which clinical finding indicates that toxicity has occurred? A. Alopecia B. Dyspnea C. Metallic taste to food D. Cardiac rhythm abnormalities

D. Cardiac rhythm abnormalities Doxorubicin is cardiotoxic, which is manifested by transient electrocardiogram (ECG) abnormalities. Alopecia is an expected side effect of doxorubicin, not a toxic effect. Dyspnea and a metallic taste to food are not effects of doxorubicin.

Which advice would the nurse include in a teaching plan to reduce the side effects of diltiazem? A. Lie down after meals. B. Avoid dairy products in diet. C. Take the medication with an antacid. D. Change slowly from sitting to standing.

D. Change slowly from sitting to standing Changing positions slowly will help prevent the side effect of orthostatic hypotension. Diltiazem decreases esophageal tone, so lying down after meals can lead to acid reflux. Avoiding dairy products and taking the medication with an antacid are not necessary.

How would the nurse determine if a client is experiencing the therapeutic effect of valsartan? A. Check a lipid profile. B. Assess an apical pulse. C. Measure urinary output D. Check the blood pressure.

D. Check the blood pressure. Angiotensin II receptor blockers (ARBs) are antihypertensive medications that lower the blood pressure. ARBs do not directly affect lipid profile, apical pulse, or urinary output.

A child is treated with succimer for lead poisoning. Which of these assessments is the priority? A. Check the client's serum potassium level. B. Check the client's blood calcium level. C. Test the client's deep tendon reflexes. D. Check the client's complete blood count with differential.

D. Check the client's complete blood count with differential. Succimer is used in the management of lead or other heavy metal poisoning. Although it is generally well-tolerated and has a relatively low toxicity, it may cause neutropenia. Succimer therapy should be withheld or discontinued if the absolute neutrophil count (ANC) is below 1,200/mm3. The normal range for an ANC is 1.5 to 8.0 (1,500 to 8,000/mm3). Therefore, the assessment priority in this scenario is checking the complete blood count (CBC) with differential which includes an ANC value.

A client is diagnosed with multiple myeloma. Which intervention would the nurse expect the plan of care to include? A. Radiotherapy on an outpatient basis B. Human leukocyte interferon therapy C. Surgery to remove the invasive lesions D. Chemotherapy employing a combination of medications

D. Chemotherapy employing a combination of medications Chemotherapy employing a combination of drugs is the treatment of choice; a variety of chemotherapeutic medications affect rapidly dividing cells at different stages of cell division. Although radiotherapy on an outpatient basis may be used to alleviate pain and treat acute vertebral lesions, it is not the primary approach. Although human leukocyte interferon therapy may be done, it is not the primary treatment. Multiple myeloma is a diffuse disorder of the bone, and no single lesion can be removed.

The health care provider prescribes alprazolam 5 mg by mouth three times a day for a client with anxiety. Which intervention will the nurse take before administering this prescription? A. Assess the apical pulse. B. Check the blood pressure. C. Encourage the ventilation of feelings. D. Clarify the prescription with the health care provider.

D. Clarify the prescription with the health care provider. The prescribed dosage is excessive, and it must be questioned before its administration. Checking the apical pulse and/or blood pressure is not indicated. Encouraging the client to ventilate feelings does not affect the need to question the prescription. Therapeutic dosages of alprazolam range from 0.75 mg to 4 mg daily; the maximal daily dose for acute anxiety is 8 mg.

A nurse is providing care to an older adult client with newly diagnosed heart failure. The nurse receives a prescription for digoxin PO 1.5 mg daily. Which action does the nurse perform next? A. Instruct the client to take the heart rate before administration B. Educate the client on the purpose of digoxin C. Administer the medication to the client D. Clarify the prescription with the healthcare provider

D. Clarify the prescription with the healthcare provider Rationale: Older adult clients (geriatric) have a high sensitivity to the toxic effects of digoxin. A dose of 1.5 mg daily is above the recommended range for adults. The initial daily dose for a geriatric client should not exceed 0.125 mg. Educating the client on the purpose of digoxin and performing related assessments are expected interventions. However, the nurse should clarify the dose first. Administering the prescribed medication dose to the client may result in significant side effects.

The nurse is completing a health history of a client diagnosed with Alzheimer's disease. The nurse reviews a list of the client's medications and supplements routinely taken at home. Which treatment should be a cause for concern by the nurse? A. Donepezil B. Ginkgo biloba C. Omega-3 fatty acids D. Coconut oil

D. Coconut oil Rationale: Donepezil, rivastigmine, and galantamine are most commonly used in the treatment of Alzheimer's disease (AD). Complementary and integrative therapies used to treat AD include Gingko biloba (a plant extract) and omega-3 fatty acids. While there isn't sufficient research to support using these treatments, continued use won't necessarily be harmful. However, coconut oil, which is a source of caprylic acid, is a concern. While there has been limited research on Katasyn (an experimental drug containing caprylic acid), there is no scientific evidence that coconut oil is safe and effective or prevents cognitive decline.

Which mechanism of action explains the ability of nitrogen mustard to interfere with growth of cancer cells? A. Interference of the cellular protein synthesis B. Inhibition of the synthesis of purine and pyrimidine C. Binding with DNA to interfere with RNA production D. Combining with DNA strands and interfering with cell replication

D. Combining with DNA strands and interfering with cell replication Alkylating agents such as nitrogen mustard combine with DNA strands and interfere with cell replication. Some chemotherapeutic medications are believed to act by interfering with cellular protein synthesis, but nitrogen mustard does not. Inhibiting the synthesis of purine and pyrimidine is the mechanism of action of antimetabolites. Antibiotics, not nitrogen mustard, used in cancer chemotherapy are believed to act by binding with DNA to interfere with RNA production.

Which of the following instructions is most important for the nurse to include when discharging a client with an infection caused by staphylococcus? A. Schedule follow-up blood cultures B. Monitor for signs of recurrent infection C. Visit the provider in a few weeks D. Complete the full course of the antibiotic

D. Complete the full course of the antibiotic Staphylococcus is a bacteria and to rid the body of the infection, it is most important to instruct the client to complete the full course of antibiotics. Not completing the full course of antibiotics can lead to antibiotic resistant infections. At this point, there is no indication for the need for blood cultures. The client will need a follow-up appointment with the provider, and will need to monitor for signs of recurrent infections, but these are not as high a priority as completing the full course of antibiotics.

The nurse provides discharge teaching to a client with tuberculosis. Which treatment measure would the nurse reinforce as the highest priority? A. Getting sufficient rest B. Getting plenty of fresh air C. Maintaining a healthy lifestyle D. Consistently taking prescribed medication

D. Consistently taking prescribed medication Tubercle bacilli are particularly resistant to treatment and can remain dormant for prolonged periods; medication must be taken consistently as prescribed. Although getting sufficient rest, getting plenty of fresh air, and maintaining a healthy lifestyle are important, to heal the microorganisms must be eliminated with medication.

The nurse teaches a client's family about the administration of donepezil for treatment of dementia of the Alzheimer type. Which side effect identified by the caregiver indicates to the nurse that further teaching is needed? A. Nausea B. Dizziness C. Headache D. Constipation

D. Constipation Donepezil, a cholinesterase inhibitor, may cause nausea, vomiting, increased salivation, diarrhea, and involuntary defecation related to the increase in gastrointestinal secretions and activity caused by parasympathetic nervous stimulation; it does not cause constipation. Common side effects of donepezil include anorexia, nausea, and vomiting that result from stimulation of the parasympathetic nervous system. Dizziness and headache are common side effects of donepezil that result from central nervous system cholinergic effects.

A client with myasthenia gravis is receiving pyridostigmine bromide to control symptoms. Recently, the client has begun experiencing increased difficulty in swallowing. Which nursing action is effective in preventing aspiration of food? A. Place a tracheostomy set in the client's room. B. Assess respiratory status after meals. C. Request for the diet to be changed from soft to clear liquids. D. Coordinate mealtimes with the peak effect of the medication.

D. Coordinate mealtimes with the peak effect of the medication. Dysphagia should be minimized during peak effect of pyridostigmine bromide, thereby decreasing the probability of aspiration. A tracheostomy set is a treatment for, rather than equipment to prevent, aspiration. Although it is vital that the client's respiratory function be monitored, assessing the client's respiratory status will not prevent aspiration. There are insufficient data to determine whether changing the diet from soft foods to clear liquids is appropriate; also, liquids are aspirated more easily than semisolids.

The nurse instructs a postpartum client on the administration of an iron supplement. Which drink selected by the client indicates the teaching was effective? A. Milk B. Water C. Cream soda D. Cranberry juice

D. Cranberry juice Iron is absorbed best when given in an acidic medium. One cup of cranberry juice contains 90 mg of vitamin C (ascorbic acid). Milk, water, and cream soda will all decrease the acidity of the stomach.

Which nursing assessment is most important for a child receiving cyclophosphamide? A. Extent of alopecia B. Changes in appetite C. Hyperplasia of gums D. Daily intake and output

D. Daily intake and output Hemorrhagic cystitis is a potentially serious adverse reaction to cyclophosphamide that can sometimes be prevented with increased fluid intake because the fluid flushes the bladder. The extent of hydration can be measured with hourly documentation of intake and output. Alopecia is expected; however, it is a benign side effect, and the hair will regrow when therapy is completed. A change in appetite is expected but is not a serious side effect of cyclophosphamide administration. Hyperplasia of the gums is unrelated to cyclophosphamide administration.

A client is prepared for a supratentorial craniotomy with burr holes, and an intravenous infusion of mannitol is instituted. The nurse concludes that this medication is primarily given for which purpose? A. Lower blood pressure B. Prevent hypoglycemia C. Increase cardiac output D. Decrease fluid in the brain

D. Decrease fluid in the brain Osmotic diuretics remove excessive cerebrospinal fluid (CSF), reducing intracranial pressure. Osmotic diuretics increase, not decrease, the blood pressure by increasing the fluid in the intravascular compartment. Osmotic diuretics do not directly influence blood glucose levels. Although there is an increase in cardiac output when the vascular bed expands as CSF is removed, it is not the primary purpose of administering the medication.

The nurse is caring for a client who has been prescribed atropine preoperatively. The nurse understands the intended purpose for administering this preoperatively is to induce which effect? A. Reduce heart rate B. Elevate blood pressure C. Enhance sedation D. Decrease secretions

D. Decrease secretions Atropine is a common anesthesia adjunct. It decreases the amount of secretions which, in turn, decreases the risk of aspiration during the operative procedure.

The nurse is caring for a client who has been taking furosemide for the past week. Which manifestation would indicate that the client may be experiencing a negative side effect? A. Edema of the ankles B. Gastric irritability C. Weight gain of five pounds D. Decreased appetite

D. Decreased appetite Furosemide (Lasix) causes a loss of potassium if a supplement is not taken. Findings of hypokalemia include anorexia, fatigue, nausea, decreased gastrointestinal motility, muscle weakness and dysrhythmias.

Captopril is prescribed for a client. Which effect would the nurse anticipate? A. Increased urine output B. Decreased anxiety C. Improved sleep D. Decreased blood pressure

D. Decreased blood pressure Captopril is an angiotensin-converting enzyme (ACE) inhibitor antihypertensive. It does not have diuretic, sedative, or hypnotic properties. Diuretics promote fluid excretion. Sedatives reduce muscle tension and anxiety. Hypnotics promote sleep.

Leucovorin calcium is prescribed and is to be administered immediately after an infusion of methotrexate. Which result of laboratory testing indicates that leucovorin has been effective? A. Potassium level normalizes B. Folic acid level within normal limits C. Improved white blood cell count D. Decreased methotrexate level

D. Decreased methotrexate level The laboratory measurement of the client's methotrexate level is the most objective measure of leucovorin calcium's effectiveness. Leucovorin calcium is considered a 'rescue' medication because it minimizes the effects of methotrexate on healthy cells by competing with methotrexate at the cellular level, thus neutralizing it and causing it to be excreted. Its purpose is not to affect folic acid levels nor to affect potassium or white blood cell counts.

A client is admitted to the hospital for diagnostic testing for possible myasthenia gravis. The nurse prepares for intravenous administration of edrophonium chloride (Tensilon). What is the expected outcome for this client following administration of this pharmacologic agent? A. Progressive difficulty with swallowing. B. Decreased respiratory effort. C. Improvement in generalized fatigue. D. Decreased muscle weakness.

D. Decreased muscle weakness. Administration of edrophonium chloride (Tensilon), a cholinergic agent, will temporarily reduce muscle weakness, the most common complaint of newly-diagnosed clients with myasthenia gravis. This medication is used to diagnose myasthenia gravis due to its short duration of action. This drug would temporarily reverse difficulty in swallowing and respiratory effort.

A health care provider prescribes famotidine for a client with dyspepsia. Which statement is important to include in a teaching session about famotidine? A. Lowers the stress level B. Neutralizes gastric acidity C. Reduces gastrointestinal peristalsis D. Decreases secretions in the stomach

D. Decreases secretions in the stomach Famotidine inhibits histamine at H 2-receptor sites in the stomach, inhibiting gastric acid secretion. Famotidine does not affect stress levels. Famotidine inhibits, rather than neutralizes, gastric secretion. Famotidine inhibits gastric secretion, not peristalsis.

The nurse is monitoring the client who is taking newly prescribed antihypertensive medication. Which finding should indicate to the nurse that the client might be experiencing an allergic reaction to the medication? A. Mild decrease in blood pressure B. Increased urine output C. Left-sided weakness D. Development of a rash

D. Development of a rash Rationale: Allergic reactions are often manifested by the presence of a rash, urticaria, gastrointestinal symptoms, and itching. A mild decrease in blood pressure is the intended effect of the medication. Increased urinary output and unilateral weakness are not indications of an allergic reaction.

A client has been given a prescription for furosemide 40 mg every day in conjunction with digoxin. Which concern would prompt the nurse to ask the health care provider about potassium supplements? A. Digoxin causes significant potassium depletion. B. The liver destroys potassium as digoxin is detoxified. C. Lasix requires adequate serum potassium to promote diuresis. D. Digoxin toxicity occurs rapidly in the presence of hypokalemia.

D. Digoxin toxicity occurs rapidly in the presence of hypokalemia. Furosemide promotes potassium excretion, and low potassium (hypokalemia) increases cardiac excitability. Digoxin is more likely to cause dysrhythmias when potassium is low. Digoxin does not affect potassium excretion. Furosemide causes potassium excretion. Potassium is excreted by the kidneys, not destroyed by the liver. Furosemide causes diuresis and consequent potassium loss regardless of the serum potassium level.

A nurse is assessing a client receiving intravenous potassium chloride. The client verbalizes pain to the IV site. The site appears swollen and is warm to touch. Which action does the nurse perform? A. Decrease the rate of the infusion B. Apply ice to the IV access site C. Inform the client that this is an expected finding D. Discontinue the IV catheter

D. Discontinue the IV catheter Rationale: The nurse should discontinue the IV catheter. The client's symptoms are indicative of phlebitis, inflammation of the vein. Decreasing the rate of the infusion will not treat the swelling or injury to the vein. Applying ice to the access site does not address the possible vein injury caused by the medication. Pain, swelling, and warmth are not expected findings for a patent IV access site.

A client receiving doxorubicin (Adriamycin) intravenously (IV) complains of pain at the insertion site, and the nurse notes edema at the site. Which intervention is most important for the nurse to implement? A. Assess for erythema. B. Administer the antidote. C. Apply warm compresses. D. Discontinue the IV fluids.

D. Discontinue the IV fluids. Doxorubicin is an antineoplastic agent that causes inflammation, blistering, and necrosis of tissue upon extravasation. First, all IV fluids should be discontinued at the site to prevent further tissue damage by the vesicant.

The nurse is assessing a postpartum client who is taking labetalol. Which client report should the nurse identify as a potential adverse effect of the medication? A. Nausea B. Ankle edema C. Abdominal pain D. Dizziness

D. Dizziness Rationale: Labetalol is a beta-blocker that is used for blood pressure management in postpartum clients. The mechanism of action for labetalol is to vasodilate, which could lead to a decrease in blood pressure. A client with a sudden drop in blood pressure could report dizziness. Report of nausea or ankle edema is normal during pregnancy. Abdominal pain in pregnancy could be from active labor or constipation.

A health care provider has prescribed isoniazid for a client. Which instruction will the nurse give the client about this medication? A. Prolonged use can cause dark, concentrated urine. B. The medication is best absorbed when taken on an empty stomach. C. Take the medication with aluminum hydroxide to minimize gastrointestinal (GI) upset. D. Drinking alcohol daily can cause medication-induced hepatitis.

D. Drinking alcohol daily can cause medication-induced hepatitis. Alcohol may increase hepatotoxicity of the medication; instruct client to avoid drinking alcohol during treatment; monitor for signs of hepatitis before and while taking medication. Prolonged use does not cause dark, concentrated urine. The client should take isoniazid with meals to decrease GI upset. Clients should avoid taking aluminum antacids at the same time as this medication because it impairs absorption.

Which intervention would the nurse include in the plan of care for a client receiving antibiotics and antifungal medication for treatment of a vaginal infection? A. Avoid spicy foods. B. Drink more fruit juices. C. Take a multivitamin every day. D. Eat yogurt with active cultures daily.

D. Eat yogurt with active cultures daily. Yogurt contains Lactobacillus acidophilus, which replaces the intestinal flora destroyed by antibiotics. Spicy foods, fruit juices, and multivitamins are not relevant to antibiotics or intestinal flora.

After receiving streptomycin sulfate for 2 weeks as part of the medical regimen for tuberculosis, the client reports feeling dizzy and having some hearing loss. Which part of the body is the medication affecting? A. Pyramidal tracts B. Cerebellar tissue C. Peripheral motor end plates D. Eighth cranial nerve's vestibular branch

D. Eighth cranial nerve's vestibular branch Streptomycin sulfate is ototoxic and may cause damage to auditory and vestibular portions of the eighth cranial nerve. Pyramidal tracts, cerebellar tissue, and peripheral motor end plates are not affected by streptomycin.

Which instructions will the nurse give a client for whom nitroglycerin tablets are prescribed? A. Limit the number of tablets to four per day. B. Discontinue the medication if a headache develops. C. Increase the number of tablets if dizziness is experienced. D. Ensure that the medication is stored in its original dark container.

D. Ensure that the medication is stored in its original dark container. Nitroglycerin is sensitive to light and moisture, so it must be stored in a dark, airtight container. Limit the number of tablets to four per day, taken as needed. If more than three tablets are necessary in a 15-minute period, emergency medical attention should be received. A headache may be an expected side effect, and the medication should not be discontinued. Dizziness indicates the dosage may need to be decreased by the health care provider.

A nurse is preparing to administer prescribed maintenance dose of digoxin to a client who has heart failure. Which action should the nurse to take? A. Withhold the medication if the heart rate is above 100/min B. Instruct the client to eat foods that are low in potassium C. Measure apical pulse rate for 30 seconds before administration D. Evaluate the client for nausea, vomiting, and anorexia

D. Evaluate the client for nausea, vomiting, and anorexia Rationale: A client with heart failure who is prescribed digoxin should be assessed for digoxin toxicity. Manifestations of digoxin toxicity include nausea, vomiting, and anorexia. Digoxin is used to decrease heart rate and should be held if the heart rate is less than 60 beats per minute. Digoxin toxicity can occur when the client has low potassium. When administering digoxin, the nurse should measure the client's apical pulse for a full minute.

Which action would the nurse perform when beginning pyridostigmine bromide therapy for a client with myasthenia gravis? A. Administer the medication after meals. B. Administer the medication on an empty stomach. C. Evaluate the client's psychological responses between medication doses. D. Evaluate the client's muscle strength every hour after the medication is given.

D. Evaluate the client's muscle strength every hour after the medication is given. The onset of action of pyridostigmine is 30 to 45 minutes after administration, and the effects last up to 6 hours; the client's response will influence dosage levels. Pyridostigmine usually is administered before meals to promote mastication. Pyridostigmine should be administered with food to prevent gastric irritation. There are no psychological side effects associated with pyridostigmine.

Which alteration is the likely cause of thyrotoxic crisis (thyroid storm) in a client who has had treatment with propylthiouracil for hyperthyroidism followed by thyroid ablation with 131I? A. Deficiency of iodine B. Decreased serum calcium C. Increased sodium retention D. Excessive hormone replacement

D. Excessive hormone replacement Thyrotoxic crisis (thyroid storm) is the body's response to excessive circulating thyroid hormones. A deficiency of iodine results in a deficiency in thyroid hormone production. A decreased serum calcium causes tetany. Sodium retention is unrelated to thyrotoxic crisis; thyrotoxic crisis is caused by excessive circulating thyroid hormones.

The nurse prepares to administer extended-release metformin to an older adult who has asked that it be crushed because it is difficult to swallow. Which rationale will prompt the nurse to ask the provider for a different form of metformin? A. This medication has a wax matrix frame that is difficult to crush. B. The medication has an unpleasant taste, which most clients find intolerable if crushed. C. If crushed, this medication irritates mucosal tissue and can cause oral and esophageal ulcer formation. D. Extended-release formulations are designed to be released slowly and crushing the tablet will prevent this from occurring.

D. Extended-release formulations are designed to be released slowly and crushing the tablet will prevent this from occurring. The slow-release formulary will be compromised, and the client will not receive the entire dose if it is chewed or crushed. The capsules are not difficult to crush. Irritation of the mucosal tissue is not the reason the medication should not be crushed; however, this medication should be given with meals to prevent gastrointestinal irritation. Although taste could be a factor, it is not the priority issue.

client who has been diagnosed with Raynaud's disease and hypertension is prescribed nifedipine. For which side effect should the nurse monitor the client? A. Cyanosis of the lips B. Decreased urine output C. Increased pain in fingers D. Facial flushing

D. Facial flushing Nifedipine is a calcium channel blocker (CCB) used in the treatment of Raynaud's disease and hypertension by producing vasodilation. As a result of this vasodilating effect, facial flushing can occur. Cyanosis of the lips and decreased urinary output are not expected findings with nifedipine. Raynaud's disease causes vasoconstriction, resulting in pain in the fingers that should decrease when nifedipine is taken.

Which condition is treated with a proton pump inhibitor (PPI)? A. Diarrhea B. Vomiting C. Cardiac dysrhythmias D. Gastroesophageal reflux disease (GERD)

D. Gastroesophageal reflux disease (GERD) PPIs are effective in decreasing the secretion of gastric acid, helping alleviate the symptoms of GERD. PPIs are not used for the treatment of diarrhea, vomiting, or cardiac dysrhythmias.

After the nurse provides education about hydrochlorothiazide, the client will agree to notify the health care provider regarding the development of which symptom? A. Insomnia B. Nasal congestion C. Increased thirst D. Generalized weakness

D. Generalized weakness Generalized weakness is a symptom of significant hypokalemia, which may be a sequela of diuretic therapy. Insomnia is not known to be related to hypokalemia or hydrochlorothiazide therapy. Although a stuffy nose is unrelated to hydrochlorothiazide therapy, it can occur with other antihypertensive medications. Increased thirst is associated with hypernatremia. Because this medication increases excretion of water and sodium in addition to potassium and chloride, hyponatremia, not hypernatremia, may occur.

A 2 year-old child is being treated with amoxicillin suspension, 200 milligrams per dose, for acute otitis media. The child weighs 33 lb (15 kg) and the daily dose range is 20 to 40 mg/kg of body weight, in three divided doses every eight hours. Using principles of safe drug administration, what should a nurse do next? A. Recognize that antibiotics are over-prescribed B. Call the health care provider to clarify the dose C. Hold the medication because the dosage is too low D. Give the medication

D. Give the medication as ordered Amoxicillin continues to be the drug of choice in the treatment of acute otitis media. The dose range is 20 to 40 mg/kg/day divided every eight hours; 15 kg x 40 mg = 600 mg, divided by 3 = 200 mg per dose. The prescribed dose is correct and should be given as ordered.

A nurse is reviewing prescribed medications for a client diagnosed with diabetic ketoacidosis. Which medication will the nurse clarify with the healthcare provider? A. Regular insulin Your Answer B. Potassium C. 0.9% sodium chloride D. Glipizide

D. Glipizide Rationale: Glipizide is an oral antidiabetic medication used in the treatment of type 2 diabetes mellitus. The intended effect of glipizide is to lower glucose levels and maintain adequate management of the disease. Oral antidiabetic agents are contraindicated in clients with diabetic ketoacidosis (DKA). Glucose levels must be carefully lowered and monitored following insulin therapy. Regular insulin, potassium, and 0.9% sodium.

A health care provider prescribes milrinone for a client with congestive heart failure. Which action would the nurse perform first? A. Administer the loading dose over 10 minutes. B. Monitor the electrocardiogram (ECG) continuously for dysrhythmias during infusion. C. Assess the heart rate and blood pressure continuously during infusion. D. Have the prescription, dosage calculations, and pump settings checked by a second nurse.

D. Have the prescription, dosage calculations, and pump settings checked by a second nurse. Accidental overdose can cause death. Another nurse would verify accuracy of the prescription, dose, and pump settings to prevent harm to the client. Although administering the loading dose over 10 minutes is an appropriate intervention, it is not the first thing the nurse would do. Although monitoring for dysrhythmias is important because they are common with this medication and may be life threatening, it is not the first thing the nurse would do. Although taking the vital signs continuously during the infusion is important because the dose needs be slowed or discontinued if the blood pressure decreases excessively, it is not the first thing the nurse would do.

Which nursing action is appropriate when administering imipramine? A. Telling the client steroids will not be prescribed B. Warning the client not to eat cheese C. Monitoring the client for increased tolerance D. Having the client checked for increased intraocular pressure

D. Having the client checked for increased intraocular pressure Glaucoma is one of the side effects of imipramine, and the client should be taught about the symptoms. The prescribing of steroids and avoiding cheese are true of monoamine oxidase inhibitors (MAOIs); imipramine is not an MAOI. Tolerance is not an issue with tricyclic antidepressants such as imipramine.

A client is prescribed 1 mcg/kg/min of dobutamine hydrochloride via IV infusion. Which client's condition would benefit the most from an administration of dobutamine hydrochloride? A. Shock. B. Asthma. C. Hypotension. D. Heart failure.

D. Heart failure. Dobutamine is a beta-1 adrenergic agonist that is indicated for short term use in cardiac decompensation or heart failure related to reduced cardiac contractility due to organic heart disease or cardiac surgical procedures.

The nurse is assessing a client who began taking omeprazole a month ago. Which finding by the nurse, indicates that the drug has had the desired effect? A. Blood pressure readings are lower B. Feelings of depression are not as severe C. Chronic pain level is markedly decreased D. Heartburn discomfort is lessened

D. Heartburn discomfort is lessened Omeprazole is a proton pump inhibitor used to decrease stomach acid and relieve symptoms of gastroesophageal reflux disorder (GERD), such as heartburn. Omeprazole is also used to treat gastric ulcers and esophagitis. Omeprazole does not affect blood administration of medication. Omeprazole is not indicated for depression. Although omeprazole can alleviate abdominal pain in an individual who has a gastric ulcer or suffers from gastric bleeding, the option does not specify what type of pain is being discussed. Secondly, omeprazole is not typically indicated for chronic pain. The desired outcome for this client is to have a decrease in symptoms of GERD within 4 weeks.

Which health history would the nurse consider a contraindication to administering the second diphtheria/tetanus/pertussis (DTaP) immunization to a 4-month-old infant? A. Allergy to eggs B. Lactose intolerance C. Infectious dermatitis D. High fever after the first dose

D. High fever after the first dose A temperature of 105°F (40.6°C) or higher after a diphtheria/tetanus/pertussis (DTaP) immunization is a contraindication to further DTaP immunizations. An allergy to eggs is not a contraindication to the administration of the DTaP vaccine because eggs are not used in the production of the vaccine. Lactose intolerance is not a contraindication to the administration of DTaP vaccine, nor is infectious dermatitis.

The nurse receives an order to administer intravenous gentamicin to a client. For which finding should the nurse contact the health care provider to clarify the order? A. Low serum albumin B. Low serum blood urea nitrogen C. High gastric pH D. High serum creatinine

D. High serum creatinine Gentamicin is an aminoglycoside antibiotic that is excreted primarily by the kidneys. If there is reduced renal function as evidenced by the elevated serum creatinine level, the client is at greater risk for drug toxicity and further renal damage.

The nurse is monitoring a client who received a first dose of intravenous ampicillin. Which finding should indicate to the nurse that the client may be experiencing an allergic reaction? A. Abdominal pain B. Increase in blood pressure C. Hypotensive bowel sounds D. Hives on the extremities

D. Hives on the extremities Rationale: If the client experiences an allergic reaction to medications they may display systemic signs such as hives, pruritus, dyspnea, etc. Abdominal pain, hypertension, and hyperactive bowel sounds do not indicate an allergic reaction.

A client who receives NPH insulin every morning reports feeling nervous at 4:30 PM. The nurse observes that the client's skin is moist and cool. Which condition is the client likely experiencing? A. Hyperosmolar hyperglycemic nonketotic state B. Ketoacidosis C. Glycogenesis D. Hypoglycemia

D. Hypoglycemia The time of the client's response corresponds to the expected peak action (4-12 hours after administration) of the intermediate-acting insulin that was administered in the morning, which can result in hypoglycemia. Hypoglycemia triggers the sympathetic nervous system; epinephrine causes diaphoresis and nervousness. Hyperosmolar hyperglycemic nonketotic state is a severe hyperglycemia state that occurs in clients with type 2 diabetes secondary to severe illness or stress. Warm, dry, flushed skin and lethargy are associated with hyperglycemia and ketoacidosis. Glycogenesis, the formation of glycogen in the liver, is unrelated to nervousness and cool, moist skin.

A client with type 1 diabetes receives 30 units of neutral protamine Hagedorn (NPH) insulin at 7:00 AM. At 3:30 PM, the client becomes diaphoretic, weak, and pale. With which condition would the nurse determine that these physiological responses are associated? A. Diabetic coma B. Hyperosmolar hyperglycemic nonketotic syndrome C. Diabetic ketoacidosis D. Hypoglycemic reaction

D. Hypoglycemic reaction These are sympathetic nervous system responses to hypoglycemia; the peak action of NPH insulin is 8 to 12 hours after administration, and 8.5 hours have elapsed since it was given. The signs and symptoms of diabetic coma are dry mucous membranes; hot, flushed skin; deep, rapid respirations (Kussmaul breathing); acetone odor to the breath; nausea and vomiting; and, as with hypoglycemia, weakness. Hyperosmolar hyperglycemic nonketotic syndrome (HHNS) is a hyperglycemic state and this client has symptoms of a hypoglycemic state. Ketoacidosis results from excess use of fats for energy when carbohydrates cannot be used. Lipids are metabolized incompletely, and dehydration, acidosis (both ketotic and lactic), and electrolyte imbalance result. It is not the result of insulin administration.

The health care provider prescribes atenolol for a client with angina. Which potential side effect will the nurse mention when instructing the client about this medication? A. Headache B. Tachycardia C. Constipation D. Hypotension

D. Hypotension Atenolol competitively blocks stimulation of beta-adrenergic receptors within vascular smooth muscles, which lowers the blood pressure. This medication does not cause headaches; this medication may be used to relieve vascular headaches. This medication may cause bradycardia, not tachycardia. This medication may cause diarrhea, not constipation.

A client with a myocardial infarction receives intravenous nitroglycerin to relieve pain. The nurse will assess for which medication side effect? A. Nausea B. Delirium C. Bradycardia D. Hypotension

D. Hypotension The major action of intravenous nitroglycerin is venous and then arterial dilation, leading to a decrease in blood pressure and resulting in decreased cardiac workload. Nausea is not a common side effect of intravenous nitroglycerin. Nitroglycerin does not cause delirium. Reflex tachycardia may occur with the decrease in blood pressure.

A 6-year-old child is receiving an intravenous solution of 10% glucose and mannitol to reduce cerebral edema. Which complication would the nurse monitor the child for? A. Overhydration B. Seizure activity C. Acute heart failure D. Hypovolemic shock

D. Hypovolemic shock Both hypertonic glucose and mannitol cause diuresis; the child should be monitored for excessive fluid loss. Hypertonic glucose and mannitol will cause fluid loss, not gain. Seizure activity is not anticipated as a result of this infusion. An increased fluid volume can lead to heart failure; however, hypertonic glucose and mannitol cause fluid loss, not gain.

A client who had an organ transplant is receiving cyclosporine. The nurse would monitor for which serious adverse effect of cyclosporine? A. Hirsutism B. Constipation C. Dysrhythmias D. Increased creatinine level

D. Increased creatinine level A life-threatening effect of cyclosporine is nephrotoxicity. Creatinine and blood urea nitrogen levels should be monitored. Although abnormal hairiness (hirsutism) is an effect of cyclosporine, it is not life threatening. Diarrhea, not constipation, is a response to cyclosporine. Cyclosporine does not cause life-threatening cardiovascular effects.

How does sodium biphosphate, prescribed for a client before a colonoscopy, accomplish its therapeutic effect? A. Irritates the intestinal mucosa B. Provides water-absorbing bulk C. Softens stool by exerting a detergent effect D. Increases osmotic pressure in the intestines

D. Increases osmotic pressure in the intestines Sodium biphosphate is a saline (hypertonic) cathartic that increases osmotic pressure within the intestine so that body fluids are drawn into the bowel, stimulating bowel stretching, peristalsis, and defecation. Intestinal stimulants increase peristalsis by irritating the mucosa. Bulk-forming laxatives are cellulose derivatives that remain in the intestinal tract and absorb water; they stimulate peristalsis by increasing bulk. Emollients have a detergent action, softening stool by facilitating its absorption of water.

health care provider prescribes simvastatin 20 mg daily for elevated cholesterol and triglyceride levels for a female client. Which advice is important for the nurse to teach when the client initially takes the medication? A. Take the medication with breakfast. B. Have liver function tests every 6 months. C. Wear sunscreen to prevent photosensitivity reactions. D. Inform the health care provider if you wish to become pregnant.

D. Inform the health care provider if you wish to become pregnant. Simvastatin is a teratogen that is contraindicated in pregnancy because it is capable of causing fetal damage. Simvastatin should be taken in the evening because most cholesterol is synthesized between midnight and 3:00 AM. Liver function tests should be done at 6 to 12 weeks initially and only then every 6 months. Although wearing sunscreen should be taught, sensitivity reactions are a rare occurrence, and this is not as important.

Which mechanism is specifically responsible for the action of the medication ranitidine? A. Inhibiting proton pumps B. Promoting the release of gastrin C. Regenerating the gastric mucosa D. Inhibiting the histamine at H 2 receptors

D. Inhibiting the histamine at H 2 receptors Ranitidine inhibits histamine at H 2 receptor sites in parietal cells, which limits gastric secretion. It does not inhibit proton pumps. Promoting the release of gastrin is undesirable; gastric hormones increase gastric acid secretion. Ranitidine does not regenerate the gastric mucosa; the medication prevents its erosion by gastric secretions.

Famotidine is prescribed for a client with peptic ulcer disease. Which mechanism of action is a characteristic of this medication? A. Increases gastric motility B. Neutralizes gastric acidity C. Facilitates histamine release D. Inhibits gastric acid secretion

D. Inhibits gastric acid secretion Famotidine decreases gastric secretion by inhibiting histamine at H 2 receptors. Increasing gastric motility, neutralizing gastric acidity, and facilitating histamine release are not actions of famotidine.

A nurse receives a prescription to administer intravenous cefepime to a client with a bacterial infection. The client has a history of lung cancer and is on a continuous cisplatin infusion. How will the nurse administer the prescribed medication? A. Piggyback the cefepime onto the cisplatin infusion B. Wait for the cisplatin infusion to finish before administering cefepime C. Infuse the cefepime via IV push at the proximal port D. Initiate a new intravenous line for the cefepime infusion

D. Initiate a new intravenous line for the cefepime infusion Rationale: Cefepime is an antibiotic medication used to treat bacterial infections. Cisplatin is an antineoplastic medication used in the treatment of various cancers. Cefepime and cisplatin are not compatible and should not be mixed. The nurse should initiate a new intravenous line for the administration of cefepime. Piggybacking the cefepime will cause the medication to mix with cisplatin. The medications are not compatible. A continuous cisplatin infusion is administered over 24 hours to 5 days. The nurse should not wait to administer other medications. Cefepime should be administered as an infusion, not an IV push.

The oncology nurse is caring for a female client who is being treated for metastatic breast cancer. The client is scheduled to receive their first dose of trastuzumab. Which assessment finding is most important to notify the health care provider of? A. Absolute neutrophil count 2.5 (2,500 mm3) B. Intermittent nausea and vomiting C. Blood glucose 130 mg/dL D. Irregular apical pulse

D. Irregular apical pulse Trastuzumab is a monoclonal antibody used as anticancer therapy for women with HER2-positive breast cancer. The main concern in administering trastuzumab is cardiotoxicity, manifesting as ventricular dysfunction and congestive heart failure. Therefore, the irregular apical pulse is the most important assessment findings. An ejection fraction is obtained as a baseline before treatment and may be monitored every few months while the client is receiving this medication. The other findings are to be expected, normal or near normal and not as important as the irregular apical pulse.

A client being treated for hyperthyroidism with propylthiouracil (PTU) asks the nurse how the medication works. Which is the best response to give the client? A. It decreases the amount of thyroid-stimulating hormone circulating in the blood. B. It increases the amount of thyroid-stimulating hormone circulating in the blood. C. It enhances the amount of T4 and diminishes the amount of T3 produced by the thyroid. D. It inhibits the synthesis of T3 and T4 by the thyroid gland.

D. It inhibits the synthesis of T3 and T4 by the thyroid gland. Propylthiouracil (PTU) is an adjunct therapy used to control hyperthyroidism by inhibiting the production of thyroid hormones. It is often prescribed in preparation for thyroidectomy or radioactive iodine therapy.

) The nurse plans to teach a fifth-grader with type 1 diabetes how to self-administer lispro and glargine insulin. Which action would the nurse include in the teaching plan? A. Alternate the sites until the best one to use is found. B. Self-administer the injections after being taught the technique. C. Draw up the insulin glargine and then draw up the insulin lispro. D. Learn to use the needle and syringe by practicing on an insulin pad first.

D. Learn to use the needle and syringe by practicing on an insulin pad first. Practice using a syringe builds confidence. The child's confidence, readiness, and skill for giving self-injections are essential for long-term management of diabetes. Injection sites must be rotated. Learning responsibility for injections should be a gradual process with continual support and guidance. Insulin glargine should not be mixed with other insulins; it should be prepared and administered separately.

The nurse identifies a nontender 5-cm indurated region on the upper arm of a client with type 1 diabetes. The client says to the nurse, 'That is where I give myself insulin shots.' The nurse concludes that the nodule is a result of which condition? A. Callus B. An allergy C. An infection D. Lipodystrophy

D. Lipodystrophy Lipodystrophy is a noninflammatory reaction causing localized atrophy or hypertrophy and a localized increase in collagen deposits. Injections of insulin will not cause a horny growth such as a wart or callus. An allergic response will precipitate a localized or systemic inflammatory response. Hyperthermia and localized heat, erythema, and pain are associated with an infection.

When caring for a client on isoniazid therapy for tuberculosis, the nurse would focus on which diagnostic testing for this client? A. Creatinine B. Hearing tests C. Electrocardiogram D. Liver function tests

D. Liver function tests Isoniazid can damage the liver enough to lead to death, so liver function should be monitored. Creatinine would be tracked for renal dysfunction, which is not a focus of isoniazid therapy because isoniazid is metabolized by the liver. Aminoglycosides can cause ototoxicity, causing hearing loss. Bedaquiline can cause prolonged QT, detected through electrocardiogram.

Which education would the nurse provide parents about the side effects of the Haemophilus influenzae (Hib) vaccine? A. Lethargy B. Urticaria C. Generalized rash D. Low-grade fever

D. Low-grade fever The Hib vaccine may cause a low-grade fever. Lethargy is not expected. Urticaria is more likely to occur with the tetanus and pertussis vaccines. There may be a mild reaction at the injection site, but a generalized rash is not expected.

The nurse is reviewing a plan of care for a client who was admitted with dehydration as a result of prolonged watery diarrhea. Which new prescription will the nurse question? A. Oral psyllium B. Oral potassium supplement C. Intravenous normal saline D. Magnesium citrate

D. Magnesium citrate Oral psyllium will absorb the watery diarrhea, giving more bulk to the stool. An oral potassium supplement is appropriate because diarrhea causes potassium loss. Intravenous normal saline is the intervention of choice to manage dehydration due to diarrhea. Magnesium citrate has a laxative effect and would worsen the client's dehydration.

The nurse is caring for a child diagnosed with seizures. While teaching the family and the child about the medication phenytoin, which concept should the nurse emphasize? A. Omit the medication if the child is seizure-free B. Serve a diet that is high in iron C. A rash is normal with this medication D. Maintain good oral hygiene and dental care

D. Maintain good oral hygiene and dental care Gingival hyperplasia may occur with this medication. It is important that good oral hygiene is maintained. The medication should never be stopped, even if the child is seizure-free. A sudden discontinuation could result in status epilepticus. A diet high in iron interferes with phenytoin absorption and will reduce the effectiveness. A blister-like rash is not normal with this medication and could indicate medication-related Stevens-Johnson syndrome, which is a serious disorder of the skin and mucous membranes.

The nurse is caring for a pregnant client who has contracted a trichomonal protozoan infection. For which oral medication would the nurse anticipate preparing to provide education? A. Penicillin G B. Acyclovir C. Nystatin D. Metronidazole

D. Metronidazole Metronidazole is a potent amebicide that is safe in pregnancy. It is effective in eradicating the protozoan Trichomonas vaginalis. Penicillin is administered for its effect on bacterial, not protozoal, infections. Acyclovir is an antiviral medication; therefore, it would not be effective in treating protozoal infections such as trichomonas. Nystatin is an antifungal for infections caused by Candida albicans.

Which potential side effect of docusate sodium would a nurse include in discharge teaching of a client who had repair of an inguinal hernia? A. Rectal bleeding B. Fecal impaction C. Nausea and vomiting D. Mild abdominal cramping

D. Mild abdominal cramping Mild abdominal cramping is the only side effect of docusate sodium; this emollient laxative permits water and fatty substances to penetrate and mix with fecal material. Rectal bleeding is more likely to occur with a saline-osmotic laxative. Docusate sodium promotes defecation, not constipation. Nausea and vomiting are more likely to occur with a saline-osmotic laxative.

Which primary reason identifies why oxazepam is given during detoxification? A. Prevents injury when seizures occur B. Enables the client to sleep better during periods of agitation C. Encourages the client to accept treatment for alcoholism D. Minimizes withdrawal symptoms the client may experience

D. Minimizes withdrawal symptoms the client may experience Oxazepam potentiates the actions of gamma-aminobutyric acid, especially in the limbic system and reticular formation, and thus it minimizes withdrawal symptoms. This medication helps reduce the risk for seizures but does not prevent injury during a seizure. Enabling the client to sleep better during periods of agitation is not the purpose of the medication. The ability of the client to accept treatment depends on the person's readiness to accept the reality of the problem.

The nurse considers that the safe administration of high-dose methotrexate therapy would include which intervention? A. Maintaining an acidic urine B. Restricting intravenous fluids C. Providing a diet high in folic acid D. Monitoring plasma levels of the medication

D. Monitoring plasma levels of the medication Plasma levels indicate whether therapeutic or toxic levels are present. Methotrexate crystallizes in the kidneys if urine becomes acidic. The regimen would include hydration with a minimum of intravenous fluids of 125 mL/h 6 to 12 hours before and during therapy. The effectiveness of methotrexate, a folic acid antagonist, is minimized by a diet high in folic acid.

A school-age child with a seizure disorder has been on long-term carbamazepine therapy. Which intervention would the nurse incorporate into the plan of care? A. Assessing the mouth for gingivitis B. Checking the pupillary reaction to light C. Keeping an accurate intake and output record D. Monitoring the child's complete blood cell counts

D. Monitoring the child's complete blood cell counts The side effects of carbamazepine include blood dyscrasias (e.g., thrombocytopenia, aplastic anemia, leukopenia, agranulocytosis). A side effect of long-term phenytoin, not carbamazepine, therapy is hyperplasia of the gingiva. Carbamazepine does not influence pupillary response directly. Keeping an accurate intake and output is unnecessary.

The nurse is caring for a client who is prescribed erythromycin 500 mg orally every six hours for the treatment of pneumonia. The nurse should monitor the client for which common side effect? A. Esophagitis B. Tendon rupture C. Orange-red discoloration of urine D. Nausea and vomiting

D. Nausea and vomiting Erythromycin is a macrolide anti-infective medication used that interferes with protein synthesis in susceptible bacteria. Nausea, vomiting and gastrointestinal (GI) upset are common with erythromycin. The other side effects are not commonly seen with this drug.

Which side effect would the nurse assess for in a child receiving vincristine? A. Hemolytic anemia B. Irreversible alopecia C. Hyperglycemia D. Neurological complications

D. Neurological complications Vincristine is highly neurotoxic, causing paresthesias, muscle weakness, ptosis, diplopia, paralytic ileus, vocal cord paralysis, and loss of deep tendon reflexes. Hematologic effects are rare; mild anemia may occur, but hemolytic anemia is not anticipated. Alopecia is reversible with cessation of the medication. Hyperglycemia is not an anticipated adverse effect.

A nurse is reviewing laboratory data of a client taking paclitaxel for ovarian cancer. Which finding would the nurse report to the healthcare provider before administering the next dose of medication? A. Platelet count of 475,000/mm³ B. Eosinophil level of 400/mm³ C. Red blood cell count of 6.5 million/mm³ D. Neutrophil count of 1,200/mm³

D. Neutrophil count of 1,200/mm³ Rationale: Paclitaxel is an antineoplastic medication used in the treatment of various cancers. Paclitaxel causes neutropenia and is contraindicated in clients with a neutrophil count below 1,500/mm³. A platelet count of 475,000/mm³ is above normal. Paclitaxel can cause thrombocytopenia (low platelet count). Eosinophils are white blood cells that fight infectious organisms. An eosinophil level of 400/mm³ is a normal finding. A red blood cell (RBC) count of 6.5 million/mm³ is above normal. Paclitaxel can cause anemia (low RBCs).

A client taking oral contraceptives for 3 months tells the nurse she has breakthrough bleeding between menstrual cycles. For which causative factor would the nurse first assess in the client? A. Illness B. Anorexia nervosa C. Ectopic pregnancy D. Nonadherence to protocol

D. Nonadherence to protocol Nonadherence to the instructions for taking the oral contraceptive can alter hormone levels, and breakthrough bleeding may occur as a result. Illness and anorexia nervosa are more likely to cause amenorrhea, not breakthrough bleeding. If nonadherence is determined not to be a concern, then the nurse would assess for far less likely causes such as ectopic pregnancy.

A client has a continuous IV infusion of dopamine (Intropin) and an IV of normal saline at 50 mL/hour. The nurse notes that the client's urinary output has been 20 mL/hour for the last 2 hours. Which intervention should the nurse initiate? A. Stop the infusion of dopamine. B. Change the normal saline to a keep open rate. C. Replace the urinary catheter. D. Notify the healthcare provider of the urinary output.

D. Notify the healthcare provider of the urinary output. The main effect of dopamine is adrenergic stimulation used to increase cardiac output, which should also result in increased urinary output. A urinary output of less than 20 mL/hour is oliguria and should be reported to the healthcare provider, so the dose of dopamine can be adjusted. Depending on the current rate of administration, the dose may need to be increased or decreased.

The client is newly diagnosed with type 1 diabetes mellitus. Which of these approaches would be the best strategy for the nurse to use when teaching insulin injection techniques? A. Give written pre and post tests B. Allow another diabetic to assist C. Ask questions during practice D. Observe a return demonstration

D. Observe a return demonstration Learning to inject oneself is a challenging task and the nurse should first demonstrate the injection and then ask for a return demonstration from the client. Giving a written test is not appropriate for this teaching. Asking questions during practice is important, but the nurse still needs to see the client self-inject. Asking another diabetic to assist is not appropriate.

A health care provider prescribes teriparatide for a client with osteoporosis. Which statement about this medication would the nurse recognize as accurate? A. It requires increased intake of vitamin A. B. It prevents existing bone from being destroyed. C. Sunscreen should be used to prevent vitamin D absorption. D. Osteoblastic activity is stimulated more than osteoclastic activity.

D. Osteoblastic activity is stimulated more than osteoclastic activity. Teriparatide is a 34-amino acid polypeptide that represents the biologically active part of human parathyroid hormone; it enhances bone microarchitecture and increases bone mass and strength by stimulating activity by osteoblasts. Supplemental intake of vitamin A should not exceed recommended daily requirements; too much vitamin A has been associated with bone loss and an increased incidence of fractures. Alendronate sodium, a regulator of bone metabolism, not teriparatide, inhibits osteoclast-mediated bone resorption, minimizing bone destruction and loss of bone density. Sunscreen should be avoided to promote exposure to the sun so that vitamin D can be converted in the skin; vitamin D helps the body absorb calcium. Sunscreen should be used after 5 to 20 minutes of exposure to prevent the negative effects of prolonged exposure to ultraviolet rays.

The nurse understands that megestrol is most often used to treat which issue? A. Menopause B. Birth control C. Premenstrual syndrome (PMS) D. Palliative management of cancer

D. Palliative management of cancer Megestrol is most often used for the palliative management of recurrent, inoperable, or metastatic endometrial or breast cancer because it can cause appetite stimulation and weight gain. Megestrol is not typically used in the treatment of menopause symptoms, as birth control, or for PMS.

A child with pinworms is prescribed mebendazole. Which expected response to the medication would the nurse teach the parents watch for? A. Blood B. Constipation C. Yellow stools D. Passage of worms

D. Passage of worms Passage of worms is the expected response because the medication causes the death of the worms. Neither the medication nor the worms cause intestinal bleeding. Transient diarrhea, not constipation, may occur. The medication may color the stool red, not yellow.

A client with leukemia who is receiving vincristine reports lower leg numbness. Which statement about vincristine explains this occurrence? A. Vincristine acts on enlarged lymph nodes in the groin. B. Vincristine affects peripheral vascular circulation. C. Vincristine increases the risk for vascular occlusion. D. Peripheral neuropathies can result from vincristine chemotherapy.

D. Peripheral neuropathies can result from vincristine chemotherapy. Muscle weakness, tingling, and numbness are related to medications like vincristine; neuropathies usually are transient if the medication is stopped or reduced. Nodal enlargement produces vascular rather than neural side effects. Most chemotherapeutic regimens do affect the nervous or peripheral vascular system; neuropathies and peripheral vascular adaptations are potential side effects of chemotherapy. Tingling and numbness are characteristic of neuropathy, not vascular occlusion.

A school-age child with leukemia is receiving treatment with vincristine. Which toxic response would the nurse assess the child for? A. Diarrhea B. Alopecia C. Hemorrhagic cystitis D. Peripheral neuropathy

D. Peripheral neuropathy Neurotoxicity is a specific response to vincristine; the child may become numb and ataxic. Vincristine causes adynamic ileus, resulting in constipation; diarrhea occurs with other antineoplastics and radiation therapy. Alopecia is an expected side effect rather than a toxic response; it is not considered serious, and hair will regrow after the treatment is completed. Hemorrhagic cystitis is a toxic response to cyclophosphamide, not vincristine.

Which effect has resulted in the avoidance of tetracycline use in children under 8 years old? A. Birth defects B. Allergic responses C. Severe nausea and vomiting D. Permanent tooth discoloration

D. Permanent tooth discoloration Tetracycline use in children under the age of 8 years has been discontinued because it causes permanent tooth discoloration. Birth defects, allergic responses, and severe nausea and vomiting are not prevalent reasons for the discontinuation of tetracycline medications in children under 8 years old.

The nurse administers sodium polystyrene sulfonate to a client with chronic renal failure. Which finding provides evidence that the intervention is effective? A. Pruritus decreases. B. Mental status improves. C. Sodium decreases to 137 mEq/L (137 mmol/L). D. Potassium decreases to 4.2 mEq/L (4.2 mmol/L).

D. Potassium decreases to 4.2 mEq/L (4.2 mmol/L). This resin exchanges sodium ions for potassium in the large intestine to lower the serum potassium level; 4.2 mEq/L (4.2 mmol/L) is in the expected range for potassium. Mental status improvement and relief of pruritus are not therapeutic effects of the medication. Sodium retention is an adverse effect; 137 mEq/L (137 mmol/L) is in the expected range for sodium.

Which purpose of insulin would a nurse identify when caring for a client prescribed insulin added to a solution of 10% dextrose in water after an intravenous solution containing potassium inadvertently was infused too rapidly? A. Glucose with insulin increases metabolism, which accelerates potassium excretion. B. Increased potassium causes a temporary slowing of the pancreatic production of insulin. C. Increased insulin accelerates the excretion of glucose and potassium, thereby decreasing the serum potassium level. D. Potassium follows glucose into the cells of the body, thereby raising the intracellular potassium level.

D. Potassium follows glucose into the cells of the body, thereby raising the intracellular potassium level. Potassium follows glucose into the cells of the body, thereby raising the intracellular potassium level and preventing fatal dysrhythmias. Potassium is not excreted as a result of this therapy; it shifts into the intracellular compartment. The potassium level has no effect on pancreatic insulin production. Insulin does not cause the excretion of these substances.

A client is scheduled to have a thyroidectomy. Which medication is indicated for decreasing the size and vascularity of the thyroid gland before surgery? A. Vasopressin B. Levothyroxine C. Propylthiouracil D. Potassium iodide

D. Potassium iodide Potassium iodide adds iodine to the body fluids, exerting negative feedback on the thyroid tissue and decreasing its metabolism and vascularity. Vasopressin is a pituitary hormone. Propylthiouracil interferes with production of thyroid hormone but causes increased vascularity and size of the thyroid. Levothyroxine is a thyroid hormone that may be administered after a thyroidectomy if the client develops hypothyroidism.

Which reason would an intravenous infusion of 5% dextrose with 0.45% sodium chloride and 20 mEq of potassium be prescribed for a client with a nasogastric (NG) tube set to low intermittent suction? A. Prevent constipation B. Prevent dehydration C. Prevent vomiting D. Prevent electrolyte imbalance

D. Prevent electrolyte imbalance When clients do not receive nutrients or fluids by mouth and have a loss of electrolytes through the removal of gastric secretions via an NG tube, electrolyte imbalance is a primary concern. Constipation is usually not a concern in this situation. Although dehydration is a possible effect of an NG tube that removes gastric secretions and fluid, electrolyte balance is still the priority. An NG tube set to low intermittent suction usually relieves nausea and vomiting.

A 25-year-old woman on estrogen therapy has a history of smoking. Which complication would the nurse anticipate in the client? A. Osteoporosis B. Hypermenorrhea C. Endometrial cancer D. Pulmonary embolism

D. Pulmonary embolism Estrogen therapy increases the risk of pulmonary embolism in clients who have a history of smoking because the medication affects blood circulation and hemostasis. Osteoporosis may be caused by reduced bone density observed in postmenopausal woman. Hypermenorrhea (excessive menstrual bleeding) is treated with estrogen therapy. Endometrial cancer is a complication of estrogen therapy seen in postmenopausal woman.

The nurse is caring for a client who is receiving azathioprine, cyclosporine, and prednisone before receiving a kidney transplant. Which medication action would the nurse identify as the purpose of these medications? A. Stimulate leukocytosis B. Provide passive immunity C. Prevent iatrogenic infection D. Reduce antibody production

D. Reduce antibody production These drugs suppress the immune system, decreasing the body's production of antibodies in response to the new organ, which acts as an antigen. These medications decrease the risk of rejection. These medications inhibit leukocytosis. These medications do not provide immunity; they interfere with natural immune responses. Because these medications suppress the immune system, they increase the risk of infection.

A client takes an oral hypoglycemic agent daily. For which condition is an oral hypoglycemic agent indicated? A. Ketosis B. Obesity C. Pancreatitis D. Reduced insulin production

D. Reduced insulin production Oral hypoglycemics may be helpful when some functioning of the beta cells exists, as in type 2 diabetes. Rapid-acting regular insulin is needed to reverse ketoacidosis. Obesity does not offer enough information to determine the status of beta cell function. Oral hypoglycemics are not routinely indicated for the treatment of pancreatitis.

Which reason will the nurse explain is the purpose for neomycin being prescribed to a client with cirrhosis? A. Prevents an infection B. Limits abdominal distention C. Minimizes intestinal edema D. Reduces the blood ammonia level

D. Reduces the blood ammonia level Reducing the blood ammonia level decreases the effect of bacterial activity on blood and wastes in the gastrointestinal tract. Although neomycin is an aminoglycoside antimicrobial, it is not administered to prevent infection. Neomycin does not reduce abdominal distention. Neomycin has little or no effect on intestinal edema.

Sodium nitroprusside is prescribed for a client with a blood pressure of 260/120 mm Hg. The nurse recalls that sodium nitroprusside decreases blood pressure by which mechanism? A. Decreasing the heart rate B. Increasing cardiac output C. Increasing peripheral resistance D. Relaxing venous and arterial smooth muscles

D. Relaxing venous and arterial smooth muscles This medication decreases blood pressure by relaxing venous and arteriolar smooth muscles and is used for immediate reduction of blood pressure. This medication may increase the heart rate as a response to vasodilation. It decreases cardiac workload by decreasing preload and afterload. It decreases peripheral resistance by dilating peripheral blood vessels.

Which pain characteristic would the nurse expect to observe when a client is experiencing anginal pain? A. Unchanged by rest B. Precipitated by light activity C. Described as a knifelike sharpness D. Relieved by sublingual nitroglycerin

D. Relieved by sublingual nitroglycerin Relief by sublingual nitroglycerin is a classic reaction because it causes vasodilation of peripheral veins and arteries, thereby decreasing oxygen demand by decreasing preload. To a lesser extent, sublingual nitroglycerin dilates coronary arteries, which increases oxygen to the myocardium, thereby decreasing pain. Immediate rest frequently relieves anginal pain. Angina usually is precipitated by exertion, emotion, or a heavy meal. Angina usually is described as tightness, indigestion, or heaviness.

The nurse administers carbidopa-levodopa to a client with Parkinson's disease. Which activity describes the mechanism of action of this medication? A. Increase in acetylcholine production B. Regeneration of injured thalamic cells C. Improvement in myelination of neurons D. Replacement of a neurotransmitter in the brain

D. Replacement of a neurotransmitter in the brain Carbidopa-levodopa is used because levodopa is the precursor of dopamine. It is converted to dopamine in the brain cells, where it is stored until needed by axon terminals; it functions as a neurotransmitter. Regeneration of injured thalamic cells is not an action of this medication; neurons do not regenerate. Increase in acetylcholine production and improvement in myelination of neurons are not actions of this medication.

Which advice will the nurse include when teaching a client about digoxin for left ventricular failure? A. Sleep flat in bed. B. Follow a low-potassium diet. C. Take the pulse three times a day. D. Report increasing fatigue.

D. Report increasing fatigue. Treatment with digoxin should improve fatigue associated with heart failure; if fatigue increases, it may reflect complications of therapy. Sleeping with the head slightly elevated facilitates respiration. The client needs potassium. A low-potassium diet when the client is taking digoxin predisposes the client to toxicity and dangerous dysrhythmias. To avoid becoming obsessed with the pulse rate, the client should take the pulse less often; once daily is adequate.

Which adverse effect will the nurse monitor for in a client who has been administered a neuromuscular blocking agent before electroconvulsive therapy? A. Seizures B. Vomiting C. Loss of memory D. Respiratory depression

D. Respiratory depression A neuromuscular blocker such as succinylcholine produces respiratory depression because it inhibits contractions of respiratory muscles. This medication does not cause seizures. Because the client is not permitted anything by mouth for 8 to 10 hours before the treatment, vomiting is not a major problem. The loss of memory results from the electroconvulsive therapy, not from the neuromuscular blocking agent.

Which physiological function is the last function lost during anesthesia induction? A. Gag reflex B. Eyelid reflexes C. Voluntary control D. Respiratory movement

D. Respiratory movement There is no respiratory movement in stage 4 of anesthesia; before this stage, respirations are depressed but present. The gag reflex is lost in stage 3 of anesthesia. Eyelid reflexes are lost in stage 2 of anesthesia. Voluntary control is lost in stage 2 of anesthesia.

The nurse is caring for a client who has taken atenolol for 2 years. The healthcare provider recently changed the medication to enalapril to manage the client's blood pressure. Which instruction should the nurse provide the client regarding the new medication? A. Take the medication at bedtime. B. Report presence of increased bruising. C. Check pulse before taking medication. D. Rise slowly when getting out of bed or chair.

D. Rise slowly when getting out of bed or chair. The client's new medication is an angiotensin-converting enzyme (ACE) inhibitor, which has the side effect of orthostatic hypotension. Instructing the client to rise slowly from a sitting or lying down position is important to teach the client to avoid dizziness and potentially falling.

Which rationale explain why intravenous (IV) potassium is prescribed in addition to regular insulin for clients in diabetic ketosis? A. Potassium loss occurs rapidly from diaphoresis present during coma. B. Potassium is carried with glucose to the kidneys to be excreted in the urine in increased amounts. C. Potassium is quickly used up during the rapid series of catabolic reactions stimulated by insulin and glucose. D. Serum potassium levels will decrease as potassium ions shift from the extracellular fluid to the intracellular fluid compartment.

D. Serum potassium levels will decrease as potassium ions shift from the extracellular fluid to the intracellular fluid compartment. Insulin stimulates cellular uptake of glucose and stimulates the sodium/potassium pump, leading to the influx of potassium into cells. The resulting hypokalemia is offset by parenteral administration of potassium. Potassium is not lost from the body by profuse diaphoresis. Potassium moves from the extracellular to the intracellular compartment rather than being excreted in the urine. Anabolic reactions are stimulated by insulin and glucose administration; potassium is drawn into the intracellular compartment, necessitating a replenishment of extracellular potassium.

Which complication will occur as the result of administering sildenafil to a client who takes isosorbide dinitrate? A. Constipation B. Protracted vomiting C. Respiratory distress D. Severe hypotension

D. Severe hypotension Concurrent use of sildenafil and a nitrate, which causes vasodilation, may result in severe, potentially fatal hypotension. Protracted vomiting and respiratory distress are not medication interaction effects associated with concurrent use of sildenafil and a nitrate. Sildenafil may cause diarrhea; adding a nitrate will not cause constipation.

A client is receiving imatinib for chronic myelogenous leukemia (CML). The nurse will assess for which complication of this protein-tyrosine kinase inhibitor? Select all that apply. One, some, or all responses may be correct. A. Hair loss B. Stomatitis C. Dehydration D. Signs of infection E. Bleeding tendencies

D. Signs of infection E. Bleeding tendencies Imatinib affects the bone marrow, causing neutropenia; an adequate number of neutrophils are necessary to fight bacterial infections. Imatinib affects the bone marrow, causing thrombocytopenia; an adequate number of thrombocytes are necessary to prevent bleeding. Hair loss is a complication associated with antimetabolites. Stomatitis is a complication associated with antimetabolites and antitumor antibiotics. Severe fluid retention is a side effect, not dehydration.

A 48-year-old male client who is being admitted to the emergency department with an acute myocardial infarction (MI) gives the following list of medications to the nurse. Which medication would the nurse recognize as having the most immediate implications for the client's care? A. Losartan B. Captopril C. Furosemide D. Sildenafil

D. Sildenafil The nurse will need to avoid giving nitrates to the client because nitrate administration, commonly prescribed for clients experiencing an acute MI, is contraindicated in clients who are using sildenafil (a PDE5 inhibitor) because of the risk of severe hypotension caused by vasodilation. The other medications the client is taking should also be documented and reported to the health care provider (HCP) but do not have as immediate an impact on decisions about the client's treatment.

A client with heart failure is to receive digoxin. Which therapeutic effect is associated with this medication? A. Reduces edema B. Increases cardiac conduction C. Increases rate of ventricular contractions D. Slows and strengthens cardiac contractions

D. Slows and strengthens cardiac contractions Digoxin improves cardiac function by increasing the strength of myocardial contractions (positive inotropic effect) and, by altering the electrophysiological properties of the heart, slows the heart rate (negative chronotropic effect). Digoxin increases the strength of the contractions but decreases the heart rate. Although a reduction in edema may result from the increased blood supply to the kidneys, it is not the reason for administering digoxin. Digoxin decreases, not increases, cardiac impulses through the conduction system of the heart.

A client is prescribed rifampin after being exposed to active tuberculosis. Which finding would the nurse immediately report to the health care provider? Select all that apply. One, some, or all responses may be correct. A. Reddish-orange color urine B. Yellow-colored teeth stains C. Orange-colored sweat and tears D. Small, red, pinpoint areas on the arms E. Numbness, tingling, and burning of extremities

D. Small, red, pinpoint areas on the arms Pinpoint red areas that appear on the arms, legs, or trunk of the body are known as petechiae. The petechiae are tiny hemorrhages that occur under the skin as a result of a low circulating platelet count (thrombocytopenia). Thrombocytopenia occurs with liver stress or damage. As hepatotoxicity is a possible adverse reaction to rifampin, the health care provider must be notified of the appearance of petechiae. Reddish-orange colored urine or stool is a normal effect of the rifampin. Yellow-colored stains on the teeth are side effects that are not dangerous; however, there is no way to reverse the staining, and they may be permanent. Orange-colored sweat and tears are also normal side effects of rifampin, but they are not dangerous. Numbness, tingling, and burning of the extremities could indicate peripheral neuropathy, which can be treated with vitamin B 12, so this is not an immediate emergency.

When a client's cells are deprived of oxygen during a cardiac arrest, which medication corrects for deleterious effects of anaerobic energy production? A. Regular insulin B. Calcium gluconate C. Potassium chloride D. Sodium bicarbonate

D. Sodium bicarbonate In the absence of oxygen, the body derives its energy anaerobically; this results in a buildup of lactic acid. Sodium bicarbonate, an alkaline medication, will help neutralize the acid, raising the pH. Insulin is used to treat diabetes; it lowers blood sugar by facilitating transport of glucose across cell membranes. Calcium gluconate is used to treat hypocalcemia. Although potassium is essential for cardiac function, it will not correct acidosis. With acidosis, serum hydrogen ions will exchange with intracellular potassium, leading to a temporary hyperkalemic state; therefore potassium chloride is contraindicated until acidosis is corrected.

Which action is responsible for the therapeutic effect of docusate sodium? A. Lubricates the feces B. Creates an osmotic effect C. Stimulates motor activity D. Softens the feces

D. Softens the feces The detergent action of docusate sodium promotes the drawing of fluid into the stool, which softens the feces. Lubricating the feces in the gastrointestinal (GI) tract is the action of lubricant laxatives such as mineral oil. Creating an osmotic effect in the GI tract is the action of saline laxatives, such as magnesium hydroxide, or other osmotics, such as lactulose. Stimulating motor activity of the GI tract is the action of peristaltic stimulants, such as cascara.

A primary health care provider prescribes venlafaxine for a client with a diagnosis of major depressive disorder who has been taking herbal medications. Which herbal supplement is contraindicated when taking venlafaxine? A. Ginseng B. Valerian C. Kava-kava D. St. John's wort

D. St. John's wort A client who takes venlafaxine, a selective serotonin reuptake inhibitor, and St. John's wort concurrently is at risk for serotonin syndrome, a medication-induced excess of intrasynaptic serotonin. Ginseng can precipitate a hypertensive crisis in clients taking a monoamine oxidase inhibitor. Valerian (valerian root) can enhance sedation in clients taking a tricyclic antidepressant. Kava-kava can increase the risk of dystonic reactions in clients taking an antipsychotic medication.

A child is prescribed tetracycline. The nurse understands which possible medication-related reaction is associated with this medication? A. Kernicterus B. Gray syndrome C. Reye syndrome D. Staining of teeth

D. Staining of teeth Tetracycline causes staining or discoloration of developing teeth in children. Sulfonamides may cause kernicterus in neonates. Chloramphenicol may cause Gray syndrome in infants. Aspirin may cause Reye syndrome in pediatric clients with a history of chickenpox or influenza.

A health care provider prescribes bisacodyl for a client with constipation. The nurse explains to the client that this medication acts by which mechanism? A. Producing bulk B. Softening feces C. Lubricating feces D. Stimulating peristalsis

D. Stimulating peristalsis Bisacodyl stimulates nerve endings in the intestinal mucosa, precipitating a bowel movement. Bisacodyl is not a bulk cathartic. Bulk-forming laxatives, such as psyllium hydrophilic mucilloid, form soft, pliant bulk that promotes physiological peristalsis. Bisacodyl is not a stool softener. Stool softeners, such as docusate sodium, permit fat and water to penetrate feces, which softens and delays the drying of the feces. Bisacodyl is not an emollient. Emollient laxatives, such as mineral oil, lubricate the feces and decrease absorption of water from the intestinal tract.

A nurse receives a prescription to administer regular insulin U-500 to a client with diabetes mellitus. How will the nurse administer this medication? A. Intravenously using an infusion pump B. Subcutaneously using an insulin pump C. Intramuscularly using a U-100 syringe D. Subcutaneously using a U-500 syringe

D. Subcutaneously using a U-500 syringe Rationale: Insulin U-500 is a highly concentrated form of insulin that is five times stronger than regular insulin. This medication should only be administered subcutaneously with a specialized U-500 syringe for accurate measurement. Concentrated insulin should not be administered intravenously, intramuscularly, or via an insulin pump due to the high risk of hypoglycemic episodes.

Which technique would the nurse use to administer ferrous sulfate to a 12-month-old infant? A. Through a straw B. Crushed in applesauce C. In an intramuscular injection D. Syringe directed toward the back of the mouth

D. Syringe directed toward the back of the mouth Very young children should receive ferrous sulfate elixir through a syringe or medicine dropper placed in the back of the mouth; this limits staining of teeth by the ferrous sulfate. A 12-month-old infant may not be able to suck on a straw. A 12-month-old infant cannot swallow a tablet, and ferrous sulfate should not be crushed. Ferrous sulfate is not available in an injectable form.

Levofloxacin is prescribed for a woman who has been experiencing urinary frequency and burning for the past 24 hours. The nurse concludes the teaching has been effective when the client states she will make which change in her routine? A. Limit her fluid intake. B. Strain her urine for calculi. C. Monitor her urine output. D. Take mineral supplements 2 hours before or after levofloxacin.

D. Take mineral supplements 2 hours before or after levofloxacin. Mineral substances taken within 2 hours of a levofloxacin dose decrease the medication's effectiveness. Fluid intake should be increased to prevent crystalluria. Although the urine should be inspected for crystals, straining is not necessary. It is unnecessary to monitor urine output.

A client with giardiasis is taking metronidazole (Flagyl) 2 grams PO. Which information should the nurse include in the client's instruction? A. Notify the clinic of any changes in the color of urine. B. Encourage the use of over-the-counter cough/cold syrup when a cough/cold develops. C. Stop the medication after the diarrhea resolves. D. Take the medication with food.

D. Take the medication with food. Flagyl, an amoebicide and antibacterial agent, may cause gastric distress, so the client should be instructed to take the medication on a full stomach. Urine may be red-brown or dark from Flagyl, but this side effect is an expectant finding and not necessary to report to the healthcare provider. The client should also avoid using alcohol-containing products such as cough or cold syrups or mouthwash while taking the medication and for at least three days after stopping it.

Pyridostigmine is prescribed for a client with myasthenia gravis. Why would the nurse instruct the client to take pyridostigmine about 1 hour before meals? A. This timing limits first pass metabolism. B. Taking it on an empty stomach increases absorption. C. Taking it before meals decreases gastric irritation. D. Taking it before meals improves the ability to chew.

D. Taking it before meals improves the ability to chew. Peak action of the medication will occur during meals to promote chewing and swallowing and prevent aspiration. It should be given with a small amount of food to prevent gastric irritation. First pass metabolism is a process of metabolism, which is not affected by medication timing. Absorption is not affected significantly by the presence of food in the stomach. Gastric irritation is reduced best by the administration of medications with food, not on an empty stomach.

The nurse is teaching a client about an oral hypoglycemic medication. The nurse should place priority emphasis on which of the following points? A. Consulting with the health care provider about dose changes based on blood glucose B. Distinguishing signs and symptoms of hypoglycemia and hyperglycemia C. Adherence with recommended diet plan D. Taking the medication at specified times

D. Taking the medication at specified times A regular interval between doses should be maintained because oral hypoglycemics stimulate the islets of Langerhans to produce insulin. If doses are not spaced correctly, insulin levels may increase, causing hypoglycemia or decrease, causing hyperglycemia. The other actions are important and would be discussed after this initial point.

An adolescent with diabetes had a 6:30 AM fasting blood glucose level of 180 mg/dL (10.0 mmol/L). Which nursing action is a priority? A. Encourage the adolescent to start exercising. B. Ask the adolescent to obtain an immediate glucometer reading. C. Inform the adolescent that a complex carbohydrate such as cheese should be eaten. D. Tell the adolescent that the prescribed dose of rapid-acting insulin should be administered.

D. Tell the adolescent that the prescribed dose of rapid-acting insulin should be administered. A blood glucose level of 180 mg/dL (10.0 mmol/L) is above the average range, and the prescribed rapid-acting insulin is needed. Although exercise does decrease insulin requirements and does lower the blood glucose level, the immediate action of insulin is needed. Asking the adolescent to obtain an immediate glucometer reading is an action that will not correct the problem; the blood glucose level is already known. Food intake at this time will increase the level of blood glucose.

Which action would the nurse take to ensure client safety when caring for client with human immunodeficiency virus-associated Pneumocystis jiroveci pneumonia that is to receive pentamidine IV daily? Select all that apply A. Monitor for decreased serum K levels. B. Administer the medication over 30 minutes. C. Monitor blood pressure hypertension during therapy. D. Tell the client to report any evidence of bleeding immediately. E. Assess blood glucose levels daily and several times after therapy

D. Tell the client to report any evidence of bleeding immediately. E. Assess blood glucose levels daily and several times after therapy is completed. Any signs of bleeding (e.g., bleeding gums or blood in the urine, stool, or emesis), unusual bruising, or petechiae should be reported to the health care provider. Pentamidine may cause hypoglycemia or hyperglycemia even after therapy has been discontinued; therefore blood glucose levels should be monitored. Pentamidine may increase, not decrease, serum potassium levels. Administering the medication over a period of 30 minutes is too quick; the medication should be given over at least 60 minutes. Clients should be monitored closely for sudden, severe hypotension; they should lie flat when receiving the medication.

Permethrin 1% lotion is prescribed for a 5-year-old child with pediculosis capitis. Which instruction would the nurse include when teaching the parents about treatment? A. Personal belongings must be discarded. B. Side effects are nonexistent with the medicated shampoo. C. Other children should be kept away from the child for a week. D. The child's hair must be combed with a fine-toothed comb to remove nits.

D. The child's hair must be combed with a fine-toothed comb to remove nits. A fine-toothed comb removes any nits that remain after the application of permethrin 1% lotion. Personal belongings do not need to be disposed of; clothing and linens should be laundered in hot water and dried in a hot dryer, and other personal items may be soaked in a pediculicidal solution. Excessive use of permethrin 1% lotion may cause the lice to develop resistance to the shampoo. Once the hair has been shampooed, there is no reason to isolate the child.

A client with Parkinson's disease is prescribed benztropine (Cogentin). For which of the following should the nurse call the health care provider immediately? A. The client is complaining of dizziness when standing up. B. The client is exhibiting bradykinesia and slurred speech. C. The client's heart rate increased from 80 to 95 beats per minute. D. The client has a history of primary angle-closure glaucoma.

D. The client has a history of primary angle-closure glaucoma. The nurse must be able to recognize adverse drug effects and contraindications of medications commonly prescribed for the client with Parkinson's disease. Common clinical manifestations of Parkinson's disease include bradykinesia (slow movement), dysarthria (slurred speech) and orthostatic hypotension, caused by the loss of the neurotransmitter dopamine. The goal of pharmacotherapy is to restore the functional balance of dopamine and acetylcholine. This is achieved by giving dopaminergic drugs and cholinergic blockers.

A client received 40 mg of furosemide by mouth at 10 am. Which information is most important for the nurse to provide to the next nurse in the change-of-shift report? A. The client lost two pounds in the last 24 hours. B. The client is to receive another dose of furosemide at 10 pm. C. The client's potassium level was 4.0 mEq/L prior to administration. D. The client's urine output was 1500 mL over nine hours.

D. The client's urine output was 1500 mL over nine hours. Although all of the information is important to include, a diuresis of 1,500 mL is a very large amount and could cause hypokalemia, fluid volume deficit and hypotension. Therefore, it is the most important information to provide to the nurse on the next shift.

A health care provider prescribes Lactobacillus granules to a 3-month-old infant to manage postantibiotic diarrhea. Which explanation would the nurse give to the infant's parents about the reason for giving lactobacilli? A. They diminish the inflammatory mucosal edema. B. The discomfort caused by gastric hyperacidity is lessened. C. They relieve the pain caused by gas in the gastrointestinal tract. D. The flora that inhabit a healthy gastrointestinal tract must be recolonized.

D. The flora that inhabit a healthy gastrointestinal tract must be recolonized. Lactobacilli are part of the flora in the healthy gastrointestinal tract. The purpose of administering lactobacilli granules is to help recolonize the normal gastrointestinal flora that were destroyed with antibiotic therapy. The other options (diminishing mucosal edema, lessening discomfort from gastric hyperacidity, and pain relief from gas) are not the actions of lactobacilli granules.

Which rationale explains why the nurse would question a benzodiazepine prescription for an individual experiencing acute grief? A. The depression is magnified, and the risk of suicide increases. B. Brain activity is suppressed, and the risk of depression increases. C. Lethargy results, and it prevents a return to interpersonal activity. D. The period of denial is extended, and the grieving process is suppressed.

D. The period of denial is extended, and the grieving process is suppressed. With this sedating medication, the individual does not face the reality of the loss and merely delays the onset of the pain associated with it. Because most support is available at the time of the death and the funeral, a benzodiazepine at this time denies the individual the opportunity to use this assistance. This class of medications does not magnify the risk of suicide or cause or prevent depression. Although sedation and muscle relaxation initially may occur with these medications, they are not reasons for the medications not to be ordered.

The nurse is giving instructions to the parents of a child who has cystic fibrosis. Which information should the nurse emphasize about administration of pancreatic enzymes? A. Administer each time a high-carbohydrate meal is eaten B. Crush the tablet and sprinkle on food three times a day C. Dispense once daily with breakfast D. They are to be taken with every meal or snack

D. They are to be taken with every meal or snack Pancreatic enzymes are necessary for digesting fat, starch and protein. They should be taken with each meal and most snacks to allow for the proper digestion of the food. If taken on an empty stomach, they may cause gastric irritation and possibly ulcers. Enzyme capsules should be swallowed whole, not crushed or chewed, and the microspheres should not be sprinkled on or mixed with the whole meal.

The chemotherapy protocol prescribed for a client with tuberculosis includes vitamin B 6 and isoniazid (INH). Which would the nurse identify as the reason for prescribing vitamin B 6? A. To improve the nutritional status of the client B. To enhance the tuberculostatic effect of INH C. To accelerate the destruction of dormant tubercular bacilli D. To counteract the peripheral neuritis that INH may cause

D. To counteract the peripheral neuritis that INH may cause One of the most common side effects of INH is peripheral neuritis, and vitamin B 6 will counteract this problem. It does help nutrition, but that is not the specific reason it is given. It counters the side effects of isoniazid; it does not act to enhance its action. It does not speed the destruction of the causative organism.

Which purpose would the nurse identify as the reason for prescribing vitamin B 6 when a chemotherapy protocol prescribed for a client with tuberculosis includes vitamin B 6 and isoniazid (INH)? A. To improve the nutritional status of the client B. To enhance the tuberculostatic effect of INH C. To accelerate the destruction of dormant tubercular bacilli D. To counteract the peripheral neuritis that INH may cause

D. To counteract the peripheral neuritis that INH may cause One of the most common side effects of INH is peripheral neuritis, and vitamin B 6 will counteract this problem. It does help nutrition, but that is not the specific reason it is given. It counters the side effects of isoniazid; it does not act to enhance its action. It does not speed the destruction of the causative organism.

Which adverse effect of imipramine requires further assessment and possible immediate medical intervention? A. Dry mouth B. Weight gain C. Blurred vision D. Urinary hesitancy

D. Urinary hesitancy Urinary hesitancy and retention are adverse effects of imipramine that may require immediate medical intervention. Dry mouth, weight gain related to increased appetite, and blurred vision may occur as side effects of imipramine; they usually decrease over time or can be managed through nursing interventions.

Which step would the nurse include during the administration of epoetin prescribed to a client with acquired immunodeficiency syndrome (AIDS)? A. Administer the medication via the Z-track technique. B. Shake the vial before withdrawing the solution. C. Obtain the client's pulse rate before administration. D. Use a syringe that has a 1-inch (2.5-cm), 25-gauge needle.

D. Use a syringe that has a 1-inch (2.5-cm), 25-gauge needle. Epoetin is administered via the subcutaneous or intravenous route; a 1-inch (2.5-cm), 25-gauge needle is appropriate for either method of administration. Epoetin is not administered via the intramuscular route, so the Z-track technique is not used. Shaking the vial denatures the glycoprotein, making the medication biologically inactive and therefore ineffective. The client's vital signs, particularly the blood pressure, need to be monitored only routinely to determine the effectiveness of the medication.

The nurse is preparing to apply nitroglycerin ointment. Before applying the ointment, which action will the nurse take? A. Assess the client's pulse rate. B. Prepare the site with an alcohol swab. C. Shave the client's chest in the area for application. D. Use the dose measuring application paper and spread the ointment in a thin layer to the prescribed amount.

D. Use the dose measuring application paper and spread the ointment in a thin layer to the prescribed amount. The nurse would use the dose measuring application paper supplied with the ointment and spread in a thin layer to the prescribed amount and place side down on the desired skin. The nurse would assess blood pressure reading, not pulse rate. There is no need to clean the site with alcohol before administration. Shaving is not recommended; a hairless site on the chest, back, abdomen, or anterior thigh should be selected.

A client has been admitted for the second time to treat tuberculosis (TB). Which referral does the nurse initiate as a priority? A. Psychiatric nurse liaison to assess reasons for noncompliance B. Infection control nurse to arrange testing for drug resistance C. Social worker to see if the client can afford the medications D. Visiting nurses to arrange for directly observed therapy (DOT)

D. Visiting nurses to arrange for directly observed therapy (DOT) Clients with TB must take multiple drugs for six months or longer, making adherence a very real problem. Non-adherence is the most common cause of treatment failure and relapse. This client has a risk of non-adherence, as evidenced because this is their second admission to treat TB. When the client is discharged, they most likely will need to be placed on DOT to ensure compliance. This is the priority referral in order to prevent transmission of TB to others in the community. The other referrals may also be appropriate depending on the client's needs.

A client with coronary artery disease who is taking digoxin (Lanoxin) receives a new prescription for atorvastatin (Lipitor). Two weeks after initiation of the Lipitor prescription, the nurse assesses the client. Which finding requires the most immediate intervention? A. Heartburn. B. Headache. C. Constipation. D. Vomiting.

D. Vomiting. Vomiting, anorexia, and abdominal pain are early indications of digitalis toxicity. Since Lipitor increases the risk for digitalis toxicity, this finding requires the most immediate intervention by the nurse.

Which prescription would the nurse anticipate for the client who takes a medication that interferes with fat absorption? A. High-fat diet B. Supplemental cod liver oil C. Total parenteral nutrition (TPN) D. Water-miscible forms of vitamins A and E

D. Water-miscible forms of vitamins A and E Vitamins A, D, E, and K are known as fat-soluble vitamins because bile salts and other fat-related compounds aid their absorption. A high-fat diet will not achieve the uptake of fat-soluble vitamins in this client. Supplemental cod liver oil will not achieve the uptake of fat-soluble vitamins in this client. TPN is unnecessary; a well-balanced diet is preferred. Water-miscible forms of vitamins A and E can be absorbed with water-soluble nutrients.

A client with laryngeal cancer is receiving chemotherapy. Which laboratory report is most important for the nurse to monitor when considering the effects of chemotherapy? A. Platelets B. Hemoglobin level C. Red blood cell count D. White blood cell count

D. White blood cell count Antineoplastic medications depress bone marrow, which results in leukopenia; the client must be protected from infection, which is a primary cause of death in the client with cancer. Platelets may decrease rapidly, but complications may be limited by infusions of platelets. Although the hemoglobin level diminishes, a transfusion with packed red blood cells (PRBCs) will alleviate the anemia. RBCs diminish slowly and may be replaced with a transfusion of PRBCs.

A client who is immunosuppressed is receiving filgrastim. When monitoring effectiveness, the nurse will check for an increase in which blood component? A. Platelets B. Erythrocytes C. Lymphocytes D. White blood cells

D. White blood cells Filgrastim, a granulocyte colony-stimulating factor, increases the production of neutrophils with little effect on the production of other blood components. The production of platelets is not stimulated by filgrastim. The production of erythrocytes is not stimulated by filgrastim. Neutrophils, not lymphocytes, are the white blood cells whose production is stimulated by filgrastim.

A complete blood count is prescribed before each round of a client's cancer chemotherapy. Which component of the complete blood count is of greatest concern to the nurse? A. Platelets B. Hematocrit C. Red blood cells (RBCs) D. White blood cells (WBCs)

D. White blood cells (WBCs) Antineoplastic medications depress bone marrow, which causes leukopenia; the client must be protected from infection, which can be life-threatening. RBCs diminish slowly and can be replaced with a transfusion of packed RBCs. Platelets decrease as rapidly as WBCs, but complications can be limited with infusions of platelets.

To minimize the side effects of the vincristine that a client is receiving, which diet would the nurse advise? Low in fat High in iron High in fluids Low in residue

High in fluids A common side effect of vincristine is a paralytic ileus that results in constipation. Preventative measures include high-fiber foods and fluids that exceed minimum requirements. These will keep the stool bulky and soft, thereby promoting evacuation. Dietary plans that are low in fat, high in iron, and low in residue will not provide the roughage and fluids needed to minimize the constipation associated with vincristine.

The nurse is planning an evening snack for a child receiving NPH insulin. The nurse offers a snack for which reason? It encourages the child to stay on the diet. Energy is needed for immediate utilization. Extra calories will help the child gain weight. Nourishment helps counteract late insulin activity.

Nourishment helps counteract late insulin activity. A bedtime snack is needed for the evening. NPH insulin is intermediate-acting insulin, which peaks 4 to 12 hours later and lasts for 18 to 24 hours. Protein and carbohydrate ingestion before sleep prevents hypoglycemia during the night when the NPH is still active. The snack is important for diet-insulin balance during the night, not encouragement. There are no data to indicate that extra calories are needed; a bedtime snack is routinely provided to help cover intermediate-acting insulin during sleep. The snack must contain mainly protein-rich foods, not simple carbohydrates, to help cover the intermediate-acting insulin during sleep.

Which finding would the nurse report to the health care provider when caring for a client prescribed temazepam at bedtime? Anxiety Drowsiness Sleep driving Morning headache

Sleep driving Benzodiazepines are frequently used at bedtime for treating insomnia. However, some clients taking the medications may experience the side effect of performing tasks without remembering the events. When these side effects occur at night, dangerous sleep-related behaviors, such as sleep driving or preparing and consuming meals, can occur. If a client reports these side effects, the nurse will need to report the finding to the health care provider immediately to ensure the client's safety. Anxiety, drowsiness, and morning headaches are common findings during treatment with benzodiazepines and do not need to be reported to the health care provider.

Which instruction about phenytoin will the nurse provide during discharge teaching to a client with epilepsy who is prescribed phenytoin for seizure control? A. "Antiseizure medications will probably be continued for life." B. "Phenytoin prevents any further occurrence of seizures." C. "This medication needs to be taken during periods of emotional stress." D. "Your antiseizure medication usually can be stopped after a year's absence of seizures."

A. "Antiseizure medications will probably be continued for life." Seizure disorders usually are associated with marked changes in the electrical activity of the cerebral cortex, requiring prolonged or lifelong therapy. Seizures may occur despite medication therapy; the dosage may need to be adjusted. A therapeutic blood level must be maintained through consistent administration of the medication irrespective of emotional stress. Absence of seizures will probably result from medication effectiveness rather than from correction of the pathophysiological condition.

The nurse prepares discharge instructions for a client who will take enalapril for hypertension. Which instruction would the nurse include in the client's teaching? A. 'Change to a standing position slowly.' B. 'This may color your urine green.' C. 'The medication may cause a sore throat for the first few days.' D. 'Schedule blood tests weekly for the first 2 months.'

A. 'Change to a standing position slowly.' Enalapril is classified as an angiotensin-converting enzyme (ACE) inhibitor. Like many antihypertensives, it can cause orthostatic hypotension. Clients should be advised to change positions slowly to minimize this effect. This medication does not alter the color of urine or cause a sore throat the first few days of treatment. Presently, there are no guidelines that suggest blood tests are required weekly for the first 2 months.

A client with acute myocardial infarction is admitted to the coronary care unit. Which medication should the nurse administer to lessen the workload of the heart by decreasing the cardiac preload and afterload? A. Nitroglycerin. B. Propranolol C. Morphine. D. Captopril

A. Nitroglycerin. Nitroglycerin is a nitrate that causes peripheral vasodilation and decreases contractility, thereby decreasing both preload and afterload.

Which adverse effect would the nurse continually assess for in a client receiving valproic acid? A. Yellow sclerae B. Motor restlessness C. Ringing in the ears D. Torsion of the neck

A. Yellow sclerae Yellow sclerae are a sign of jaundice; pancreatitis and hepatic failure are life-threatening adverse effects of valproic acid. The client must have frequent liver function tests. Motor restlessness (akathisia) is associated with antipsychotic medications. Ringing or buzzing in the ears (tinnitus) is associated with aspirin. Torsion of the neck (torticollis) because of contracted cervical muscles is associated with antipsychotic medications.

The nitrate isosorbide dinitrate is prescribed for a client with angina. Which instruction should the nurse include in this client's discharge teaching plan? A. Quit taking the medication if dizziness occurs. B. Do not get up quickly. Always rise slowly. C. Take the medication with food only. D. Increase your intake of potassium-rich foods.

B. Do not get up quickly. Always rise slowly. An expected side effect of nitrates is orthostatic hypotension and the nurse should instruct the client to prevent it by rising slowly.

Which medication is unsafe to administer as an intravenous (IV) bolus? A. Saline flush B. Potassium chloride C. Naloxone D. Adenosine

B. Potassium chloride Potassium chloride given as an IV bolus can cause cardiac arrest. It must be diluted and infused slowly through an IV infusion pump. Saline flush, naloxone, and adenosine are appropriate to be given as an IV bolus undiluted.

Which nursing diagnosis is important to include in the plan of care for a client receiving the angiotensin-2 receptor antagonist irbesartan (Avapro)? A. Fluid volume deficit. B. Risk for infection. C. Risk for injury. D. Impaired sleep patterns.

C. Risk for injury. Avapro is an antihypertensive agent, which acts by blocking vasoconstrictor effects at various receptor sites. This can cause hypotension and dizziness, placing the client at high risk for injury.

A health care provider prescribes furosemide for a client with hypervolemia. The nurse recalls that furosemide exerts its effects in which part of the renal system? A. Distal tubule B. Collecting duct C. Glomerulus of the nephron D. Loop of Henle

D. Loop of Henle Furosemide acts in the ascending limb of the loop of Henle in the kidney. Thiazides act in the distal tubule in the kidney. Potassium-sparing diuretics act in the collecting duct in the kidney. Plasma expanders, not diuretics, act in the glomerulus of the nephron in the kidney.

Which sign of hypokalemia will the nurse monitor for in a client receiving furosemide? A. Chvostek sign B. Muscle weakness C. Anxious behavior D. Abdominal cramping

Muscle weakness With hypokalemia, failure occurs in myoneural conduction and smooth muscle functioning, resulting in fatigue and muscle weakness. Chvostek sign, the contraction of the facial muscles in response to a light tap over the facial nerve in front of the ear, is associated with hypocalcemia; low calcium levels allow sodium to move into excitable cells, increasing depolarization and nerve excitability. Anxiety and irritability are associated with hyperkalemia. Hyperkalemia affects the nervous and muscular systems; fatigue, weakness, and lethargy are associated with hypokalemia. Decreased gastrointestinal motility occurs with hypokalemia; abdominal cramping is associated with hyperkalemia and is caused by hyperactivity of smooth muscles.

A client is admitted to the emergency department after experiencing a seizure. Which action would the nurse take first? Ask the emergency provider for a prophylactic anticonvulsant. Obtain a history of seizure type and incidence. Ask the client to remove any dentures and eyeglasses. Observe the client for increased restlessness and agitation.

Obtain a history of seizure type and incidence. Data collection is an essential first step for a client with a seizure disorder; it should always include a history of the seizures (e.g., type and incidence). Because different seizure medicines are used to control different seizure types, it is important to determine the type before treating. Although dentures and eyeglasses may be removed during a seizure, the client's normal routines should be respected. Increased restlessness may be evidence of the prodromal phase of a seizure in some individuals, but signs and symptoms vary so widely that the client's history should be obtained.


Related study sets

Cisco Module 3: Protecting your privacy & data

View Set

Lecture 3 Exam: TRUE / FALSE QUESTIONS

View Set

Information Security Framework, Infrastructure & Architecture

View Set

Health - Group Accident And Health Insurance - Quiz

View Set

Corporate Financial Management Exam 1

View Set